{"question": "According to NYHA grading, Patients with marked limitation to physical activity belongs to", "exp": "New York Hea Association Functional Classification of Cardiac diseaseGrade I: Patients have no limitation of physical activityGrade II: Patients have slight limitation of physical activityGrade III: Patients have marked limitation of physical activityGrade IV: Patients have an inability to carry on the physical activity without symptomsReference: Practical Guide to High-Risk Pregnancy and Delivery; Chapter 20; Cardiac Disease and Pregnancy", "cop": 3, "opa": "Grade I", "opb": "Grade II", "opc": "Grade III", "opd": "Grade IV", "subject_name": "Gynaecology & Obstetrics", "topic_name": "Medical, surgical and gynaecological illness complicating pregnancy", "id": "e3b8b163-13c9-4d15-9339-215c551dfeee", "choice_type": "single"} {"question": "Coaptation is", "exp": "Submucosal vascular network aids in Urethral mucosal approximation called as coaptation, which improves continence.", "cop": 1, "opa": "Urethral mucosal approximation.", "opb": "Adaptation of bladder to large volumes of filtrate.", "opc": "Stretch of urethra while voiding.", "opd": "Constriction of ureters in response to low volumes of urine.", "subject_name": "Gynaecology & Obstetrics", "topic_name": null, "id": "1e865ed7-ce5f-468b-b496-decb872f2932", "choice_type": "single"} {"question": "Contraindication of induction of labour among the following is", "exp": "Methods to induce or augment labor are contraindicated by most conditions that preclude spontaneous labor or delivery. The few maternal contraindications are related toPrior uterine incisionType contracted or distoed pelvic anatomy,Abnormally implanted placentas, and Uncommon conditions such as active genital herpes infection or cervical cancer. Fetal factors include Appreciable macrosomiaSevere hydrocephalusMalpresentationNonreassuring fetal statusRef: Willaim&;s Obstetrics; 25th edition", "cop": 1, "opa": "Contracted pelvis", "opb": "Bad obstetrical history", "opc": "Diabetes", "opd": "Hea disease", "subject_name": "Gynaecology & Obstetrics", "topic_name": "General obstetrics", "id": "2c2932a5-7d82-451b-9734-67a5c7499fa1", "choice_type": "single"} {"question": "Germ cells appear in yolk sac at", "exp": "By 11th to 12th-day cells appear between the inner surface of the cytotrophoblast and the outer surface of the yolk sac. \nThese cells lie loosely and are called extraembryonic mesoderm. A cavity formed within it called the extraembryonic coelom or chorionic cavity. \nThis covers both the yolk sac and amniotic cavity except where the embryoblast is connected to trophoblast through the connecting stalk", "cop": 3, "opa": "9 weeks", "opb": "6 weeks", "opc": "3 weeks", "opd": "5 weeks", "subject_name": "Gynaecology & Obstetrics", "topic_name": null, "id": "2e01723b-027f-4eb9-aad6-643f9636f29c", "choice_type": "single"} {"question": "Oral hypoglycemic safely given in pregnancy is", "exp": "Oral hypoglycemic drugs have found to be effective and safe. Commonly used drugs are Glibenclamide and metformin. Both the drugs cross the placenta However, No teratogenic effect has been observed yet. Glibenclamide appears to be superior when compared to metformine. Reference: DC.Duttas textbook of OBG,9th edition,page 267", "cop": 1, "opa": "Metformin", "opb": "Sitagliptin", "opc": "Glimepride", "opd": "Pioglitazone", "subject_name": "Gynaecology & Obstetrics", "topic_name": "Medical, surgical and gynaecological illness complicating pregnancy", "id": "d979104a-ea10-4062-9a98-679212d43d34", "choice_type": "single"} {"question": "Best investigation endometriosis is", "exp": "Laparoscopy is an operation performed in the abdomen or pelvis through small incisions with the aid of a camera. \nThe laparoscope aids diagnosis or therapeutic interventions with a few small cuts in the abdomen", "cop": 1, "opa": "Laparoscopy", "opb": "USG", "opc": "X-ray pelvis", "opd": "CT Scan", "subject_name": "Gynaecology & Obstetrics", "topic_name": null, "id": "673cf6dc-2a99-4843-bfc6-561f2de65d0f", "choice_type": "single"} {"question": "Most common abnormality in first trimester abortion", "exp": "A. i.e. (Defective genes) (161 - Dutta)* Genetic factor (50%) of early miscarriage are due to chromosomal abnormality in the conceptus. Autosomal trisomy is the commonest (50%) cytogenic abnormality in FIRST TRIMESTER*** Cervical incompetence (congenital or acquired) is the most common abnormality in SECOND TRIMESTER*** The cause of fetal death in ectopic pregnancy is postulated as vascular accident*** Most valuable diagnostic test in a case of suspected ectopic pregnancy is - Transvaginal USG**", "cop": 1, "opa": "Defective genes", "opb": "Cervical incompetence", "opc": "Placental & membrane abnormality", "opd": "Retroversion", "subject_name": "Gynaecology & Obstetrics", "topic_name": "Miscellaneous (Gynae)", "id": "747c876a-b665-4c4e-acb3-99778abaf32b", "choice_type": "single"} {"question": "Test used to differentiate maternal from fetal blood", "exp": "The Apt test is most commonly used in cases of vaginal bleeding late during pregnancy (antepaum haemorrhage) to determine if the bleeding is from the mother or the fetus.", "cop": 3, "opa": "Osmotic fragility test", "opb": "Water bulb test", "opc": "Apt test", "opd": "Kleihauer Betke test", "subject_name": "Gynaecology & Obstetrics", "topic_name": "All India exam", "id": "058bf897-54aa-4560-a583-185f9b4eb28f", "choice_type": "single"} {"question": "Daily Iodine requirement during pregnancy is", "exp": "Recommendation of the daily requirement of iodine during pregnancy is 220mcg and during lactation is 290mcg.(Ref: William's Obstetrics; 25th edition)", "cop": 3, "opa": "75mcg", "opb": "150mcg", "opc": "220mcg", "opd": "500mcg", "subject_name": "Gynaecology & Obstetrics", "topic_name": "All India exam", "id": "0ff23e5d-5011-4ace-a77b-89922d5b50ed", "choice_type": "single"} {"question": "Leiomyomatosis is", "exp": "Extra-uterine smooth - muscle tumors, Which are benign yet infiltrative, may develop in women with concurrent or prior uterine leiomyoma ⇒ Leiomyomatosis.", "cop": 3, "opa": "Malignant tumor", "opb": "Non-infiltrative tumor", "opc": "Extra uterine tumor", "opd": "uterine fibroid", "subject_name": "Gynaecology & Obstetrics", "topic_name": null, "id": "475e75cc-5c47-4d92-a410-866bfcb0c864", "choice_type": "single"} {"question": "Ambiguous genitalia in males is due to", "exp": "5 - α Reductase deficiency causes ambiguous genitalia in males. Rest all options cause ambigous genitalia in females.", "cop": 3, "opa": "Congenital adrenal hyperplasia", "opb": "Cushing's disease", "opc": "5 - alpha reductase deficiency", "opd": "Aromatase deficiency", "subject_name": "Gynaecology & Obstetrics", "topic_name": null, "id": "a95be71c-431a-4330-8541-81d8f8f85324", "choice_type": "single"} {"question": "Condition not responsive to medical therapy", "exp": "Ans. is'c' i.e., Hydatidiform moleIt is an abnormal condition o[ the placenta where there are paly degenerative and paly proliferative changes in the young chorionic villi.It is best regarded benign neoplasia of the chorion with rnalignant potential.Management: Suction evacuation", "cop": 3, "opa": "Invasive mole", "opb": "Choriocarcinoma", "opc": "Hydatidiform mole", "opd": "Persistent gestational trophoblastic disease", "subject_name": "Gynaecology & Obstetrics", "topic_name": null, "id": "4822ee17-9cb5-4a05-b6c8-d0177eb763dc", "choice_type": "single"} {"question": "Caudal regression syndrome is seen in babies of the mother having", "exp": "The caudal regression sequence is a rare malformation frequently associated with maternal diabetes.Ref: William&;s Obstetrics; 24th edition; Chapter 57; Diabetes Mellitus", "cop": 4, "opa": "PIH", "opb": "Anemia", "opc": "Cardiac disease", "opd": "Diabetes", "subject_name": "Gynaecology & Obstetrics", "topic_name": "Medical, surgical and gynaecological illness complicating pregnancy", "id": "b07952ad-c2cc-4471-922e-6775ace8e573", "choice_type": "single"} {"question": "The most sensitive method for detecting cervical chlamydia trachomatis infection is", "exp": "PCR", "cop": 3, "opa": "Direct fluorescent antibody test", "opb": "Enzyme immunoassay", "opc": "PCR", "opd": "Culture on irradiated McConkey cells", "subject_name": "Gynaecology & Obstetrics", "topic_name": null, "id": "93818020-e4e4-4817-ace2-a4340f93d019", "choice_type": "single"} {"question": "Emergency presentation of imperforate hymen is", "exp": "Imperforate hymen present with Amenorrhea with secondary sexual characters Cyclical abdominal discomfo Retention of urine Palpation of midline hypogastric lump Ref: Shaw Gynecology 17 e pg 67.", "cop": 1, "opa": "Retention of urine", "opb": "Mass abdomen", "opc": "Cyclic pain", "opd": "Hematocolpos", "subject_name": "Gynaecology & Obstetrics", "topic_name": "Congenital malformations", "id": "2d2a0f6e-aaf9-4817-bd94-cdc818e94353", "choice_type": "single"} {"question": "After delivery of Hea disease patient the following should be given", "exp": "Inj. furosemide is given to decrease preload in hea disease patient post delivery. Labour Management of Cardiac Patient Limit I.V. fluids Position - semi recumbent Pain relief - opioids and epidural analgesia avoid straining in 2nd stage of labour- Cut sho it by forceps/vaccum Inj. furosemide and I.V./ I.M. Oxytocin given to decrease preload methyl ergometrine is C/I Observation in high risk ward for 24 hrs.", "cop": 3, "opa": "Ace inhibitors", "opb": "Methergine", "opc": "Furosemide", "opd": "IVF fluids 1000 ml over next 4 hours", "subject_name": "Gynaecology & Obstetrics", "topic_name": "Medical Illness Complicating Pregnancy", "id": "aff81ecd-f251-4a45-84c1-c4d67da81255", "choice_type": "single"} {"question": "Ideal contraceptive for lactating mothers is aEUR'", "exp": "Lactational amenorrhoea An almost similar question was asked previously in 'All India' examination. Note that previously 'Most effective' contraceptive was asked. Now the question is different -Ideal' contraceptive is asked. Practically, there are no ideal contraceptives (neither for lactating women, nor for non-lactating women). An Ideal contraceptive is one which is - 100% effective, 100% safe, reversible with no side effects, and minimal effect on lactation. The Barrier methods and Combined OCPs can be easily ruled out; as barrier methods have high failure rates if used alone, while combined OCPs have a negative effect on breast milk production. Lactational amenorrhoea is the most ideal contraceptive for a lactating women, as it has viually no side effects, rather it has plethora of benefits for both mother and child. About effectiveness, it is almost equally effective to Progesterone only pills (POPs) when used perfectly. (see the below table) Failure rates: Contraceptive method Typical use Perfect use LAM (6 months only) 2.0 0.5 IUD 0.8 0.6 POP 'mini pill' 8.0 0.5 Combined oral contraceptive 6-8 0.1 Barrier method 14 3.0 Lactational amenorrhoea (LAM) is a natural, safe and effective contraceptive method which can be used upto 6 months post paum if the following conditions are met. (Note that LAM is not pure simple exclusive breast feeding; following conditions need to be fulfilled) I. The mother should not experience vaginal bleeding after the 8 weeks postpaum. The baby is less than 6 months old. The baby is exclusively breastfed. Breast feeding is done at least every 4 hrs. during the day and every 6 hrs during night. If the above conditions are fulfilled then LAM can give more than 98% protection from pregnancy. It has the added benefit of promoting exclusive breast feeding which is the ideal nutrition for infants. Lactational Amenorrhoea Advantages Disadvantages * Very effective if used perfectly * Can be used only for a sho period * Has no side effects (6 months post paum) * Promotes exclusive breast feeding, and therefore enhances maternal and infant health. * Does not require inseion of any device at the time of sexual intercourse * Can be initiated immediately postpaum * Is economical and requires no commodities or supplies * Reduces mother's lifetime risk of breast cancer. Physiology of LAM: Infant's suckling is the stimulus that initiates the state of lactational amenorrhea for breastfeeding women. The act of suckling elevates prolactin levels, and reduces GnRH from the hypothalamus. This reduces LH release thus inhibiting .follicular maturation. This prevents ovulation. But, even with continued nursing, ovulation eventually returns after 6 months, so another method of contraception should be used 6 months postpaum, or sooner if the menses resume. Ovalation has been seen to resume even earlier in some patients, so some books advice 'Rule of 3' i.e. - another contraceptive method should be use along with LAM beginning from the 3rd postpaum month. - and if the women is paially breastfeeding or not breastfeeding, the contraceptive should begin by the 3rd postpaum week. Progesterone only pills are the most effective contraceptive during lactation amongst the options provided. They have drawbacks like: - Have to be taken regularly at the same time - Cause spotting or irregular bleeding, ectopic pregnancy, headache, nausea, acne, hirsuitism and weight gain", "cop": 3, "opa": "Barrier method", "opb": "Combined OCP", "opc": "Lactational amenorrhoea", "opd": "Progesterone only pill", "subject_name": "Gynaecology & Obstetrics", "topic_name": null, "id": "50d1f284-0e5e-4b08-817c-2046fcfa87d4", "choice_type": "single"} {"question": "37 weeks primi with uterine contraction for 10 hours, cervix is 1 cms dilated and poorly effaced. Management is", "exp": "The patient is presenting at 37 weeks with mild labor pains for 10 hours. Cervix is 1 cms dilated and is not effaced. Now this can either be a case of false labor pains or it can be prolonged latent phase of labor.\nLack of progress during the latent phase is defined as lack of change or minimal change in cervical effacement and dilatation during a 2 hour period in a woman having regular uterine contractions.\nThe patient in the question has been having mild labor pains for 10 hours with no change in cervical dilatation and effacement. But again the question does not specify whether she is having regular uterine contractions or not.\n\nFalse labor can be differentiated from latent phase of labor by therapetuic rest i.e., patient is sedated with morphine.", "cop": 4, "opa": "Cesarean section", "opb": "Amniotomy", "opc": "Oxytocin drip", "opd": "Sedation and wait", "subject_name": "Gynaecology & Obstetrics", "topic_name": null, "id": "55a2945d-6799-48a4-ab14-4359f355b467", "choice_type": "single"} {"question": "The following complication is seen in", "exp": "Couvelaire Uterus: Seen in cases of placental abruption. At the time of cesarean delivery, it is not uncommon to find widespread extravasation of blood into the uterine musculature and beneath the serosa. It is named after Couvelaire, who in the early 1900s termed it uteroplacental apoplexy. Effusions of blood are also seen beneath the tubal serosa, between the leaves of the broad ligaments, in the substance of the ovaries, and free in the peritoneal cavity. These myometrial hemorrhages seldom cause uterine atony, and alone they are not an indication for hysterectomy(Ref: William's Obstetrics; 25th edition)", "cop": 2, "opa": "Diabetes complicating pregnancy", "opb": "Hypeension complicating pregnancy", "opc": "Placenta pre complicating pregnancy", "opd": "Preterm delivery", "subject_name": "Gynaecology & Obstetrics", "topic_name": "All India exam", "id": "ff925fc3-09a8-480c-82fe-3d40ad785f0d", "choice_type": "single"} {"question": "Gas most commonly used in laparoscopy is", "exp": "Ans. is a i.e. CO2 CO2 is the gas used to create pneumoperitoneum during laparoscopy. Other option is N20. But it is expensive, less soluble in blood and suppos combustion. Also know : Instrument used for creating pneumoperitoneum is veress needle. Flow Rate of CO2 for creating pneumoperitoneum 200 -- 2000 ml/min and pressure between 15 -- 25 mm of Hg.", "cop": 1, "opa": "CO2", "opb": "SO2", "opc": "N2", "opd": "O2", "subject_name": "Gynaecology & Obstetrics", "topic_name": null, "id": "61dd21cd-674e-4428-9cb5-08788d44bc0f", "choice_type": "single"} {"question": "Infertility in women with endometriosis is most likely due to", "exp": "Adhesions causing tubal obstruction and thereby impairing oocyte - pickup is responsible to cause infertility in women with endometriosis.", "cop": 1, "opa": "Tubal obstruction", "opb": "Ovulatory dysfunction", "opc": "Defect in implantation", "opd": "Anti - sperm antibodies", "subject_name": "Gynaecology & Obstetrics", "topic_name": null, "id": "4a2cf580-8990-4e5d-9c89-4e38f2cb2aaf", "choice_type": "single"} {"question": "The risk of endometrial cancer is highest with the following histological pattern of endometrial hyperplasia", "exp": "Ans. is d i.e. Complex hyperplasia with atypia Friends, let's first try to figure out this question on the basis of our basic knowledge of neoplasia. We all have read time and again that in 'Neoplasia' atypical cells are present, therefore obviously out of the given options, either option \"b\"or -d\"is correct and since neoplasia is most likely to occur in complex situations therefore out of the given options even if I don't know anything about Endometrial hyperplasia I would have gone for option \"d\". Let's now have a look at Endometrial hyperplasia. Endometrial Hyperplasia It represents a spectrum of morphological and biological alteration of endometrium (both glandsdeg +Stromadeg), ranging from an exaggerated physiological state to carcinoma in situ. They are clinically impoant because : They cause abnormal bleeding.deg Either preceede or occur simultaneously with Endometrial carcinoma. Endometrial Hyperplasia can be : If atypical cells are present in either of the two types - chances of malignancy increase. Simple atypical hyperplasia is a relatively uncommon diagnosis. In general, most atypical hyperplasias have a complex architecture. Characteristics of Atypical cells :deg You must have already learnt in pathology, but I will brief it up here for convenience : Large in sizedeg Loss of polaritydeg Irregular shapedeg Hyperchromatic nucleusdeg and prominent nucleolideg Altered nucleus/cytoplasmic ratio.deg Chances of Progression to Carcinoma : Type of hyperplasia Simple without atypia Complex without atypia Simple with atypia Complex with atypia Thus, minimum chances of progression to carcinoma are with simple-hyperplasia without atypia and maximum chances of carcinoma are with complex hyperplasia with atypia.deg", "cop": 4, "opa": "Simple hyperplasia without atypia", "opb": "Simple hyperplasia with atypia", "opc": "Complex hyperplasia without atypia", "opd": "Complex hyperplasia with atypia", "subject_name": "Gynaecology & Obstetrics", "topic_name": null, "id": "7b453585-f4d9-4501-9708-2c6c54c27893", "choice_type": "single"} {"question": "Fecundability is", "exp": "Fecundability is probability to achive pregnancy within one menstrual cycle.", "cop": 2, "opa": "Probability to achieve a live birth within one menstrual cycle", "opb": "Probability to achive pregnancy within one menstrual cycle", "opc": "Probability to achive a twin pregnancy within one menstrual cycle", "opd": "Probability to achive a live birth within one year of treatment for infertility", "subject_name": "Gynaecology & Obstetrics", "topic_name": null, "id": "b93b4e22-d0b7-420e-b944-ab5812292ad6", "choice_type": "single"} {"question": "Serum testosterone levels exceeding 200ng /dL is suggestive of", "exp": "Very high testosterone levels are suggestive of ovarian or adrenal androgen-producing tumor.", "cop": 2, "opa": "PCOD", "opb": "Adrenal tumor", "opc": "Metabolic syndrome", "opd": "Ovarian Atresia", "subject_name": "Gynaecology & Obstetrics", "topic_name": null, "id": "cb696c45-ea90-4624-98cb-407ece8ecfba", "choice_type": "single"} {"question": "CPD is best assessed by", "exp": "Methods of assesment of Pelvis \n\nClinically\nImaging studies\n\nClinical Pelvimetry :\n\nDone manually\nCan be done beyond 37th week, better done after onset of labour\nFollowing are to be observed \n\n\nSacrum \nSacrosciatic notch\nIscial spines \nSide walls\nPubic arch\nPubic angle\nTransverse diameter of outlet\n\nImaging :\n\nX-ray pelvimetry \nUSG\nCT \nMRI\n\nX-ray pelvimetry : \n\nLimited value in diagnosis\nCan only assess size and shape of pelvis \nUseful in cases with pelvic fractures \nDisadvantage : Radiation exposure to both mother and foetus (885 milirad)\nErect lateral view \n\nCT \n\nEasier to perform\nRadiation exposure(45-425milirad)\nMore accurate\n\nMRI\n\nMore accurate to assess bony pelvis\nAssess pelvic capacity in different planes\nAssess foetal size and head volume, pelvic soft tissues\nNo radiation risk\nExpensive,limited availability, more time taking\n\nUSG : \n\nAccurately measures the biparietal diameter", "cop": 4, "opa": "CT scan", "opb": "Ultrasound", "opc": "Radio pelvimetry", "opd": "Pelvic assessment", "subject_name": "Gynaecology & Obstetrics", "topic_name": null, "id": "4c7b1230-cb81-4b8c-9ede-95d4311b8527", "choice_type": "single"} {"question": "Most specific marker for neural tube defects is", "exp": "Both alpha-fetoprotein and acetylcholinesterase in amniotic fluid are markers for neural tube defects. But acetylcholinesterase is the most specific marker. Ref : Dutta book of obstetrics 8th Ed", "cop": 3, "opa": "Alpha-fetoprotein", "opb": "Unconjugated estriol", "opc": "Acetylcholinesterase", "opd": "Pseudocholinesterase", "subject_name": "Gynaecology & Obstetrics", "topic_name": "All India exam", "id": "777e4501-6a4a-418c-8059-d38880722d15", "choice_type": "single"} {"question": "Gonads do not acquire male or female characteristics until", "exp": "Gonads acquire male or female characteristics only after 6 weeks of development (that is in the 7th week)", "cop": 1, "opa": "7th week of development", "opb": "9th week of development", "opc": "12th week of development", "opd": "4th week of development", "subject_name": "Gynaecology & Obstetrics", "topic_name": "Intersex", "id": "94af8c01-7653-4be6-853c-8af0f8f21892", "choice_type": "single"} {"question": "EMACO regimen of chemotherapy is used in the treatment of", "exp": "EMA-CO a cyclic non, cross-resistant combination of etoposide, methotrexate, and actinomycin D alternating with cyclophosphamide and vincristine.Etoposide induces complete remission in 95% of patients with nonmetastatic and low-risk metastatic GTN. The use of EMA-CO induced an 83% remission rate in patients with metastasis and a high-risk score.Ref: Berek and Novak&;s Gynecology; 15th edition; Chapter 39 Gestational Trophoblastic Disease", "cop": 2, "opa": "Carcinoma cervix", "opb": "Gestational trophoblastic neoplasia", "opc": "Malignant Ovarian germ cell tumor", "opd": "Endometrial carcinoma", "subject_name": "Gynaecology & Obstetrics", "topic_name": "Gynaecological oncology", "id": "9196611e-9db3-48bf-bfb8-7556a27b291e", "choice_type": "single"} {"question": "Formula used for estimation of the total iron requirement is", "exp": "(4.4 x body weight (kg) x Hb deficit (g/dl) (269-Dutta 6th) (549-KDT)Estimation of the total requirement* 0.3 x W (100-Hb%) mg of elemental iron, where W= patients weight in pounds, Hb% = observed haemoglobin concentration in percentage. Additional 50% as to be added for partial replenishment of the body store iron.Examples - The total requirement of iron in anaemic patients weighing 100 lb with haemoglobin 50% is calculated as follows 0.3 x 100 (100-50) = 3/10 x 100 x 50 = 1500. Add 50% = 750 mg. Total iron requirement = 2250 mg.Iron requirement (mg) = 4.4 x body weight (kg) X Hb deficit (g/dl)", "cop": 2, "opa": "4 x body weight (kg) x Hb deficit (g/dl)", "opb": "4.4 x body weight (kg) x Hb deficit (g/dl)", "opc": "3 x body weight (kg) xHb deficit (g/dl)", "opd": "3.3 x body weight (kg) x Hb deficit (g/dl)", "subject_name": "Gynaecology & Obstetrics", "topic_name": "Miscellaneous (Gynae)", "id": "751bc95a-4d1e-4c00-8486-eb8307605e05", "choice_type": "single"} {"question": "46, XY chromosomal pattern with uterus and cervix but poorly developed breasts are seen in", "exp": "Complete Gonadal Dysgenesis (Swyer Syndrome)Swyer syndrome is an uncommon form of gonadal dysgenesis, characterized by a 46, XY karyotype.Despite the presence of a Y chromosome, the phenotype is female because the dysgenetic (streak) gonads produce neither AMH nor androgens. Consequently, the vagina, cervix, uterus, and fallopian tubes develop normally and the internal and external genitalia do not masculinize but due to hypoestrogenism breasts are poorly developed.Reference: Clinical gynecologic endocrinology; 10th edition; Chapter 9; Normal and Abnormal Sexual Development; Page no: 365", "cop": 1, "opa": "Swyer syndrome", "opb": "Turner syndrome", "opc": "Androgen insensitivity syndrome", "opd": "Klinefelter syndrome", "subject_name": "Gynaecology & Obstetrics", "topic_name": "Sexuality and intersexuality", "id": "2fdfe2df-032c-4da5-b19d-3c2b62b13e48", "choice_type": "single"} {"question": "Intrauterine Infections most commonly associated with Neural tube defects is", "exp": "Toxoplasmosis and rubella are most commonly associated with Neural tube defects Toxoplasmosis in fetus occurs mostly if the infection is in the later pa of the pregnancy and is associated with Neurologic abnormalities, hydrocephalous, cerebral calcification, chorioretinal scars with or with out severe visual impairment. Rubella is one of the most complete teratogens, and sequelae of fetal infection are worst during organogenesis", "cop": 2, "opa": "CMV and Toxoplasma", "opb": "Toxoplasma and Rubella", "opc": "Rubella and herpes", "opd": "CMV and herpes", "subject_name": "Gynaecology & Obstetrics", "topic_name": "Medical Illness Complicating Pregnancy", "id": "82b4049f-1484-4761-a053-6450f8501d85", "choice_type": "single"} {"question": "A 52 years postmenopausal female presents unilocular with a ovarian cyst of 6 cms with normal CA 125 levels management is;", "exp": "A simple unilocular ovarian cyst less than 5 cm can be observed with repeat ultrasound and CA-125 every 3 months.Many resolve in 6 months.A persistant cyst calls for its removal laparoscopically or by laparotomy.Aspiration of the cyst is contraindicated because of low yield of malignant cells and possibility of spread of malignancy if the cyst proves malignant histologically.Many perform bilateral oophorectomy and hysterectomy SHAW&;S TEXTBOOK OF GYNECOLOGY,Pg no:388,15th edition", "cop": 2, "opa": "USG guided ovarian tapping", "opb": "Wait and watch", "opc": "Surgery", "opd": "OCP", "subject_name": "Gynaecology & Obstetrics", "topic_name": "Gynaecological oncology", "id": "3e12e546-61a3-4b82-be51-db338859660d", "choice_type": "single"} {"question": "Cervix to uterine body ratio in a prepubeal girl", "exp": "The pubeal uterus has the adult pear configuration (fundus larger than cervix) (fundus-to-cervix ratio \u0001 2/1 to 3/1) (4-8) (Fig 3) and is 5-8cm long, 3 cm wide, and 1.5 cm thick. The endometrial lining is seen and varies with the phases of the menstrual cycle. Overfilling of the bladder can modify the uterine shape", "cop": 1, "opa": "1:01", "opb": "2:01", "opc": "1:02", "opd": "3:01", "subject_name": "Gynaecology & Obstetrics", "topic_name": "Anatomy of the female genital tract", "id": "19e1eae1-6303-4cba-b4f1-cf847e8ea983", "choice_type": "single"} {"question": "Fetal sex can be detected by USG at", "exp": "Male development is completed by 12 weeks and female development by 15 weeks\nBy 28 weeks,the testis descends to the internal inguinal ring.By 36 weeks, one testis and by 38-40 weeks both testes descend into scrotum\nIn female, the genital tubercle becones clitoris, the genital folds become the labia minora and the genital swellings became the labia majora", "cop": 4, "opa": "20 weeks", "opb": "16 weeks", "opc": "18 weeks", "opd": "14 weeks", "subject_name": "Gynaecology & Obstetrics", "topic_name": null, "id": "4cabff06-ffbc-458f-93b6-8ab1396f921c", "choice_type": "single"} {"question": "During pregnancy, GFR", "exp": "GFR increases by 50% as filtering capacity of kidney increases due to - a) Decrease oncotic pressure due to hemodilution b) Increased renal blood flow by 80% in 2nd trimester and 50% in 3rd trimester.", "cop": 1, "opa": "Increases", "opb": "Decreases", "opc": "Remains Unchanged", "opd": "Increases only in multiple gestations", "subject_name": "Gynaecology & Obstetrics", "topic_name": "Physiological Changes of Pregnancy", "id": "e5039f0b-eedf-4663-9241-59e094f629a1", "choice_type": "single"} {"question": "Pelvic outlet is said to be contracted if", "exp": "8cm is threshold to define pelvic outlet contraction.", "cop": 2, "opa": "Interischial tuberous diameter is < 7 cm", "opb": "Interischial tuberous diameter is < 8 cm", "opc": "Interischial tuberous diameter is < 9 cm", "opd": "Interischial tuberous diameter is < 10 cm", "subject_name": "Gynaecology & Obstetrics", "topic_name": null, "id": "5b4de56b-939f-4724-b95e-78963d299cd5", "choice_type": "single"} {"question": "Termination of pregnancy in placenta previa is indicated in a) Active bleedingb) Active labourc) Gestational age > 34 weeks with live fetusd) Fetal malformatione) Unstable lie", "exp": null, "cop": 2, "opa": "acd", "opb": "abd", "opc": "abc", "opd": "ab", "subject_name": "Gynaecology & Obstetrics", "topic_name": null, "id": "fd4d9cb3-7012-4e37-aa51-4756d0a04832", "choice_type": "single"} {"question": "Ligamentum teres is formed after", "exp": "Umbilical vein closure occurs shortly after closure of umbilical arteries and forms the ligamentum teres.", "cop": 1, "opa": "Obliteration of the umbilical vein", "opb": "Obliteration of the ductus venous", "opc": "Obliteration of the ductus arteriosus", "opd": "Obliteration of the hypogastric artery", "subject_name": "Gynaecology & Obstetrics", "topic_name": null, "id": "2c14bec0-2cbe-4ebb-ade4-b5a32b628b2f", "choice_type": "single"} {"question": "Endometrial cancer involving 50% of endometrium, extending to vagina, lymph nodes negative with positive peritoneal cytology is staged as", "exp": "3b FIGO STAGING OF ENDOMETRIAL CANCER STAGE I Cancer confined to corpus uteri IA --> Limited to endometrium IB --> < 1/2 myometrial thickness IC > 1/2 myometrial thickness STAGE II Tumour involves cervix but does not extend beyond uterus (la --> Cervical glandular involvement llb --> Cervical stromal involvement STAGE III Local and/or regional spread Ilia --> Uterine serosa, positive peritoneal washings Adnexal involvement Ilib -4 Vaginal involvement (lymph node absent) 111c --> Positive lymph node no pelvis and paraaoic lymph node STAGE IV Tumour widespread IVa - Bladder or bowel mucosa IVb --> Distant metastasis", "cop": 2, "opa": "3a", "opb": "3b", "opc": "3c1", "opd": "3c2", "subject_name": "Gynaecology & Obstetrics", "topic_name": null, "id": "2c6d0837-72f1-4dad-ab58-0ff17f3c37f2", "choice_type": "single"} {"question": "Periconceptional use of the following agent leads to reduced incidence of neural tube defects", "exp": "Women should begin taking folic acid supplements before becoming pregnant and continue taking the supplements of at least 0.4 mg/day throughout the first trimester to reduce the risk of neural tube defects. It has been shown to reduce the occurrence and recurrence of neural tube defects . For low-risk women, folic acid supplementation of 400 micrograms per day currently is recommended For women at high risk of NTDs or who have had a previous pregnancy with an NTD, folic acid supplementation of 4 mg per day is recommended.", "cop": 1, "opa": "Folic acid", "opb": "Iron", "opc": "Calcium", "opd": "Vitamin A", "subject_name": "Gynaecology & Obstetrics", "topic_name": "Intra Uterine Growth Restriction, Intrapaum and Antepaum Fetal Surviellance", "id": "9921d989-7978-48a8-aa51-43800fc253af", "choice_type": "single"} {"question": "Hidradenoma of vulva occurs arises from", "exp": "Ans. is 'a' i.e., Apocrine glandsHidradenoma - arises from the apocrine glands ofvulva.It rarely exceeds 1 cm sizeHistologically it shows cystic spaces enclosing a papillary adenomatous massIt may undergo malignant change and thus requires excision.", "cop": 1, "opa": "Apocrine glands", "opb": "Sebaceous glands", "opc": "Subcutaneous tissue", "opd": "Hair follicle", "subject_name": "Gynaecology & Obstetrics", "topic_name": null, "id": "b4a605b9-fa54-43da-bf0c-d3fd55c9db91", "choice_type": "single"} {"question": "Karyotype of Paial mole", "exp": "Paial moles have a triploid karyotype (69 chromosomes); the extra haploid set of chromosomes usually is derived from the father.When a fetus is present in conjunction with a paial mole, it generally exhibitsthe stigmata of triploidy, including growth retardation and multiple congenital malformations such as syndactyly and hydrocephaly.Ref: Berek and Novak&;s Gynecology; 15th edition; Chapter 39", "cop": 4, "opa": "45XO", "opb": "46XX", "opc": "46XY", "opd": "69XXX", "subject_name": "Gynaecology & Obstetrics", "topic_name": "General obstetrics", "id": "5af47ec9-9f89-4a44-a4d0-3d50f4204b78", "choice_type": "single"} {"question": "The major contribution of the amniotic fluid after 20 weeks of gestation", "exp": "Amniotic fluid: - * pH = 7.1 -7.3 * Specific gravity= 1.008-1.010 * Osmolality = 250 mOsm/L * Rate of amniotic fluid turnover is 500cc/hr * Completely changed and replaced in every 3 hours * Main source- fetal urine (overall) * Main source after 20 weeks- fetal urine Ref:- Mudaliar and Menon's 12th Edition; Pg num:- 28.", "cop": 4, "opa": "Fetal lung fluid", "opb": "Fetal skin", "opc": "Ultrafiltrate and maternal plasma", "opd": "Fetal urine", "subject_name": "Gynaecology & Obstetrics", "topic_name": "recent papers obst", "id": "8d2b7a83-52f7-4b91-a1f8-82922ec3baf5", "choice_type": "single"} {"question": "The relation of fetal axis with uterine axis is known as", "exp": "Ans. b (Lie). (Ref. Dutta, Obstetrics 4th ed.78, William's Obst 21st ed.- 291)TermDefinitionLieRelationship of the long axis of the fetus to the long axis of centralized uterus or maternal spine. The commonest being longitudinal.PresentationPart of fetus which occupies lower pole of uterus.Presenting partPart of presentation which overlies internal os.AttitudeRelationship of different parts of fetus to one another.DenominatorArbitrary bony fixed point on the presenting part which comes in relation with various quadrants of maternal pelvis.PositionRelation of denominator to different quadrants of pelvis.", "cop": 2, "opa": "Attitude", "opb": "Lie", "opc": "Presentation", "opd": "Position", "subject_name": "Gynaecology & Obstetrics", "topic_name": "Miscellaneous (Gynae)", "id": "28f0b965-c79e-4500-9484-a1dee14b5bc0", "choice_type": "single"} {"question": "Most common endocrine disorder of reproductive aged women is", "exp": "PCOD is most common endocrine disorder in reproductive age woman.", "cop": 2, "opa": "Hypothyroidism", "opb": "Polycystic ovarian syndrome", "opc": "Type-2 Diabetes mellitus", "opd": "Congenital adrenal hyperplasia", "subject_name": "Gynaecology & Obstetrics", "topic_name": null, "id": "850df5d2-8f8f-4f60-a398-eaaf80147f3d", "choice_type": "single"} {"question": "The commonest cause of primary Amenorrhoea is", "exp": "Ans. is b i.e. Ovarian dysgenesis Primary amenorrhea is a condition when a female has not attained menarche by the age of 14deg in the absence of growth or development of secondary sexual characteristics. OR No menarche by the age of 16 yearsdeg regardless of the presence of normal growth and development of secondary sexual characteristics. \"Mullerian agenesis - is a relatively common cause of primary amenorrhea. more frequent than congenital androgen insensitivity and second only to gonadal dysgenesis.\" Remember-Most common cause of Primary Amenorrhea = Gonadal dysgenesisdeg Ilnd most common cause of Primary Amenorrhea = Mullerian Agenesis (Mayer Rokitansky Kuster Hauser Syndrome). deg Also know : Causes of Primary Amenorrhea Friends most of the textbooks have given an exhaustive list of the causes of Primary amenorrhea which is quite difficult to mug, but if you try to understand and then learn. I think you will be able to recall them better during exams. Let's first see what are the basic requirements for a female to menstruate normally. An intact out flow tract which connects the uterine cavity with outside and a normally developed uterus with its endometrium lining. Proper quantity and sequence of steroid hormones i.e. Estrogen and Progesterone which inturn originate from ovary. The maturation of follicular apparatus is guided by Gonadotropins - FSH and LH (released by pituitary). The secretion of these hormones is inturn dependant on Gonadotropin Releasing Hormone (released by hypothalamus). So, broadly we can classify the causes of amenorrhea into the following compaments.", "cop": 2, "opa": "Genital tuberculosis", "opb": "Ovarian dysgenesis", "opc": "Mullerian duct anomalies", "opd": "Hypothyroidism", "subject_name": "Gynaecology & Obstetrics", "topic_name": null, "id": "f4b2936b-0e86-4a2f-ab0d-c0da274ade89", "choice_type": "single"} {"question": "Absence of corkscrew glands is seen in", "exp": "Ans. D. Metropathia hemorrhagicaEndometrial histology - secretory phasea. The characteristic feature of endometrium in Metropathia hemorrhagica is cystic glandular hyperplasia.b. Another important feature is absence of secretory endometrium with absence of corkscrew glands.c. Endometrium is usually thick and polypoidal.d. Thin polypi project into the uterine cavity.", "cop": 4, "opa": "Halban's disease", "opb": "Polycystic ovarian disease", "opc": "Irregular ripening", "opd": "Metropathia hemorrhagica", "subject_name": "Gynaecology & Obstetrics", "topic_name": "Miscellaneous (Obs)", "id": "41f370e3-deda-4c23-9577-ababa1a8800a", "choice_type": "single"} {"question": "Best treatment for severe stress incontinence without prolapse is", "exp": "Students, Please read this: From Novaks Gynecology 15th edition Comparing various surgeries for the treatment of stress incontinence, Numerous panels and studies conducted in the late 90's concluded that colposuspension (e.g., Burch, Marshall- Marchetti-Krantz ) and slings were more effective than transvaginal needle suspensions or anterior repairs for long-term success (48-month cure/dry rates).... Now, there were many journals which came out with the information that the TVT/TOT are now better, but since it never came in a book, it was actually never asked of you for the last 5-6 years Now, the 2020 Novaks gynecology 16th writes this: Over the past two decades, there has been an accumulation of a large body of evidence on mid-urethral slings from randomized controlled trials, including several meta-analyses and Cochrane reviews . The overarching summary is that mid-urethral slings have become the procedure of choice in treating women with stress urinary incontinence. They have a good safety profile with a significant improvement of QOL measures, and irrespective of route of placement, they are highly effective in the sho, medium, and long term. There is evidence to suggest that the mid-urethral slings are more effective than the Burch colposuspension procedure, and as effective as the autologous pubovaginal sling ... SO yes, now we change our answer to: that the mid urethral TVT/TOT slings are the best procedures..", "cop": 4, "opa": "Pelvic floor exercise", "opb": "Kelly's repair", "opc": "Burch colposuspension", "opd": "Tension free (Transvaginal) tape", "subject_name": "Gynaecology & Obstetrics", "topic_name": "Gynaecology", "id": "e15e4073-d7d9-4416-b5d2-4ee595209c53", "choice_type": "single"} {"question": "Carcinoma cervix is commonly associated with following sub type of HPV", "exp": "(A) HPV - 16, 18# HUMAN PAPILLOMAVIRUS (HPV):> HPV plays an important role in the development of CIN. HPV infected cells (koilocytes) are characterized by enlarged cells with perinuclear halos. The nucleus is large, irregular and hyperchromatic. Depending on their oncogenic potential, HPV types are broadly grouped into two. More than 130 HPV types have been identified.> High oncogenic risk - Types 16, 18, 31, 58> Low oncogenic risk - Types 6, 11,42, 43> Over 90% of patients with CIN and invasive cancer are found to be positive with HPV DNA. HPV DNA detection in cervical tissues may be a screening procedure as that of pap smear. Polymerase chain reaction or southern blot or hybrid capture (HC) technique is used for HPV DNA detection.", "cop": 1, "opa": "HPV-16, 18", "opb": "HPV-42, 44", "opc": "HPV - 62, 66", "opd": "HPV - 6, 11", "subject_name": "Gynaecology & Obstetrics", "topic_name": "Miscellaneous (Gynae)", "id": "10934cd2-5bff-48e4-8c0a-df39fefef398", "choice_type": "single"} {"question": "Commonest cause of the Postpartum haemorrhage is", "exp": "(Atonic uterus): Ref: 411-DPost partum haemorrhage the amount of blood loss in excess of 500 ml following birth of the babyCAUSES1. Atonic uterus: (80%) commonest cause of PPHGrand multipara, overdistension of the uterus, Malnutrition and anaemia, APH, prolong labour, Anaesthesia,Initiation or augmentation of delivery by oxytocin, Persistent uterine distensionMalformation of the uterus, uterine fibroids, Mismanaged third stage labour, constriction ring, Precipitate labour,2. Traumatic (20%), similarly blood loss in C.S. amounting to 800 - 1000 ml is most often ignored, episiotomy and lacerations3. Combination of atonic and traumatic causes4. Blood coagulation disorders, acquired or congenital: may be due to diminished procagulants (washout phenomenon) or increased fibrinolytic activity.The conditions where such disorders may occur are abruptioplacentae, jaundice in pregnancy, thrombocytopenia purpura, HELLP syndrome or in IVD", "cop": 1, "opa": "Atonic uterus", "opb": "Traumatic", "opc": "Combination of atonic and traumatic", "opd": "Blood coagulation disorders", "subject_name": "Gynaecology & Obstetrics", "topic_name": "Miscellaneous (Gynae)", "id": "4aa5e9a3-14ae-4f36-affe-7946cc806cb8", "choice_type": "single"} {"question": "Most vulnerable period for teratogenicity during fetal development is", "exp": "The preimplantation period is the 2 weeks from feilisation to implantation and is known as the \"all or none\" period. As the zygote undergoes cleavage, an insult damaging a large number of cells typically causes embryonic death.However if few cells are injured compensation may be possible, with normal development. The embryonic period is from the 2nd -8th week post feilisation, encompasses organogenesis and is thus the most critical period with regard to structural malformations. The fetal period, which is beyound 8 weeks, is characterised by continued maturition and functional development and during this time, ceain organs remain vulnerable. Reference: William's Textbook of Gynaecology 4th edition page 242", "cop": 2, "opa": "1st and 2nd week post feilization", "opb": "3rd to 8th week post feilization", "opc": "10th to 12th week post feilization", "opd": "18th to 20th week post feilization", "subject_name": "Gynaecology & Obstetrics", "topic_name": "General obstetrics", "id": "3338df7e-4a55-40a5-a616-56cf8657e347", "choice_type": "single"} {"question": "Type of seizures in Eclampsia", "exp": "Eclampsia is an extremely severe form of preeclampsia characterized by the sudden onset of generalized tonic-clonic seizures.Reference: Reference: Practical guide to High-risk pregnancy and delivery; Chapter 16; Hypeensive DIsorders of Pregnancy", "cop": 3, "opa": "Focal seizures", "opb": "Absence seizures", "opc": "Generalised Tonic-Clonic Seizures", "opd": "Myoclonic seizures", "subject_name": "Gynaecology & Obstetrics", "topic_name": "Medical, surgical and gynaecological illness complicating pregnancy", "id": "e5d034b4-40ee-4666-ad7c-a0c88188fc54", "choice_type": "single"} {"question": "Marker for ovarian cancer is", "exp": "CA 125", "cop": 1, "opa": "CA 125", "opb": "Ca 19 - 19", "opc": "Ca 15 - 5", "opd": "Hypercalcemia", "subject_name": "Gynaecology & Obstetrics", "topic_name": null, "id": "1e98bc03-ef7f-412a-b9a2-c2dadee5cfd5", "choice_type": "single"} {"question": "Biggest fetal skull diameter is", "exp": "Ans. (c) Mento vertical", "cop": 3, "opa": "Biparietal", "opb": "Bitemporal", "opc": "Mento vertical", "opd": "Submentobregmatic", "subject_name": "Gynaecology & Obstetrics", "topic_name": "Fetal Skull and Maternal Pelvis", "id": "7088de81-996f-4e6d-8ca5-3e20e8a4400c", "choice_type": "single"} {"question": "After a full term normal delivery patient went into shock. Most probable cause is aEUR'", "exp": "Inversion of uterus ?? 'c' i.e., Amniotic fluid embolism The patient went into unexplained shock after postpaum. Both uterine inversion and amniotic fluid embolism can cause unexplained shock. Uterine inversion is more common and impoant cause of unexplained shock than amniotic fluid embolism. Munrokerr's operative obstetrics says \"In all cases of unexplained shock after delivery make a careful vaginal examination to exclude the possibility of occult rupture of the uterus, paial uterine inversion or pelvic hematonza\". Munrokerr fuher adds Uterine inversion is the most impoant cause of unexplained shock Three degrees of inversion are generally described. i) Where the inveed .fundus reaches the internal os ii) Where the whole body of the uterus is inveed upto the internal os. iii) Where uterus, cervix and vagina are completely inveed Symptoms of uterine inversion Symptoms of uterine inversion are not always well defined. If however the inversion occurs quickly or is of severe degree there is .feelling of something coming down and pain followed by collapse and hemorrhage of variable severity. When the inversion is of lesser degree and is contained within the vagina or when the .fundus does not come beyond the os externum, only pain and hemorrhage may be present. Inversion is also the most impoant cause of unexplained shock - In these cases the shock is not primarily due to loss of blood or trauma but is really of neurogenic origin. - In this connection it has been suggested by several authors that as one or both ovaries may be pulled into the invagination, pressure on them may be responsible for shock. - The traction on the infundibulopelvic ligaments, peritoneum and broad ligaments stimulate the \"parasympathetic system\" producing shock. Amniotic fluid embolism may also cause unexplained shock -Amniotic fluid embolism is brought about by the uterine contractions forcing the liquor amni with its contained paicles of meconium, lanugo, and vernix caseosa into a patent uterine sinus from where it is carried into the lungs. - We have opted for uterine inversion as it is the more common cause for unexplained shock. Postpaum collapse from Ian Donald 6/e edited by Renu Mishra Postpaum maternal collapse is serious obstetric complication Majority of these cases occur as hemorrhage that is revealed. But a small minority of cases of collapse after deliver occur without obvious vaginal bleeding but these are the ones that pose a greater challenge .for diagnosis and management. The conzmon causes of postpaum collapse without bleeding are -Eclampsia - Uterine rupture - Vasovagal .syncope - Pulmonary embolism (Amniotic fluid embolism) - Septic shock - Uterine inversion Renu Mishra fuher adds - \"Acute inversion of the uterus should be one of the first possibilities to cross the obstetricians mind in case of unexplained shock\" Amniotic fluid embolism It is a life threatening complication of pregnancy with an extremely high moality rate. 300 The pathogenesis of the condition is not clear. Amniotic fluid probably enter the maternal venous circulation through a breach in the barrier b/w the maternal vasculature and the amniotic fluid most likely from the placental site or at the site of a uterine trauma after the membranes have ruptured. Classically amniotic fluid embolism presents during labour or immediately after the delivery with sudden onset maternal collapse associated with tachyapnea, cyanosis, hypotension, altered mental DIC. So, there is nothing to differentiate between uterine inversion and amniotic fluid embolism in terms of clinical presentation. We have opted for uterine inversion as it seems to be the more impoant and more common cause of unexplained shock. But we are still not sure - A patient with amniotic fluid embolism will always present in this manner i.e., unexplained death following normal de livery . - On the other hand uterine inversion commonly present as life threatening post paum hemorrhage. - It may present as unexplained shock but this is not its usual presentation. - If there is a case of postpaum collapse and no other significant findings are present the first diagnosis that comes to the mind is embolism. - All other causes of postpaum collapse will have some clinical .feature which would point towards to cause. Clinically most of the obstetricianl would suspect amniotic fluid embolism if any other abnormality is absent in case post paum collapse.", "cop": 1, "opa": "Inversion of uterus", "opb": "PPH", "opc": "Amniotic fluid embolism", "opd": "Eclapsia", "subject_name": "Gynaecology & Obstetrics", "topic_name": null, "id": "700fe9ee-c619-43fe-9272-60fbc09370bc", "choice_type": "single"} {"question": "Herniation of meninges through cranial defect is known as", "exp": "Anencephaly is characterized by the absence of the cranium and telencephalic structures, with the skull base and orbits covered only by angiomatous stroma. Acrania is the absence of the cranium, with protrusion of disorganized brain tissue.Cephalocele is the herniation of meninges through a cranial defect, typically located in the midline occipital region. When brain tissue herniates through the skull defect, the anomaly is termed an encephalocele(Ref: William's Obstetrics; 25th edition)", "cop": 3, "opa": "Acrania", "opb": "Anencephaly", "opc": "Cephalocele", "opd": "Encephalocele", "subject_name": "Gynaecology & Obstetrics", "topic_name": "All India exam", "id": "f2436fb2-1141-4671-9d30-32a7a2fefae3", "choice_type": "single"} {"question": "Treatment of choice for extragenital endometriosis is", "exp": "Treatment of extragenital endometriosis will depend on the site. If complete excision is possible, this is the treatment of choice. When this is not possible, long-term medical treatment is necessary using the same principles of medical treatment for pelvic endometriosis. Surgical treatment of Bladder endometriosis -excision of the lesion and primary closure of the bladder wall. Ureteral lesions may be excised after stenting the ureter, Abdominal wall and perineal endometriosis -complete excision of the nodule. Appendicular endometriosis-Appendectomy.", "cop": 1, "opa": "Complete excision", "opb": "Medical treatment only", "opc": "Medical + surgical treatment", "opd": "Steroids", "subject_name": "Gynaecology & Obstetrics", "topic_name": "Endometriosis & H", "id": "eca6c3b8-ed43-4a7b-90e0-4839b896470d", "choice_type": "single"} {"question": "GnRH agonist therapy for leiomyoma is given for", "exp": "GnRH agonist treatment is continued for 3 → 6 months.", "cop": 2, "opa": "2 → 4 months", "opb": "3 → 6 months", "opc": "4 → 8 months", "opd": "5 → 10 months", "subject_name": "Gynaecology & Obstetrics", "topic_name": null, "id": "af50b446-99fa-4dc2-80ae-c3b11434e55e", "choice_type": "single"} {"question": "Fetal anomaly characterized by the absence of the cranium is", "exp": "Anencephaly is characterized by the absence of the cranium and telencephalic structures, with the skull base and orbits covered only by angiomatous stroma. Acrania is the absence of the cranium, with protrusion of disorganized brain tissue. Both are generally grouped together, and anencephaly is considered to be the final stage of acrania.Ref: William&;s Obstetrics; 24th edition; Chapter: 10", "cop": 3, "opa": "Cephalocele", "opb": "Holoprosencephaly", "opc": "Anencephaly", "opd": "Dandy-Walker Malformation", "subject_name": "Gynaecology & Obstetrics", "topic_name": "Fetus", "id": "735ec612-ea6d-40f4-8b43-ada85272900e", "choice_type": "single"} {"question": "Battledore placenta is associated with highest risk of", "exp": "Battle dore placenta - marginal insertion of umbilical cord, occasionally results in cord avulsion during delivery of placenta.", "cop": 2, "opa": "Fetal anomalies", "opb": "Cord avulsion", "opc": "Uterine inversion", "opd": "Single umbilical artery", "subject_name": "Gynaecology & Obstetrics", "topic_name": null, "id": "50ca7400-45cb-47a7-9c52-7fe52c657299", "choice_type": "single"} {"question": "Following ectopic pregnancy, recurrence is seen approximately in", "exp": "Following an ectopic pregnancy, approximately 15% of women will have a subsequent ectopic pregnancy.Ref: Berek and Novak&;s Gynecology; 15th edition; Chapter 20", "cop": 3, "opa": "5%", "opb": "10%", "opc": "15%", "opd": "20%", "subject_name": "Gynaecology & Obstetrics", "topic_name": "General obstetrics", "id": "98baefeb-c692-4351-98a7-44dcd1621939", "choice_type": "single"} {"question": "Normal sperm concentration in semen analysis should be", "exp": "Normal Semen Analysis: World Health Organization Volume>=1.5 mLSperm concentration>=15 million/mL Sperm motility>=32% progressive>25% rapidly progressiveMorphology (strict criteria)>=4% normal formWhite blood cells<1 million/mLReference: Novak's Gynecology; 14th edition; Chapter 32; Infeility and Assisted Reproductive Technology", "cop": 2, "opa": ">= 5 millilon/ml", "opb": ">= 15 millilon/ml", "opc": ">= 20 millilon/ml", "opd": ">= 25 millilon/ml", "subject_name": "Gynaecology & Obstetrics", "topic_name": "Infertility", "id": "c1c6da02-069f-499b-8f53-2a5e52f24824", "choice_type": "single"} {"question": "Recurrent miscarriage is defined as", "exp": "Recurrent miscarriage is classically defined as three or more consecutive pregnancy losses at <= 20 weeks or with a fetal weight < 500 grams. (Ref: William's Obstetrics; 25th edition)", "cop": 3, "opa": "One or more consecutive pregnancy losses at <= 20 weeks or with a fetal weight < 500 grams", "opb": "Two or more consecutive pregnancy losses at <= 20 weeks or with a fetal weight < 500 grams", "opc": "Three or more consecutive pregnancy losses at <= 20 weeks or with a fetal weight < 500 grams", "opd": "Four or more consecutive pregnancy losses at <= 20 weeks or with a fetal weight < 500 grams", "subject_name": "Gynaecology & Obstetrics", "topic_name": "All India exam", "id": "d99a481b-378f-4940-a435-3b94d4f28395", "choice_type": "single"} {"question": "Beneficial effects of hormone replacement therapy in early menopause", "exp": null, "cop": 1, "opa": "Increased Nitric oxide", "opb": "Increased Endothelin", "opc": "Decreased COX-2", "opd": "Increased TNF-α", "subject_name": "Gynaecology & Obstetrics", "topic_name": null, "id": "d59bba84-5124-4d1f-9758-d8d1b74f6bdd", "choice_type": "single"} {"question": "Treatment of deep transverse arrest is", "exp": "Ans. a (LSCS). (Ref. Williams, Obstetrics, 21st ed., 301)Deep transverse arrest (DTA)# Occipitotransverse (OT) head position frequently results in a deep transverse arrest and resultant cephalopelvic disproportion.# DTA occurs in the second stage of labor where the fetus maintains an OT position at a low pelvic station.# In most cases, this requires rotation of the fetal head to OA or OP to allow vaginal delivery, but occasionally, patients with a platypelloid pelvis will spontaneously deliver a baby with a transverse head.# The options open to the modem obstetrician faced with a deep transverse arrest in an OT position include:- Digital or manual rotation- Rotation with traction using a vacuum extractor- Rotation with forceps- Cesarean section# Unless the operator has had sufficient training and experience with rotational delivery using forceps, cesarean section is frequently the most prudent choice.", "cop": 1, "opa": "LSCS", "opb": "Induction of labour", "opc": "Trial of labour", "opd": "Craniotomy", "subject_name": "Gynaecology & Obstetrics", "topic_name": "Miscellaneous (Gynae)", "id": "7b8be832-e21d-4438-be9f-1f68c32a6eae", "choice_type": "single"} {"question": "Primary Amenorrhea with Absent uterus, Normal breasts and Scanty pubic hair is seen in", "exp": "Testicular Feminising Syndrome Complete AIS (testicular feminization) is a form of male pseudohermaphroditism Gonadal sex (male) and the Contrasting phenotype (female) Normal male karyotype (46,XY) and testes that produce both testosterone and AMH An inactivating mutation in the gene encoding the intracellular androgen receptor (located on the long arm of the X chromosome, Xq) results in an end organ insensitivity to androgen actions that prevents normal masculinization of the internal and external genitalia during embryonic development Consequently, the external genitalia are those of a female (absent androgen action), the cervix and uterus are absent (due to normal AMH action), and the vagina is sho and ends blindly (derived only from the urogenital sinus) Reference: Clinical Endocrinologic Gynecology and Infeility; Eigth Edition; Chapter 11", "cop": 4, "opa": "Mayer Rokitansky Kuster hauser Syndrome", "opb": "Turner Syndrome", "opc": "Noonan Syndrome", "opd": "Testicular Feminizing Syndrome", "subject_name": "Gynaecology & Obstetrics", "topic_name": "Disorders of menstruation", "id": "8ec0de8f-1e89-4cdd-8308-723ee3d95c20", "choice_type": "single"} {"question": "In a young female of reproductive age an absolute contraindication for prescribing oral contraceptive pills is", "exp": "Ans. is d i.e. Impaired liver function Contraindications of OCP, Absolute contraindications include : I have a mnemonic which might prove helpful in learning the absolute contraindlactions : Relative contraindications of OCP's Migraine with Aura Diabetes mellitus/Gestational diabetes Hypeension (mild) Smoking Uterine leiomyoma Elective surgery (OCP should be stopped four weeks before any scheduled surgery) Seizure disorders Obstructive Jaundice in pregnancy Sickle cell disease Gall bladder disease SLE Mitrel valve prolapse Hyperlipidemia Hepatic disease", "cop": 4, "opa": "Diabetes", "opb": "Hypeension", "opc": "Obesity", "opd": "Impaired liver function", "subject_name": "Gynaecology & Obstetrics", "topic_name": null, "id": "aae72d5a-a4c6-4880-aee0-ca7b76a4c12f", "choice_type": "single"} {"question": "Diethylstilbesterol causes the following defects expect", "exp": "Diethylstilbesterol defects Neural tube defects Vaginal and cervical adenosis Clearcell adenocarcinoma Uterine anomalies Cryptorchidism Testicular hypoplasia TEXTBOOK OF OBSTETRICS,SHEILA BALAKRISHNAN,PG NO:565,2nd edition", "cop": 1, "opa": "Renal anomalies", "opb": "Incompetent cervix", "opc": "T saped uterus", "opd": "Vaginal adenosis", "subject_name": "Gynaecology & Obstetrics", "topic_name": "Congenital malformations", "id": "74ad26ed-3bbc-4bc6-b4db-531eddfbbdb4", "choice_type": "single"} {"question": "Aspiration of sperms from testes is done in", "exp": "TESA (Testicular sperm aspiration)- It is a method of obtaining sperm for ICSI (Intracytoplasmic sperm injection).\nMESA (Microsurgical epididymal sperm aspiration) it is another method of sperm collection from the epididymis.", "cop": 1, "opa": "TESA", "opb": "MESA", "opc": "ZIFT", "opd": "GIFT", "subject_name": "Gynaecology & Obstetrics", "topic_name": null, "id": "5df6d8da-04ba-4050-919e-2b159555e3bf", "choice_type": "single"} {"question": "A patient is diagnosed to have CIN II. She approaches you for advice. You can definitely tell her the risk of malignancy as", "exp": "CIN I and CIN II, the lesions have a 5-10% chance to progress to a high grade lesion CIN III progressing to cervical cancer is almost 10-30% Transformation Rates Form low grade CIN to High grade CIN takes around 5-6 years From High Grade CIN to invasive cancer takes around 10 years time. HPV related cervical lesions occur more on the cervical anterior lip than the lower lip (2:1 incidence).", "cop": 4, "opa": "15%", "opb": "60%", "opc": "30%", "opd": "5%", "subject_name": "Gynaecology & Obstetrics", "topic_name": "Cervical Carcinoma", "id": "afa90c7c-3331-43f1-b2b4-b85596a83b4e", "choice_type": "single"} {"question": "Contraction of uterine pain is carried by", "exp": "In the early stages of labour pains are mainly uterine in origin because of painful uterine contractions It is distributed along the cutaneous nerve distribution of T10 - L 1. In the later stages pain is due to cervical dilatation This pain is referred to back through sacral plexus. Ref: Dutta Obs 9e pg 112.", "cop": 3, "opa": "L 1,2,3", "opb": "T 12, L 1,2,3", "opc": "T 10,11,12 L 1", "opd": "L 3,4,5 S 1,2", "subject_name": "Gynaecology & Obstetrics", "topic_name": "General obstetrics", "id": "d9e62e26-8c12-416d-b2f9-4d991774a03c", "choice_type": "single"} {"question": "Virus responsible for non immune hydrops fetalis is", "exp": "Hydrops is defined as the accumulation of extracellular fluid in tissues and serous cavities. Hydrops fetalis is more common because of non-immune reasons rather than Rh isoimmunization. Most common cause of non-immune hydrops is due to congenital hea block (cardiac causes) Most common infection causing non immune hydrops- Parvovirus B19 infections", "cop": 4, "opa": "Cytomegalovirus", "opb": "Herpes simplex virus", "opc": "Hepatitis B virus", "opd": "Parvovirus", "subject_name": "Gynaecology & Obstetrics", "topic_name": "Medical Illness Complicating Pregnancy", "id": "bf03dcdb-43e0-4464-8cfa-3ee79e2d7991", "choice_type": "single"} {"question": "Polycystic ovarian disease not related to", "exp": "(A) (Hypertension) (459,469, Dutta Gynae 6th)POLYCYSTIC OVARIAN SYNDROME (PCOS)Multifactorial and polygenic conditionsMost obese woman with oligomenorrhea have the polycystic ovarian syndrome (PCOS), with its associated anovulation and ovarian hypcrandrogenism 40% of women with PCOS are absess (Not always). Most non obese with PCOS are also insulin resistance, hyperinsulinemia or the combination of the two are causative or contribute to the ovarian pathophysiology in PCOS in both obese and lean individuals (476-H17th)PCOS (Novak's Gynaecology)Clinical characteristicDiagnostic Criteria* Hirsutism* Obesity* Oligomenorrhea or AmenorrheaMajor criteria Minor criteria* Chronic anovulation* Pre menarchial onset of* Infertility* Hyper androgcncmia (in absence of any notable cause)- Hirsutism- Obesity* Insulin resistance (Abnormal glucose tolerance)* Elevated LH (LH/FSH ratio>2)* Intermittent anovulation associated with hyperandrogenemia (| ed free estosterone, |ed DHEAS)* Multiple cysts are located peripherally along the surface of the ovary giving a ''Necklace\" appearance in UCG is diagnostic* The increased conversion of androstenedione to estrogen which occurs to a greater degree in women with lower body obesity may contribute to the increased incidence of uterine cancer in post menopausal women with obesity (467-HI 7th)* PCOS is most strongly associated with a risk of endometrial carcinoma and may also be associateed with an increased risk of ovarian and breast cancer**** Long term sequelae of PCOS- Diabetes mellitus- CVS disorders- MI- Lipidaemia- Hypertension- Endometrial cancer- Breast cancer- Premature ovarian failure following surgery* Autosomal dominant inherited factor** were added as aetiological factor in the development of PCOS CYP21 gene mutation has been discovered in this connectionSerum level in PCOSAndrogens (|)Estrogens (|)LH/FSHInsulinAndroeen excessEstroeen excessLH levels are |edInsulin |ed* Excess production of androgens is seen from both the ovaries and adrenals, (postulated to be due to abnormal regulation of androgen forming enzyme)* Estrogen levels are increased due to peripheral aromatization of androgens* EstrogenQ levels are raised(Estrone>Estradiol)* Enhanced estrogen levels exert a- Positive feedback on LH levels (|ed)- Negative feedback on FSH levels (|ed)* | ed insulin resistance LH/FSH Ratio is characteristically >2", "cop": 1, "opa": "Hypertension", "opb": "DM", "opc": "Obesity", "opd": "Hirsutism", "subject_name": "Gynaecology & Obstetrics", "topic_name": "Miscellaneous (Gynae)", "id": "28e169d8-1b35-41b6-a5b1-3e97cf195c65", "choice_type": "single"} {"question": "Normal pregnancy can be continued in", "exp": "Hea disease in which Termination of pregnancy is advised Pulmonary hypeension- primary or secondary Marfans syndrome with aoa involvement Eisenmenger syndrome Any hea disease which belongs to NYHA Class 3 or 4 Ejection fraction < 40% Severe aoic stenosis or Severe mitral stenosis Previous peripaum cardiomyopathy with any residual impairment of LV function ABSOLUTE INDICATIONS FOR TERMINATION OF PREGNANCY Pulmonary HTN Eisenmenger's Syndrome Pulmonary veno-occlusive disease. Reference: William's Obstetrics; 24th edition; Chapter 49 Reference: DC.Duttas textbook of OBG,9th edition,page 259", "cop": 2, "opa": "Primary pulmonary hypeension", "opb": "Wolf-Parkinson-White syndrome", "opc": "Eisenmenger syndrome", "opd": "Marfan syndfome with dilated aoic root", "subject_name": "Gynaecology & Obstetrics", "topic_name": "Medical, surgical and gynaecological illness complicating pregnancy", "id": "b76ecb08-501a-49c2-8167-b57c8dc3a0d7", "choice_type": "single"} {"question": "The best way to diagnose the degree of placenta pre is", "exp": "Trans vaginal sonography", "cop": 1, "opa": "Trans vaginal sonography", "opb": "Double set-up examination", "opc": "Observation during C.S.", "opd": "Examination of placenta after delivery", "subject_name": "Gynaecology & Obstetrics", "topic_name": null, "id": "023c7862-ae74-4462-a686-d6f414132e1d", "choice_type": "single"} {"question": "Commonest cause of pyometra", "exp": "Ans. is b i.e. Carcinoma endometrium Pyometra is collection of pus or mixture of pus and blood within the uterus. Causes : * Most common cause is Carcinoma endometriumdeg 2'ld most common cause is Senile endometritts. Other causes : * Congenital atresia of the vagina/cervix Stenosis of cervix/vagina following : - Operations - Burns - Radiotherapy Endometritis : - Senile - Tuberculous - Puerperal Carcinoma : Ca endometrium (most common) Ca cervix Ca corporis", "cop": 2, "opa": "Carcinoma cervix", "opb": "Carcinoma endometrium", "opc": "Vaginitis", "opd": "Senile endometritis", "subject_name": "Gynaecology & Obstetrics", "topic_name": null, "id": "09f319af-8d32-4f64-bb72-c812af413dba", "choice_type": "single"} {"question": "Most common cause of breech presentation is", "exp": "Causes of breech presentation : Prematurity - Most common cause Undue mobility of the fetus Fetal abnormality Favorable adaptation Ref: Dutta Obs 9e pg 352.", "cop": 1, "opa": "Prematurity", "opb": "Contracted pelvis", "opc": "Oligohydramnios", "opd": "Placenta pre", "subject_name": "Gynaecology & Obstetrics", "topic_name": "Abnormal labor", "id": "4b587d4a-9081-4e63-b54c-aa354d07a900", "choice_type": "single"} {"question": "In terms of birth defect potential, the safest of the following drugs is", "exp": "Alcohol is an enormous contributor to otherwise preventable birth defects. Sequelae include retardation of intrauterine growth, craniofacial abnormalities, and mental retardation. The occasional drink in pregnancy has not been proved to be deleterious. Isotretinoin (Accutane) is a powerful drug for acne that has enormous potential for producing congenital anomalies when ingested in early pregnancy; it should never be used in pregnancy. Tetracyclines interfere with development of bone and can lead to stained teeth in children. Progesterone have been implicated in multiple birth defects, but controlled studies have failed to demonstrate a significant association with increased risk. Patients who have inadvertently become pregnant while on birth control pills should be reassured that the incidence of birth defects is no higher for them than for the general population. Phenytoin (Dilantin) is used for epilepsy and can be associated with a spectrum of abnormalities, including digital hypoplasia and facial abnormalities.", "cop": 4, "opa": "Alcohol", "opb": "Isotretinoin (Accutane)", "opc": "Tetracyclines", "opd": "Progesterone", "subject_name": "Gynaecology & Obstetrics", "topic_name": "Fetus & New Born and their Diseases", "id": "f415c0a6-f689-4e82-afd5-2c135eee772f", "choice_type": "single"} {"question": "Steroids used in treatment of HELLP syndrome", "exp": "In patients with HELLP syndrome with dangerously low platelet count, steroids might serve to increase platelets.", "cop": 3, "opa": "Decrease materral mortality rate.", "opb": "Decrease rate of acute renal failure.", "opc": "Increase platelet count in severe thrombocytopenia.", "opd": "Increase rate of recovery of liver enzymes.", "subject_name": "Gynaecology & Obstetrics", "topic_name": null, "id": "a0b99f62-3957-4acb-b705-660a50649eae", "choice_type": "single"} {"question": "Most common cause of infertility in females is due to the disorder of", "exp": "(B) Ovarian# Common causes of infertility of females include:> Ovulation problems (E.g. Polycystic ovarian syndrome, PCOS, the leading reason why women present to fertility clinics due to anovulatory infertility, tubal blockage, pelvic inflammatory disease caused by infections like tuberculosis> Age-related factors, Uterine problems, Previous tubal ligation, Endometriosis, Advanced maternal age", "cop": 2, "opa": "Uterus", "opb": "Ovarian", "opc": "Vagina", "opd": "Cervix", "subject_name": "Gynaecology & Obstetrics", "topic_name": "Miscellaneous (Obs)", "id": "ad61a9dd-ca0b-4d12-a4ad-54ff7b35a896", "choice_type": "single"} {"question": "Bilateral ovarian cancer with; capsule breached; ascites positive for malignant cells. Stage is;", "exp": "Stage 1: tumor confined to ovaries or fallopian tubes. 1c: tumor limited to one or both ovaries. : Surgical spill : Capsule rupture before surgery or tumor on ovarian surface : Malignant cells present in ascites or peritoneal washings Reference : textbook of gynaecology Sheila balakrishnan, 2nd edition, pg no:277 <\\p>", "cop": 1, "opa": "l", "opb": "ll", "opc": "Ill", "opd": "lV", "subject_name": "Gynaecology & Obstetrics", "topic_name": "Gynaecological oncology", "id": "dea49320-9e76-4ea3-8efb-4eada45b1b33", "choice_type": "single"} {"question": "Vaginal artery arising from anterior division of hypogastric artery corresponds to", "exp": "Vaginal artery usually arises from uterine artery. In some cases, it arises from anterior division of hypogastric artery where it corresponds to inferior vesical artery in male.", "cop": 1, "opa": "Inferior vesical artery in male", "opb": "Superior vesical artery in male", "opc": "Deep dorsal artery of penis", "opd": "Dorsal artery of penis.", "subject_name": "Gynaecology & Obstetrics", "topic_name": null, "id": "b669865b-9881-43d9-b3a2-a540321ce5d6", "choice_type": "single"} {"question": "Maximum chances of ureteric injury are seen with", "exp": "Wertheim's hysterectomy : Its a radical procedure, hence more chances of injuries while dissecting ureter in parametrial tissue", "cop": 3, "opa": "TAH", "opb": "Anterior colporrhaphy", "opc": "Wertheims hysterectomy", "opd": "Vaginal hysterectomy", "subject_name": "Gynaecology & Obstetrics", "topic_name": null, "id": "872def7c-ecbd-441a-97d9-66d66b4febec", "choice_type": "single"} {"question": "With increasing gestational age, the S/D (Systolic / Diastolic ratio) of umbilical aery ...", "exp": "The systolic/diastolic (S/D) ratio of flow velocities was measured as an index of peripheral resistance. The S/D ratio normally declines from appx. 4 at 20 weeks gestation, to generally <3 after 30 weeks and finally to 2 at bih Doppler indices show a continuous reduction throughout the second half of pregnancy without any plateau or increase near term.", "cop": 2, "opa": "Increases", "opb": "Decreases", "opc": "Shows a persistent diastolic notch", "opd": "Remains constant", "subject_name": "Gynaecology & Obstetrics", "topic_name": "Intra Uterine Growth Restriction, Intrapaum and Antepaum Fetal Surviellance", "id": "ed2fe9e6-2009-4915-89a9-0c7d475eabc3", "choice_type": "single"} {"question": "Clue cell is seen in aEUR'", "exp": "Bacterial vaginosis Clue cells are seen in bacterial vaginosis -Clue cells are vaginal epithelial cells coated with coccobacillary organisms which have granular appearance and indistinct borders, on a wet mount prepared by mixing vaginal secretions with normal saline in a ratio of -1:1. Amsel criteria for the diagnosis of bacterial vaginosis Includes any three of the following four clinical abnormalities i) Objective signs of increased white homogenous vaginal discharge. ii) Vaginal discharge pH of >4.5 iii) Liberation of distinct .fishy odour (attributable to volatile amines such as trimethylamine) immediately after vaginal secretions are mixed with a 10% solution of KOH. iv) Microscopic demonstration of clue cells.", "cop": 1, "opa": "Bacterial vaginosis", "opb": "Candidial vaginosis", "opc": "Chlamydiasis", "opd": "Trichomonas", "subject_name": "Gynaecology & Obstetrics", "topic_name": null, "id": "5e2f71c2-ac05-4d30-a114-d3d9049736ae", "choice_type": "single"} {"question": "Levels of sex hormone binding globulin in PCOD", "exp": "In PCOD, SHBG Levels are decreased.", "cop": 2, "opa": "Increased", "opb": "Decreased", "opc": "Unchanged", "opd": "Variable", "subject_name": "Gynaecology & Obstetrics", "topic_name": null, "id": "10ec3a61-6774-4ebe-bebc-1c30f2f4987d", "choice_type": "single"} {"question": "Most common etiology of abnormal uterine bleeding in childhood age group", "exp": "Vulvovaginitis is commonest etiology of AUB in childhood age group.", "cop": 1, "opa": "Vulvovaginitis", "opb": "Anovulation", "opc": "Endometrial polyp", "opd": "endometrial cancer", "subject_name": "Gynaecology & Obstetrics", "topic_name": null, "id": "4c038d90-5a91-4bfb-bc01-fe5f28c2928d", "choice_type": "single"} {"question": "Kidney size increases in pregnancy by", "exp": "Kidney size increases approximately 1.5 cm, returns to normal size in postpaum.(Ref: William's Obstetrics; 25th edition)", "cop": 2, "opa": "0.5cm", "opb": "1.5cm", "opc": "2cm", "opd": "2.5cm", "subject_name": "Gynaecology & Obstetrics", "topic_name": "All India exam", "id": "9706788b-43d5-4bb7-bf82-223dfdf85a5c", "choice_type": "single"} {"question": "A woman presents with a fluctuant non tender swelling at the introitus. The best treatment is", "exp": "Ans. is a i.e. Marsupilisation Fluctuant non tender swelling at the introitus suggests a diagnosis of baholins cyst. Baholins cyst 7 It is the commonest cyst of vulva. Baholins cyst are produced from accumulation of secretions of Baholins gland. The cyst may develop either in the duct (more common) or in the gland Aetiology : cyst formation occurs due to the obstruction of the main duct or opening of an acinus. The cause of obstruction is usually fibrosis which follows either infection or trauma. It was formerly believed that the infection was invarialy gonococcal but almost any orgnaism can be responsible. Left Baholins gland is more often affected than the right. Presentation : Usually presents as a unilateral swelling that bulges across the vaginal introitus. Size of the cyst rarely exceeds hens egg. Swelling is present characteristically on the inner side of the junction of the anterior 2/3rd with posterior 1/3rd of the labium majus. The swelling is fluctuant and usually non tender. May present with discomfo, dyspareunia or infection. Treatment of choice is Marsupialization : Marsupialization is the gratifying surgery for Baholins cyst. It is preferred over traditional exicision operations.", "cop": 1, "opa": "Marsupilisation", "opb": "Incision and drainage", "opc": "Surgical resection", "opd": "Aspiration", "subject_name": "Gynaecology & Obstetrics", "topic_name": null, "id": "4fe70053-7adb-435a-9855-bbe9dd567650", "choice_type": "single"} {"question": "Treatment recommended for secondary syphilis is", "exp": null, "cop": 2, "opa": "Benzathine penicillin G 2.4 mIU i.m weekly, 3 doses", "opb": "Benzathine penicillin G 2.4 mIU i.m single dose", "opc": "Benzathine penicillin G 2.4 mIU i.m twice weekly, 4 doses", "opd": "Benzathine penicillin G 2.4 mIU i.m twice daily for 2 weeks.", "subject_name": "Gynaecology & Obstetrics", "topic_name": null, "id": "859a92cd-2229-4a00-830d-1cded7f001ca", "choice_type": "single"} {"question": "Most common uterine malignancy", "exp": "75 to 80 % of endometrial tumors are type1, which are endometroid adenocarcinomas.", "cop": 2, "opa": "Leiomyosarcoma of uterus", "opb": "Endometroid adeno carcinoma", "opc": "Serous Uterine carcinoma", "opd": "Clear cell carcinoma", "subject_name": "Gynaecology & Obstetrics", "topic_name": null, "id": "4001eb2b-8513-4d40-b92b-d5772ede5086", "choice_type": "single"} {"question": "Least likely physiological change in pregnancy", "exp": "Normal pregnancy is characterized by increased plasma volume (preload), increased Cardiac Output and decreased PVR. These changes result in a physiologic decrease in mean blood pressure during the second trimester.", "cop": 4, "opa": "Increase in intravascular volume", "opb": "Increase in cardiac output", "opc": "Increase in stroke volume", "opd": "Increase in peripheral vascular resistance", "subject_name": "Gynaecology & Obstetrics", "topic_name": "Physiological Changes of Pregnancy", "id": "26375c9e-d07b-423a-9859-f6884370c012", "choice_type": "single"} {"question": "A9 month old pregnant lady presents with Jaundice and distension, pedal edema after delivering normal baby. Her clinical condition deteriorates with increasing abdominal distension and severe ascites. Her bilirubin is 5 mg/dl, S. alkaline phosphatase was 450 u/L and ALT (345 Iu). There is tender hepatomegaly 6 cm below costal margin and ascitic fluid shw protein less than 2 mg%. Diagnosis is", "exp": "Budd chiari syndrome The most common cause of an acute severe livery injury in a young pregnant women are - Viral hepatitis (HAV, HBV) - Eclampsia, preeclampsia (HELLP syndrome) - Acute fatty liver of pregnancy - Budd chiary syndrome Let us discuss the options Preeclampsia and Eclampsia - It is the most common cause of abnormal liver chemistry test in women. Liver enzyme tests are only mildly abnormal in patients with liver diseases. Aminotransferases are modestly elevated. Alkaline phosphatase is subtantially increased But in these cases delivery of the fetus is followed by rapid normalization of the hepatic abnormalities. Moreover there is no mention of history of P.I.H, hemolysis and thrombocytopenia (HELLP syndrome) Acute Fatty liver of pregnancy Acute fatty liver develops after 34th week of pregnancy Jaundice develops a few days after the onset, the serum bilirubin is rarely above 10 mg/dl. Alkaline phosphate is markedly elevated. Aminotransferases are moderately elevated. A markedly raised serum ammonia is the most diagnostic finding in establishing the diagnosis of acute fatty liver-of pregnancy Fulminant hepatic failure The patient presents with features of severe acute hepatitis leading to the development of hepatic encephalopathy within 8 weeks of onset. The bilirubin increases to 20-30 mg/d1. The aminotransferase levels are very high (> 1000) alkaline phosphatase moderately elevated. Delivery is usually the best treatment. Paurition usually results in resolution. Budd chiary syndrome It is a disorder characterized by thrombotic occlusion of the hepatis veins. It is a rare complication of pregnancy. Most of the cases presents within few weeks of delivery but in several cases onset occurs during pregnancy. Clinical triad of B.C. syndrome includes sudden onset of abdominal pain, hepatontegaly and ascites near tee_ rn or sholy after delivery. Tender hepatomegaly is one of the hallmark of Budd chiari syndrome. Ascites with a high protein content is always present. Aminotransferases are mildly elevated. Jaundice is seen in only half of the cases.", "cop": 4, "opa": ">Acute fatty liver of pregnancy", "opb": ">HELLP syndrome", "opc": ">Acute fulminant, liver failure", "opd": ">Budd chiari syndrome", "subject_name": "Gynaecology & Obstetrics", "topic_name": null, "id": "43a68008-e182-4535-aeb9-46a3d1851718", "choice_type": "single"} {"question": "Decidualization of endometrium is due to", "exp": "Progesterone is responsible for decidualization of endometrium.", "cop": 2, "opa": "Estrogen", "opb": "Progesterone", "opc": "Inhibin", "opd": "FSH", "subject_name": "Gynaecology & Obstetrics", "topic_name": null, "id": "5f553572-bc84-4089-ae40-90eac8bfff34", "choice_type": "single"} {"question": "Most common cause of maternal anaemia in pregnancy", "exp": "According to WHO anemia in pregnancy is defined as haemoglobin level below 11g/dl Iron deficiency anemia is the most common cause of anaemia both in the non pregnant and pregnant state Reference: Text Book of Obstetrics; Sheila Balakrishnan; 2nd edition; Page no: 285", "cop": 2, "opa": "Acute blood loss", "opb": "Iron deficiency state", "opc": "Gastrointestinal blood loss", "opd": "Hemolytic anaemia", "subject_name": "Gynaecology & Obstetrics", "topic_name": "Medical, surgical and gynaecological illness complicating pregnancy", "id": "63f05900-8954-40a0-bfeb-8116462a7453", "choice_type": "single"} {"question": "Congenital anamolies are most severe in", "exp": "Ans. is 'a' Rubella infectionRubella causes numerous serious defects in the fetus, but nowhere is it specifically written that it causes most severe congenital deformities. Just have a look at the various defects caused by Rubella and you won't need any further confirmation for the answer.Various fetal defects caused d/t RubellaCardiac malformations* PDA (most common)* VSD, PS, (Coarctation of Aorta is not present)*Deafness*Cataract*Glaucoma*Retinopathy*Microcephaly*Cerebral palsy*1UGRHepatosplenomegalyMental & Motor retardationThese defects occuring singly or in combination are known as \"Congenital Rubella Syndrome\"", "cop": 1, "opa": "Rubella infection", "opb": "Mumps", "opc": "CMV", "opd": "Toxoplasma", "subject_name": "Gynaecology & Obstetrics", "topic_name": "Pathology of Conception", "id": "25505411-426c-429e-afa6-0b39a11ec3f8", "choice_type": "single"} {"question": "The below abnormality is seen in", "exp": "Ans. A. Monochorionic diamniotic twins\"All monochorionic placentas likely share some anastomotic connections. And, with rare exceptions, anastomoses between twins are unique to monochorionic twin placentas.\"\"Artery-to-artery anastomoses are most frequent and are identified on the chorionic surface of the placenta in up to 75 percent of monochorionic twin placentas. Vein-to-vein and artery-to-vein communications are each found in approximately half. One vessel may have several connections, sometimes to both arteries and veins. In contrast to these superficial vascular connections on the surface of the chorion, deep artery-to-vein communications can extend through the capillary bed of a given villus. These deep arteriovenous anastomoses create a common villous compartment or \"third circulation\" that has been identified in approximately half of monochorionic twin placentas.\" \"Whether these anastomoses are dangerous to either twin depends on the degree to which they are hemodynamically balanced. In those with significant pressure or flow gradients, a shunt will develop between fetuses. This chronic fetofetal transfusion may result in several clinical syndromes that include twin-twin transfusion syndrome (TTTS), twin anemia polycythemia sequence (TAPS), and acardiac twinning.\"", "cop": 1, "opa": "Monochorionic diamniotic twins", "opb": "Dichorionic diamniotic twins", "opc": "Monochorionic monoamniotic twins", "opd": "Conjoined twins", "subject_name": "Gynaecology & Obstetrics", "topic_name": "Multiple Pregnancy", "id": "375be968-f50f-4674-bb30-2a7a98961122", "choice_type": "single"} {"question": "Women with PCOD and Hirsutism, Management is", "exp": "Treatment of Hirsutism\n\nEstrogen-Progestin Contraceptives\nAntiandrogens \n\nMechanism of action\n\nAndrogen production in hirsuite women usually is an LH-dependent process\n\n\nEstrogen-progestin contraceptives suppress pituitary LH secretion and thus suppress ovarian androgen production\n\n\nThe high level of estrogen in oral contraceptives stimulates hepatic SHBG production, which decreases the amount of free/active androgen\nDirectly or indirectly, estrogen-progestin contraceptives decrease adrenal DHEA-S secretion\nContraceptive progestins inhibit 5 alfa-reductase activity in the skin, which decreases the production of dihydrotestosterone (DHT), the major nuclear androgen in hair follicles and sebaceous glands. Treatment with estrogen-progestin contraceptives induces regular, predictable menses and attenuates endometrial growth, thereby eliminating the risk for developing endometrial hyperplasia and neoplasia Current oral contraceptives contain ethinyl estradiol, in doses ranging from 20 mcg to 50 mcg daily, and one of a variety of progestins All low-dose oral contraceptives (containing 20–35 µg ethinyl estradiol) have similar effectiveness in the treatment of acne and hirsutism Cyproterone is a derivative of 17a-hydroxyprogesterone (17OHP), has potent progestational activity that inhibits gonadotropin secretion and also acts as competitive androgen receptor antagonist which inhibits enzymes involved in androgen synthesis\n“Dianette” or “Diane 35” contains 2 mg cyproterone acetate and 35 mg ethinyl estradiol\nAntiandrogens are an effective treatment for hirsutism but are best used in combination with oral contraceptives or another means of highly effective contraception, because they adversely affect sexual development in a male fetus if the patient conceives during treatment", "cop": 1, "opa": "Ethinyl estradiol + Cyproterone Acetate", "opb": "Ethinyl estradiol", "opc": "Levonorgestrel", "opd": "Ethinyl estradiol + Levonorgestrel", "subject_name": "Gynaecology & Obstetrics", "topic_name": null, "id": "8c6aac4d-e5d8-48ba-8a9d-9b3e5dc06904", "choice_type": "single"} {"question": "Normal blood loss in menstruation is", "exp": "Normal blood loss during menstruation is 30-40ml Ref: Internet sources", "cop": 1, "opa": "20-40ml", "opb": "40-80ml", "opc": "80-120ml", "opd": "120-150ml", "subject_name": "Gynaecology & Obstetrics", "topic_name": "Reproductive physiology and hormones in females", "id": "b31304a8-1c16-4182-88bd-498c0119e742", "choice_type": "single"} {"question": "Cervical carcinoma arises from", "exp": "Carcinoma of cervix arise from the Squamocolumnar juction Vagina and Ectocervix are covered by stratified squamous epithelium and the endocervix is covered by columnar epithelium. The two meet at squamocolumnar junction which is dynamic. Reference : Textbook of Gynaecology; Sheila balakrishnan; 2nd edition; Pg no:257", "cop": 1, "opa": "Squamocolumnar junction", "opb": "Isthmus", "opc": "Cervical anterior lip", "opd": "Internal os", "subject_name": "Gynaecology & Obstetrics", "topic_name": "Gynaecological oncology", "id": "ee3eaa23-1df8-4e37-8892-368b42e3b691", "choice_type": "single"} {"question": "The most important indication for surgical repair of a double uterus, such as a septate or bicornuate uterus, is", "exp": "Habitual abortion is the most important indication for surgical treatment of women who have a double uterus. The abortion rate in women who have a double uterus is two to three times greater than that of the general population. Therefore, women who present with habitual abortion should be evaluated to detect a possible double uterus. Hysterosalpingography, hysteroscopy, ultrasound, CT, and magnetic resonance imaging (MRI) are all potentially useful imaging modalities in this investigation. Dysmenorrhea, premature delivery, dyspareunia, and menometrorrhagia are other, less important indicators for surgical intervention.", "cop": 1, "opa": "Habitual abortion", "opb": "Dysmenorrhea", "opc": "Menometrorrhagia", "opd": "Dyspareunia", "subject_name": "Gynaecology & Obstetrics", "topic_name": "Physiology & Histology", "id": "2ecace01-a4b9-4f87-8b0f-1c521bba77f9", "choice_type": "single"} {"question": "Conjugate of the diagonal is taken as 'a' cm, then obstetric conjugate will be", "exp": "Obstetrical conjugate\n\nIt is the shortest distance between the sacral promontory and the symphysis pubis\nNormally, this measures 10 cm or more, but it cannot be measured directly with examining fingers\nThus, for clinical purposes, the obstetrical conjugate is estimated indirectly by subtracting 1.5 to 2 cm from the diagonal conjugate, which is determined by measuring the distance from the lowest  margin of the symphysis to the sacral promontory", "cop": 4, "opa": "a + 1 cm", "opb": "a + 2 cm", "opc": "a - 1 cm", "opd": "a - 2 cm", "subject_name": "Gynaecology & Obstetrics", "topic_name": null, "id": "86dff55e-8e1f-4260-8b8d-5bf18d5ee4cb", "choice_type": "single"} {"question": "'Twin peak\" sign is seen in", "exp": "(A) Dichorionic diamniotic twins # Twin peak sign indicates the presence of a dichorionic-diamniotic twin gestation. It represents the extension of placental villi into the potential space that is formed from the reflection of apposed amniotic and chorionic layers from each fetus.> By performing an obstetric ultrasound at a gestational age of 10-14 weeks, mono- chorionic-diamniotic twins are discerned from dichorionic twins.> Presence of a \"T-sign\" at the inter-twin membrane-placental junction is indicative of monochorionic-diamniotic twins (that is, the junction between the inter-twin membraneand the external rim forms a right angle), whereas dichorionic twins present with a \"lambda (I) sign\" (that is, the chorion forms a wedge-shaped protrusion into the inter-twin space, creating a rather curved junction).> The \"Lambda sign\" is also called the \"twin peak sign\".> At ultrasound at a gestational age of 16-20 weeks, the \"lambda sign\" is indicative of dichorionicity but its absence does not exclude it.", "cop": 1, "opa": "Dichorionic diamniotic twins", "opb": "Monochorionic monoamniotic twins", "opc": "Discordant twins", "opd": "Conjoined twins", "subject_name": "Gynaecology & Obstetrics", "topic_name": "Miscellaneous (Obs)", "id": "75b06fb6-05f0-491b-aa67-7d3dcafeb99b", "choice_type": "single"} {"question": "Polymenorrhoea Means", "exp": "Polymenorrhoea means regular and shoened cycles Reference: Shaw&;s textbook of gynaecology; 16th edition; Page no 293", "cop": 1, "opa": "Menses < 21 days", "opb": "Menses >35 days", "opc": "Painful menses", "opd": "DUB", "subject_name": "Gynaecology & Obstetrics", "topic_name": "Disorders of menstruation", "id": "0dc64e00-91a9-4860-8c4d-69cfc8fe0d81", "choice_type": "single"} {"question": "ln multiparous women, external os is i", "exp": "Transverse", "cop": 3, "opa": "Circular", "opb": "Longitudinal", "opc": "Transverse", "opd": "Septate", "subject_name": "Gynaecology & Obstetrics", "topic_name": null, "id": "0e4fcd70-33da-4eba-9369-ec3839b1289c", "choice_type": "single"} {"question": "Remnants of WolFion ducts in a female are in", "exp": "Ganer&;s duct remnants of mesonephric duct in female Located in Broad ligament ref : BD CHAURASIA", "cop": 2, "opa": "Pouch of Douglas", "opb": "Broad ligament", "opc": "Utero vesical pouch", "opd": "iliac fossa", "subject_name": "Gynaecology & Obstetrics", "topic_name": "All India exam", "id": "384e7abe-73b4-43cd-93d9-d37f1c958821", "choice_type": "single"} {"question": "Marker for granulosa cell tumor;", "exp": "

- inhibin B is a peptide growth factor secreted by granulosa cells. - it is a measure of diminished ovarian reserve. - inhibin B is elevated in granulosa cell tumors. Reference : textbook of gynaecology Sheila balakrishnan, 2nd edition, pg no: 135 <\\p>", "cop": 3, "opa": "CA 19-9", "opb": "CA 50", "opc": "Inhibin", "opd": "Teratoma", "subject_name": "Gynaecology & Obstetrics", "topic_name": "Gynaecological oncology", "id": "840609a5-b36b-400f-9cdb-c630411d3da0", "choice_type": "single"} {"question": "End product of progesterone metabolism found in urine", "exp": "Fate of progesterone\n\nWithin a few minutes after secretion, almost all the progesterone is degraded to other steroids that have no progesterone effect\nThe major end product of progesterone degradation is pregnanediol. Approximately 10%  is excreted in urine in this form", "cop": 4, "opa": "Pregnenolone", "opb": "17-OH Pregnenolone", "opc": "Pregnanetriol", "opd": "Pregnanediol", "subject_name": "Gynaecology & Obstetrics", "topic_name": null, "id": "2058fe37-46bb-4608-ba1f-1dc639dc4e05", "choice_type": "single"} {"question": "The sensitivity of uterine musculature is", "exp": "Progesterone reduces muscle excitability by increasing calcium binding, thereby reducing free intracellular calcium. Oestrogen has the opposite effect and increases myometrial excitability.Refer page no 106 of Textbook of obstetrics, Sheila Balakrishnan, 2 nd edition.", "cop": 4, "opa": "Enhanced by progesterone", "opb": "Enhanced by estrogen", "opc": "Inhibited by estrogen", "opd": "Enhanced by estrogen and Inhibited by progesterone", "subject_name": "Gynaecology & Obstetrics", "topic_name": "General obstetrics", "id": "832ec1a2-99db-4673-9c7b-5750d5ea2988", "choice_type": "single"} {"question": "Most common cause of postpartum haemorrhage (PPH) is", "exp": "Ans. a (Atonic uterus). (Ref. Textbook of Obstetrics by Dutta, 3rd ed., 424)*Most Common Cause Of Primary PPH is atonicity of the uterus (80%)*Most Common Cause Of secondary PPH is retained bits of cotyledon or membrane.RISK FACTORS FOR POSTPARTUM HEMORRHAGE# Multiple Gestations# Large baby# Polyhydramnios# Multiparity (particularly grand multiparity, more than 5 term pregnancies)# Prolonged labor (uterine inertia)# Labor augmented with Pitocin# General Anesthesia# Placenta Previa# Abruptio Placentae# Magnesium Sulfate infusion", "cop": 1, "opa": "Atonic uterus", "opb": "Cervical tears", "opc": "Episiotomy wound", "opd": "Vaginal tears", "subject_name": "Gynaecology & Obstetrics", "topic_name": "Miscellaneous (Gynae)", "id": "331b8d23-38ab-4e37-9ede-4bf2c4472b17", "choice_type": "single"} {"question": "Shoest diameter of foetal skull is", "exp": "Bitemporal", "cop": 1, "opa": "Bitemporal", "opb": "Biparital", "opc": "Veicomental", "opd": "Suboccipito bregmatic", "subject_name": "Gynaecology & Obstetrics", "topic_name": null, "id": "29204b05-7700-4d48-923c-00a6779e39c6", "choice_type": "single"} {"question": "Fate of Inner cell mass in embryo development...", "exp": "Cells of blastocyst differentiate into an outer trophoectoderm and inner cell mass. Just before implantation trophoectoderm fuher divided into: Cytotrophoblast & Syncytiotrophoblast Placenta and fetal membrances are derived from trophoectoderm. Inner cell mass helps in the development of embryo proper.", "cop": 3, "opa": "Placenta", "opb": "amniotic membrane", "opc": "Fetus", "opd": "Expelled out of developing embryo by 7 weeks", "subject_name": "Gynaecology & Obstetrics", "topic_name": "Obstetrics", "id": "d9baa8a7-a084-4ec9-a3ab-715c33d738e3", "choice_type": "single"} {"question": "Lymph Nodes not involved in Cervical cancer", "exp": "Lymphatic spread of Cervical Cancer includes: Pelvic nodes Parametrial Obturator Internal iliac External iliac Common iliac Sacral Para aoic nodes Reference : Textbook of Gynaecology; Sheila balakrishnan; 2nd edition; Pg no: 258", "cop": 4, "opa": "Para aoic Lymph nodes", "opb": "Obturator Lymph Nodes", "opc": "Hypogastric Lymph Nodes", "opd": "Femoral Lymph Nodes", "subject_name": "Gynaecology & Obstetrics", "topic_name": "Gynaecological oncology", "id": "4e326ed3-1937-4ffb-9329-88c7006c2ebc", "choice_type": "single"} {"question": "Cervical myoma is", "exp": "Cervical stroma being poor in smooth muscle fibers, majority of myoma arise from isthumus of uterus.\nIn contrast to uterine myomas, cervical myoma is usually solitary.", "cop": 2, "opa": "Multiple growths", "opb": "Solitary growth", "opc": "Mostly large always", "opd": "Mainly arise from cervical stroma", "subject_name": "Gynaecology & Obstetrics", "topic_name": null, "id": "6e4d81d5-2e71-4e49-87f9-9e3ca9ac390e", "choice_type": "single"} {"question": "The expectant line of management in placenta prae is contraindicated in", "exp": "Active InterferenceThe indication of active treatment in placenta pre are: Bleeding is continuingBaby is dead or known to be congenitally deformed Bleeding occurs at or beyond 38 weeks of pregnancy Patients is in emarginated state on admissionPatient is labour.", "cop": 4, "opa": "Pre term foetus", "opb": "Live foetus", "opc": "Breech presentation", "opd": "Active labour", "subject_name": "Gynaecology & Obstetrics", "topic_name": "General obstetrics", "id": "b93c40bb-b87e-4eb8-9780-a7a82944491b", "choice_type": "single"} {"question": "Conservative surgical treatment of PPH includes", "exp": "Ans. a (Internal iliac artery ligation). (Ref. Dutta, Obstetrics 4th/ 447, Williams, Obstertrics, 21st/620, 635)Traditionally, postpartum hemorrhage (PPH) has been defined as the loss of 500 mL of blood or more after completion of the third stage of labor. This is unreasonable, because nearly half of all women who are delivered vaginally shed that amount of blood or more when measured quantitatively. After the first 24 hours it is designated late postpartum hemorrhage.Scheme for the management of true PPH:# If uterus is flabby:- Massage uterus- IV methergin (Drug of choice) - MH 2006- Oxytocin- Bladder catheterization- Examine expelled placenta# Bleeding Unresponsive to Oxytocics: Following management should be initiated immediately:- Use bimanual uterine compression- Begin blood transfusions- Explore the uterine cavity manually for retained placental fragments or lacerations.- If still flabby, hot uterine douche and intrauterine packing and- If this fails, either hysterectomy or bilateral ligation of anterior division of internal iliac arteries (in exceptional cases as an alternative to hysterectomy).", "cop": 1, "opa": "Internal iliac artery ligation", "opb": "Intravenous methergin", "opc": "Packing of uterus", "opd": "Vaginal hysterectomy", "subject_name": "Gynaecology & Obstetrics", "topic_name": "Miscellaneous (Gynae)", "id": "97a82343-c65f-4228-9aa0-4cc3f16c87a8", "choice_type": "single"} {"question": "Most important criteria in semen analysis is", "exp": "Most important criteria in semen analysis is sperm morphology > Sperm motility > Sperm concentration.", "cop": 2, "opa": "Sperm concentration", "opb": "Sperm motility", "opc": "Semen volume", "opd": "Total sperm count", "subject_name": "Gynaecology & Obstetrics", "topic_name": null, "id": "65768012-8d50-4f47-abd4-2ae55a3c1e7d", "choice_type": "single"} {"question": "Polyhydramnios not associated with", "exp": "Ans. is 'a' i.e. B/L renal agenesis Polyhydramnios is defined as a state where liquour amni exceeds 2000 ml.Amniotic fluid measures about 50 ml at 12 weeks, 400 ml at 20 weeks and reaches its peak of 1 litre at 36-38 weeks.There after the amount diminishes till at term it measures about 600-800 ml Amniotic fluid formation:- Early pregnancy - Ultrafiltrate of plasma Beginning of 2nd trimester - Composed largely of extracellular fluid that diffuses through the fetal skin. After 20 weeks - Cornification of fetal skin prevents this diffusion and the amniotic fluid is composed largely of fetal urine.The fluid in the amniotic fluid is completely changed and replaced in every 3 hours.Causes of increase in amniotic fluid (Polyhydramnios)Because the fetus normally swallows the amniotic fluid and excretes it through urine it has been assumed that this mechanism is one of the ways by which the volume is controlled.Thus renal agenesis will lead to oligohydramnios not polyhydramnios.Anencephaly causes polyhydramnios because of transudation from exposed meninges and absence of fetal swallowing reflex.Open spina bifida leads to polyhydramnios because of transudation of meninges.Causes of polyhydramnios MaternalPh isoimmunisationDiabetes PlacentalPlacental chrioangiomaCircumvallate placenta syndrome FetalMultiple pregnancyFetal anomaliesCNS abnormalities (Anencephaly)G.L abnormalities (Oesophageal or duodenal atresia)G. U. abnormalitiesSkeletal malformationsFetal tumoursCardiac anomaliesChromosomal abnormalitiesGenetic syndromesHematological disordersIntrauterine infectionsMiscellaneous IdiopathicComplications of Polyhydramnios:Most of the maternal complications of polyhydramnios arise purely from pressure exerted within the overdistended uterus and upon adjacent organs. Which areRespiratory discomfortPremature rupture of membranePremature labourPregnancy induced hypertensionAbruptio placentaCord prolapsePlacental insufficiencyPost partum hemorrhageFetal complications:Fetal morbidity and mortality is significant in cases of polyhydramnios.The major causes of mortality are congenital abnormalities incompatible with life.", "cop": 1, "opa": "B/L renal agenesis", "opb": "Anencephaly", "opc": "Open spina bifida", "opd": "Tracheo esophageal fistula", "subject_name": "Gynaecology & Obstetrics", "topic_name": "Hydramnios and Oligo Hydramnios", "id": "f2bb8873-7061-4892-a25c-c1367a496a12", "choice_type": "single"} {"question": "Lichen sclerosus lesions are limited by", "exp": "Lesions of lichen sclerosus are always limited by labia majora.", "cop": 4, "opa": "Vagina", "opb": "Cervix", "opc": "Labia minora", "opd": "Labia majora", "subject_name": "Gynaecology & Obstetrics", "topic_name": null, "id": "a55ccc4d-893b-4bd7-a7c9-61a1a4ec6ac3", "choice_type": "single"} {"question": "Cervical pregnancy is confirmed by the presence of", "exp": "Ans is 'd' i.e., Histology showing presence of villi inside the cervical stromaCervical PregnancyIn cervical pregnancy, the bleeding is painless and the uterine body lies above the distended cervix. Intractable bleeding following evacuation or expulsion of the products brings about suspicion.The morbidity and moality is high because of profuse hemorrhage.Clinical diagnostic criteria (Rubin-1983) for cervical pregnancy are?Soft, enlarged cervix equal to or larger than the fundus.Uterine bleeding following amenorrhea, without cramping pain.Products of conception entirely confined within and firmly attached to endocervix.A closed internal cervical os and a paially opened external os.Sonography reveals the pregnancy in the cervical canal and an empty uterine cavity.Confirmation is done by histological evidence of the presence of villi inside the cervical stroma.", "cop": 4, "opa": "Gestational sac below internal os", "opb": "Intractable bleeding following evacuation or ex pulsion of products", "opc": "Bleeding is painless", "opd": "Histology showing presence of villi inside the cervical stroma", "subject_name": "Gynaecology & Obstetrics", "topic_name": null, "id": "fa39eb13-7ce1-46de-be7e-9b39b307f654", "choice_type": "single"} {"question": "Most frequent chromosomal abnormality found in those with heart defect", "exp": "Trisomy 21 >> Trisomy 18 >> 22q Microdeletion >> Trisomy 13 >> monosomy X.\nThis is order of occurrence of heart defect most commonly and frequently.", "cop": 2, "opa": "Trisomy 13", "opb": "Trisomy 21", "opc": "Trisomy 18", "opd": "Monosomy X", "subject_name": "Gynaecology & Obstetrics", "topic_name": null, "id": "4292b67d-0b0b-45f9-988c-5b97f531ab80", "choice_type": "single"} {"question": "At 8 weeks postpaum, uterus should weigh about", "exp": "During pregnancy, uterus weighs about 1000g At the end of 6weeks ,it's measurement is almost similar to the prepregnant state which is about 60g . Decrease in size of uterus is seen with serial MRI Ref: Dutta Obs 9e pg 137.", "cop": 1, "opa": "100 grams", "opb": "500 grams", "opc": "700 grams", "opd": "900 grams", "subject_name": "Gynaecology & Obstetrics", "topic_name": "General obstetrics", "id": "ae94acea-7902-465d-9c38-1de2a240e77f", "choice_type": "single"} {"question": "Lactational amenorrhoea is due to", "exp": "Ans. is 'a' i.e. Prolactin induced inhibition of GnRH Suppression of the female ovarian cycles in nursing mothers for many months after delivery.In most nursing mothers, the ovarian cycle and ovulation does not resume until a few weeks after cessation of nursing the babyThe cause of this seems to be that the same nervous signals from the breasts to the hypothalamus that cause prolactin secretion during suckling either because of the nervous signals themselves or because of the subsequent effect of the increased prolactin, inhibit secretion of gonadotropin releasing hormone by the hypothalamus.This in turn, suppresses the formation of the pituitary gonadotrophic hormones, leutinizing hormones and follicle stimulating hormone.Yet, after several months of lactation in some mothers, especially in those who nurse their babies only part of the time, the pituitary does begin again to secrete sufficient gonadotropic hormones to reinstate the monthly sexual cycle even though nursing continues.", "cop": 1, "opa": "Prolactin induced inhibition of GnRH", "opb": "Prolactin induced inhibition of FSH", "opc": "Oxytocin induced inhibition of GnRH", "opd": "Oxytocin induced inhibition of FSH", "subject_name": "Gynaecology & Obstetrics", "topic_name": "Endocrine Control of the Menstrual Cycle", "id": "4c7aa4e9-930f-4955-91fd-f0d260e29edc", "choice_type": "single"} {"question": "Most common cause of hirsutism is", "exp": "Ans. is 'a' Polycystic Ovary disease We have already discussed that hirsutism occurs d/t androgen excess in a female and the most common cause of androgen excess is polycystic ovary disease.", "cop": 1, "opa": "Polycystic ovary disease", "opb": "Arrhenoblastoma", "opc": "Cushings syndrome", "opd": "Congenital adrenal hyperplasia", "subject_name": "Gynaecology & Obstetrics", "topic_name": "Disorders of Ovulation - Anovulation ", "id": "7191074e-8137-40e1-9698-f00825b2ff82", "choice_type": "single"} {"question": "Genetic component of Superfemale is", "exp": "Superfemale (Triple X Chromosome)The possession of an extra X is not excessively rare since it is quite compatible with complete feminine normality. There is, however, a well-recognized triple X syndrome in which the patient, who is often mentally subnormal, suffers from scanty or irregular menstruation and infeility. Clinical examination may reveal hypoplasia of the genital tract.Reference: Shaw's textbook of gynecology; 16th edition; Page no 146", "cop": 4, "opa": "XX", "opb": "XXY", "opc": "XO", "opd": "XXX", "subject_name": "Gynaecology & Obstetrics", "topic_name": "Sexuality and intersexuality", "id": "bb7e78be-3a2c-422a-a1c5-8817280d3e0a", "choice_type": "single"} {"question": "During Dilatation and curettage, if uterine fundus is perforated, most appropriate management is", "exp": "Laparotomy or laparoscopy to examine abdominal contents is safest to rule out intraabdominal damage, perionitis due to bowel injury.", "cop": 3, "opa": "Observation", "opb": "Hysterectomy", "opc": "Abdominal exploration", "opd": "Uterine artery embolization", "subject_name": "Gynaecology & Obstetrics", "topic_name": null, "id": "8b61ae11-b1d6-488d-b4d4-62cb36d420de", "choice_type": "single"} {"question": "Endosalpingitis is best diagnosed by", "exp": "Clinical Triad for Diagnosis of PID Pelvic organ tenderness, Adnexal tenderness, Cervical motion tenderness (Suggests the presence of peritoneal inflammation, which causes pain when the peritoneum is stretched by moving the cervix and causing traction of the adnexa on the pelvic peritoneum) Direct or rebound abdominal tenderness may be present. Evaluation of both vaginal and endocervical secretions is a crucial pa of the workup of a patient with PID. Increased number of polymorphonuclear leukocytes may be detected in a wet mount of the vaginal secretions or in the mucopurulent discharge. More elaborate tests include Endometrial biopsy to confirm the presence of endometritis, Ultrasound or radiologic tests to characterize a tubo-ovarian abscess laparoscopy to confirm salpingitis visually A laparoscopy, if asked in the Questions, is indeed the best test to diagnose PID. Its not a practical option always since this is a common presentation in the OPD and will be impossible to do on every one . Regular OPD diagnosis is made by the clinical triad If a Laparoscopy is being done for some indication & the surgeon notices inflamed tubes, then it is diagnostic of a PID Also: Please note that for diagnosis of PID , any uterine procedure like hysteroscopy, HSG, sonosalpingography ,will all push in fluids through the Uterus and this can in fact increase the PID.", "cop": 1, "opa": "laparoscopy", "opb": "X-Ray abdomen", "opc": "Hysterosalpingography", "opd": "Hystero-laparoscopy", "subject_name": "Gynaecology & Obstetrics", "topic_name": "Genital Tract Infections (Too hot to handle!)", "id": "58f22ff6-8cd5-4132-b371-62f293b463d8", "choice_type": "single"} {"question": "Causative organism of strawberry cervix is", "exp": "Trichomonas Vaginitis Vulvar erythema and edema Copious frothy yellow green foul smelling discharge Punctate lesions of cervix-Strawberry cervix Vaginal pH >4.5 Ref: Shaw Gynecology 17 e pg 330.", "cop": 1, "opa": "Trichomonas", "opb": "Hemophilus", "opc": "Candida", "opd": "Herpes", "subject_name": "Gynaecology & Obstetrics", "topic_name": "Infections of the genital tract", "id": "4fde522d-9562-4f00-96f3-9b4aef19d57f", "choice_type": "single"} {"question": "According to Naegele's rule, calculate EDD of a patient with LMP 9/01/2017", "exp": "Naegel's rule = 9 months + 7 days (add to the first day of last menstrual period) - LMP = 9/01/2017- Apply naegel's rule and add 9 months + 7 days =16/10/2017 (add 7 to date and 9 to month)", "cop": 1, "opa": "16/10/2017", "opb": "16/09/2017", "opc": "16/11/2017", "opd": "9/10/2017", "subject_name": "Gynaecology & Obstetrics", "topic_name": "Diagnosis of Pregnancy", "id": "bc698970-ad4b-48d9-8195-0ddf70854ed5", "choice_type": "single"} {"question": "The weight of placenta at term is", "exp": "(500 gm): Ref: 29-D, 23-H & BPlacenta at term. The mature placenta is a Fleshy, discoid organ weighing about 500 gm usually weighing about 1/6 of the fetal weight. It is about 20 - 25 cm in diameter and 2.5 cm thickness at term, about four-fifths ofplacenta is of fetal origin.", "cop": 2, "opa": "250 gm", "opb": "500 gm", "opc": "7500 gm", "opd": "1000 gm", "subject_name": "Gynaecology & Obstetrics", "topic_name": "Miscellaneous (Gynae)", "id": "e18e1bc4-3142-455e-acf3-108aa2a348ab", "choice_type": "single"} {"question": "Absolute contraindication for Transvaginal sonography", "exp": "Imperforate hymen and patient refusal are the only absolute contraindications for Transvaginal sonography.", "cop": 2, "opa": "Placenta previa", "opb": "Imperforate hymen", "opc": "Abruptio placenta", "opd": "Abnormal uterine bleeding", "subject_name": "Gynaecology & Obstetrics", "topic_name": null, "id": "f747bf68-aa24-4ceb-a245-b439736bd960", "choice_type": "single"} {"question": "Alpha fetoprotein synthesized from", "exp": "(Yolk sac) (107-D)* AFP - is a oncofetal protein it is produced by yolksac and fetal liver* Highest level of AFT in fetal serum and amniotic fluid reached around 13 weeks and thereafter it decreases* Maternal serum level reaches a peak around 32 weeks* * Test is done between 15-18 weeksMSAFP level is elevated in(a) Wrong gestational age(b) Open neural tube defects(c) Multiple pregnancy(d) IUCD(e) Anterior abdominal wall defects(f) Renal anomaliesMSAFP level is lowin Trisomies (Down's syndrome) gestational trophoblastic disease* Triple test - includes MSAFP, hCG and UE3 (Unconjugated oestriol)", "cop": 1, "opa": "Yolk sac", "opb": "Placenta", "opc": "Foetal brain", "opd": "Foetal kidney", "subject_name": "Gynaecology & Obstetrics", "topic_name": "Miscellaneous (Gynae)", "id": "efc8e405-2ad7-41ac-96c5-36fadef0d85f", "choice_type": "single"} {"question": "M/C uterine malformation associated with infeility", "exp": "Septate uterus The two mullerian ducts are fused together but there is a persistance of septum in between the either two paially or completely Infeility and dyspareunia are often associated with vaginal septum D.C.DUTTA&;S TEXTBOOK OF GYNAECOLOGY,Pg no:44,6th edition", "cop": 3, "opa": "Bicornuate", "opb": "Unicornuate", "opc": "Septate", "opd": "Didelphys", "subject_name": "Gynaecology & Obstetrics", "topic_name": "Congenital malformations", "id": "28056b29-d9fe-4f5e-900a-aa77abeb4e00", "choice_type": "single"} {"question": "M/C site of ureteric injury during hysterectomy", "exp": "The injury to ureter occurs at the infundibulopelvic junction on the lateral pelvic wall, in the ureteric canal when the uterine vessels are ligated, near the internal cervical os and near the uterosacral ligamentIt is impoant to identify the ureter during Weheim hysterectomy, broad ligament tumour dissection and ligating internal iliac aeryRef: Shaw&;s Textbook of Gynaecology, 15th edition, page no: 14", "cop": 2, "opa": "Pelvic brim", "opb": "Where it is crossed by uterine aery", "opc": "Where it enters the bladder", "opd": "Where it is over obturator vessels", "subject_name": "Gynaecology & Obstetrics", "topic_name": "Anatomy of the female genital tract", "id": "db1c8a79-5c0e-4131-9154-46a9803eadfa", "choice_type": "single"} {"question": "Vitamin K injection given to newborn within 1 hour of birth", "exp": "Vitamin K dependent hemorrhagic disease of newborn is prevented by vitamin - K injection.", "cop": 2, "opa": "Prevents necrotizing enterocolitis", "opb": "Prevents hemorrhagic disease of newborn", "opc": "Prevents neonatal seizures", "opd": "Prevents neonatal tetanus", "subject_name": "Gynaecology & Obstetrics", "topic_name": null, "id": "a234d8a2-ba9f-4a6d-83c1-7ee30c392ec2", "choice_type": "single"} {"question": "Uterine blood flow at term is aEUR'", "exp": "500-750 ml \"Uterine blood flow is increased from 50 ml/min in non pregnant states to about 750 ml near term\". \"Uterine blood increase from 50m/min in early gestation to 500-750 m/min by term\".", "cop": 4, "opa": "50-70 ml/min", "opb": "100-150 ml/min", "opc": "175-200 ml/min", "opd": "500-750 ml/min", "subject_name": "Gynaecology & Obstetrics", "topic_name": null, "id": "2111ee61-4537-48f7-a8e6-93b60d07ca54", "choice_type": "single"} {"question": "The most common pure germ cell tumour of the ovary", "exp": "Ans. is b i.e. Dysgerminoma \"Dysgerminoma is the most common malignant germ cell tumour accounting for about 40% of all ovarian cancers of germ cell origin.\" \"Dysgerminomas are the most common malignant germ cell tumours of the ovary and have been considered the female - equivalent of seminoma.\" Most common germ cell tumour of ovary is dermoid cyst (mature teratoma). It is benign in nature. Dysgerminoma Commonest malignant germ cell tumourdeg of ovary. Primarily affect young women (average age of incidence is 20 yearsdeg). Usually unilateraldeg but they are the only germ cell malignancy with a significant rate of bilateral ovarian involvement - 15 to 20%. Can be found at gonadal as well as extra gonadal sites.deg Pathologically it is a solid neoplasm with areas of softeningdeg due to degeneration. They are the most common ovarian malignancy detected during pregnancy. Histologically as in seminoma, it mimics the pattern of primitive gonad, lymphocytic infiltration may be seen (good prognostic sign).deg Clinically as with all germ cell tumours most dysgerminoma are diagnosed at an early stage. Unlike other germ cell tumours it does not secrete AFP and HCG is only rarely secreted, however it secretes LDH and placental alkaline phosphate, which are used as tumour marker of dysgerminoma. Although dysgerminoma is most radio sensitive tumouty2treatment of choice is surgery (unilateral salpingo oophorectomy) followed by Bleomycin, Etoposide and Cisplatin (BEP) based chemotherapy as feility can be preserved. They have the best prognosis of all malignant ovarian germ cell variants. Tumours associated with it : - Immature teratoma - Choriocarcinoma - Endodermal sinus tumour. Dysgerminomas are also seen in phenotypic females with abnormal gonads like : Pure gonadal dysgenesis (46XY with bilateral streak gonads, swyer syndrome) Mixed gonadal dysgenesis Testicular feminization syndrome (46XY) Klinfelter syndrome For patients in whom karyotype reveals Y chromosome, both ovaries should be removed although uterus may be left in situ for possible future embryo transfer.", "cop": 2, "opa": "Choriocarcinoma", "opb": "Dysgerminoma", "opc": "Embryonal cell tumor", "opd": "Malignant Teratoma", "subject_name": "Gynaecology & Obstetrics", "topic_name": null, "id": "0d1d39d8-62be-4d06-90f2-3ad76a06d798", "choice_type": "single"} {"question": "Halban's sign is seen in", "exp": "Halban's sign refers to tender, softened uterus on premenstrual bimanual examination in adenomyosis.", "cop": 2, "opa": "Endometriosis", "opb": "Adenomyosis", "opc": "Cervical cancer", "opd": "Endometrial cancer", "subject_name": "Gynaecology & Obstetrics", "topic_name": null, "id": "36c1f6ed-0c77-4f50-b057-530b87a41a48", "choice_type": "single"} {"question": "Not a barrier contraceptive among the following is", "exp": "Centchroman \n\nNon-hormonal contraceptive pill\nIt is a selective estrogen receptor modulator", "cop": 4, "opa": "Diaphragm", "opb": "Condom", "opc": "Today", "opd": "Centchroman", "subject_name": "Gynaecology & Obstetrics", "topic_name": null, "id": "e3d80585-2b54-4f3c-a7a4-f5d7d1d125da", "choice_type": "single"} {"question": "In accidental hemorrhage, TOC", "exp": "Ans. is c i.e. Simultaneous emptying of uterus and blood transfusion", "cop": 3, "opa": "Induction of labour", "opb": "Rx of hypofibrinogenemia then blood transfusion", "opc": "Simultaneous emptying of uterus and blood transfusion", "opd": "Wait and watch", "subject_name": "Gynaecology & Obstetrics", "topic_name": null, "id": "082bf003-681a-4f00-8eed-555b07650596", "choice_type": "single"} {"question": "Ovarian tumor involving retroperitoneal lymph nodes is seen in stage", "exp": "-Ovarian cancer staging: Stage III Tumor involves 1 or both ovaries or fallopian tubes or primary peritoneal cancer with cytologically or histologically confirmed spread to the peritoneum outside the pelvis and/or metastasis to the retroperitoneal lymph nodes. -Stage IIIa: Involvement of retroperitoneal lymph nodes and/or microscopic metastasis beyond the pelvis. -Stage IIIa1: positive retroperitoneal lymph nodes only Stage IIIa1(i): metastasis < 10mm Stage IIIa1(ii): metastasis > or = 10mm -Stage IIIa2: Microscopic, extrapelvic peritoneal extension with or without positive retroperitoneal lymph nodes. Reference : textbook of gynaecology Sheila balakrishnan, 2nd edition, pg no: 277 <\\p>", "cop": 2, "opa": "Stage IIB", "opb": "Stage IIIA", "opc": "Stage IIIB", "opd": "Stage IIIC", "subject_name": "Gynaecology & Obstetrics", "topic_name": "Gynaecological oncology", "id": "a92ef444-9d90-45e6-962d-1aae775ef6ce", "choice_type": "single"} {"question": "A 29yrs old nulliparous woman complains of severe menorrhagia and lower abdominal pain since 3months .On examination there was a 14 weeks size uterus with fundal fibroid .The treatment of choice is", "exp": "Routes of myomectomy : Abdominal myomectomy Vaginal myomectomy: Done in case of submucosal pedunculated fibroid Hysteroscopic myomectomy: Submucosal fibroid which cannot be removed by simple vaginal route can be removed with the help of hysteroscope use when myoma <5 cms in size Laproscopic myomectomy: Indicated for pedunculated subserosal fibroids <10 cms in size Intramural fibroids can also be removed by laproscopy but it is very time consuming Myolysis or myoma coagulation using lazer: In this procedure lasers are used to drill holes into substance of intramural myoma As the patient is having severe menorrhagia and chronic abdominal pain These indications are strong enough for surgical intervention .earlier size of fibroid >12 weeks was also an indication of surgery Indications of myomectomy Myomectomy is specifically indicated in an infeile woman or woman desirous of bearing child ,uterus should be retained Ref Shaw 16/e pg 395", "cop": 1, "opa": "Myomectomy", "opb": "GnRH analogues", "opc": "Hysterectomy", "opd": "Observation", "subject_name": "Gynaecology & Obstetrics", "topic_name": "Anatomy of the female genital tract", "id": "edad9c4a-2dad-4432-92e4-c51c8d71d031", "choice_type": "single"} {"question": "Side effects of magnesium sulfate includes", "exp": "Side effects flushing perspiration headache muscle weakness rarely pulmonary edema neonatal side effects lethargy hypotonia respiratory depression D.C.DUTTA&;S TEXTBOOK OF OBSTETRICS,Pg no:508,7th edition", "cop": 1, "opa": "Hypotonia", "opb": "Anuria", "opc": "Coma", "opd": "Pulmonary edema", "subject_name": "Gynaecology & Obstetrics", "topic_name": "Medical, surgical and gynaecological illness complicating pregnancy", "id": "02a8810f-a45e-4ba9-a789-a7170d8f5ddd", "choice_type": "single"} {"question": "Fibroids cause infertility and pregnancy wastage because", "exp": "Fibroids are associated with endometrial inflammation and vascular changes that may disrupt implantation.", "cop": 2, "opa": "Inhibit normal ovulation", "opb": "Endometrial inflammation", "opc": "Pressure effect on ovaries", "opd": "Hyperestrogenic state", "subject_name": "Gynaecology & Obstetrics", "topic_name": null, "id": "8799aa84-66d3-47d4-beb2-dfb9c1bbb03c", "choice_type": "single"} {"question": "Not an indication for blood transfusion", "exp": "Indications for Blood Transfusion \n\nTo correct anemia due to blood loss and to combat postpartum hemorrhage\nPatient with severe anemia seen in later months of pregnancy (beyond 36 weeks)\nRefractory anemia: Anemia not responding to either oral or parenteral therapy in spite of correct treatment\nAssociated infection", "cop": 1, "opa": "Moderate anemia at 24-30 weeks", "opb": "Severe anemia at 36 weeks", "opc": "Blood loss anemia", "opd": "Refractory anemia", "subject_name": "Gynaecology & Obstetrics", "topic_name": null, "id": "946ee136-fa79-453b-85d5-160ed33bef66", "choice_type": "single"} {"question": "Transobturator tape approach in treatment of urinary incontinence is known to", "exp": "TOT approach was introduced with the intention to reduce vascular and lower urinary tract injury risks that can be associated with traversing retropubic space.", "cop": 2, "opa": "Increased long term efficacy than TVT.", "opb": "Reduce vascular and lower urinary tract injury risks.", "opc": "Overlie over proximal tendon of adductor magnus.", "opd": "Traverse retropubic space, causing neurovascular injury.", "subject_name": "Gynaecology & Obstetrics", "topic_name": null, "id": "93880c0a-210d-4eb8-867e-3bb68f79eb47", "choice_type": "single"} {"question": "Gestational sac can be seen earliest on transvaginal scan at", "exp": "Gestational sac Cardiac activity Transvaginal sonography 4 + weeks 5 + weeks Transabdominal sonography 5 +weeks 6 + weeks Please note: in exams the same information may be asked as 'Which week gestational sac is seen\" ? the answer choice maybe given as 5th week. so please realize that '4 wks + some days' is actually the 5th week.", "cop": 1, "opa": "4- 5 wks", "opb": "5 - 6 wks", "opc": "7 - 8 wks", "opd": "10 wks", "subject_name": "Gynaecology & Obstetrics", "topic_name": "Ectopic Pregnancy (Hello ! Where are you ?)", "id": "45f40480-8f91-4e70-ac3b-ee73cb089f62", "choice_type": "single"} {"question": "Precocious menstruation denotes the occurence of menstruation before the age of", "exp": "Precocious menstruation denotes the occurence of menstruation before the age of 10 years. Precocious menstruation is one of the menstrual cycle irregularities. Precocious pubey in a girl is the appearance of any of the secondary sexual characteristics before the age of 8 years or the occurence of menarche before the age of 10 years. Precocious pubey in boys is the onset of secondary sexual characterstics before the age of nine years. Ref: Shaw&;s textbook of Gynaecology 17th edition Pgno: 122", "cop": 3, "opa": "8 years", "opb": "7 years", "opc": "10 years", "opd": "11 years", "subject_name": "Gynaecology & Obstetrics", "topic_name": "Disorders of menstruation", "id": "90385c97-cba0-4792-a751-1de543cb8a7d", "choice_type": "single"} {"question": "Spontaneous movements of fetus begins at", "exp": "12th Gestational Week The uterus usually is just palpable above the symphysis pubis, and the fetal crown-rump length is 6 to 7 cm. Centers of ossification have appeared in most fetal bones, and the fingers and toes have become differentiated. Skin and nails have developed, and scattered rudiments of hair appear. The external genitalia are beginning to show definitive signs of male or female gender. The fetus begins to make spontaneous movements.Reference: WIlliam's Obstetrics; 24th edition; Chapter 7; Embryogenesis and Fetal Morphological Development", "cop": 2, "opa": "6 weeks", "opb": "12 weeks", "opc": "18 weeks", "opd": "28 weeks", "subject_name": "Gynaecology & Obstetrics", "topic_name": "General obstetrics", "id": "f8e5e6a1-c8cb-481d-a31c-8e4f1212e01f", "choice_type": "single"} {"question": "Treatment of transverse lies at labour is", "exp": "(Caesarean section): Ref: 393, 397-DTRANSVERSE LIE: - When the long axis of the fetus lies perpendicularly to the maternal spine or centralized uterine axis it is called transverse lies* In dorso-posterior, chance of fetal extension is common with increased risk of arm prolopse, left one being common than right* Dorso-anterior is the commonest (60%) positionManagement of Transverse lie at labourEarly labourLate labour* External cephalic version* Caesarian section is the preferred method of delivery, if version fails or contraindicated* Baby alive - caesarean section prefer* Internal version - Not only it might inflect injury to the uterus (rupture uterus) but the fetal mortality is increased to the extent of about 50%In modem obstetric practice internal version is not recommended except in the case of second twin* Baby dead - caesarean section is prefer.* There is no mechanism of labour in transverse lie", "cop": 3, "opa": "Artificial rupture of membrane", "opb": "Oxytocin infusion", "opc": "Caesarean section", "opd": "Forceps delivery", "subject_name": "Gynaecology & Obstetrics", "topic_name": "Miscellaneous (Gynae)", "id": "ff02cf15-fe9d-4a2d-95b6-f48bdf033bea", "choice_type": "single"} {"question": "In terms of bih defect potential, the safest of the following drugs is", "exp": "Alcohol sequelae include retardation of intrauterine growth, craniofacial abnormalities, and mental retardation. Isotretinoin, a drug for acne, is asoociated with congenital anomalies like Microtia, Micrognathia, cleft palate, thymic aplasia, CVS anomalies, NTDs Tetracyclines interfere with development of bone and can lead to stained teeth in children. Regarding progesterones use in pregnancy, controlled studies have failed to demonstrate a significant association with increased risk.", "cop": 4, "opa": "Alcohol", "opb": "Isotretinoin (Accutane)", "opc": "Tetracyclines", "opd": "Progesterones", "subject_name": "Gynaecology & Obstetrics", "topic_name": "Drugs in Pregnancy", "id": "041a81a9-4c26-4e55-90f6-864f9a750504", "choice_type": "single"} {"question": "Patients with organic hea disease in pregnancy most commonly die during", "exp": "Maternal moality in Hea disease complicatin Pregnancy Maternal moality is lowest with rheumatic hea lesions and acyanotic group of hea diseases ie <1% With the elevation of pulmonary vascular resistance especially with cyanotic hea diseases the moality may be raised to 50%( Eisenmengers syndrome) Most of the deaths occur due to cardiac failure and maximum deaths occurs following bih. Management Of Cardiac Failure: The principle if management is same as in the non-pregnant women. A cardiologist should be involved Propped up position O2 Adminstration Monitoring with ECG and pulse oximetry Diuretic: Frusemide (40-80mg) IV - Anticipatory aggresive diuresis is needed to avoid pulmonary congestion. Mechanical Ventilation Injection morphine 15mg IM Digoxin 0.5mg IM followed by Tab.Digoxin 0.25mg P/O Dysrhythmias -Quinidine or electrica cardiaversion Tachyarhymias -Adenosine(3-12mg) IV or DC conversion. Reference: D.C.Dutta's Textbook of Obstetrics; 9th edition; Page no: 258 & 260", "cop": 3, "opa": "20-24 weeks of pregnancy", "opb": "First stage of labour", "opc": "Soon following delivery", "opd": "Two weeks postpaum", "subject_name": "Gynaecology & Obstetrics", "topic_name": "Medical, surgical and gynaecological illness complicating pregnancy", "id": "2fd58702-776e-4bf8-b9b6-ab9997891f00", "choice_type": "single"} {"question": "Ovarian follicles produce increased amounts of Oestrogen and inhibin during", "exp": "During mid follicular phase, follicles produce increased amounts of oestrogen and inhibin, resulting in decline in FSH levels through negative feed back.", "cop": 2, "opa": "Early follicular phase", "opb": "Mid follicular phase", "opc": "Late follicular phase", "opd": "Early luteal phase", "subject_name": "Gynaecology & Obstetrics", "topic_name": null, "id": "e8022f26-908f-4149-a867-6fd29f12a7f7", "choice_type": "single"} {"question": "Risk of HIV transmission is highest during", "exp": ".", "cop": 3, "opa": "During Caesarian section", "opb": "During antepaum period", "opc": "During vaginal delivery", "opd": "Breast feeding", "subject_name": "Gynaecology & Obstetrics", "topic_name": "All India exam", "id": "2e3df92f-1380-48ac-a0ad-bcf0e5e14c83", "choice_type": "single"} {"question": "A female of 35yrs old that has completed her family is having asymptomatic 10 weeks size fibroid . The most appropriate management is", "exp": "Indications of hysterectomy : It&;s called removal of uterus .is indicated in a woman over 40years of age.multiparous woman or when associated with malignancy uncontrolled haemorrhage and unforeseen surgical difficulties during myomectomy for the above case given observation and myomectomy may be appropriate for management However in these conditions hysterectomy should be performed at any cost Abdominal benign: Menorrhagia Uterine fibromyoma Adenomyosis Tubo ovarian mass Carcinoma insitu Atypical endometrial hyperplasia Endometriosis malignancy: Ca.of cervix Ca.of endometrium Ca.ovary Uterine sarcoma mixed mesodermal tumor Choriocarcinoma Obstetric: Rupture of uterus pPH ,molar pregnancy Ca.cervix vaginal: Prolapse Ca.insitu Ca.cervix+ lymphadenectomy Menorrhagia uterine fibroid Genital prolapse In case of asymptomatic condition, wait and watch is the correct option... Following every year with scans to check the size of fibroid Ref Shaw 16/e pg 405,406", "cop": 1, "opa": "Observation only", "opb": "Hysterectomy with bilateral salpingoophorectomy", "opc": "Hysteroscopic myomectomy", "opd": "Myomectomy", "subject_name": "Gynaecology & Obstetrics", "topic_name": "Anatomy of the female genital tract", "id": "4c5f90cc-6250-482f-8802-f60e87c26a81", "choice_type": "single"} {"question": "HIV Positive Primi Near Term, Advice Given isa) Treatment should be started before labourb) Avoid mixing of blood intrapartamc) Vaginal delivery preferredd) Ceaserian section would be decrease transmission of HIV to baby", "exp": null, "cop": 4, "opa": "acd", "opb": "bcd", "opc": "abc", "opd": "abd", "subject_name": "Gynaecology & Obstetrics", "topic_name": null, "id": "47447908-eda3-4b77-bf0e-27b2320519b5", "choice_type": "single"} {"question": "Ureter pierces the following structure in pelvis", "exp": "Ureter pierces Mackenrodt's ligament where a canal, the ureteric canal is developed.", "cop": 2, "opa": "Round Ligament", "opb": "Mackenrodt's ligament", "opc": "Ovarian ligament", "opd": "Broad ligament", "subject_name": "Gynaecology & Obstetrics", "topic_name": null, "id": "df26d30b-144c-422c-b8d0-1458c876263e", "choice_type": "single"} {"question": "To avoid conception, DMPA is given", "exp": null, "cop": 4, "opa": "Monthly", "opb": "Yearly", "opc": "6 Monthly", "opd": "3 Monthly", "subject_name": "Gynaecology & Obstetrics", "topic_name": null, "id": "63b47125-62a9-463c-ab61-915db5f9c251", "choice_type": "single"} {"question": "In carcinoma cervix, the causative etiological factor is", "exp": "HPV 16,18", "cop": 2, "opa": "Condyloma accuminata", "opb": "HPV 16,18", "opc": "HPV 6, 11", "opd": "HSV 1 and 2", "subject_name": "Gynaecology & Obstetrics", "topic_name": null, "id": "201c0a65-49cc-46b1-86cf-219894219ff3", "choice_type": "single"} {"question": "Puerperium is the period", "exp": "Purperium begins as soon as the placenta is expelled and lasts for approximately 6wks when the uterus becomes regressed almost to the nonpregnant size.", "cop": 1, "opa": "6wks following delivery", "opb": "3wks following delivery", "opc": "2wks following delivery", "opd": "1wk following delivery", "subject_name": "Gynaecology & Obstetrics", "topic_name": "General obstetrics", "id": "ef37e01b-457e-4570-9fc7-4c63838cad5d", "choice_type": "single"} {"question": "Vaginal delivary is contraindicated in", "exp": "A. i.e. (Central placenta previa) (252 - Dutta 6th)Indication of caesarean section in APH are1. Severe degree of placenta praevia ( Type IIposterior,Type III (Incomplete or partil central) Type IV (central or total). This is indicated for maternal interest even where the baby is dead2. Lesser degree of placenta praevia where amniotomy fails to stop bleeding or fetal distress appears3. Complicating factors associated with lesser degrees of placenta praevia where vaginal delivery is unsafe* Type II posterior is dangerous placenta**", "cop": 1, "opa": "Central placenta previa", "opb": "Antepartum haemorrhage", "opc": "Previous LSCS", "opd": "Type II anterior placenta previa", "subject_name": "Gynaecology & Obstetrics", "topic_name": "Miscellaneous (Gynae)", "id": "97cc4efe-81e0-4f40-b429-f779ab7a7f5f", "choice_type": "single"} {"question": "Capillary invasion of corpus luteum begins", "exp": "Capillary invasion begins 2 days after ovulaton and reaches center of corpus luteum by 4th day.", "cop": 2, "opa": "One day after ovulation", "opb": "2 days after ovulation", "opc": "3 days after ovulation", "opd": "4 days after ovulation", "subject_name": "Gynaecology & Obstetrics", "topic_name": null, "id": "391d5f53-55a4-4ce9-b0e9-0203cb5914cb", "choice_type": "single"} {"question": "Most common site of metastasis in choriocarcinoma", "exp": "(Lung) (233 - Shaw's 14th)CHORIOCARCINOMA - Malignant growths arising in the body of uterus* Most common, metastases are seen in the lungs (80%), brain and liver* Less common sites are GIT, kidney, spleen, genital tract and lymphnodes (10%)* \"Cannon ball\" metastasis in lung is typical of a malignant lesion* beta - hCG is very specific marker*** Purple haemorrhagic projections either into the lower third of the vagina or around the vaginal orifice is characteristic and pathognomic of chorio carcinoma* The development of chorio carcinoma following H. mole ranges between 2 - 10%* A post molar GTN may be benign or malignant. But a GTN after non-molar pregnancy is always a choriocarcinoma (201 - Dutta 6lh)", "cop": 2, "opa": "Liver", "opb": "Lung", "opc": "Kidney", "opd": "Spleen", "subject_name": "Gynaecology & Obstetrics", "topic_name": "Miscellaneous (Gynae)", "id": "40e3c55e-e3bb-4b8f-be4a-8a705a5bd747", "choice_type": "single"} {"question": "A female with recurrent aboion and isolated prolonged APTT is most likely associated with", "exp": "Lupus anticoagulant They have asked about antiphospholipid antibody syndrome. Antiphospholipid antibodies account for 3% to 5% of patients with repetitive pregnancy losses. The frequency of fetal death and recurrent aboion in untreated patients with antiphospholipid antibodies is greater than 90%. There are several antiphospholipid antibodies. The most relevant to obstetricians are: - The lupus anticoagulant (LAC), - The anticardiolipin antibody and - The antibody that cause false positive syphilis test (BFP-ST). The name lupus anticoagulant was derived from the fact that this antibody was found first in patients with systemic lupus erythematosus and acted as an anticoagulant by prolonging the paial thromboplastin time (PTT). This name was a poor choice because soon it was found that lupus anticoagulant was present in many patients who did not have lupus and that in majority of patients the antibody was responsible for episodes of thrombosis, rather than anticoagulation. In the laboratory, LAC is not measured directly. - It is assessed by its effect on PTT and the kaolin clotting time. - Typically patients with lupus anticoagulant have a prolonged PTT and a normal PT. - Unfounately, a normal PTT does not exclude the possibility of LAC, and if the clinical suspicion is strong and the PIT is normal a kaolin clotting time or a dilute Russel viper venom time should be performed. - It has also been found that some patients with LAC and recurrent aboions have elevated values of serum 1gM. Other antiphospholipid antibodies The anticardiolipin antibody is the antiphospholipid antibody, most commonly found in patients with repetitive early pregnancy losses. Anlicardiolipin antibody is found in 90% of patients with L.A.C. but the majority of patients with positive anticardiolipin antibody do not have L.A.C. - Anticardiolipin antibody is measured in the laboratory by the ELISA test. BFP-ST - It is the less common antiphospholipid antibody. - Both BFP-ST and Anticardiolipin antibody, measure the antibody against cardiolipin but they are not the same. Clinical features of antiphospholipid antibodies. The presence of any or several of the three antiphospholipid antibodies is associated with - Recurrent early pregnancy losses. - Episodes of venous and aerial thrombosis - Severe preeclampsia - Chorea gravidarum - Pilo ischemic strokes, transient ischemic episodes, migraine headaches. - Postpaum complications such as pulmonary infiltrates, fever, and cardiac symptoms. Aboions in antiphospholipid antibody syndrome - Typically these patients give a history of a live fetus documented by ultrasound or by Doppler before demise or aboion occurs. The majority of the pregnancy losses occur between 14 and 18 weeks. Fetal death is these patients is caused by extensive thrombosis of the placental vessels and the placenta is usually smaller than expected for the gestational age.", "cop": 1, "opa": ">Lupus anticoagulant", "opb": ">DIC", "opc": ">Von wilebrand disease", "opd": ">Hemophilia", "subject_name": "Gynaecology & Obstetrics", "topic_name": null, "id": "bbc3579f-7cd2-444d-b952-cf30f454e5c6", "choice_type": "single"} {"question": "Most common cause of intrauterine infection", "exp": "Ans. is 'd' Cytomegalovirus Most common cause of intrauterine infection is cytomegalovirus.", "cop": 4, "opa": "Rubella", "opb": "Toxoplasma", "opc": "Hepatitis", "opd": "Cytomegalovirus", "subject_name": "Gynaecology & Obstetrics", "topic_name": "Pathology of Conception", "id": "72010a45-e26d-4906-94ae-d41acfab42d8", "choice_type": "single"} {"question": "Nile blue sulphatase test in amniotic fluid is for", "exp": "Nile blue sulphate test - Place on a slide, one drop of amniotic fluid and add one drop of 0.1% Nile blue sulphate. Place on slide with a cover slip and observe under high power microscope. More than 50% foetal squamous cells becoming orange in colour is considered as an indicator of foetal lung maturity. ( Please note: NBS test is NOT for Skin maturity)", "cop": 1, "opa": "Lung maturity", "opb": "Kidney maturity", "opc": "Liver maturity", "opd": "Skin maturity", "subject_name": "Gynaecology & Obstetrics", "topic_name": "Preterm Labour", "id": "d403a905-6d69-426e-88b5-2edf7e212c6a", "choice_type": "single"} {"question": "Normal menstrual cycle duration in an adolescent is", "exp": "Parameters for Normal Menstrual Cycles in AdolescentsParameterNormalMenstrual cycle frequency21 - 45 daysCycle variation from cycle to cycleLess than adultsDuration of flow4 - 8 daysVolume of flow4 - 80mlParameters for Normal Menstrual Cycles in Reproductive age womenParameterNormalMenstrual cycle frequency24 - 38 daysCycle variation from cycle to cycle2 - 20 daysDuration of flow4 - 8 daysVolume of flow4 - 80ml", "cop": 2, "opa": "21 - 35 days", "opb": "21 - 45 days", "opc": "24 - 38 days", "opd": "24 - 45 days", "subject_name": "Gynaecology & Obstetrics", "topic_name": "Disorders of menstruation", "id": "91eac6ce-b825-45d3-9c26-49243a34af66", "choice_type": "single"} {"question": "Aspermia is the term used to describe", "exp": "Aspermia- means no semen Azoospermia- no sperm in semen Asthenospermia-no motile sperms or diminished motility \nTeratospermia- abnormal morphology of sperms.", "cop": 1, "opa": "Absence of semen", "opb": "Absence of sperm in ejaculate", "opc": "Absence of sperm motility", "opd": "Occurrence of abnormal sperm", "subject_name": "Gynaecology & Obstetrics", "topic_name": null, "id": "4b87dc75-8847-4799-86a7-2c8d27d32c9c", "choice_type": "single"} {"question": "To diagnose prolonged latent phase in a nulliparous women, latent phase should be prolonged for", "exp": "ParityProlonged latent phaseNulliparous> 20 hoursMultiparous> 14 hoursReference: William's Obstetrics; 25th edition; Chapter 23; Abnormal Labor", "cop": 4, "opa": "> 6 hours", "opb": "> 10 hours", "opc": "> 14 hours", "opd": "> 20 hours", "subject_name": "Gynaecology & Obstetrics", "topic_name": "General obstetrics", "id": "ab7deb32-4778-481c-a819-edfb01886140", "choice_type": "single"} {"question": "Hormone therapy is not indicated in", "exp": "Ans. (b) CVDRef: Shaw's Textbook of Gynecology 16th Ed; Page No-74Hormone Replacement TherapyBenefits and risks* Estrogen therapy is the most effective FDA-approved method for relief of menopausal vasomotor symptoms (hot flashes), it is also used in genitourinary atrophy and dyspareunia.* Hot flashes:# Excess amount of sweating and sensation of heat is felt by 75% of menopausal women.# It is mediated through the hypothalamic thermoregulatory center.# Due to peripheral conversion of androgens to estrone in their peripheral adipose tissues; obese women are less commonly to undergo hot flashes.* The Womens Health Initiative (WHI) study of the National Institutes of Health (NIH) studied 27,000 postmenopausal women with a mean age of 63 years. These included women with a uterus on hormone therapy (HT), both estrogen and progestin, and hysterectomized women on estrogen therapy (ET) only.Critique of Women's Health Initiative StudyExcludes patients with vasomotor symptoms* Primary indication for hormone replacementMean patient age was 63 years* Missed the 10-year \"window of opportunity\"Same dose of hormone for all ages* Older women don't need as a high dose as do younger womenPatients were not all healthy* Hypertension (40%), | cholesterol (15%), diabetes mellitus (7%), myocardial infarction (3%)* Benefits: Both HT and ET groups in WHI had decreased osteoporotic fractures and lower rates of colorectal cancer.* Risks: Both HT and ET groups in WHI were found to have small increases in deep vein thrombosis (DVT). The HT group also had increased heart attacks and breast cancer, but these were not increased in the ET group.WHI-Benefit and Risk (Mean Age of 63 Years) Estrogen and progestinEstrogen onlyVaginal drynessBenefitBenefitHot flashesBenefitBenefitVasomotor symptomsBenefitBenefitOsteoporosisBenefitBenefitBreast cancerRiskNo changeHeart diseaseRiskNo changeStrokeRiskRiskContraindications* Personal history of an estrogen-sensitive cancer (breast or endometrium), active liver disease, active thrombosis, or unexplained vaginal bleedingModalities* The route of Estrogen can be administered by oral, transdermal, vaginal, or parenteral routes.* All routes will yield the benefits described. Estrogen is contraindicated in Women with uterus cancer.* All women with uterus cancer should be treated with progestin therapy to prevent endometrial hyperplasia.* The most common current regimen is oral estrogen and progestin given continuously.", "cop": 2, "opa": "Hot flashes", "opb": "CVD", "opc": "Osteoporosis", "opd": "Vaginal dryness", "subject_name": "Gynaecology & Obstetrics", "topic_name": "Pharmacotherapeutic in Obstetrics", "id": "c05ad5bf-68ac-41c1-b061-590987c04ac2", "choice_type": "single"} {"question": "Treatment of choice for adenomyosis is", "exp": "Treatment of adenomyosis: Diagnostic hysteroscopy combined with curretage is the initial management Total hysterectomy is the treatment in elderly Ref: Shaw Gynecology 17 e pg 186.", "cop": 4, "opa": "Estrogens only", "opb": "OCP", "opc": "Laser endoscopy", "opd": "Hysterectomy", "subject_name": "Gynaecology & Obstetrics", "topic_name": "Endometriosis and dysmenorrhea", "id": "a45b077f-2131-41a3-8a89-98e7cff2ce32", "choice_type": "single"} {"question": "A 28 yrs old lady, is suspected to have polycystic ovarian disease ,sample for testing LH and FSH are best taken on the following days of menstrual cycle", "exp": "In PCOD Determination of FSH and LH may help to confirm the diagnosis of poly cystic ovaries . These are assayed on the second or third day of cycle Lutenizing Hormone (LH) and Follicle Stimulating Hormone (FSH) LH and FSH are the hormones that encourage ovulation. Both LH and FSH are secreted by the pituitary gland in the brain. At the beginning of the cycle, LH and FSH levels usually range between about 5-20 mlU/ml. Most women have about equal amounts of LH and FSH during the early pa of their cycle. However, there is a LH surge in which the amount of LH increases to about 25-40 mlU/ml 24 hours before ovulation occurs. Once the egg is released by the ovary, the LH levels goes back down. While many women with PCOS still have LH and FSH still within the 5-20 mlU/ml range, their LH level is often two or three times that of the FSH level. For example, it is typical for women with PCOS to have an LH level of about 18 mlU/ml and a FSH level of about 6 mlU/ml (notice that both levels fall within the normal range of 5-20 mlU/ml). This situation is called an elevated LH to FSH ratio or a ratio of 3:1. This change in the LH to FSH ratio is enough to disrupt ovulation. While this used to be considered an impoant aspect in diagnosing PCOS, it is now considered less useful in diagnosing PCOS, but is still helpful when looking at the overall picture. Ref jeffcoate 6/e ,pg 205", "cop": 1, "opa": "4-Jan", "opb": "10-Aug", "opc": "13-15", "opd": "24-26", "subject_name": "Gynaecology & Obstetrics", "topic_name": "PCOD, hirsutism and galactorrhea", "id": "c29a2b38-eb19-44c9-850a-97d43712932a", "choice_type": "single"} {"question": "Hot flushes are experienced as a result of", "exp": "Hot flushes are caused by noradrenaline, which disturbs the thermoregulatory system. Oestrogen deficiency reduces hypothalamic endorphins, which release more norepinephrine and serotonin. This leads to inappropriate heat loss mechanism. Other causes that can be associated with the symptom of hot flushes include: thyroid disease, epilepsy, pheochromocytoma, carcinoid syndromes, autoimmune disorders, mast cell disorders, insulinoma, pancreatic tumours and even leukemias. REF : Shaw book of gynecology", "cop": 3, "opa": "Increased noradrenaline", "opb": "Decreased estrogen", "opc": "Increased noradrenaline and decreased estrogen", "opd": "Increased noradrenaline and estrogen", "subject_name": "Gynaecology & Obstetrics", "topic_name": "All India exam", "id": "285006de-c1e2-4c96-82a2-4b18a47099ab", "choice_type": "single"} {"question": "High risk of hypoplastic left heart syndrome occurs with maternal intake of", "exp": "Nitrofurantoin poses high risk of hypoplastic left heart syndrome in fetus.", "cop": 3, "opa": "Aminoglyocides", "opb": "Chloramphenical", "opc": "Nitrofurontoin", "opd": "Sulfonamides", "subject_name": "Gynaecology & Obstetrics", "topic_name": null, "id": "ac83893d-ca3f-4922-b018-77a3ed8ba888", "choice_type": "single"} {"question": "Nuchal translucency is measured between", "exp": "Nuchal translucency represents the maximum thickness of the subcutaneous translucent area between the skin and soft tissue overlying the fetal spine at the back of the neck. It is measured in the sagittal plane between 11 and 14 weeks using precise criteria. Increased NT is seen inChromosomal abnormalitiesGenetic SyndromesCardiac anomaliesSkeletal deformities(Ref: William's Obstetrics; 25th edition)", "cop": 2, "opa": "8 to 10 weeks", "opb": "11 to 14 weeks", "opc": "16 to 18weeks", "opd": "18 to 20weeks", "subject_name": "Gynaecology & Obstetrics", "topic_name": "All India exam", "id": "33315705-81bc-4159-a14f-fcaa70344689", "choice_type": "single"} {"question": "Shortest fetal skull diameter is", "exp": "Ans: A (Bimastoid) Ref: Dana textbook of obstetrics 5th ed pg 85Explanation: (See the following figure and table)", "cop": 1, "opa": "Bimastoid", "opb": "Submentovertical", "opc": "Submentobregmatic", "opd": "Occipito-fronta!", "subject_name": "Gynaecology & Obstetrics", "topic_name": "Fetal Skull and Maternal Pelvis", "id": "5bab4c84-c6db-46a9-bc47-825e33eb3816", "choice_type": "single"} {"question": "Fothergill's repair is also known as", "exp": "(Manchester operation): Ref: 327-S (305-S14th)OPERATION FOR GENITAL PROLAPSE* FothergilPs repair (Manchester operation). It is suitable for women under 40 years who are desirous of retaining their menstrual and reproductive functions* Le Fort's repair - reserved for very elderly menopausal patient with an advanced prolapse* Abdominal sling operations: designed for young women suffering from second or third degree uterovaginal prolapse(I) Abdominocervicopexy(II) Shirodkar's abdominal sling operations(III) Khann's abdominal sling operationsIn younger women desirous of retaining child bearing functions conservating surgical repair operations are indicated, where as in the perimeopausal and menopausal women, vagional hysterectomy with repair of the pelvic floor is the operation of choice.", "cop": 2, "opa": "Khann's sling operation", "opb": "Manchester operation", "opc": "Le Fort's repair", "opd": "Shirodkar's abdominal sling operation", "subject_name": "Gynaecology & Obstetrics", "topic_name": "Miscellaneous (Gynae)", "id": "ab5d3364-ca47-4ade-9f4d-5bdcd9ee55ef", "choice_type": "single"} {"question": "Adequate steroidogenesis in corpus luteum depends on serum levels of", "exp": "LH levels in serum, determine adequate steroidogenesis in corpus Luteum.", "cop": 3, "opa": "GnRH", "opb": "FSH", "opc": "LH", "opd": "Oestrogen", "subject_name": "Gynaecology & Obstetrics", "topic_name": null, "id": "8a90fd46-ccaf-4d3b-8a0c-3d670fed562f", "choice_type": "single"} {"question": "A pregnant woman presents with red degeneration of myoma. The management is", "exp": "Most fibroids do not increase in size during pregnancy. clinical symptoms and sonographic evidence of fibroid degeneration occurs in about 5% Very rarely does the presence of a fibroid during pregnancy lead to an unorable outcome Few complications of fibroids in pregnancy are: aboion preterm delivery placenta pre postpaum hemorrhage malpresentations increased operative interference Red degeneration develops most frequently in pregnancy The myoma becomes tense and tender and causes severe abdominal pain with constitutional upset and fever. The tumour itself assumes a peculiar purple red colour and develops a fishy odour. Although the patient is febrile with moderate leucocytosis and raised ESR, the condition is an aseptic one The treatment is done with conservative approach, using analgesics and bed rest", "cop": 2, "opa": "Myomectomy", "opb": "Conservative", "opc": "Hysterectomy", "opd": "Termination of pregnancy", "subject_name": "Gynaecology & Obstetrics", "topic_name": "Medical Illness Complicating Pregnancy", "id": "9c28626c-d575-4cfa-9596-f41050a839b3", "choice_type": "single"} {"question": "Percentage of tubal causes in female infertility is", "exp": "Tubal factor infertility is most commonly caused by pelvic inflammatory diseases, sexually transmitted diseases or other diseases such as endometriosis.\nThe American Society for Reproductive Medicine (ASRM) says that 25 to 35 percent of female infertility is due to tubal factors.", "cop": 3, "opa": "7%", "opb": "19%", "opc": "26%", "opd": "40%", "subject_name": "Gynaecology & Obstetrics", "topic_name": null, "id": "0d42a384-99ce-4550-a69a-5857b5c9d7bf", "choice_type": "single"} {"question": "Progesterone is produced by", "exp": "Granulosa luteal cells", "cop": 1, "opa": "Granulosa luteal cells", "opb": "Stroma ot the ovary", "opc": "Theca cells", "opd": "Seoli cells", "subject_name": "Gynaecology & Obstetrics", "topic_name": null, "id": "92fc81df-9c8a-40b3-bee7-4a054031678c", "choice_type": "single"} {"question": "Neural tube defects is prevented by", "exp": "(Folic acid) (343 - Dutta 6th)Pre pregnancy folic acid therapy (4mg daily) is given when there is any history of neural tube defects in previous birth. Therapy is started 1 month before conception and is continued in the first trimester", "cop": 1, "opa": "Folic acid", "opb": "Vitamin B12", "opc": "Vitamin B6", "opd": "Vitamin - C", "subject_name": "Gynaecology & Obstetrics", "topic_name": "Miscellaneous (Gynae)", "id": "2a1c756f-5698-4bce-9cbe-9b6c6fb6e9cb", "choice_type": "single"} {"question": "Puerperal sepsis can occur upto", "exp": "Infection of genital tract which occurs as a complication of delivery is termed puerperal sepsis Puerperium lasts until 6 weeks after delivery Ref: Dutta Obs 9e pg 406.", "cop": 4, "opa": "1 week", "opb": "2 weeks", "opc": "3 weeks", "opd": "6 weeks", "subject_name": "Gynaecology & Obstetrics", "topic_name": "General obstetrics", "id": "2cbc7158-78f8-4a18-adbf-85b2a5f51fd5", "choice_type": "single"} {"question": "Commonest cause of carcinoma endometrium is", "exp": "(Unopposed estrogen) (392-S) (374-SI4th)Predisposing factors for carcinoma endometrium* Unsupervised administration of HRT with oestrogens alone during the menopause predisposes these women to develop endometrial hyperplasia and cancer* Women with oestrogen dominance suffering from endometrial hyperplasia and presenting as cases of DUB are prone to endometrial cancer* In some families there is a strong familial predisposition to the disease. This may be due to genetic factors or dietic habits* Tamoxifen prescribed to women with breast cancer are prone to develop endometrial hyperplasia / cancer* OCP have a protective effect. Also the practice of adding progestogens to oestrogen for HRT during the last 12 to 14 days of the cycle is protective against development of endometrial cancer* Obesity, Hypertension, DM are associated with endometrial cancer in 30% cases* Infertile women, those with fewer children and subjects of PCOD are prone to the disease", "cop": 1, "opa": "Unopposed estrogen", "opb": "Multiple sex partner", "opc": "Early marriage", "opd": "Early menarchae", "subject_name": "Gynaecology & Obstetrics", "topic_name": "Miscellaneous (Gynae)", "id": "df4aea51-bf78-492b-9584-937a10f4f3fc", "choice_type": "single"} {"question": "Increased risk of endometrial and ovarian cancer is seen in", "exp": "HNPCC or Lynch II syndrome is caused by an inherited mutation in one of the following mismatch repair genes: hMSH2, hMLH1, PMS1, PMS2, or hMSH6. The disorder is characterized by early age (average age younger than 45 years) at onset of neoplastic lesions in a variety of tissues, including the colon, uterus, stomach, ureters, ovaries, and skin. The lifetime risk of endometrial cancer in women with Lynch II syndrome is 32% to 60% and the lifetime risk of ovarian cancer is 10% to 12%. Reference: Novak's gynecology; 14th edition; Chapter 35; Uterine cancer", "cop": 1, "opa": "Hereditary nonpolyposis colorectal cancer", "opb": "Peutz-jeghers syndrome", "opc": "Cowden's syndrome", "opd": "Ataxia telangiectasia", "subject_name": "Gynaecology & Obstetrics", "topic_name": "Gynaecological oncology", "id": "ea98544d-2b35-42ec-b86a-bf7b69733890", "choice_type": "single"} {"question": "Glands of littre are homologous to", "exp": "Glands of littre are the urethral glands of males Skene glands are the paraurethral glands Glands of littre are homologous to skene glands Ref: Internet sources,Shaw Gynecology 17 e pg 2.", "cop": 3, "opa": "Baholin gland", "opb": "Cowper's gland", "opc": "Skene glands", "opd": "Glands on labia", "subject_name": "Gynaecology & Obstetrics", "topic_name": "Anatomy of the female genital tract", "id": "0d5e031a-3e63-4901-b757-64bf3972ee4e", "choice_type": "single"} {"question": "LH surge occurs at", "exp": "C. i.e. (24 - 36 hours before ovulation) (37-Shaw's 14th/64-Dutta 4th)* LH surge precedes ovulation by 24 to 36 hours (mean 30 hours) and minimum of 75 ng/ml is required for ovulation (37 - Shaw's 14th)* LH levels remains almost static throughout the cycle except at least 12 hours prior to vulation. When it attains its peak, called LH surge (64 - Dutta 4th)* The main function of LH is steroidogenic, but along with FSH. It is responsible for full maturation of the graffian follicle and oocyte and ovulation", "cop": 3, "opa": "At the time of ovulation", "opb": "5-6 days before ovulation", "opc": "24-36 hours before ovulation", "opd": "24 - 72 hours after ovulation", "subject_name": "Gynaecology & Obstetrics", "topic_name": "Miscellaneous (Gynae)", "id": "74587f99-eca5-482f-9fe3-bc27ad330cdf", "choice_type": "single"} {"question": "Diagnostic criteria for ovarian pregnancy is", "exp": "Ectopic implantation of the feilized egg in the ovary is rare and is diagnosed if four clinical criteria are met. These were outlined by Spiegelberg (1878):The ipsilateral tube is intact and distinct from the ovary The ectopic pregnancy occupies the ovaryThe ectopic pregnancy is connected by the uteroovarian ligament to the uterusOvarian tissue can be demonstrated histologically amid the placental tissue. Risk factors are similar to those for tubal pregnancies, but A or IUD failure seems to be dispropoionately associated Classically, management for ovarian pregnancies has been surgical. Small lesions can be managed by ovarian wedge resection or cystectomy, whereas larger lesions require oophorectomy Reference: William's Obstetrics; 25th edition; Chapter 19; Ectopic pregnancy", "cop": 1, "opa": "Spiegelberg criteria", "opb": "Studiford criteria", "opc": "Rubin's criteria", "opd": "Timor-Tritsch criteria", "subject_name": "Gynaecology & Obstetrics", "topic_name": "General obstetrics", "id": "e5f90f45-f25a-4b1d-afb3-aff8f5542042", "choice_type": "single"} {"question": "Vaginal epithelium is derived from", "exp": "Vaginal epithelium is derived from the endoderm of the urogenital sinus Sinovaginal bulbs fuse to form vaginal plate which breaks down in centre to form lumen of vagina Peripheral cells of vaginal plate forms the vaginal epithelium Hymen is formed by invagination of the posterior wall of the urogenital sinus Ref: Essentials of embryology and bih defects; Ninth edition; chapter 13", "cop": 1, "opa": "Endoderm of urogential sinus", "opb": "Mesoderm of urogential sinus", "opc": "Endoderm of gential ridge", "opd": "Mesoderm of gential ridge", "subject_name": "Gynaecology & Obstetrics", "topic_name": "Congenital malformations", "id": "0bcbc48e-362c-4fb3-ae4f-b06b1486d9bf", "choice_type": "single"} {"question": "In Down's syndrome, the serum markers..........", "exp": "Triple test (combined test): MSAFP, Unconjugated estriol (E3) and hCG. Quadruple test: MSAFP, UE3, hCG, Inhibit-A Integrated test: NT, PAPP-A + hCG reference: D C Dutta's Textbook of obstetrics 7th edition page no 494", "cop": 2, "opa": "Increased MSAFP and UE3, Decreased PAPP-A and hCG, NT < 3mm", "opb": "Decreased MSAFP, PAPP-A and UE3 and Increased hCG, NT > 3mm", "opc": "Decreased MSAFP, PAPP-A , UE3 and hCG, NT > 3mm", "opd": "Increased MSAFP, UE3, PAPP-A and hCG, NT < 3mm", "subject_name": "Gynaecology & Obstetrics", "topic_name": "General obstetrics", "id": "e9976a37-3068-434b-8c14-dbd9b8f5d882", "choice_type": "single"} {"question": "Most sensitive diagnosis test for ectopic pregnancy", "exp": "Transvaginal ultrasound Diagnosis of Ectopic pregnancy Pregnancy is definitely diagnosed by demonstrating intrauterine sac. - A normal intrauterine sac appears regular and well defined on ultrasound. So ectopic pregnancy can be diagnosed by demonstrating the absence of the intrauterine sac. In a woman in whom ectopic pregnancy is suspected because of pain, bleeding and positive pregnancy test, performance of vaginal sonography is the logical first step. If the sonography demonstrates live intrauterine fetus then ectopic pregnancy is extremely unlikely. Alternatively if the uterus is empty, an ectopic pregnancy can be diagnosed based on the visualization of an adnexal mass separate from the ovaries. 3-hCG hormones also play an impoant role in the diagnosis of pregnancy. fi HCG is a hormone secreted during pregnancy. It is positive in viually 100% of ectopic pregnancies. However, a positive test only confirms pregnancy and does not indicate whether it is intrauterine or extrauterine. In normal pregnancy pHCG should double up every 2 days but in ectopic pregnancy the rate of increase of fl hCG is slow. 13 hCG litres and ultrasound complement one another in detecting ectopic pregnancy and have led to earlier detection. By correlating fl hCG titres with ultrasound .findings an ectopic pregnancy can often be differentiated.from intrauterine pregnancy. An intrauterine sac should be visible by transvaginal ultrasound when the p hCG is approximately 1000 ml p/m1 and by transabdominal ultrasound approximately 1 week later when the fi hCG is 1800-3600 ?nip/int. Thus when an empty uterine cavity is seen with a 13 hCG litre above this threshold, the patient is likely to have an ectopic pregnancy. An empty cavity is less of a concern when a fihCG below this threshold is obtained. Serum progesterone measuremen A single measurement of serum progesterone may sometimes clarify the diagnosis when ectopic pregnancy is suspected, but Its accuracy is crude and the customary thresholds are < 5 ng/m and > 25 'tend. i.e.a value exceeding 25 ng/m1 excludes ectopic pregnancy with 97.5% sensitivity, value below 5 ng/ml occur in only .3 percent of normal regnancies. So normal pregnancies can be excluded if the progesterone level is below 5%. Surgical diagnosis of ectopic pregnancy :- Laparoscopy Direct visualization of the ,fallopian tubes and pelvis by diagnostic laparoscopy offers a reliable diagnosis in most cases of suspected ectopic pregnancy and a ready transition to definitive operative therapy. At times, identification of an early unruptured tubal pregnancy may be difficult even if the tube is .fully visualized. Inspite of the low morbidity and quick recovery time, laparoscopy usually is performed when, on the basis of noninvasive test or curettage results, the diagnosis of ectopic pregnancy is fairly ceain and medical therapy is not planned. In these cases laporoscopy is used both for therapeutic and diagnostic purpose. Laparotomy Open abdominal surgery is preferred when the woman is hemodynamically unstable or when laparoscopy is not feasible. Laparotomy should not be delayed while laparoscopy is performed in a woman with obvious abdominal hemorrhage that required immediate definitive treatment. Culdocentesis Culdocentesis is the transvaginal passage of a needle into the posterior cul-de-sac in order to determine whether free blood is present in the abdomen. This procedure is useful in the diagnosis of intraperitoneal bleeding. This procedure will reveal nonclotting blood if intra-abdominal bleeding has occurred. If culdocentesis is positive, laparoscopy or laparotomy should be performed immediately. Indeed, some argue that the main purpose of culdocentesis is, not in diagnosis but to better prioritize patients so that those with positive culdocentensis are taken immediately to the operating room. More on imaging procedures used in the diagnosis of ectopic pregnancy Vaginal sonography There has been much improvement in the early diagnosis of ectopic pregnancy using vaginal sonography. Its use results in earlier and more specific diagnosis of uterine pregnancy than abdominal sonography and it has become the imaging method of choice in early pregnancy. Abdominal sonography:- Id entification of pregnancy products in the fallopian tube.", "cop": 1, "opa": "Transvaginal USG", "opb": "Culdocentesis", "opc": "MRI", "opd": "Serial monitoring of (-HCG", "subject_name": "Gynaecology & Obstetrics", "topic_name": null, "id": "0318cf9e-6caa-44bf-adc1-7ffffc839050", "choice_type": "single"} {"question": "First line Treatment of Abnormal Uterine bleeding is", "exp": "Managment of Abnormal uterine bleeding Medical management is the first line treatment, If medical therapy fails or is contraindicated, consider Mirena IUCD. If Mirena fails or side effects develop, go for ablative techniques. When the above methods fail, consider hysterectomy Versapoint- Bipolar electrosurgical system works in normal saline, is cheap, has excellent haemostasis NovaSure Impedance-controlled endometrial ablation is the latest and most safe procedure, taking just 90 sec. It uses bipolar radiofrequency and vaporizes endometrium upto myometrium.", "cop": 3, "opa": "Versapoint", "opb": "Novasure", "opc": "Medical mangement", "opd": "Surgical management", "subject_name": "Gynaecology & Obstetrics", "topic_name": "Normal Menstruation, Abnormal Menstruation, Menopausal Physiology and forsight of conception", "id": "2c381938-e107-4cd1-b685-eb6240a94212", "choice_type": "single"} {"question": "Depot medroxy progesterone acetate is", "exp": "DMPA is an injectable progestin, given i.m Being a progestin it doesn't increase risk of ovarian and endometrial cancer.", "cop": 3, "opa": "Subdermal progestin implant", "opb": "150 mg dose given i.v every 90days", "opc": "Known to dimnish bone mineral density", "opd": "Increases risk of endometrial cancer", "subject_name": "Gynaecology & Obstetrics", "topic_name": null, "id": "796e0e1c-32ae-4898-a9b0-ed00cf94dac1", "choice_type": "single"} {"question": "Criteria used for diagnosing Cervical pregnancy is", "exp": "Cervical ectopic pregnancyReference: Te Linde&;s Operative Gynecology, 10th Edition", "cop": 4, "opa": "Studiford", "opb": "Spigelberg", "opc": "Wrigly", "opd": "Rubin", "subject_name": "Gynaecology & Obstetrics", "topic_name": "General obstetrics", "id": "07c5924b-4b8b-4269-9a30-343f8fb06a91", "choice_type": "single"} {"question": "51 yr female with abdominal mass & ascites. On H/P ovarian Ca is +ve for..", "exp": "Ans. is 'a' i.e., Ca 125 \"Fertility preservation in early stage ovarian cancer the uterus and the contralateral ovary can be preserved in women with stage la, grade 1& 2 who desire to preserve fertility. The condition of the women should be monitored carefully with routine periodic pelvic examinations and determinations of serum CA 125 levels.\"", "cop": 1, "opa": "Ca 125", "opb": "Ca19-9", "opc": "AFP", "opd": "hCG", "subject_name": "Gynaecology & Obstetrics", "topic_name": "Tumor Markers", "id": "34fda8cb-9b9d-48d0-8014-388555f0bfb4", "choice_type": "single"} {"question": "Best management of Vault prolapse", "exp": "SACRAL COLPOPEXY Vault suspension Principle:To suspend the vaginal vault to the anterior longitudinal ligament of the 3rd sacral veebra Non absorbable suture material is used. Most common complication is stress urinary incontinence Ref: D.C.DUTTA&;S TEXTBOOK OF GYNAECOLOGY; 6th edition; Pg no:221", "cop": 1, "opa": "Sacral colpopexy", "opb": "Sacrospinous ligament fixation", "opc": "Le fos repair", "opd": "Anterior colporapphy", "subject_name": "Gynaecology & Obstetrics", "topic_name": "Urogynecology", "id": "5e7ffc41-05e1-47db-a300-dd92443d8cf5", "choice_type": "single"} {"question": "The desired level of HBA1C in diabetic pregnant women is", "exp": "Self-Monitored capillary blood glucose GoalsFasting<= 95mg/dlPremeal<= 100mg/dl1-hr postprandial<= 140mg/dl2-hr postprandial<= 120mg/dlMean100mg/dlHBA1C<= 6%(Ref: William's Obstetrics; 25th edition)", "cop": 3, "opa": "<= 5%", "opb": "<= 5.5 %", "opc": "<= 6%", "opd": "<= 7%", "subject_name": "Gynaecology & Obstetrics", "topic_name": "All India exam", "id": "d8d8b6c1-a6cf-4e84-b576-ad93a22a724b", "choice_type": "single"} {"question": "Postpaum tubectomy in a hea disease patient is done at", "exp": "Sterilization should be considered with the completion of the family at the end of first week in the puerperium under local anaesthesia throught abdominal route by minilap technique. If the hea is not well compensated the husband advised for vasectomy.", "cop": 2, "opa": "48 hours", "opb": "1 week", "opc": "2 weeks", "opd": "Immediately", "subject_name": "Gynaecology & Obstetrics", "topic_name": "Medical, surgical and gynaecological illness complicating pregnancy", "id": "2f1cce80-9fe2-4c5f-a5b4-d4a2f46f33cb", "choice_type": "single"} {"question": "Most hereditary ovarian cancers result from germline mutations in the", "exp": "Most hereditary ovarian cancers result from germline mutations in the BRCA1 and BRCA2 genes. The mutations are inherited in an autosomal dominant fashion, and therefore a full pedigree analysis (i.e., both maternal and paternal sides of the family history for both breast and ovarian cancer) must be carefully evaluated in all patients with epithelial ovarian cancer, and those withfallopian tube cancer and peritoneal cancer", "cop": 1, "opa": "BRCA1 and BRCA2", "opb": "MSH2", "opc": "PMS1", "opd": "MLH1", "subject_name": "Gynaecology & Obstetrics", "topic_name": "Gynaecological oncology", "id": "405c703b-4ae0-4f07-8da6-c6d919922a82", "choice_type": "single"} {"question": "The volume of amnionic fluid peaks at", "exp": "The volume of amnionic fluid at each week is variable. In general, the volume increases by 10 mL per week at 8 weeks and increases to 60 mL per week at 21 weeks, then peaks at 34 weeks(Ref: William's Obstetrics; 25th edition)", "cop": 3, "opa": "21 weeks", "opb": "28 weeks", "opc": "34 weeks", "opd": "36 weeks", "subject_name": "Gynaecology & Obstetrics", "topic_name": "All India exam", "id": "0a05a0d2-afef-4a2e-a54d-3ddc75b81381", "choice_type": "single"} {"question": "Earliest symptom of carcinoma cervix is", "exp": "Ans. is a i.e. Irregular vaginal bleeding \"In its very early stage, invasive carcinoma of cervix causes no symptoms and is discovered accidently or as a result of routine search. Symptoms come with surface ulceration and consist only of irregular uterine bleeding or discharge or both. These being pe-or postmenopausal in half the cases. The first episode of bleeding commonly follows coitus, straining at stool or trauma.\" \"For those with symptoms, however, early stage cervical cancer may create a watery, blod tinged vagina! discharge. Intermittent vaginal bleeding that follows coitus or douching may also be noted.\" Thus earliest symptom is irregular vaginal bleeding which usually follows coitus. So I am taking option 'a' i.e. irregular vaginal bleeding as the correct answer. Also Know : CIN is most commonly detected in women in their 20's. Peak incidence of carcinoma in situ : 25 - 35 yearsdeg Ca cervix has bimodal peak - first at 35-39 yearsdeg and second at 60-64 yearsdeg. Histology of Ca cervix : Squamous cell carcinoma (Epidermoid carcinoma) Accounts for 80% of carcinoma cervixdeg Arises from squamocolumnar junctiondeg Squamous cell carcinoma can be fuher classified as : Adenocarcinoma Accounts for 20% of carcinoma cervixdeg Arises from Endocervixdeg Recently increased in incidence because of use of OCP, Progesterone pills for long time. MC suply be of adenocarcinoma is mucinous endocervical adenocarcinoma. Discharge : -- It is at first creamy and later becomes diy brown in colour and is very offensive.deg -- The odour is caused by infection of necrotic tissue with saprophytes. Symptoms of Advanced Stage : Deep pelvic paindeg often unilateral and radiating to hip or thigh. Urinary incontinence, dysuria, increased urinary frequency, ureteric colic. Rectal pain Low backache Sciatica Oedama of legs Weight loss, anorexia, malaise. Adenocarcinoma Accounts for 20% of carcinoma cervixdeg Arises from Endocervixdeg Recently increased in incidence because of use of OCP, Progesterone pills for long time. MC suply be of adenocarcinoma is mucinous endocervical adenocarcinoma.", "cop": 1, "opa": "Irregular vaginal bleed", "opb": "Post coital bleed", "opc": "Foul smelling discharge", "opd": "Pain", "subject_name": "Gynaecology & Obstetrics", "topic_name": null, "id": "d8c81389-12b0-4288-b94b-4eafd3031e24", "choice_type": "single"} {"question": "The first line of treatment of infeility in PCOD is", "exp": "In pcos , ovulation is usually induced with combination of clomiphene citrate and hMG.The patient is advised to take CC 50-100mg/dl from day 2-6 of cycle for 5 days. Injecting hMG 75units IM is added on day 3,5 and 7 and more if required. Shaws Textbook of Gynaecology 16th edition page no 254.", "cop": 3, "opa": "Cabergoline", "opb": "GNRH", "opc": "Clomiphene citrate", "opd": "mifepristone", "subject_name": "Gynaecology & Obstetrics", "topic_name": "Infertility", "id": "aadcdb57-3ea6-48be-a1f4-ffc3a2984283", "choice_type": "single"} {"question": "Normal pregnancy can be continued in aEUR'", "exp": "Wolf Parkinson-While syndrome Marfan's syndrome Ideally, a patient with Marfan's syndrome contemplating pregnancy should have a preconceptional echocardiography to determine the diameter of the aoic root risk depends upon the diagmeter of aoic root. If it is greater than 4.0 cm, she is at significant risk for aoic dissection and she should be offered surgery. If the patient is in early pregnancy, she should be informed that termination of pregnancy is an option. Eisenmengher syndrome & Pulmonary hypeension Pulmonary vascular disease whether secondary to a reversed large left to right shunt such as VSD, Eisenmenger's syndrome or lung or connective tissue disease (e.g., scleroderma) or due to primary pulmonary hyeension is extremely dangerous in pregnancy and women known to have significant pulmonary vascular disease should be adviced from an early age to avoid pregnancy and be given appropriate contraceptive advice. Arias says \"Pregnancy is deleterious to patients with primary pulmonary hypeension. The maternal moality is approximately 40% and the fetal outcome is also poor with frequent spontaneous aboions and fetal demises secondary to maternal deaths.\" According to Dutta, Absolute indications for termination of pregnancy are:? a) Primary pulmonary hypeension b) Eisenmengers syndrome c) Pulmonary veno-occlusive disease Relative indications are a) Parous woman with grade HI and grade IV cardiac lesions. b) Grade I or II with previous history of cardiac failure in early month or in between pregnancy. The termination should be done within 12 weeks by suction evacuation (MVA) or by conventional D & E.", "cop": 2, "opa": "Primary pulmonary hypeension", "opb": "Wolf-Parkinson-White syndrome", "opc": "Eisenmenger syndrome", "opd": "Marfan syndrome with dilated aoic root", "subject_name": "Gynaecology & Obstetrics", "topic_name": null, "id": "bb3dcf10-7035-4856-ad8a-7ab2f82bed35", "choice_type": "single"} {"question": "karyopyknotic index is", "exp": "Karyopyknotic index is ratio of mature squamous cells over the intermediate and basal cells.", "cop": 1, "opa": "Ratio of mature squamous cells over the intermediate and basal cells.", "opb": "Ratio of dysplastic squamous cells over the normal squamous cells.", "opc": "Ratio of Koilocytes over normal squamous cells.", "opd": "Ratio of immature basal cells over total number of cells.", "subject_name": "Gynaecology & Obstetrics", "topic_name": null, "id": "b644fac1-4688-4543-a2d3-4f85e17a501e", "choice_type": "single"} {"question": "The Hymen is the membranous vestige of junction between", "exp": "Hymen is the Partition between sino vaginal bulbs and urogenital sinus.", "cop": 1, "opa": "Sinovaginal bulbs and urogenital sinus", "opb": "Cloaca and mullerian duct", "opc": "Mullerian duct and urogenital sinus", "opd": "Mullerian duct and sinovaginal bulbs", "subject_name": "Gynaecology & Obstetrics", "topic_name": null, "id": "574ade4c-e479-44d9-9424-4e87da1f75d0", "choice_type": "single"} {"question": "Most common cause of first trimester aboion is.", "exp": "Ans. is a i.e. Chromosomal abnormalities Aboion is spontaneous termination of pregnancy before the period of bility There are various definitions of aboion depending on the criteria for bility or survival of fetus. In the past, the criterion for bility was considered from 28 weeks onwards, but due to improvement in neonatal survival, this has been changed to 20th completed weeks\". Another commonly used definition is the \"-delivery of a fetus or any pa of product of conception with weight less than 500gms.'deg Incidence : About 15% of all conceptions end up in spontaneous aboions. Out of these 80% occur before 12 weeks i.e. in 1st trimester and among these 50-75% are due to chromosomal anomalies. Common causes of aboion : Petal factors Chromosomal abnormalities Hydropic degeneration of villi Multiple pregnancy Maternal factors Maternal infections like : TORCH infections, malaria,ureoplasma, chlamydia, brucella, spirochaetes Maternal medical disorders like : -- Hypeension -- Chronic renal disease -- Cyanotic hea disease -- Hemoglobinopathies Environmental factors like : -- Alcohol, caffeine. -- Exposure to radiation (> 5 rads) & anaesthetic gases Endocrine problems like : - Luteal phase defect (deficiency of progesterone) -- Thyroid abnormalities - hypothyroidism. - Poorly controlled diabetes mellitus -- PCOD Immunological causes : -- Antiphospholipid antibody syndrome -- Inherited thrombophilias Uterine factors like -- Cervical incompetence -- Mullerian anomalies - Large and multiple submucous leiomyoma -- Ashermann syndrome -- DES exposure in utero Others : Trauma; Subchorionic hematoma; Defective placentation Mnemonic to remember maternal causes of aboion--Mnemonic T = Trauma 12 = Infections/immunological causes M = Maternal medical diseases E' = Environmental factors/Endocrine problem D = Developmental/anatomical problems Mnemonic to remember maternal causes of aboion--Mnemonic T = Trauma 12 = Infections/immunological causes M = Maternal medical diseases E' = Environmental factors/Endocrine problem D = Developmental/anatomical problems", "cop": 1, "opa": "Chromosomal abnormalities", "opb": "Syphilis", "opc": "Rhesus isoimmunization", "opd": "Cervical incompetence", "subject_name": "Gynaecology & Obstetrics", "topic_name": null, "id": "c8ec5fc7-ee07-4cdd-97e7-482ca71c931c", "choice_type": "single"} {"question": "Test for ovarian reserve", "exp": "FSH Ovarian reserve is the capability of ovary to produce egg. The ovary is generally thought of as an egg bank from which the woman draws during the reproductive life. During the .flrst few days of each monthly cycle, under the influence of FSH and LH there is rapid growth of several follicles in the ovary. After a week or more of growth one of the follicles begin to outgrow. All the others and the remaining follicles involute (a process called atresia), and these follicles are said to become atretic. The process of atresia is impoant because it normally allows only one of the follicles to grow large enough, each month, to ovulate. This usually prevents more than one child from developing with each pregnancy. The single follicle reaches a diameter of 1-1.5 centimeters at the time of ovulation and is called the mature follicle. With advancing age the capability of the ovary to produce eggs declines. Methods of assessing ovarian reserve: With advancing age the capability of the ovary to produce egg declines i.e. ovarian reserve decrease with age. - Usually age is the best predictor of ovarian reserve but approximately 10% of women have an accelarated loss of ovarian reserve by their mid-30's. - So age alone does not tell the whole story. Consequently more refined methods .for predicting ovarian reserve have been developed. The methods for assessing ovarian reserve are classified into two groups:- a)Passive testing b)Dynamic testing The goal of both the approaches is to provide information regarding oocyte (egg) quality and quantity. Passive testing FSH level Early follicular phase FSH levels play an impoant role in pregnancy outcomes. As woman ages FSH becomes elevated in an attempt to force the aging ovary to respond. However, the exact mechanism responsible for this adaptive response remains unknown. A rise in early follicular phase FSH is also accompained by a decline in oocyte quality. High FSH has been correlated with poor ovarian reserve but it is difficult to establish absolute values that define how high an FSH level can be and still achieve pregnancy. Generally FSH level are expected to be below 10 lu/mlin women withreproductive potential - In one study it was .found out that 3 day FSH of less than 15 mlu/ml were twice as likely to conceive than women with FSH values between 15 and 24.9 mlu/ml. - Cycle day 3 is chosen because at this time the estrogen level is expected to be low, a critical feature as FSH levels are subject to a negative feedback. - Thus any determination of FSH needs to include corresponding estradiol level to indicate that the FSH level was drawn when estrogen level was low. Other methods for passive ovarian reserve testing Measurement of Inhibin B Inhibit B is an ovarian hormone that inhibits FSH release. Although present in ovulating women it is not normally .found in postmenopausal women. - Low inhibin levels suggests poor ovarian reserve. Transvaginal ultrasound Diminished ovarian reserve means that fewer follicles are available for stimulation and recruitment by _feility drugs. Transvaginal ultrasound allows the observation of ovaries and assess the number of follicles (ovarian volume). Dynamic ovarian reserve testing Clomiphene citrate challenge test ? Clomiphene is an antiestrogen which inhibits the negative feedback of estrogen on hypothalamus i.e. the hypothalamus gets false signals that the patient does nor have enough estrogen. In response, the hypothalamus induces the pituitary gland to produce more FSH and LH. This is turn initiates follicular growth to produce estrogen. Clomiphene is able to exe this effect only when the estrogen level is low or the ovarian reserve is low. In a normal woman with adequate ovarian reserve there is enough inhibin and estrogen to suppress FSH production through negative feedback and resist the effects clomiphene. - When undergoing clomiphene citrate challenge test, the .first step is to measure day 3 FSH and estrogen. Then 100 ing of clomiphene is administered on cycles day 5 through 9 and FSH and estrogen measurements are repeated on cycle day 10. In general, a high day 10 FSH suggests poor ovarian reserve.", "cop": 3, "opa": "LH", "opb": "LH/FSH ratio", "opc": "FSH", "opd": "Estradiol", "subject_name": "Gynaecology & Obstetrics", "topic_name": null, "id": "73b8c881-2b67-4fdb-a642-4377c4945d70", "choice_type": "single"} {"question": "Antibody to cross placenta", "exp": "Ans. (b) Ig GRef: Robbin's 9th ed. /198-199", "cop": 2, "opa": "IgA", "opb": "IgG", "opc": "IgE", "opd": "IgM", "subject_name": "Gynaecology & Obstetrics", "topic_name": "Endocrinology in Relation to Reproduction", "id": "fef82f77-74c8-4801-b733-48c973d2a85a", "choice_type": "single"} {"question": "Changes in the respiratory system in pregnancy;", "exp": "Anatomical changes:The lower ribs flare out,the subcoastal angles increases from 68 to 103 degree,transverse diameter of chest increases by 2 cm and the diaphragm rises about 4cm in pregnancy. Functional changes:Increase in ventilation in pregnancy is achieved by an increase in tidal volume by 40%.Minute ventilatory volume also increases.There is no change in vital capacity.The functional residual capacity and residual volume are decreased due to elevated diaphragm. TEXT BOOK OF OBSTETRICS,Sheila Balakrishnan,2nd edition,page no.80", "cop": 4, "opa": "Vital capacity is increased", "opb": "Subcostal angle remains unchanged", "opc": "Tidal volume remains unaltered", "opd": "Residual volume Is decreased", "subject_name": "Gynaecology & Obstetrics", "topic_name": "Medical, surgical and gynaecological illness complicating pregnancy", "id": "3bf214da-c0b2-477e-ac1b-394229c84f20", "choice_type": "single"} {"question": "serum prolactin level are highest", "exp": "proactin is more in NREM sleep in ovulation its low ref : dutta 9th ed", "cop": 1, "opa": "After 24 hour of paurition", "opb": "After 24 hour of ovulation", "opc": "After REM sleep", "opd": "After running for 1 hour", "subject_name": "Gynaecology & Obstetrics", "topic_name": "All India exam", "id": "ff9f5698-ee7c-43ab-b9b2-f4064f6cba32", "choice_type": "single"} {"question": "Bih is defined as", "exp": "Bih: The complete expulsion or extraction from the mother of a fetus after 20 weeks gestation. In the absence of accurate dating criteria, fetuses weighing < 500 g are usually not considered as bihs but rather are termed abouses for purposes of vital statistics(Ref: William's Obstetrics; 25th edition)", "cop": 1, "opa": "The complete expulsion or extraction from the mother of a fetus after 20 weeks gestation", "opb": "The complete expulsion or extraction from the mother of a fetus after 28 weeks gestation", "opc": "The complete expulsion or extraction from the mother of a fetus after 38 weeks gestation", "opd": "The complete expulsion or extraction from the mother of a fetus after 40 weeks gestation", "subject_name": "Gynaecology & Obstetrics", "topic_name": "All India exam", "id": "77d8e0ce-9e8e-4e5a-afc4-87543ac99909", "choice_type": "single"} {"question": "Most common complaints of ectopic pregnancy", "exp": "Ans. A. Pain* An ectopic pregnancy is one in which the fertilized ovum is implanted and develops outside the normal endometrial cavity.* Classic Triad of Ectopic Pregnancy* Pain (100%)* Amenorrhea (75%)* Bleeding per vaginam (70%)* NOTE: Triad is present only in 50% case of ectopic* Most common/Consistent finding of Ectopic Pregnancy PainSymptoms* Nausea and Vomiting* Lower Abdominal pain/generalized Abdomen pain* Shoulder tip pain (25%)* Syncopal attacks (10%)", "cop": 1, "opa": "Pain", "opb": "Vaginal bleeding", "opc": "Infection", "opd": "Amenorrhea", "subject_name": "Gynaecology & Obstetrics", "topic_name": "Ecotopic Pregnancy", "id": "e52a25a6-054c-40ea-8026-92bad5ae722e", "choice_type": "single"} {"question": "\"Schiller Dual\" bodies are seen in", "exp": "Ans. a (Endodermal sinus tumor) (Ref. Dutta Gynecology 4th/pg. 354)SCHILLER-DUVAL BODIES are characteristic structures resembling fetal glomeruli composed of a central blood vessel surrounded by embryonal cells lying within a space also lined by embryonal cells. They are seen in endodermal sinus tumors (yolk sac tumors). AFP is its tumor marker.", "cop": 1, "opa": "Endodermal sinus tumor", "opb": "Granulosa cell tumor", "opc": "Dysgerminoma", "opd": "Dermoid", "subject_name": "Gynaecology & Obstetrics", "topic_name": "Pathology (Carcinoma Ovary)", "id": "5b262f5f-5b1f-4daa-af56-8464489f8b32", "choice_type": "single"} {"question": "Rate of recurrence of breech presentation in the second pregnancy is", "exp": "Following one breech delivery, recurrence rate for second pregnancy to have breech is 10% and that for a subsequent third pregnancy is 27%.", "cop": 2, "opa": "5%", "opb": "10%", "opc": "15%", "opd": "20%", "subject_name": "Gynaecology & Obstetrics", "topic_name": null, "id": "ebec7819-2b16-472e-beb4-9ab4622e484a", "choice_type": "single"} {"question": "a and 13 subunits are not seen in", "exp": "Ans. is c i.e. Prolactin HCG (Human chorionic gonadotropic hormone) has alpha and beta subunits. its alpha subunit is similar to that of LH (leutinizing hormone), FSH (Follicular stimulating hormone) and TSH (Thyroid stimulating hormone) whereas beta is specific. We have also studied that insulin hormone has alpha and beta subunits. Remember : Hormones with alpha and beta subunits HOGdeg * LHdeg FSHdeg * TSHdeg Insulindeg", "cop": 3, "opa": "FSH", "opb": "HCG", "opc": "Prolactin", "opd": "Insulin", "subject_name": "Gynaecology & Obstetrics", "topic_name": null, "id": "c93f621d-4cb3-49e8-b832-3d8823f9fe1b", "choice_type": "single"} {"question": "Currently, the threshold of bility lies between", "exp": "Threshold of ViabilityBihs once considered to be \"abouses\" because the fetus weighed <500 g are now classified as live bihs. For those newborns delivered before 33 weeks' gestation, perinatal and neonatal care has advanced tremendously. As a result, the threshold of bility, which is the lower limit of fetal maturation compatible with extrauterine survival, has been reassessed. Currently, the threshold of bility lies between 20 and 26 weeks' gestationReference: William's Obstetrics; 25th edition; Chapter 42; Preterm bih", "cop": 1, "opa": "20 and 26 weeks", "opb": "24 and 28 weeks", "opc": "26 and 30 weeks", "opd": "28 and 30 weeks", "subject_name": "Gynaecology & Obstetrics", "topic_name": "General obstetrics", "id": "5f1cbdb6-e8d4-44e2-9937-34e85bf064e7", "choice_type": "single"} {"question": "Snowstorm appearance in sonography is characteristic of", "exp": "Sonographically, a complete mole appears as an echogenic uterine mass with numerous anechoic cystic spaces but without a fetus or amniotic sac. The appearance is often described as a \"snowstorm\"Paial mole has features that include a thickened, multicystic placenta along with a fetus or at least fetal tissueReference: William's Obstetrics; 25th edition, chapter 20; Gestational trophoblastic diseases", "cop": 1, "opa": "Complete molar pregnancy", "opb": "Ectopic pregnancy", "opc": "Incomplete aboion", "opd": "Missed aboion", "subject_name": "Gynaecology & Obstetrics", "topic_name": "General obstetrics", "id": "e1e895fb-9cc0-4feb-ad47-7c1eddde6802", "choice_type": "single"} {"question": "During active labor cervIcal dilatation per hour in a primigravida is", "exp": "In primigravida dilatation of cervix at the rate of 1cm/hr and in multigravida -1.5cm/hr.D.C.DUTTA&;S TEXTBOOK OF OBSTETRICS,Pg no:130,7th edition", "cop": 1, "opa": "1.0 cm", "opb": "1.5 cm", "opc": "1-7 cm", "opd": "2 cm", "subject_name": "Gynaecology & Obstetrics", "topic_name": "General obstetrics", "id": "85d47ef1-626a-4ec7-92b8-1d3d4ef0915b", "choice_type": "single"} {"question": "Drug that is used for fetal lung maturity is", "exp": "* Betamethasone and dexamethasone are coicosteroids, also called glucocoicoids, that are given before bih (antenatally) to speed up a preterm fetuses lung development. * Either is used when a mother is in preterm labor and bih may occur in 24 to 48 hours REF : DUTTA BOOK OF OBESTETRICS", "cop": 1, "opa": "Dexamethasone", "opb": "Folic acid", "opc": "eclomethasone", "opd": "one", "subject_name": "Gynaecology & Obstetrics", "topic_name": "All India exam", "id": "0a7c149e-a41b-475e-b24e-87e434dacdcf", "choice_type": "single"} {"question": "Laparoscopy is contraindicated in", "exp": "Contraindications of Laparoscopy Severe cardiopulmonary disease Patient hemodynamically unstable Generalised peritonitis Significant hemoperitoneum Extensive peritoneal adhesions Large pelvic tumor Pregnancy >16 weeks Advanced malignancy On anticoagulation therapy D.C.DUTTA&;S TEXTBOOK OF GYNECOLOGY,Pg no:615,6th edition", "cop": 4, "opa": "Ectopic pregnancy", "opb": "PID", "opc": "Endometriosis", "opd": "Peritonitis", "subject_name": "Gynaecology & Obstetrics", "topic_name": "Gynaecological diagnosis and operative surgery", "id": "d9e21382-97d9-4c16-86bf-0272d4a5f10b", "choice_type": "single"} {"question": "Paramesonephric ducts develop at", "exp": "Paramesonephric ducts develop from invagination of intermediate mesoderm at 6th week of gestation.", "cop": 2, "opa": "4 weeks of gestation", "opb": "6 weeks of gestation", "opc": "8 weeks of gestation", "opd": "10 weeks of gestation", "subject_name": "Gynaecology & Obstetrics", "topic_name": null, "id": "38f5bfba-45af-4ef3-b41f-0fc0fff7fd81", "choice_type": "single"} {"question": "Incomplete Androgen insensitivity syndrome is", "exp": "Incomplete AIS =\n\nLess severe than complete AIS.\nAs testosterone gets converted to estrogen, breast development is proper.\nKaryotype is 46 XY.\nClitoromegaly is the characterisitic feature.", "cop": 2, "opa": "More severe than complete androgen insensitivity.", "opb": "Clitoromegaly and partial fusion of Labia.", "opc": "Breast development is absent.", "opd": "Karyotype is 46XX.", "subject_name": "Gynaecology & Obstetrics", "topic_name": null, "id": "42d8eefc-25d6-4732-868e-f069dee7db6e", "choice_type": "single"} {"question": "Most common site for metastasis in choriocarcinoma", "exp": null, "cop": 1, "opa": "Lungs", "opb": "Brain", "opc": "Liver", "opd": "Spine", "subject_name": "Gynaecology & Obstetrics", "topic_name": null, "id": "cf758358-bca8-491e-ad62-8ec786359fb5", "choice_type": "single"} {"question": "Ideal contraceptive for newly married couple is", "exp": "Combined oral contraceptive pill [Ref. Dutta 7/e p. 5591 Best contraceptive for newly married couple is combined oral contraceptive pill (COC). -Combined oral contraceptive has lesser failure rate than progestin only pill. - IUCD's should not be used in newly married couple. Also know Best contraceptive for parous young women --) IUCD (considered the best method for spacing child bih).", "cop": 2, "opa": "Barrier method", "opb": "Combined OCP", "opc": "IUCD", "opd": "Progesterone only pill", "subject_name": "Gynaecology & Obstetrics", "topic_name": null, "id": "2fc8433c-c4e8-44cb-8271-d0a4ed81c535", "choice_type": "single"} {"question": "Type IIIB Endometrial cancer (FIGO) is", "exp": null, "cop": 2, "opa": "Endometrial cancer invading cervical stroma", "opb": "Endometrial cancer with vaginal involvment", "opc": "Endometrial cancer involving uterine adnexa", "opd": "Endometrial cancer invading Pelvic lymph nodes", "subject_name": "Gynaecology & Obstetrics", "topic_name": null, "id": "81c90529-2161-4cdf-84ef-ff43d8e0e7b6", "choice_type": "single"} {"question": "Mrs. A, 30years with the previous history of preterm delivery at 24weeks and baby is stillborn, now presently at 30 weeks of gestation, her obstetric score is", "exp": "Nullipara: a woman who has never completed a pregnancy beyond 20 weeks' gestation. She may not have been pregnant or may have had a spontaneous or elective aboion(s) or an ectopic pregnancy.Primipara: a woman who has been delivered only once of a fetus or fetuses born alive or dead with an estimated length of gestation of 20 or more weeks.Multipara: a woman who has completed two or more pregnancies to 20 weeks' gestation or more. Parity is determined by the number of pregnancies delivered reaching 20 weeks. It is not increased to a higher number if multiples are delivered in a given pregnancy. Moreover, stillbih does not lower this number.Reference: William's obstetrics; 25th edition", "cop": 3, "opa": "G1Ab1", "opb": "G2Ab1", "opc": "G2P1L0", "opd": "G2P2L0", "subject_name": "Gynaecology & Obstetrics", "topic_name": "General obstetrics", "id": "9885806d-bd8f-474e-84d3-9a5939e6acbc", "choice_type": "single"} {"question": "Varicoceles appear to cause male infertility by", "exp": "The incidence of varicoceles in the general population is about 15%, but 40% of males with infertility are found to have varicoceles. Because of the anatomy and physiology, varicoceles are more likely to occur on the left side. There is no correlation between the size of the varix and prognosis for fertility. The characteristic stress pattern seen with varicoceles is decreased number of sperm, decreased motility, and increased abnormal forms. How the varicocele causes abnormal semen quality, and the relationship between varicocele, semen abnormalities, and male infertility (especially when semen quality appears normal) is unclear.", "cop": 3, "opa": "Interfering with sperm production", "opb": "Blocking epididymal sperm motility activation", "opc": "Increasing the likelihood of sperm antibody formation", "opd": "Interfering with sperm movement through cervical mucus", "subject_name": "Gynaecology & Obstetrics", "topic_name": "Physiology & Histology", "id": "a21b4bab-c9bd-4fff-a786-8fabec5dd6e9", "choice_type": "single"} {"question": "Vaginal delivery is contraindicated in", "exp": "Vaginal delivery is contraindicated in Classical C.S because there is more chance of rupture of the uterus. 8-10% risk of uterine rupture is there in next pregnancy. This is because this poion of the uterus is taking pa in active contractions and will predispose to uterine rupture.", "cop": 1, "opa": "Previous classical CS", "opb": "Preterm labour", "opc": "Previous LSCS", "opd": "Face presentation", "subject_name": "Gynaecology & Obstetrics", "topic_name": "Labour - III", "id": "6e303368-f59b-413d-88c7-d2faf3c1ba10", "choice_type": "single"} {"question": "Superfecundation refers to", "exp": "Mechanism of monozygotic twinning:The outcome of the monozygotic twinning process depends on when division occurs. If zygotes divide within the first 72 hours after feilization, two embryos, two amnions, and two chorions develop, and a diamniotic, dichorionic twin pregnancy evolves.If division occurs between the fouh and eighth day, a diamniotic, monochorionic twin pregnancy results. By approximately 8 days after feilization, the chorion and the amnion have already differentiated, and division results in two embryos within a common amniotic sac, that is, a monoanionic, monochorionic twin pregnancy. Conjoined twins result if twinning is initiated later.(Ref: William's Obstetrics; 25th edition)", "cop": 2, "opa": "Feilization of one ovum with two spermatozoa", "opb": "Feilization of two ova in the same menstrual cycle", "opc": "Feilization of two ova in different menstrual cycles", "opd": "Feilization of three ova in the same menstrual cycle", "subject_name": "Gynaecology & Obstetrics", "topic_name": "All India exam", "id": "1b0d68a2-1755-4026-8e05-a1d0a05f3507", "choice_type": "single"} {"question": "Total amount of iron needed by the fetus during entire pregnancy", "exp": "Iron needs in pregnancy Fetus and placenta 300mg Red cell expansion 500mg Net need in pregnancy 900mg Reference: Textbook of Obstetrics; Sheila Balakrishnan; 2nd edition; Page no: 285", "cop": 4, "opa": "500 mg", "opb": "1000 mg", "opc": "800 mg", "opd": "300 mg", "subject_name": "Gynaecology & Obstetrics", "topic_name": "Medical, surgical and gynaecological illness complicating pregnancy", "id": "a160dfe2-deab-4af6-9538-c555e8d653fb", "choice_type": "single"} {"question": "A 20yr old P1L1 with RhD negative pregnancy had a spontaneous vaginal delivery. Her baby is RhD positive. Anti D Ig should be administered within", "exp": "* For successful immunoprohylaxis anti D Ig should be given as soon as possible after the sensitizing event but always within 72hours * If not given within 72hours, effo should still be made to give anti D Ig, as a dose given within 9 to 10 dose will offer some protection * Women who is already sensitized should not be given anti D Ig Reference: Greentop guidelines no 22", "cop": 1, "opa": "72 hours", "opb": "7 days", "opc": "10 days", "opd": "30 days", "subject_name": "Gynaecology & Obstetrics", "topic_name": "Medical, surgical and gynaecological illness complicating pregnancy", "id": "040f7443-0678-47ca-9512-3521b908c9af", "choice_type": "single"} {"question": "Length of Umbilical cord is", "exp": "Umbilical cord - Normal length- 40-70 cm long- Cord has three vessels : 1 vein(left) and 2 aeries - Length is influenced by amniotic fluid volume and fetal mobility.", "cop": 3, "opa": "10-20 cm", "opb": "20-40 cm", "opc": "40-70 cm", "opd": "80-100 cm", "subject_name": "Gynaecology & Obstetrics", "topic_name": "Labour - III", "id": "bf73c025-3970-4532-9b76-6aee7828abb0", "choice_type": "single"} {"question": "IUCD having the longest life span is", "exp": "The time periods for replacement for various IUDs are: - Copper T 200:3years- Multiload Cu 250:3 years - Multiload Cu 375: 5 years- LNG-IUS/Mirena : 5 years- Cu T 380A : 10 years ( also known as Paraguard)- Nova : 3years - Progestase : 1 years", "cop": 2, "opa": "Progestase", "opb": "Cu T 380A", "opc": "Mirena", "opd": "Nova T", "subject_name": "Gynaecology & Obstetrics", "topic_name": "Contraceptives", "id": "22b85eb5-bdab-463e-999b-51e8803254bf", "choice_type": "single"} {"question": "Induction at term is not done in", "exp": "Preeclampsia is associated with significant maternal and fetal morbidity and moality. No pregnancy complicated by preeclampsia should be allowed to cross 38 weeks and earlier terminations will often have to be considered depending on maternal and fetal status. Refer page no 559 of Textbook of obstetrics, Sheila Balakrishnan, 2 nd edition.", "cop": 1, "opa": "Preeclampsia", "opb": "Diabetes", "opc": "Hea disease", "opd": "Renal disease", "subject_name": "Gynaecology & Obstetrics", "topic_name": "General obstetrics", "id": "8845d5fb-1445-492d-877d-68f59ec7c0ab", "choice_type": "single"} {"question": "Drug of choice for maternal toxoplasmosis", "exp": "Ref: Text book of Obstetrics by D.C.Dutta. 7th ed. Pg. 297.Explanation:Treatment of choice for toxoplasmosis during pregnancy:Spiramycin 3 g/dayPyrimethamine 25mg + sulphadiazine 1 gQlD for 4-6 weeksPyrimethamine is not given in 1' trimesterMode of infection : Eating raw, Uncooked meat/ through cat feces/ Through placentaEffects on baby:HydrocephalusChorioretinitisCerebral calcificationsMicrocephalyMental retardation", "cop": 1, "opa": "Spiramycin", "opb": "Pyrimethamine", "opc": "Doxy + sulfadiazine", "opd": "Trimethorpim sulphamethoxazole", "subject_name": "Gynaecology & Obstetrics", "topic_name": "Infectious Diseases", "id": "fc2ca07f-e3f5-4911-b280-7f85f4ed95ac", "choice_type": "single"} {"question": "Bonney&;s test demonstrates", "exp": "MARSHALL AND BONNEY&;S TEST To demonstrate stress incontinence Procedure: Two fingers are placed in the vagina at the urethrovesical junction, on either side of urethra and the bladder neck region is elevated, on straining or coughing absence of leakage of urine indicates a positive test Positive stress test is indicative of beneficial outcome following surgical repair Ref: SHAW&;S TEXTBOOK OF GYNAECOLOGY; 15th edition; Pg no:189", "cop": 1, "opa": "Stress urinary incontinence", "opb": "Urge incontinence", "opc": "Overflow", "opd": "Prolalpse uterus", "subject_name": "Gynaecology & Obstetrics", "topic_name": "Urogynecology", "id": "26b386c0-526c-450e-8dd3-3382c5e5125b", "choice_type": "single"} {"question": "Condyloma acuminata in pregnancy is treated by", "exp": "Chemical cauterization is usually done with liquified phenol or concentrated trichloroacetic acid", "cop": 3, "opa": "Podophyllin", "opb": "Podophyllin toxin", "opc": "Trichloroacetic acid", "opd": "5-FU cream", "subject_name": "Gynaecology & Obstetrics", "topic_name": "All India exam", "id": "e7759f65-7c04-44eb-841f-2b45d40abec0", "choice_type": "single"} {"question": "The normal range of AFI is", "exp": "AFI : It is the sum of the single deepest pocket from four quadrants of the uterus. Normal range: 5 to 24cm(Ref: William's Obstetrics; 25th edition)", "cop": 4, "opa": "2 to 8cm", "opb": "5 to 8cm", "opc": "2 to 24cm", "opd": "5 to 24cm", "subject_name": "Gynaecology & Obstetrics", "topic_name": "All India exam", "id": "b4fee28f-d40c-4918-8d00-ef5a6e55402d", "choice_type": "single"} {"question": "Treatment of choice in a postmenopausal lady with atypical endometrial hyperplasia is ;", "exp": "Hysterectomy", "cop": 2, "opa": "Estrogens", "opb": "Hysterectomy", "opc": "Progestogens", "opd": "Radiotherapy", "subject_name": "Gynaecology & Obstetrics", "topic_name": null, "id": "bda9fc2c-5048-4457-8da6-25ff247f3785", "choice_type": "single"} {"question": "Test used to differentiate between maternal and fetal blood in a given sample", "exp": "Ans. c. Apt test Both Apt test and Kleihauer-Betke test can be used to detect the presence of fetal blood within a sample but test used to differentiate between maternal and fetal blood in a given sample is Apt test. Apt Test: Used to detect the presence or absence of fetal blood (qualitative) in a vaginal discharge to rule out vasa pre late in pregnancy or to detect the origin of a neonatal bloody vomiting, whether it is a genuine upper GI hemorrhage/ hemoptysis or simply swallowed maternal blood during delivery or from cracked nipple. Kleihauer-Betke Test: The sample is maternal peripheral smear and is used to see how much of fetal blood (quantitative) has been transfused into the maternal serum in order to assess the risk of isoimmunization and then the risk of hemolytic disease of newborn. Both of them relies on the fact that HbF is resistant to alkali (Apt) and acids (Kleihauer-Betke) and so the HbA containing RBCs (Maternal) will be hemolyzed but not the fetal RBCs as they contain the HbF. When fetal blood needs to be differentiated from maternal blood Apt test is usedQ (Qualitative estimation) When the amount of fetal bleeding needs to be estimated Kleihauer-Betke test is usedQ (Quantitative estimation) Apt test Kleihauer-Betke test Source of sample Maternal or neonatal MaternalQ Principle Adding 1% NaOH destroys adult HbA but not fetal HbF Adding acid destroys adult HbA but not fetal Assessment Type QualitativeQ QuantitativeQ Result Positive means blood is of fetal origindeg Repoed in estimated milliliters of fetal", "cop": 3, "opa": "Kleihauer-Betke test", "opb": "Osmotic fragility test", "opc": "Apt test", "opd": "Bubbling test", "subject_name": "Gynaecology & Obstetrics", "topic_name": null, "id": "56f47af5-5a30-4625-9860-0d660b2a7da2", "choice_type": "single"} {"question": "For \"Postcoital douche method\" of contraception, select the most appropriate rate of use effectiveness (failure rate or percentage of pregnancies per year of actual patient use). (SELECT 1 RATE)", "exp": "There are two methods of describing the effectiveness of contraceptive agents: the theoretical or method effectiveness rate and the actual use effectiveness rate. When comparing different methods, it is important to use comparable figures. The effectiveness of the rhythm method is influenced by the woman's ability to predict the time of ovulation from the regularity of her menses, and by her motivation to successfully abstain from intercourse during the 10 days around suspected ovulation. The menstrual and ovulatory irregularities and lapses in the woman's motivation account for a pregnancy rate of 40% with the rhythm method. In contrast to the rhythm method, the IUD requires little or no action on the part of the woman. For this reason the device's actual use effectiveness approaches its maximal theoretical effectiveness, with a pregnancy rate of 3 to 10%. Unrecognized expulsion or misplaced insertion of the IUD are responsible for most failures. The vaginal diaphragm and the condom are barrier contraceptives in that for each act of sexual intercourse they pose a barrier between the sperm ejaculate and the endocervical canal. In theory, both can be very effective. However, both require recurrent motivation for application with each act of intercourse. Lapses in motivation are not uncommon, and there is a pregnancy rate of 15 to 25% for each of these two methods. The condom used with a spermicidal agent is very effective, more so than either used alone. The pregnancy rate with postcoital douching is almost the same as that for unprotected intercourse (80%). This lack of effectiveness is readily explained by the extremely rapid progression of motile sperm into the endocervical canal. Within several minutes of coitus, sperm have ascended the female reproductive tract and can be found within the endocervical mucus, uterus, and fallopian tubes. Coupled with the failure of a vaginal douche to reach the endocervix, this method is essentially useless. Combined oral contraceptive birth control pills are clearly the most effective reversible contraceptive currently available. With correct use, many studies report a contraceptive effectiveness that approaches 100% (pregnancy rate less than 0.1%). This extreme effectiveness is best explained by the pill's multiplicity of actions, i.e., suppression of ovulation, hostility of cervical mucus to sperm penetration, and hostility of atrophic endometrium to the implantation of a conceptus. Failure to take the pills regularly is responsible for most failures, and in practice pregnancy rates of at least 5% are common.", "cop": 1, "opa": "80%", "opb": "40%", "opc": "15 to 25%", "opd": "5 to 15%", "subject_name": "Gynaecology & Obstetrics", "topic_name": "Contraceptives", "id": "819ea3ee-14dc-42a1-b497-c16361977366", "choice_type": "single"} {"question": "Antimullerian hormone levels, during menopausal transition", "exp": "Anti mullerian hormone levels decrease markedly and progressively across menopausal transition.", "cop": 2, "opa": "Remain constatn", "opb": "Decrease", "opc": "Increase", "opd": "Varies", "subject_name": "Gynaecology & Obstetrics", "topic_name": null, "id": "c1d186dc-df1b-469a-9b59-b11a07ae94e2", "choice_type": "single"} {"question": "Most commonly uterine fibroid presents as heavy menstrual bleeding. Most likely cause is", "exp": "Dysregulation of local vasoactive growth factors is known to cause dilatation of endometrial venules which causes heavy menstrual bleeding.", "cop": 2, "opa": "Hormonal imbalance", "opb": "Dilated endometrial venules", "opc": "Ruptured fibroids", "opd": "Torsion of fibroids", "subject_name": "Gynaecology & Obstetrics", "topic_name": null, "id": "192e740b-2620-4e44-b8d6-d0d4b451e7f3", "choice_type": "single"} {"question": "Moschcowitz repair is done for", "exp": "In Moschcowitz repair, several purse string sutures staing from below are used to obliterate the cul-de-sac of the pouch of Douglas. Care should be taken not to include the ureter in the stitch. Ref: Shaw's Textbook of Gynaecology,18th edition, p342.", "cop": 2, "opa": "Vault prolapse", "opb": "Enterocoele", "opc": "Chronic inversion of uterus", "opd": "Adenomyosis", "subject_name": "Gynaecology & Obstetrics", "topic_name": "All India exam", "id": "18c14d87-6efb-4ecc-9ac7-fe1db2f0a41b", "choice_type": "single"} {"question": "NOT included in third stage of labour", "exp": "(C) (IV - Methergin) (142 - Dutta 7th)Methergin 0.2 mg IM (Intramuscular) to the mother within one minute of delivery of the babyOyxtocin may be given with crowning of the head, with delivery of the anterior shoulder of the baby or after the delivery of the placenta.Components of Active Management of Third stage of Labour (WHO)* Administration of uterotonic (oxytocin/Ergometrine) soon after birth of baby* Delayed cord clamping and cutting* Controlled Cord traction for delivery of placenta* Uterine massageCurrent evidence show that delayed cord clamping is beneficial for baby, immediate cord clamping has been shown to increase the incidence of iron deficiency anemia for premature and LBW babies immediate cord clamping can also increase the risk of intraventricular haemorrhage and late onset sepsis", "cop": 3, "opa": "Controlled cord traction", "opb": "lM-oxytocin", "opc": "IV methergin", "opd": "Uterine massage", "subject_name": "Gynaecology & Obstetrics", "topic_name": "Miscellaneous (Gynae)", "id": "8a26ae07-2a40-4081-95a7-e0b7a0f776bf", "choice_type": "single"} {"question": "Halban's disease is due to", "exp": "Halban's disease: Rare, self-limiting process. Also called Irregular shedding endometrium Due to persistent corpus luteum due to incomplete withdrawal of LH even on 26 in day of cycle. The corpus luteum continues to secret progesterones Menstruation comes on time, is prolonged but not heavy. Slight bleeding continues intermittently for several days after proper flow. On D& C done on 5-6th of cycle - endometrial tissue shows presence of proliferative endometrium along with secretory endometrium", "cop": 1, "opa": "Persistent corpus luteum", "opb": "Deficient corpus luteum", "opc": "Persistent trophoblast", "opd": "Deficient trophoblast", "subject_name": "Gynaecology & Obstetrics", "topic_name": "Normal Menstruation, Abnormal Menstruation, Menopausal Physiology and forsight of conception", "id": "51e1f516-73e7-49b5-9f73-003d93ce3699", "choice_type": "single"} {"question": "In twin pregnancy, Vaginal delivery is contraindicated in", "exp": "Monoamniotic twins are at a great risk of developing cord accidents like entanglement which can lead to sudden fetal death. Therefore all monoamniotic twins have to be delivered by a Cesarean sections between 32-34 weeks , after ensuring lung maturation", "cop": 1, "opa": "Monochorionic monoamniotic", "opb": "First twin veex, 2nd breech", "opc": "Second twin transverse", "opd": "First twin Mentoanterior", "subject_name": "Gynaecology & Obstetrics", "topic_name": "Twin Pregnancy, Molar Pregnancy, Gestational Trophoblastic disease and contraception in special situations (Sour Grapes!)", "id": "0216aaf7-bd27-47de-9682-247a1c3c9da7", "choice_type": "single"} {"question": "In Peripaum cardiomyopathy, cardiac failure can develop anytime during", "exp": "Diagnostic criteria for peripaum cardiomyopathy: Development of cardiac failure in the last month of pregnancy or within 5 months after deliveryThe absence of an identifiable cause for the cardiac failure, The absence of recognizable hea disease prior to the last month of pregnancy, and Left ventricular systolic dysfunction demonstrated by classic echocardiographic criteria, such as depressed ejection fraction or fractional shoening along with a dilated left ventricle (Ref: William's Obstetrics; 25th edition)", "cop": 4, "opa": "Anytime during antenatal period", "opb": "Only during third trimester", "opc": "Only during 6 weeks postpaum", "opd": "Last month of pregnancy or within 5 months after delivery", "subject_name": "Gynaecology & Obstetrics", "topic_name": "All India exam", "id": "64c4a70e-2841-4bbe-b2ad-a8f6a71fe2aa", "choice_type": "single"} {"question": "Vertical transmission of HIV is most likely to occur", "exp": "Most of the vertical transmission of HIV is likely to occur during labor.", "cop": 3, "opa": "Early antepartum", "opb": "Late antepartum", "opc": "Intrapartum", "opd": "Postpartum", "subject_name": "Gynaecology & Obstetrics", "topic_name": null, "id": "6596c784-7ce7-4d29-9cc6-055d98b52f78", "choice_type": "single"} {"question": "Labetalol is an", "exp": "Labetalol is a commonly used a 1 and non-selective b-adrenergic blockerDose: 200-2400mg/day, orally in two to three divided dosesIn an emergency: 20mg IV, followed by 20-80mg every 20min to a maximum of 220mg. Avoid in patients with Asthma, congestive cardiac failure, Liver failure(Ref: William's Obstetrics; 25th edition)", "cop": 4, "opa": "Alpha 1 blocker", "opb": "Alpha 2 blocker", "opc": "Alpha 1 and selective beta 1 blocker", "opd": "Alpha 1 and non-selective beta blocker", "subject_name": "Gynaecology & Obstetrics", "topic_name": "All India exam", "id": "a843da51-7ae7-464f-9c2b-3bb7efe762b0", "choice_type": "single"} {"question": "Endometriomas on ultasound appear as", "exp": "Endometrioma  classically is cystic with homogenous, low-level internal echoes, i.e ground-glass echogenicity.", "cop": 3, "opa": "Solid with diffuse internal low-level echoes.", "opb": "Solid with focal internal low-level echoes.", "opc": "Cystic with diffuse internal low-level echoes.", "opd": "Cystic with focal hyperechoic internal echoes.", "subject_name": "Gynaecology & Obstetrics", "topic_name": null, "id": "20ee69f6-cd95-4442-8c8b-c9326ed31a34", "choice_type": "single"} {"question": "In utero, the fetal human ovary has capacity to produce oestrogen by", "exp": "By 8 weeks, fetal human ovary has capacity to produce oestrogen.", "cop": 2, "opa": "6 weeks", "opb": "8 weeks", "opc": "10 weeks", "opd": "12 weeks", "subject_name": "Gynaecology & Obstetrics", "topic_name": null, "id": "06cf4b4d-b129-48f3-9d49-fbfb892912db", "choice_type": "single"} {"question": "Asthenozoospermia is", "exp": "Asthenozoospermia is decreased sperm motility.", "cop": 2, "opa": "Increased abnormal forms of sperm", "opb": "Decreased sperm motility", "opc": "Absence of sperms in semen", "opd": "Increased white cells in semen", "subject_name": "Gynaecology & Obstetrics", "topic_name": null, "id": "89f71932-afc7-495d-974d-0c8b39051f54", "choice_type": "single"} {"question": "Impetigo herpetiformes in pregnancy is treated with", "exp": "Generalized pustular psoriasis of pregnancy (impetigo herpetiformes) is treated with systemic steroids along with antimicrobials for secondary infection.", "cop": 3, "opa": "Acyclovir", "opb": "Acyclovir + antimicrobials for secondary infections", "opc": "Systemic corticosteroids + antimicrobials for secondary infection", "opd": "Systemic corticosteroids + acyclovir + antimicrobials for secondary infection", "subject_name": "Gynaecology & Obstetrics", "topic_name": null, "id": "2f45175e-2ec1-4507-ad9b-2d95f734fa54", "choice_type": "single"} {"question": "Mrs. Y, 25years with the previous history of twin pregnancy delivered at 36weeks, now at 10weeks of gestation, her obstetric score is", "exp": "Gravida: a woman who currently is pregnant or has been in the past, irrespective of the pregnancy outcome.With the establishment of the first pregnancy, she becomes a primigravida, and with successive pregnancies, a multigravida.Nullipara: a woman who has never completed a pregnancy beyond 20 weeks' gestation. She may not have been pregnant or may have had a spontaneous or elective aboion(s) or an ectopic pregnancy.Primipara: a woman who has been delivered only once of a fetus or fetuses born alive or dead with an estimated length of gestation of 20 or more weeks.Multipara: a woman who has completed two or more pregnancies to 20 weeks' gestation or more. Parity is determined by the number of pregnancies delivered reaching 20 weeks. It is not increased to a higher number if multiples are delivered in a given pregnancy. Moreover, stillbih does not lower this number.Reference: William's obstetrics; 25th edition", "cop": 3, "opa": "G1P1L2", "opb": "G1P2L2", "opc": "G2P1L2", "opd": "G2P2L2", "subject_name": "Gynaecology & Obstetrics", "topic_name": "General obstetrics", "id": "4884b55a-2e29-4fee-a781-fa98820b235e", "choice_type": "single"} {"question": "Induction of labor by amniotomy can lead to the following complications;", "exp": "Cord prolapse is te condition where the umbilical cord lies below the presenting pa after rupture of membranes. Aetiological factors includes,Mapresentations(20% transverse lie,12% footling,6% flexed breech),contracted pelvis,multiple pregnancy,polyydraminos when themembrane ruptures,exclusively long cord,battledore placenta as the cord is attached to the margin of placenta,multipara,iatrogenicaificial rupture of membranes with a mobile presenting pa, ,manual rotationor version Refer page no 386 of Text book of obstetrics ,sheila balakrishnan.", "cop": 1, "opa": "Cord prolapse", "opb": "Abruptio placenta", "opc": "Rupture uterus", "opd": "Infection", "subject_name": "Gynaecology & Obstetrics", "topic_name": "Abnormal labor", "id": "00732a9d-4d99-4ae1-9945-535ebc7d1a5e", "choice_type": "single"} {"question": "Bih trauma is a risk factor for", "exp": "A bh injury is impoant etiological factor for prolapse A perineal tear is less harmful than the excessive stretching of the pelvic floor muscles and ligaments that occur during childbih because overstretching cause atonicity whereas torned muscle could be stitched Squatting position used during delivery may cause excessive stretching of the pelvic floor muscles and ligaments and leads to genital prolapse a few years later. Ref: SHAW&;S TEXTBOOK OF GYNAECOLOGY; 15th edition; Pg no:332", "cop": 1, "opa": "Prolapse uterus", "opb": "Endometriosis", "opc": "PID", "opd": "Aboions", "subject_name": "Gynaecology & Obstetrics", "topic_name": "Urogynecology", "id": "6af0c634-97d2-492d-b4c4-a0c69d59a2bc", "choice_type": "single"} {"question": "Whiff test is positive in", "exp": "Bacterial vaginosis (BV) is an alteration of normal vaginal bacterial flora that results in the loss of hydrogen peroxide-producing lactobacilli and an overgrowth of predominantly anaerobic bacteria. Diagnosis A fishy vaginal odor, which is paicularly noticeable following coitus, and vaginal discharge are present.Vaginal secretions are gray and thinly coat the vaginal walls. The pH of these secretions is higher than 4.5 (usually 4.7 to 5.7). Microscopy of the vaginal secretions reveals an increased number of clue cells, and leukocytes are conspicuously absent. In advanced cases of BV, more than 20% of the epithelial cells are clue cells.The addition of KOH to the vaginal secretions (the \"whiff\" test) releases a fishy, amine-like odor. Treatment Metronidazole, an antibiotic with excellent activity against anaerobes but poor activity against lactobacilli, is the drug of choice for the treatment of BV. A dose of 500 mg administered orally twice a day for 7 days should be used. Ref: Berek and Novak's Gynecology; 15th edition; Chapter 18", "cop": 1, "opa": "Bacterial Vaginosis", "opb": "Trichomonas Vaginitis", "opc": "Vulvovaginal Candidiasis", "opd": "Inflammatory Vaginitis", "subject_name": "Gynaecology & Obstetrics", "topic_name": "Infections of the genital tract", "id": "bd81159c-9fd8-475a-9e69-022c2b21fd15", "choice_type": "single"} {"question": "Strawberry vaginitis is caused by", "exp": "Ans. d (Trichomonas vaginalis) (Ref. Shaw's Text book of Gynaecology, 12th/pg. 97, 98)TRICHOMONAS VAGINALIS# Trichomoniasis is disease of child-bearing age and its ingress into vagina is favoured by lowered general resistance and when pH is raised as during the menstrual period (pH 5 -6).# The vaginal discharge is profuse, thin creamy or slightly green in colour, irritating and frothy.# The vaginal walls are tender and discharge causes pruritus and inflammation of vulva. There are often multiple small punctate strawberry spots on vaginal vault and portio vaginalis of cervix-- the so-called strawberry cervix/vagina.# Actively motile Trichomonads can be seen on wet film preparation or parasite can be cultured on feinberg-Whittington medium.# Treatment is metronidazole 200 mg tid for 7 days for both partners.# In first trimester of pregnancy -Trichofuran suppositories, Vinegar douche and betadine gel are useful. STDCausative agentDiagnosisTreatment1.Chancroid (Soft sore)H. ducreyi\"School of fish\"/Rail road trackErythromycin/ Cotrimazole2.LGVChlamydia trachomatis(LI, L2, L3)Gram's stain with > 4 neutrophils per fieldTetracycline3.Syphilis (Hard chancre)T pallidumDark ground microscopyPenicillin4.Genital herpesH. simplex 2Tzanck preparation or Pap stainAcyclovir5.Donovaniasis (Granuloma inguinale)C.granulomatisDemonstration of Donovan bodies in mononuclear cells in smear.Erythromycin6.GonorrhoeaN. gonorrhoeaeRapid diagnosis in male is by Gram's staining of urethral exudatesPenicillin, Ceftriaxone7.TrichomoniasisT. vaginalis\"Strawberry vagina'' ; Wet film preparationMetronidazole8.Bacterial vaginosisGadnerella vaginalis'Clue cells', Leucocytopenia, Reduced lactobacilliMetro/ClindaStrawberry cervix: diffusely visible punctate cervical erythema, seen with trichomonas vaginitis.Cobblestone vagina: Thickening of the natural vaginal folds due to chronic vaginitis.", "cop": 4, "opa": "Candida albicans", "opb": "Chlamydia", "opc": "Herpes genitalis", "opd": "Trichomonas vaginalis", "subject_name": "Gynaecology & Obstetrics", "topic_name": "Examination for Genital Infections", "id": "b409cf64-b8e7-49c9-82a1-fadde23ff4da", "choice_type": "single"} {"question": "Twin peak sign seen in", "exp": "Twin peak or lambda sign it is characterized of dichorionic pregnancies and is due to the chorionic tissue between the two layers of the intewin membrane at the placental origin . twin peak appears as a triangle with base at chorionic surface and apex in intewin membrane.", "cop": 3, "opa": "Monochorionic Monoamniotic", "opb": "Monochorionic Diamniotic", "opc": "Dichorionic Diamniotic", "opd": "Dichorionic Monoamniotic", "subject_name": "Gynaecology & Obstetrics", "topic_name": "JIPMER 2018", "id": "67bfe8a2-4378-4392-a291-212aac6bbda5", "choice_type": "single"} {"question": "Uterosacral ligament contains", "exp": "Uterosacral ligaments are composed almost entirely of connective tissue and very little plain muscle. They contain sympathetic and parasympathetic nerves.", "cop": 2, "opa": "Mainly plain muscle", "opb": "Sympathetic and parasympathetic nerves", "opc": "Contain uterine vessels", "opd": "Ureters", "subject_name": "Gynaecology & Obstetrics", "topic_name": null, "id": "0d78d5b8-55b4-43e2-9c5d-5b95886e6c1d", "choice_type": "single"} {"question": "A lady presents with headache, recurrent abortions, and cavernous sinus thrombosis. The diagnosis is", "exp": "(A) APLA CLINICAL FEATURES OF ANTIPHOSPHOLIPID ANTIBODY SYNDROMEManifestation%Venous Thrombosis and Related Consequences Deep vein thrombosis39Livedo reticularis24Pulmonary embolism14Superficial thrombophlebitis12Thrombosis in various other sites11Arterial Thrombosis and Related Consequences Stroke20Cardiac valve thickening/dysfunction and/or Libman- Sacks vegetations14Transient ischemic attack11Myocardial ischemia (infarction or angina) and coronary bypass thrombosis10Leg ulcers and/or digital gangrene9Arterial thrombosis in the extremities7Retinal artery thrombosis/amaurosis fugax7Ischemia of visceral organs or avascular necrosis of bone6Multi-infarct dementia3Manifestation%Neurologic Manifestations of Uncertain Etiology Migraine20Epilepsy7Chorea1Cerebellar ataxia1Transverse myelopathy0.5Renal Manifestations Due to Various Reasons(Renal Artery/Renal Vein/Glomerular Thrombosis,Fibrous Intima Hyperplasia)3Osteoarticular Manifestations Arthralgia39Arthritis27Obstetric Manifestations (Referred to the Number of Pregnancies) Preeclampsia10Eclampsia4Fetal Manifestations (Referred to the Number of Pregnancies) Early fetal loss (<10 weeks)35Late fetal loss (10 weeks)17Premature birth among the live births11Hematologic Manifestations Thrombocytopenia30Autoimmune hemolytic anemia10", "cop": 1, "opa": "APLA", "opb": "Cervical incompetence", "opc": "Unexplained", "opd": "Pre eclampsia", "subject_name": "Gynaecology & Obstetrics", "topic_name": "Miscellaneous (Obs)", "id": "a595c108-e75b-4ecb-bab5-e654ec0604ee", "choice_type": "single"} {"question": "Scale for sexual maturity is", "exp": "(A) Tanners' scale # TANNER STAGES (TANNER SCALE) are stages of physical development in children, adolescents and adults. The stages define physical measurements of development based on external primary and secondary sex characteristics, such as the size of the breasts, genitalia, and development of pubic hair.> Due to natural variation, individuals pass through the Tanner stages at different rates, depending in particular on the timing of puberty.> In HIV treatment, Tanner staging is used to determine which treatment regimen to follow (adult, adolescent, or pediatric).> The Tanner stages were first identified by James Mourilyan Tanner and thus bear his name.> Pubic hair (both male and female) Tanner I: no pubic hair at all (prepubertal state) Tanner II: small amount of long, downy hair with slight pigmentation at the base of the penis and scrotum (males) or on the labia majora (females) Tanner III: hair becomes more coarse and curly, and begins to extend laterally Tanner IV: adult-like hair quality, extending across pubis but sparing medial thighs Tanner V: hair extends to medial surface of the thighs> Genitals (male) Tanner I: prepubertal (testicular volume less than 1.5 ml; small penis of 3 cm or less) Tanner II: testicular volume between 1.6 and 6 ml; skin on scrotum thins, reddens and enlarges; penis length unchanged Tanner III: testicular volume between 6 and 12 ml; scrotum enlarges further; penis begins to lengthen to about 6 cm Tanner IV: testicular volume between 12 and 20 ml; scrotum enlarges further and darkens; penis increases in length to 10 cm and circumference Tanner V: testicular volume greater than 20 ml; adult scrotum and penis of 15 cm in length> Breasts (female) Tanner I: no glandular tissue; areola follows the skin contours of the chest (prepubertal) Tanner II: breast bud forms, with small area of surrounding glandular tissue; areola begins to widen Tanner III: breast begins to become more elevated, and extends beyond the borders of the areola, which continues to widen but remains in contour with surrounding breast Tanner IV: increased breast size and elevation; areola and papilla form a secondary mound projecting from the contour of the surrounding breast Tanner V: breast reaches final adult size; areola returns to contour of the surrounding breast, with a projecting central papilla.", "cop": 1, "opa": "Tanners' scale", "opb": "Glasgow scale", "opc": "Goniometer", "opd": "Moss scale", "subject_name": "Gynaecology & Obstetrics", "topic_name": "Miscellaneous (Gynae)", "id": "b7e6cc74-e0d1-4d4f-b5b9-aea9f9a292c7", "choice_type": "single"} {"question": "Eclampsia is a complication of", "exp": "Eclampsia is an extremely severe form of Preeclampsia characterized by the sudden onset of generalized tonic-clonic seizures.Reference: Practical guide to High-risk pregnancy and delivery; Chapter 16; Hypeensive disorders of Pregnancy", "cop": 3, "opa": "Anaemia complicating Pregnancy", "opb": "Diabetes complicating Pregnancy", "opc": "Hypeensive complicating Pregnancy", "opd": "Cardiac disease complicating Pregnancy", "subject_name": "Gynaecology & Obstetrics", "topic_name": "Medical, surgical and gynaecological illness complicating pregnancy", "id": "5b7e75b5-494f-41fb-b5d5-86d07c7c8520", "choice_type": "single"} {"question": "Mrs. X, Primigravida at 36weeks complains of generalized pruritis with increased intensity in soles. Diagnosis is", "exp": "Clinical presentation of Intrahepatic Cholestasis of Pregnancy:Pruritus develops in late pregnancy, although it occasionally manifests earlier. There are no constitutional symptoms, and generalized pruritus shows a predilection for the soles. Skin changes are limited to excoriations from scratching. Biochemical tests may be abnormal at presentation, but pruritusmay precede laboratory findings by several weeks. Approximately 10 percent of women have jaundice.Ref: William&;s Obstetrics; 24th edition; Chapter 55", "cop": 1, "opa": "Intrahepatic cholestasis of pregnancy", "opb": "HELLP", "opc": "Viral hepatitis", "opd": "Acute fatty liver of pregnancy", "subject_name": "Gynaecology & Obstetrics", "topic_name": "Medical, surgical and gynaecological illness complicating pregnancy", "id": "d1dd13f9-ee85-4f5a-979e-0f9f00b1d81e", "choice_type": "single"} {"question": "A woman with a history of repeated aboions gave bih to a low bih weight baby. The child is having rashes over palm and rhinitis. The most probable diagnosis is September 2009", "exp": "Ans. B: Congenital Syphilis Symptoms in newborns with congenital syphilis may include: Failure to gain weight or failure to thrive Fever Irritability No bridge to nose (saddle nose) Early rash -- small blisters on the palms and soles Later rash -- copper-colored, flat or bumpy rash on the face, palms, and soles Rash of the mouth, genitalia, and anus Watery discharge from the nose Congenital rubella predominantly include cochlear (sensori-neural deafness), cardiac (septal defects, PDA), hematologic, ophthalmic (cataracts, retinopathy) and chromosomal abnormalities.", "cop": 2, "opa": "Congenital HIV", "opb": "Congenital syphilis", "opc": "Congenital rubella", "opd": "Pemphigus", "subject_name": "Gynaecology & Obstetrics", "topic_name": null, "id": "d06467ec-a7ef-4ca8-a771-6de4f2ca7772", "choice_type": "single"} {"question": "P4 pf vagina during reproductive life is", "exp": "Vaginal secretion is acidic during reproductive life which is oestrogen dependent, and inhibits growth of pathogenic organisms.", "cop": 2, "opa": "Alkaline", "opb": "Acidic", "opc": "Neutral", "opd": "Variable", "subject_name": "Gynaecology & Obstetrics", "topic_name": null, "id": "1f62809c-4a05-4aa9-8ab3-2940d7ce5791", "choice_type": "single"} {"question": "In brachytherapy of carcinoma cervix, dose of radiation to Point A is", "exp": "Point A: 2cm cephalic and lateral to the external os It is the point of crossing of uterine aery and ureter It receives 7000-8000cGy Point B: 2cm cephalic and 5cm lateral at the same plane and is approximately the site of obturator gland It receives 2000cGy initially and rest of dose supplemented by external beam irradiation of 4000cGy spreading over another 3 weeks total of 6000cGy Ref: D.C.DUTTA'S TEXTBOOK OF GYNECOLOGY; 6th edition; Pg no:350", "cop": 4, "opa": "1000 rad", "opb": "4000 rad", "opc": "6000 rad", "opd": "8000 rad", "subject_name": "Gynaecology & Obstetrics", "topic_name": "Gynaecological oncology", "id": "4cb68580-f965-4bfc-af39-846eb20cec53", "choice_type": "single"} {"question": "Colposcopy is best performed during", "exp": "Cervix is moist with mucus and external os is slightly patulous in proliferative phase and exposes squamocolumnar junction adequately.", "cop": 1, "opa": "Proliferative phase", "opb": "Secretory phase", "opc": "Menstrual phase", "opd": "Any day of cycle", "subject_name": "Gynaecology & Obstetrics", "topic_name": null, "id": "4081711b-e516-4f8e-b298-9c443674d247", "choice_type": "single"} {"question": "Submucosal fibroid is detected by", "exp": "USG: Ultrasound is the most readily available, least costly imaging technique to diagnose fibroid. It checks the number, location, and size of fibroids and helps to reduce overlooking small fibroids during surgery (which might lead to persistence or recurrence of symptoms).\nSonohysterography – is instillation of saline into endometrial cavity during TVS\nHysteroscopy or hysterosalpingography: These methods are useful to detect submucous fibroid in unexplained infertility and repeated pregnancy wastage. The presence and site of submucous fibroid can be diagnosed by direct visualization during hysteroscopy or indirectly as a filling defect on HSG. Hysteroscopy also allows its excision under direct vision.\nUterine Curettage: It can also help in diagnosis of submucous fibroid by feeling of a bump during curettage.\nLaparoscopy: is helpful if uterine size is less than 12 weeks, for detection of subserous fibroid and not submucous. It can also differentiate a pedunculated fibroid from an ovarian tumor not revealed by clinical examination and ultrasound.", "cop": 1, "opa": "Hysteroscopy", "opb": "Hysterosalpingography", "opc": "USG(Transabdominal)", "opd": "Laparoscopy", "subject_name": "Gynaecology & Obstetrics", "topic_name": null, "id": "b254244b-6bf2-4e11-a60e-431733236034", "choice_type": "single"} {"question": "Time taken for spermatogenesis is", "exp": "The process whereby the germ cells is converted into spermatids is called male gametogenesis or spermatogenesis. \nShortly before puberty, the testicular cords become canalized and form the seminiferous tubules. \nAccompanying this change is the conversion of the germ cells into a group of spermatogonial cells. At regular intervals, spermatogonia arise from this stem cell cluster to start the process of spermatogenesis", "cop": 4, "opa": "50-60 days", "opb": "60-70 days", "opc": "80-90 days", "opd": "70-80 days", "subject_name": "Gynaecology & Obstetrics", "topic_name": null, "id": "c9e76b70-fa47-4695-bbaf-9c36a203538d", "choice_type": "single"} {"question": "Cells in vaginal cytology increase in reproductive age under the influence of", "exp": "Ans: d (Oestrogen)EXPLANATION:A majority of cells observed in a normal vaginal smear are, not surprisingly, vaginal epithelial cells. In addition, varying numbers of leukocytes, erythrocytes and bacteria are usually evident, and small numbers of other contaminating cells and microorganisms are sometimes observed.Analyzing a vaginal smear is largely an exercise in classifying the epithelial cells into one of three fundamental types: parabasal, intermediate or superficial cells. Parabasal cells are the smallest epithelial cells seen on a typical vaginal smear.They are round or nearly round and have a high nuclear to cytoplasmic ratio.Intermediate CellsIntermediate cells vary in size and shape, but typically have a diameter two to three times that of parabasal cells. Many cytologists subclassify these cells into:Small intermediates - nearly round or oval shape with large, prominent nucleiLarge intermediates - polygonal shape with a small nuclear/cytoplasmic ratio Intermediate cells are prevalent during all stages of the cycle except estrus.Superficial CellsSuperficial cells are the largest cells seen on a vaginal smear.They are polygonal in shape and distinctly flat, sometimes having the appearance of being rolled up.Their nuclei are either absent or pyknotic (very small and dark).Superficial cells without nuclei are often referred to as being \"fully comified\".", "cop": 4, "opa": "Progesterone", "opb": "FSH", "opc": "LH", "opd": "Oestrogen", "subject_name": "Gynaecology & Obstetrics", "topic_name": "Endocrine Control of the Menstrual Cycle", "id": "e55d376d-2d82-4345-960d-d6dd763d02c1", "choice_type": "single"} {"question": "Increased gonadal production of estrogen is characteristic of", "exp": "In persons with testicular feminization, estradiol secretion by the testes is markedly increased (but not to the level produced by normal ovaries); the mechanism is lack of suppression of LH by testosterone and consequently increased stimulation of gonadal testosterone and estradiol secretion. Ovaries containing follicle cysts may be a source of increased estrogen production, paicularly during the postmenopausal years, when gonadotropin levels are very high. The increase in estrogen production that is characteristic of PCOD is the consequence of the peripheral conversion of androstenedione to estrogen, not of direct gonadal production. During the third trimester of pregnancy estrogen production is increased because of the formation of estrogen by the placenta rather than by the ovary. Arrhenoblastoma is a virilizing ovarian tumor and does not secrete estrogen. Congenital adrenal hyperplasia causes virilization due to the increased adrenal output of androgens.", "cop": 1, "opa": "Testicular feminization", "opb": "Polycycstic ovarian disease", "opc": "Congenital adrenal hyperplasia", "opd": "Third trimester of pregnancy", "subject_name": "Gynaecology & Obstetrics", "topic_name": "Disorders of menstruation", "id": "115e5944-a995-48b1-9881-4bb604111d2c", "choice_type": "single"} {"question": "Schwangershaft protein is the other name of", "exp": "Schwangershaft Protein:\n\nIt is the other name for pregnancy specific B1 glycoprotein.\nProduced by trophoblast.\nCan be detected 18 days after ovulation.\nIts concentration rises steadily and reaches 200 mg/ml at term.\nRole-measure of placental function for fertility control.", "cop": 3, "opa": "HCG", "opb": "Papp-1", "opc": "Pregnancy specific betal 1 glycoprotein", "opd": "Activin", "subject_name": "Gynaecology & Obstetrics", "topic_name": null, "id": "7eaa9ce8-b4cf-462a-bf5b-ce6ed75556a6", "choice_type": "single"} {"question": "Drugs Supplied by NACO For Prevention of Mother to Child transmission", "exp": "NACO is National AIDS Control Organization which was launched in India in the year 1987.The Ministry of Health and Family Welfare had setup NACO as a separate wing to implement and closely monitor the various components of the National Aids Control Programme.\nNACO has established many integrated counselling and testing centres (ICTCs ),where pregnant women are provided counselling and testing facilities.\n“Women who are found to be HIV positive are given single dose of prophylactic Nevirapine at the time of labour and new born infant is also given a single dose of Nevirapine within 72 hrs of birth”.\nPark 20/e, p 373", "cop": 1, "opa": "Nevirapine", "opb": "Zidovudine", "opc": "Nevirapine + Zidovudine", "opd": "Nevirapine + Zidovudine + 3tc", "subject_name": "Gynaecology & Obstetrics", "topic_name": null, "id": "50e93ffb-bebf-43b2-b30a-e88a0f66b571", "choice_type": "single"} {"question": "Contraceptive method of choice in lactating mothers is", "exp": "Progesterone only pills or mini pill Pills that contain very low doses of progestin, do not contain estrogen, and so can be used throughout the period of breastfeeding.These pills work primarily by thickening cervical mucus, disrupting the menstrual cycle, and preventing the ovulation.Reference: Mudaliar and Menon&;s clicks obstetrics,12th edition, page no:452", "cop": 2, "opa": "Barrier method", "opb": "Progesterone only pills", "opc": "Oral contraceptive pills", "opd": "Lactational amenorrhea", "subject_name": "Gynaecology & Obstetrics", "topic_name": "General obstetrics", "id": "791f74c6-c2b0-404e-938f-dbfdec9a007d", "choice_type": "single"} {"question": "Late deceleration indicates", "exp": "Deceleration pattern - Three types Early deceleration - due to head compression Late deceleration - chronic placental insufficiency Variable deceleration - cord compression Sinusoidal pattern - It is stable baseline FHR with fixed base line variability without any acceleration. If is often associated with fetal anaemia, feto-maternal haemorrhage, fetal hypoxia, and when narcotics are given to mothers", "cop": 3, "opa": "Head compression", "opb": "Cord compression", "opc": "Foetal hypoxia", "opd": "Breech presentation", "subject_name": "Gynaecology & Obstetrics", "topic_name": "All India exam", "id": "71c43978-0dab-4812-b124-868d11c2dc69", "choice_type": "single"} {"question": "Pawlik's grip is done in obstetrics for assessment of", "exp": "(C) Presenting part", "cop": 3, "opa": "Gestational age", "opb": "Fetal maturity", "opc": "Presenting part", "opd": "Labour", "subject_name": "Gynaecology & Obstetrics", "topic_name": "Miscellaneous (Obs)", "id": "7a624926-6e2d-4e48-95a0-403936180171", "choice_type": "single"} {"question": "Corpus luteum supports endometrium in pregnancy till", "exp": "(A) 6-10 weeks# At 6-8 weeks there is transfer of function of the corpus luteum to the placenta.", "cop": 1, "opa": "6-10 weeks", "opb": "12-24 weeks", "opc": "36 weeks", "opd": "42 weeks", "subject_name": "Gynaecology & Obstetrics", "topic_name": "Miscellaneous (Obs)", "id": "71a00b95-f77c-4885-acdb-083e11ba35a6", "choice_type": "single"} {"question": "Best prenatal treatment CAH is", "exp": "Prenatal diagnosis of CAH is possible with chorion villus sampling at early weeks of pregnancy using DNA probes\n\nThe 17-OHP level in the amniotic fluid is elevated\nPrenatal therapy with dexamethasone or termination of pregnancy is the option", "cop": 4, "opa": "Betamethasone", "opb": "Prednisolone", "opc": "Hydrocortisone", "opd": "Dexamethasone", "subject_name": "Gynaecology & Obstetrics", "topic_name": null, "id": "a7b4feb3-754f-4609-9f61-5c456f39c5d2", "choice_type": "single"} {"question": "Young lady presents with acute abdominal pain and history of 11/2 months amenorrhoea, on USG examination there is collection of fluid in the pouch of douglas and empty gestational sac. Diagnosis is", "exp": "Ans. is a i.e. Ectopic pregnancy", "cop": 1, "opa": "Ectopic pregnancy", "opb": "Pelvic hematocele", "opc": "Threatened aboion", "opd": "Twisted ovarian cyst", "subject_name": "Gynaecology & Obstetrics", "topic_name": null, "id": "c8642155-6f4f-4179-8b20-24761e00d647", "choice_type": "single"} {"question": "1st meiotic division of oogenesis gets arrested at", "exp": "The germ cell reach gonadal ridge by the 6th week to form oogonia which multiply by mitosis\nBy 12th week the oogonia are arranged in clusters and are covered by a layer of follicular cells to form the primordial follicle\nThe oogonia enter into the1st meiotic division but are arrested in diplotene stage of prophase and doesn't enter into metaphase but rest until puberty", "cop": 1, "opa": "Diplotene stage of prophase", "opb": "Pachytene stage of prophase", "opc": "Leptotene stage of prophase", "opd": "Metaphase stage of prophase", "subject_name": "Gynaecology & Obstetrics", "topic_name": null, "id": "2b40cbc4-317e-49bf-866b-9f4f37189ade", "choice_type": "single"} {"question": "Chassar Moir technique is used in", "exp": "Vesico vaginal fistula", "cop": 1, "opa": "Vesico vaginal fistula", "opb": "Stress incontinence", "opc": "Urethrocoele", "opd": "Enterocoele repair", "subject_name": "Gynaecology & Obstetrics", "topic_name": null, "id": "e74f2708-075a-44ba-8fd9-b36ac19b44e6", "choice_type": "single"} {"question": "Second stage of labour begins with", "exp": "Ans: a (Complete cervical Dilatation)Ref: William's Obstetrics, 23r ed. Chapter 17The dilatation and the effacement of the cervix occurs in the first stage of the labour. The patient is in active stage of labour once the cervix is effaced and dilated till 3 cm, following this the rate of dilatation is 1 cm/hour in cases of primigravidas and 1.5 cm/hour in cases of multigravidasThe beginning of the 2 stage of labour is characterized by the complete dilatation and the effacement of the cervixThe main aim of the 2 stage of the labor is the expulsion of the foetus through the female reproductive tract. The 2 stage consists of engagement, progressive flexion and descent, internal rotation, restitution, external rotation and lateral flexion for the delivery of the trunk.Engagement occurs when the greatest possible transverse diameter of the foetus enters the pelvic brim.Flexion and descent are the processes which occurs constantly during the 2 stage.Internal rotation is the most important step occurring in the 2 stage.Expulsion of the placenta is the 3 stage of labor which immediately follows the expulsion of the foetus.Once the baby is expelled, active management of the third stage of labour is recommended. In this immediately post the delivery of the baby, and after ensuring that the uterus is adequately contracted, controlled cord traction, uterine massage and administration of uterotonics is carried out.", "cop": 1, "opa": "Complete cervical dilatation", "opb": "Expulsion of foetus", "opc": "Expulsion of placenta", "opd": "Internal rotation", "subject_name": "Gynaecology & Obstetrics", "topic_name": "Miscellaneous (Gynae)", "id": "3797c8f6-4b6d-4513-92cb-304b471a6d1b", "choice_type": "single"} {"question": "A female with recurrent abortion and isolated prolonged APTT is most likely associated with", "exp": "Ans. is 'a' i.e. Lupus anticoagulant They have asked about antiphospholipid antibody syndrome.Antiphospholipid antibodies account for 3% to 5% of patients with repetitive pregnancy losses.The frequency of fetal death and recurrent abortion in untreated patients with antiphospholipid antibodies is greater than 90%.* There are several antiphospholipid antibodies. The most relevant to obstetricians are:The lupus anticoagulant (LAC),The anticardiolipin antibody andThe antibody that cause false positive syphilis test (BFP-ST). The name lupus anticoagulant was derived from the fact that this antibody was found first in patients with systemic lupus erythematosus and acted as an anticoagulant by prolonging the partial thromboplastin time (PTT).This name was a poor choice because soon it was found that lupus anticoagulant was present in many patients who did not have lupus and that in majority of patients the antibody was responsible for episodes of thrombosis, rather than anticoagulation.In the laboratory, LAC is not measured directly.It is assessed by its effect on PTT and the kaolin clotting time.Typically patients with lupus anticoagulant have a prolonged PTT and a normal PT.Unfortunately, a normal PTT does not exclude the possibility of LAC, and if the clinical suspicion is strong and the PTT is normal a kaolin clotting time or a dilute Russell viper venom time should be performed.It has also been found that some patients with LAC and recurrent abortions have elevated values of serum IgM.Other antiphospholipid antibodiesThe anticardiolipin antibody is the antiphospholipid antibody, most commonly found in patients with repetitive early pregnancy losses. Anticardiolipin antibody is found in 90% of patients with L.A.C. but the majority of patients with positive anticardiolipin antibody do not have L.A.C.Anticardiolipin antibody is measured in the laboratory by the ELISA test.BFP-STIt is the less common antiphospholipid antibody.Both BFP-ST and Anticardiolipin antibody, measure the antibody against cardiolipin but they are not the same.Clinical features of antiphospholipid antibodies.* The presence of any or several of the three antiphospholipid antibodies is associated withRecurrent early pregnancy losses.Episodes of venous and arterial thrombosisSevere preeclampsiaChorea gravidarumH/o ischemic strokes, transient ischemic episodes, migraine headaches.Postpartum complications such as pulmonary infiltrates, fever, and cardiac symptoms.Abortions in antiphospholipid antibody syndromeTypically these patients give a history of a live fetus documented by ultrasound or by Doppler before demise or abortion occurs.The majority of the pregnancy losses occur between 14 and 18 weeks.Fetal death is these patients is caused by extensive thrombosis of the placental vessels and the placenta is usually smaller than expected for the gestational age.", "cop": 1, "opa": "Lupus anticoagulant", "opb": "DIC", "opc": "Von wilebrand disease", "opd": "Hemophilia", "subject_name": "Gynaecology & Obstetrics", "topic_name": "Recurrent Miscarriage", "id": "e129e427-86ef-472f-b92b-f7e43187d6df", "choice_type": "single"} {"question": "Site of placement of tension free vaginal tapes in stress urinary incontinence", "exp": "Tension-free vaginal tape A knitted 11 mm x 40 cm polypropylene mesh tape is inseed transvaginally at the level of the midurethra Complications: Sho term Voiding difficulties , Bladder perforation, de novo Urgency and Bleeding Ref: SHAW&;S TEXTBOOK OF GYNAECOLOGY; 15th edition; Pg no:194", "cop": 4, "opa": "Ureterovaginal junction", "opb": "Urethrovaginal junction", "opc": "Upper pa of urethra", "opd": "Middle pa of urethra", "subject_name": "Gynaecology & Obstetrics", "topic_name": "Urogynecology", "id": "2b3ce6e8-b033-45af-8317-05a736e4cd26", "choice_type": "single"} {"question": "Cervical cerclage is offered to pregnant women with prior history of preterm bih if cervical length is less than", "exp": "Cervical length screening is now recommended for women with prior history of preterm bih. Between 16 and 24weeks gestation sonographic cervical measurement is completed every 2weeks. If initial or subsequent cervical length is 25 to 29mm, then a weekly interval is considered. If the cervical measurement is <25mm, cerclage is offered. Women without a history of preterm bih but with a sho cervix incidentally identified sonographically, progesterone therapy is to be offered instead of cerclage(Ref: William's Obstetrics; 25th edition)", "cop": 1, "opa": "25mm", "opb": "30mm", "opc": "35mm", "opd": "40mm", "subject_name": "Gynaecology & Obstetrics", "topic_name": "All India exam", "id": "d8b6713d-0345-4407-a979-13fe99249479", "choice_type": "single"} {"question": "In child with ambiguous genitalia, first test to be done is", "exp": "Karyotyping is first test to be done in child with ambiguous genitalia.", "cop": 2, "opa": "Serum 17 hydroxy progesterone", "opb": "Karyotyping", "opc": "Serum 17 ketosteroid", "opd": "Ultra sound pelvis", "subject_name": "Gynaecology & Obstetrics", "topic_name": null, "id": "c114f5ae-f545-4be5-baaa-0cd748f05239", "choice_type": "single"} {"question": "Fetal heart starts contracting at", "exp": "(C) 3-5 weeks[?]The Beginning:The human heart begins to beat and pump blood through the embryo around day 22 of gestation.The electric stimulus that triggers contractions in the myocardium arise spontaneously within the myocardium itself, and propagate from cell to cell. Input from the central nervous system can modify the heart rate (the frequency of heart beats), but it does not initiate beats.The ability of cardiac myocytes to beat is an intrinsic property of these cells. It has been found that myocytes removed from the early heart and grown in culture will beat sporadically, and if they become connected to each other, will then begin to beat rhythmically, in unison.As a functional organ, the heart begins to beat very early, even before it has assumed its final form.Interestingly, the heart begins to beat even before structures such as valves and septa (singular: septum; the muscular walls that divide the chambers) have formed. The initial contractions are peristaltic - that is, they proceed in a wave like fashion along the length of the heart. IMPORTANT EVENTS FOLLOWING FERTILIZATION0' hourFertilization (day-15 from LMP)30 hours2 cell stage (blastomeres)40-50 hours4 cell stage72 hours12 cell stage96 hours16 cell stage. Morula enters the uterine cavity5th dayBlastocyst4-5th dayZona pellucida disappears5-6th dayBlastocyst attachment to endometrial surface6-7th dayDifferentiation of cyto and syncytiotrophoblast layers10th daySynthesis of hCG by syncytiotrophoblast9-10th dayLacunar network forms10-11th dayTrophoblasts invade endometrial sinusoids establishing uteroplacental circulationInterstitial implantation completed with entire decidual coverage13th dayPrimary villi16th daySecondary villi21st dayTertiary villi21st-22nd dayFetal heart, Fetoplacental circulation", "cop": 3, "opa": "10-12 days", "opb": "10-12 weeks", "opc": "3-5 weeks", "opd": "3- 5 month", "subject_name": "Gynaecology & Obstetrics", "topic_name": "Diagnosis of Pregnancy", "id": "34ce9372-2d85-4e05-b579-65efb1f34eea", "choice_type": "single"} {"question": "the distention media used in diagnostic hysteroscopy with bipolar couesy is", "exp": "ref : gynecology shaw", "cop": 2, "opa": "CO2", "opb": "NS", "opc": "Dextran 70", "opd": "Glycine", "subject_name": "Gynaecology & Obstetrics", "topic_name": "All India exam", "id": "d044bb53-7c7a-47b0-9d31-ce91e142383c", "choice_type": "single"} {"question": "Previous ectopic pregnancy is a contraindication for", "exp": "LNG-IUS is considered to be contraindicated in women with history of previous ectopic pregnancy.", "cop": 3, "opa": "Copper containing IUD", "opb": "Depot medroxy progesterone acetate", "opc": "Levonorgestrel- releasing IUS", "opd": "Progestin containing subdermal implant", "subject_name": "Gynaecology & Obstetrics", "topic_name": null, "id": "33502299-4267-40a4-a45c-11ed7b408a02", "choice_type": "single"} {"question": "The condition listed below that would most likely pursue a benign course is", "exp": "Adenomyosis is the presence of benign glands and stroma within the myometrium. It is a common cause of diffuse uterine enlargement and menstrual irregularities; however, it is not a precancerous lesion. Ref: Shaw&;s Textbook of Gynaecology; 16th edition; Chapter 30; Endometriosis and Adenomyosis", "cop": 2, "opa": "Vaginal adenosis", "opb": "Adenomyosis", "opc": "Extramammary Paget's disease", "opd": "Endometrial hyperplasia", "subject_name": "Gynaecology & Obstetrics", "topic_name": "Endometriosis and dysmenorrhea", "id": "c8bc2660-0530-4798-bb1f-da9c36d9c142", "choice_type": "single"} {"question": "The GnRH (Gonadotrophic Releasing hormone) reaches the anterior pituitary from the hypothalamus along", "exp": "The poal hypophyseal vessels form a direct vascular link between the hypothalamus and the anterior pituitary. Hypothalamic hormones like GnRH (Gonadotrophic Releasing hormone) reach the anterior pituitary from the hypothalamus along the hypophyseal poal system", "cop": 3, "opa": "Its neurological connections", "opb": "Through systemic circulation", "opc": "Its poal vascular connections", "opd": "Through CSF", "subject_name": "Gynaecology & Obstetrics", "topic_name": "Maternal Anatomy and Physiology", "id": "a076daa8-d366-421f-aea4-e308c35679ec", "choice_type": "single"} {"question": "Complete failure of mullerian duct fusion will result in", "exp": null, "cop": 1, "opa": "Uterus didelphys", "opb": "Arcuate uterus", "opc": "Subseptate uterus", "opd": "Bicornuate uterus", "subject_name": "Gynaecology & Obstetrics", "topic_name": null, "id": "6b105874-3dd5-4133-831c-67e826c9ed7a", "choice_type": "single"} {"question": "Selective reduction of multi chorionic multifetal gestation is performed around", "exp": "Selective Reduction Reduction of a selected fetus or fetuses in a multi chorionic multifetal gestation may be chosen as a therapeutic intervention to enhance survival of the remaining fetuses (ACOG, 2013a).Transabdominal fetal reductions are typically performed between 10 and 13 weeks. This gestational age is chosen because most spontaneous aboions have already occurred, the remaining fetuses are large enough to be evaluated sonographically, the amount of devitalized fetal tissue remaining after the procedure is small, and the risk of aboing the entire pregnancy as a result of the procedure is low. The smallest fetuses and any anomalous fetuses are chosen for reduction. Potassium chloride is then injected under sonographic guidance into the hea or thorax of each selected fetus(Ref: William's Obstetrics; 25th edition)", "cop": 3, "opa": "4 to 6 weeks", "opb": "8 to 10 weeks", "opc": "10 to 13 weeks", "opd": "16 to 20 weeks", "subject_name": "Gynaecology & Obstetrics", "topic_name": "All India exam", "id": "2905cd2f-8d1b-471a-b976-1507acfa158b", "choice_type": "single"} {"question": "PIH can be prevented by", "exp": "Smoking is the only environmental factor that reduces the risk of pre-eclampsia Smoking during pregnancy has adverse effects on other organs It may have effects on angiogenic factors, endothelial function and immune system reducing the risk of pre-eclampsia. Ref: Internet sources.", "cop": 4, "opa": "Polyhydramnios", "opb": "Diabetes mellitus", "opc": "Twin pregnancy", "opd": "Smoking", "subject_name": "Gynaecology & Obstetrics", "topic_name": "Medical, surgical and gynaecological illness complicating pregnancy", "id": "3355c0c4-2804-488e-a517-4eee052c833a", "choice_type": "single"} {"question": "Drug not used in the management of PPH", "exp": "Misoprostol is another prostaglandin that increases uterine tone and decreasespostpaum bleeding.36 Misoprostol is effective in the treatment of postpaum hemorrhage, but side effects may limit its use.28,37 It can be administered sublingually, orally, vaginally, and rectally Ref mudaliar textbook of obstetrics 6e", "cop": 1, "opa": "Mifepristone", "opb": "Misoprostol", "opc": "Oxytocin", "opd": "Ergotamine", "subject_name": "Gynaecology & Obstetrics", "topic_name": "Anatomy of the female genital tract", "id": "6ac3a018-cad6-431e-9fb0-6912cb1ac31f", "choice_type": "single"} {"question": "A unilateral ovarian tumour with visible peritoneal implants belong to", "exp": "Ovarian cancer staging:Stage III Tumor involves 1 or both ovaries or fallopian tubes or primary peritoneal cancer with cytologically or histologically confirmed spread to the peritoneum outside the pelvis and/or metastasis to the retroperitoneal lymph nodes. III b: macroscopic extrapelvic peritoneal metastasis < or = 2cm with or without positive retroperitoneal lymph nodes. Includes extension to capsule of liver/ spleen. III c: macroscopic extrapelvic peritoneal metastasis > 2cm with or without positive retroperitoneal lymph nodes. Includes extension to capsule of liver/ spleen. Reference : textbook of gynaecology Sheila balakrishnan, 2nd edition, pg no: 277", "cop": 4, "opa": "Stage IB", "opb": "Stage IC", "opc": "Stage IIA", "opd": "Stage IIIB", "subject_name": "Gynaecology & Obstetrics", "topic_name": "Gynaecological oncology", "id": "f927b187-1d57-4adc-9832-a64f45a432cb", "choice_type": "single"} {"question": "Fibroid with a typical \"Lantern on top of St Paul's cathedral\" appearance is", "exp": "Cervical Fibroid is compared to \"Lantern on dome of st Paul's catherdral\".", "cop": 3, "opa": "Submucosal fibroid", "opb": "subserosal fibroid", "opc": "Cervical fibroid", "opd": "Interstitial fibroid", "subject_name": "Gynaecology & Obstetrics", "topic_name": null, "id": "b594271d-ac4b-4ae7-afbc-96fee364ae28", "choice_type": "single"} {"question": "Granuloma inguinale", "exp": "Granuloma inguinale is caused by Klebsiella granulomatis. It is painless. It has incubation period of weeks to months (long).", "cop": 2, "opa": "Is caused by gardnerella vaginalis", "opb": "Beefy red ulcers that bleed easily", "opc": "Painful inflammatory nodules", "opd": "Short incubation period of 3 days", "subject_name": "Gynaecology & Obstetrics", "topic_name": null, "id": "75997290-d27f-47db-9d5a-96a6d2922856", "choice_type": "single"} {"question": "Definition of Prolonged pregnancy is", "exp": "The international definition of prolonged pregnancy, endorsed by the American College of Obstetricians and Gynecologists (2013a), is 42 completed weeks, 294 days or more from the first day of the last menstrual period.Reference: William's Obstetrics; 24th edition; Chapter 43; Postterm Pregnancy", "cop": 4, "opa": "Completed 38weeks", "opb": "Completed 40weeks", "opc": "Completed 41weeks", "opd": "Completed 42weeks", "subject_name": "Gynaecology & Obstetrics", "topic_name": "General obstetrics", "id": "4e0ccde9-9122-47eb-8c7c-7b902046c736", "choice_type": "single"} {"question": "Hormonal therapy is indicated in menopausal women for", "exp": "(A) Hot flashes[?]Hot Flashes:They are sudden, intense feelings of heat in the neck face, and torso. They can be accopanied by:Heart palpitations; Sleep disturbances; Profuse perspiration; Cold chills; Anxiety; FlushingHot flashes are: Mainly women over 40 as they transition into menopause as well as pregnant women.Other causes of Hot Flushes:Thyroid disease, epilepsy, autoimmune disorders, insulinomas, pancreatic tumors, leukemias.Hot flushes are waves of vasodilation affecting face & neck last for 2-5 min each more severe during night, disturb sleep.Mechanism:Estrogen deficiency - Reduces hypothalamine endorphine - Release more norepinephrine & serotonin - Leads to inappropriate heat loss mechanism.Hormonal Replacement Therapy:Benefit of HRT for Relief of Symptoms:Hot flashes: Highly effective; Usually required for a relatively short period of time, E.g., 1 to 3 years.Mood disturbance: Improve irritability & anxiety in many menopausal women; Relieve mild depressive symptoms.Hormone Replacement Therapy (HRT) is indicated in menopausal women to overcome the short-term and long-term consequences of estrogen deficiency. HRT can be administered orally (in pill form), vaginally (as a cream) or trans dermally (in patch form) because it replaces female hormones produced by the ovaries, hormone replacement therapy minimize menopause symptoms. Used before, during & after menopause.Who needs HRT:Symptomatic women who suffer from estrogen deficiency (Therapeutic); High risk cases for menopausal complications such as cardiovascular disease, osteoporosis, stroke, Alzheimer's disease, colonic cancer (prophylactic)Premature & surgical menopause; Gonadal dysgenesis in adolescents.Treatment of Hot Flashes: If mild symptoms & signs advised exercise.Initiate low dose HRT if:Moderate or severe symptoms: Non-hormonal treatments have failed; No interest in non-hormonal therapyTitrate estrogen dosage upward if needed.When estrogen can't be used, offer:SSRI or SNRI; Gabapentin, clonidine, alpha-methyldopa; MPA or MegesterolAttempt discontinuation after 1-2 yearsTREATMENT OPTIONS FOR MENOPAUSAL SYMPTOMS SHOWN TO HAVE CLINICAL BENEFITSSymptomsHormone TherapyNon-hormonal TreatmentsLifestyle ChangeVasomotorOral conjugated estrogens and MPAParoxetineSmoking cessationContinuous and sequentialVenlafaxineDecrease alcohol consumptionOral estradiol and norgestimatelGabapentinRelaxation techniquesTransdermal*. 17 beta estradiol-norethindrone acetate*. 17 beta estradiol - levonogestre*. 17 beta estradiol*. EstradiolClonidine Oral conjugated estrogen (unopposed)Oral 17 beta estradiolEstradiol vaginal ringVaginal atrophyEstradiol vaginal creamVaginal moisturizersSmoking cessationConjugated estrogen vaginal creamVaginal lubricantsregular sexual activityEstradiol vaginal ring OsteoporosisHT not recommended as first-line therapy for prevention or treatmentCalcium & Vitamin D supplementsWeight bearing exercisesBisphosphonatesSmoking cessationRaloxifeneDecrease alcohol consumptionCalcitonin Teriparatide", "cop": 1, "opa": "Hot flashes", "opb": "Carcinoma breast", "opc": "Endometriosis", "opd": "Uterine bleeding", "subject_name": "Gynaecology & Obstetrics", "topic_name": "Gynaecological Disorders in Obs.", "id": "e56947ba-242a-4a0a-b709-6c252206ebaa", "choice_type": "single"} {"question": "Not used in emergency contraceptions aEUR'", "exp": "LNG intrauterine device Levonorgestrel alone, 0.75 mg stat F/B another 0.75 mg 12 hrs later, taken within 72 hrs of unprotected intercourse is the method of choice .for emergency contraception Emergency contraception is also k/a postcoital morning after contraception. Basically the methods used interfere with the physiological events before implantation, for e.g. inhibition or delaying of ovulation or interference with postovulatory events necessary for implantation & longevity of the blastocyst. Agents used for emergency contraception are known as 'interceptives'. Methods of emergency contraception Drugs Dosage Time frame Preg. rate * High dose estrogens (not used now) - Ethinyl estradiol (EE) 2.5 mg BD x 5 days Within 72 hrs of coitus -L. 0.15% - Conjugated equine estrogen (CEE) 15 mg BD x 5 days 0-0.6% * Estrogen & progestin combination pill - Yuzpe method EE 50 lag + d, 1-NG 0.5 mg (Ovral) 2 tabs stat & 2 tabs after 12 hrs (Total dose EE 0.2 mg & LNG 2 mg) \" 0-2% - Low dose pills EE 30 lig + LNG 0 0.15mg (Ovral L, Mala-N etc.) 4 tabs stat & 4 tabs after 12 hors 0-2% * Levonorgestrel (LNG) alone (Ecee 2, i-pill, E-pill, unwanted-72, Plan-B) 0.75 mg stat FIB 0.75 mg after 12 hrs or 1.5 mg stat Preferably within 72 hrs but can be given upto 5 days 0-1% * Centchroman 30mg (Saheli) 2 tab BD x 1 day 400/800 x BD mg m 3 Within 5 days not known * Danazol days or 1200 mg BD x 2 days Within 72 hrs 0.8 - 1.7% * Mifepristone (RU 486) 600 mg/200 mg / 10 mg single dose Within 5 days to 27th day of cycle 1.3% * Copper HID (more effective than s steroids) ) To be inseed within 7 days < 1%", "cop": 1, "opa": "LNG intra uterine device", "opb": "Oral mifepristone", "opc": "Oral levonorgesterol", "opd": "Cu T intrauterine device", "subject_name": "Gynaecology & Obstetrics", "topic_name": null, "id": "359e8863-6e0d-4365-a9dd-85f749b94036", "choice_type": "single"} {"question": "Oral anticoagulants given to pregnant women can cause", "exp": ".", "cop": 2, "opa": "Long bones limb defect", "opb": "Craniofacial malformation", "opc": "CVS malformation", "opd": "Costo chondrodysplasia", "subject_name": "Gynaecology & Obstetrics", "topic_name": "All India exam", "id": "15e536c4-78b0-4a0b-9b39-078c3dda5875", "choice_type": "single"} {"question": "First line of Rx of osteoporosis in postmenopausal women", "exp": "REF : SHAW GYNECOLOGY REF REF :", "cop": 2, "opa": "Estrogen", "opb": "Bisphosphonates", "opc": "Raloxifene", "opd": "Ullipsistal", "subject_name": "Gynaecology & Obstetrics", "topic_name": "All India exam", "id": "9e167709-7f18-4e54-8330-562388fc58a5", "choice_type": "single"} {"question": "Probability to achieve a live birth within a single cycle is", "exp": "Probability to achieve a live birth within a single cycle is called Fecundity.", "cop": 2, "opa": "Fertility", "opb": "Fecundity", "opc": "Fecundability", "opd": "Sterility", "subject_name": "Gynaecology & Obstetrics", "topic_name": null, "id": "e07d7aa8-c31c-4958-bc8f-804949a0de85", "choice_type": "single"} {"question": "Congenital hea block develops in fetuses of women suffering from", "exp": "Congenital hea block develops almost exclusively in fetuses of women with antibodies to the SS-A or SS-B antigens. (Ref: William's Obstetrics; 25th edition)", "cop": 1, "opa": "SLE", "opb": "Epilepsy", "opc": "Rheumatoid ahritis", "opd": "Ankylosing spondylitis", "subject_name": "Gynaecology & Obstetrics", "topic_name": "All India exam", "id": "5e78c8f9-1868-402c-9cf3-96dcd40defc2", "choice_type": "single"} {"question": "Maximum dose of radiation exposure during pregnancy is", "exp": "The biological effects of x-rays are caused by an electrochemical reaction that can damage tissue. Embryo-fetal risks from low-dose diagnostic radiation appear to be minimal. Current evidence suggests that there are no increased risks for malformations, growth restriction, or aboion from a radiation dose of less than 0.05 Gy (5 rad). Reference: William&;s Obstetrics; 24th edition; Chapter 46; General Considerations and Maternal Evaluation", "cop": 4, "opa": "0.5 rad", "opb": "1 rad", "opc": "1.5 rad", "opd": "5 rad", "subject_name": "Gynaecology & Obstetrics", "topic_name": "Fetus", "id": "697ef2bb-2379-4103-a636-ef683b0c99f7", "choice_type": "single"} {"question": "Gland homologous to prostate in females is", "exp": "Masculine counterpas: Labia majora - scrotum clitoris - penis Labia minora - penile urethra and a pa of skin of penis in males Baholin's glands - bulb of penis Skene's glands - prostrate vestibular bulb - bulb of penis and corpus spongiosum DC Dutta's Textbook of Obstetrics 7th edition page no 1", "cop": 3, "opa": "Ganer's gland", "opb": "Baholin's gland", "opc": "Skene's gland", "opd": "Cowper's gland", "subject_name": "Gynaecology & Obstetrics", "topic_name": "Anatomy of the female genital tract", "id": "5fc35c20-0b79-4944-b8c2-6ffb0bb3ec9a", "choice_type": "single"} {"question": "Time taken for capacitation of sperms is", "exp": "Capacitation is the process of reconditioning of the sperm, making it more suitable for penetration into the ovum. \nIt takes about 6-8 hours and occurs in FGT chiefly in the tubes.", "cop": 3, "opa": "2-4 hours", "opb": "4-6 hours", "opc": "6-8 hours", "opd": "8-10 hours", "subject_name": "Gynaecology & Obstetrics", "topic_name": null, "id": "6e4d5d67-9cf8-4edc-8ce5-1f5282b94318", "choice_type": "single"} {"question": "Bleeding that follows paial or complete placental separation and dilation of the cervical os in first 20 weeks is termed as", "exp": "Incomplete Aboion: Bleeding that follows paial or complete placental separation and dilation of the cervical os is termed incomplete aboion. The fetus and the placenta may remain entirely within the uterus or paially extrude through the dilated os. Management options of incomplete aboion include curettage, medical aboion, or expectant management in clinically stable women.(Ref: William's Obstetrics; 25th edition)", "cop": 3, "opa": "Threatened aboion", "opb": "Complete aboion", "opc": "Incomplete aboion", "opd": "Preterm labor", "subject_name": "Gynaecology & Obstetrics", "topic_name": "All India exam", "id": "f70683c6-b7ae-4d6d-9001-28746e4a5615", "choice_type": "single"} {"question": "Secondary amenorrhea is defined as", "exp": "A woman who previously menstruated can develop secondary amenorrhea.Which is defined as absence of menstruation for three normal menstrual cycles. A woman with regular cycles and a delay of menses of even a week may warrant assessment with a pregnancy test.It is reasonable to evaluate a woman who has fewer than nine cycles per year.Reference: Berek and Novak's gynecology; 14th edition; Chapter 30; Amenorrhea", "cop": 2, "opa": "Absence of menstruation for one normal menstrual cycle", "opb": "Absence of menstruation for three normal menstrual cycles", "opc": "Absence of menstruation for six normal menstrual cycle", "opd": "Absence of menstruation for one year", "subject_name": "Gynaecology & Obstetrics", "topic_name": "Sexuality and intersexuality", "id": "7adfcf61-6b65-40ee-a530-337f48412d0c", "choice_type": "single"} {"question": "Most effective treatment for Hirsutism is", "exp": "Low-dose OCPs are most effective treatment for Hirsutism.", "cop": 2, "opa": "Metformin", "opb": "Lowdose OCPs", "opc": "Spironolactone", "opd": "Danazol", "subject_name": "Gynaecology & Obstetrics", "topic_name": null, "id": "442d8efd-bec7-4ce5-89fd-78ca61a223af", "choice_type": "single"} {"question": "Diagnosis of Down syndrome at 11 weeks in the best assessed by", "exp": "Ans is c i.e. Chorionic viii biopsy", "cop": 3, "opa": "Ultrasonography", "opb": "Aminocentesis", "opc": "Chorionic villous biopsy", "opd": "Doppler ultrasound", "subject_name": "Gynaecology & Obstetrics", "topic_name": null, "id": "52619108-909d-4c76-96ba-31377d237403", "choice_type": "single"} {"question": "Goniometer is used for", "exp": "Goniometer\n\nIt is used to measure baseline urethral angle and maximum strain angle of urethra with a cotton tip swab in place\nThe bladder forms an angle of 100 degree with posterior urethral wall which is also responsible for maintaining urinary continance", "cop": 4, "opa": "Amount o vaginal secretions", "opb": "To measure width of genital mhiatus", "opc": "Gonococcal colony count", "opd": "Urethrovesical Angle", "subject_name": "Gynaecology & Obstetrics", "topic_name": null, "id": "8f8ef90f-548b-4ca5-b388-1dff96ba7ce7", "choice_type": "single"} {"question": "Extrophy of the bladder is associated with", "exp": "Bladder extrophy is associated with bifid clitoris and labial fusion.", "cop": 2, "opa": "Congenital adrenal hyperplasia", "opb": "Bifid clitoris", "opc": "Rectal prolapse", "opd": "Imperforate anus", "subject_name": "Gynaecology & Obstetrics", "topic_name": null, "id": "9a9affe8-6635-4d1d-98e1-3fdca4751710", "choice_type": "single"} {"question": "Following vasectomy,", "exp": "Sterility following vasectomy is not immediate nor is its onset predictable. Thus another form of contraception is recommended until azoospermia is documented.", "cop": 3, "opa": "Sterility is achieved immediately", "opb": "Onset of sterility is predictable", "opc": "Semen has to be analyzed till 2 consecutive sperm counts are zero", "opd": "No need to use additional contraception after 1 month", "subject_name": "Gynaecology & Obstetrics", "topic_name": null, "id": "549822f1-8d0d-4b26-814d-1c411e81796e", "choice_type": "single"} {"question": "Smt. Gupta presented with bleeding PV and enlarged inguinal nodes. She was diagnosed as a case of endometrial carcinoma. State the stage", "exp": "Ans. is c i.e. Stage IV", "cop": 3, "opa": "Stage II", "opb": "Stage III", "opc": "Stage IV", "opd": "Stage I", "subject_name": "Gynaecology & Obstetrics", "topic_name": null, "id": "f0ddc8e5-3424-439f-91bc-7cc6c10bc271", "choice_type": "single"} {"question": "37 weeks primigravida presents with irregular uterine contractions, on examination the cervix is 1cm dilated and poorly effaced, management is", "exp": "The Uterine activity that causes discomfo, but that does not represent true labor, may develop at any time during pregnancy. False labor often stops spontaneously, or it may proceed rapidly into effective contractions. Sedation can be given and watch for some time. The patient may proceed into true labor or pain may subside.Ref: William&;s Obstetrics; 24th edition", "cop": 4, "opa": "Cesarean section", "opb": "Amniotomy", "opc": "Oxytocin drip", "opd": "Sedation and wait", "subject_name": "Gynaecology & Obstetrics", "topic_name": "General obstetrics", "id": "b5110194-ed4e-45ab-aa7c-143794ae5e7b", "choice_type": "single"} {"question": "Hysteroscopic resection can be done in submucosal myomas of size", "exp": "For submucosal myomas Grade 0-1, hysteroscopic resection (for <4 cm diameter) or abdominal myomectomy (for >4 cm diameter) is the recommended treatmentReference: NICE Guidelines", "cop": 2, "opa": "Any size can be done", "opb": "< 4 cm", "opc": "4-8 cm", "opd": "< 10 cm", "subject_name": "Gynaecology & Obstetrics", "topic_name": "Disorders of menstruation", "id": "e1f1cabf-509f-4013-91b5-2868ebfb0f4e", "choice_type": "single"} {"question": "Single pelvic ala is seen in", "exp": "Naegele's pelvis - Ala on one side is absent Robe's pelvis - Ala on both sides are absent Osteomalacic pelvis - the shape of inlet is triradiate Rachitic pelvis - shape of inlet is reniform", "cop": 2, "opa": "Robe's pelvis", "opb": "Naegele's pelvis", "opc": "Osteomalacia pelvis", "opd": "Rickets pelvis", "subject_name": "Gynaecology & Obstetrics", "topic_name": "Maternal Anatomy and Physiology", "id": "d84cdf94-ead5-4d2d-8605-4aa4903e8657", "choice_type": "single"} {"question": "Cause of secondary PPH is", "exp": "Causes of secondary PPH: Retained bits of cotyledon or membranes Infection and separation of slough over a deep cervicovaginal laceration Endometritis Hemorrhage from cesarean section wound Withdrawal bleeding following estrogen therapy for suppression of lactation Ref: Dutta Obs 9e pg 392.", "cop": 2, "opa": "Atonic uterus", "opb": "Retained placental bits", "opc": "Ruptured uterus", "opd": "Cervical laceration", "subject_name": "Gynaecology & Obstetrics", "topic_name": "General obstetrics", "id": "523b04e3-7030-4435-be04-2bc780a69adf", "choice_type": "single"} {"question": "Bacterial vaginosis is named as", "exp": "Bacterial vaginosis is also named as:\n\nHaemophilus vaginitis\nCorynebacterium vaginitis\nGardnerella vaginitis\nAnaerobic vaginitis\nNon - specific vaginitis", "cop": 2, "opa": "Clostridium vaginitis", "opb": "Haemophilus vaginitis", "opc": "Citrobacter vaginitis", "opd": "Staphylococcal vaginitis", "subject_name": "Gynaecology & Obstetrics", "topic_name": null, "id": "a493cad3-fe86-493f-ae82-d9ea846c3483", "choice_type": "single"} {"question": "Fibromyoma is seen in age group", "exp": "Ans. is'a'i.e., ReproductiveFibroids are the most common benign solid tumours in females.It is the most common Pelvic tumourMost commonly affects the reproductive age group.Most commonly seen in nulliparous common", "cop": 1, "opa": "Reproductive", "opb": "Early adolescence", "opc": "Latepostmenopausal", "opd": "Prepubeal", "subject_name": "Gynaecology & Obstetrics", "topic_name": null, "id": "add43c36-dee2-49fa-86bd-cc17e7ee1f99", "choice_type": "single"} {"question": "Cervical smear fixation is done by", "exp": "Ans. is 'a' i.e. Ethyl alcohol The fixative used to prevent air drying in case of Pap smear is ethyl alcohol.Ordinarily when a slide is prepared it is air dried but when a slide prepared for pap smear is air dried it gets damaged.''Thus while preparing Pap smear slide ethanol is used as a fixative to prevent air drying\".Procedure:-The techniques of collection of a pap smear may vary but the following is a common procedure.The patient should not have douched for at least 24 hours before the examination and should not be menstruating.The speculum is placed in the vagina after being lubricated with water only.With the cervix exposed, either a cotton-tipped applicator slightly dampened with saline solution or a specially designed plastic or wooden spatula is applied to the cervix and rotated 360 degrees to abrade the surface slightly and to pick up cells from the squamocolumnar area of the cervical os.Care should be taken to ascertain that endocervical cells are also obtained.If this is not accomplished with the spatula, a cotton tipped applicator or small brush is now available for this purpose, it can be inserted into the cervical canal of most of the women and rotated picking up cervical cells more efficiently.These 2 specimens may be mixed or put on the slide, separately according to the preference of the examiner.A preservative is applied immediately to prevent air drying, which will compromise the interpretation.The preservative or the fixative used is 95% ethyl alcohol (Note: To prevent the air drying, slide is transported in the bottle containing the fixative).", "cop": 1, "opa": "Ethyl alcohol", "opb": "Acetone", "opc": "Xyline", "opd": "Formalin", "subject_name": "Gynaecology & Obstetrics", "topic_name": "Staging, Investigation and Prognostic Factors", "id": "133df9aa-5fe8-4dbe-9b65-a42b134db6be", "choice_type": "single"} {"question": "A 35 yr old P 3+0 is observed to have CIN grade III on colposcopic biopsy. Best treatment will be", "exp": "LEEP Repeat All India 2009 \"Although CIN can be treated with a variety of techniques, the preferred treatment for CIN 2 and 3 has become LEE?\" - Novak's Gynecology p582 Though the patient is 35 yrs old and has completed her family, still hysterectomy won't be treatment of choice as-\"Hysterectomy is currently considered too radical .for treatment of CIN\"Novak's Gynecology p.585 Following are some situations in which hysterectomy remains a valid and appropriate method of treatment for CIN Microinvasion CIN 3 at limits of conizution specimen in selected patients Poor compliance with follow-up Other gynecologic problems requiring hysterectomy, such as fibroids, prolapse, endometriosis, and pelvic inflammatory disease Cervical intraepithelial neoplasia(CIN) Invasive squamous cell cervical cancers are preceded by a long phase of preinvasive disease, collectively referred to as cervical intraepithelial neoplasia (CIN). Histopathologically a pa or the full thickness of cervical squamous epithelium is replaced by cells showing varying degree of dysplasia, with intact basement membrane. CIN may be suspected through cytological examination using the Pap smear test or through colposcopic examination. Cervical cytology is the most efficacious and cost-effective method for cancer screening. Final diagnosis of CIN is established by the histopathological examination of a cervical punch biopsy or excision specimen. Additionally, human papilloma virus (HPV) testing can be performed in order to better triage women with early cytologic changes. Cervical Cytology Screening (American College of Obstetricians and Gynecologists Guideline) Initial screening Age 21 or 3 y after vaginal sex Interval Every year Every 2-3 y after age 30 with 3 consecutive normals Discontinue No upper limit of age Comparison of Cytology Classification Systems (in simplified form) Bathesda CIN llysplasia Limit of histologic changes *LSIL CIN 1 Mild Basal 1/3rd of sq. epithelium HSIL ON 2 Moderate Basal 1/2 to 2/3rd CIN3 Severe Whole thickness except one or two superficial layers CIS Whole thickness LSIL - Low grade squamous intraepithelial lesion HSIL - High grade squamous intraepithelial lesion CIS - Carcinoma in situ *LSIL incorporates HPV changes (koilocytotic atypia) along with CIN I. Role of HPV HPV infection is found in approx. 90% cases of intraepithelial neoplasia. Type H&18 are most commonly associated. HPV-18 is more specific than HPV-16 for invasive tumors. In most women, the HPV infection clears in 9 to 15 months. Only a minority of women exposed to HPV develop persistent infection that may progress to CIN. Type-16 is the most common HPV type found in women with normal cytology. Treatment CIN 1 Spontaneous regression of CIN 1 is seen in 60% to 85% of cases, typically within 2yrs. So patients who have biopsy diagnoses of C1N 1 are kept under observation with: Pap testing performed at 6 and 12 months Or HPV DNA testing at 12 months After two negative test results or a single negative HPV DNA test, annual screening may be resumed. Women with persistent CIN 1 after 24 months should be treated with a local ablative method. CIN 2 and3 CIN 2&3 carries a much higher probability of progressing to invasive cancer. All CIN 2 and 3 lesions require t reatment. LEEP ( loop electrosurgical excision procedure) is the preferred treatment for CIN 2 and 3. Because all therapeutic modalities carry an inherent recurrence rate of upto 10%. cytologic follow-up at about 3-month intervals for 1 year is necessary. Cryotherapy Considered acceptable therapy when the following criteria are met: - Cervical intraepithelial neoplasia, grade 1 to 2 - Small lesion Ectocervical location only - Negative endocervical sample - No endocervical gland involvement on biopsy Laser Ablation It has been used effectively for the treatment of CIN .But because of the expense of the equipment as well as necessity for special training, laser ablation has fallen out of or. Laser has been widely replaced by LEEP. Laser Excisional Conization Rather than using laser for vaporization leading to ablation, it can be used to excise a conization specimen. The ease of LEEP conization has significantly reduced the indications of laser conization. Loop electrosurgical excision( LEEP) LEEP, variably known as simply loop excision or LLETZ (large loop excision of the transformation zone), is a valuable tool for the diagnosis and treatment of CIN. It uses low-voltage, high-frequency, thin wire loop electrodes to perform a targeted removal of a cervical lesion, an excision of the transformation zone, or a cervical conization. This technique can be used in the outpatient setting Cold knife conization (scalpel) Conization is both a diagnostic and therapeutic procedure and has the advantage over ablative therapies of providing tissue for fuher evaluation to rule out invasive cancer. Conization is indicated for CIN 2&3 in following conditions: - Limits of the lesion cannot be visualized with colposcopy. - The squat-no-columnar junction (SCE) is not seen at colposcopy. - Endocervical curettage (ECC) histologic findings are positive for CIN 2 or CIN 3. - There is a substantial lack of correlation between cytology, biopsy, and colposcopy results. - Microinvasion is suspected based on biopsy, colposcopy, or cytology results. - The colposcopist is unable to rule out invasive cancer.", "cop": 3, "opa": "Cryosurgery", "opb": "Conization", "opc": "LEEP", "opd": "Hysterectomy", "subject_name": "Gynaecology & Obstetrics", "topic_name": null, "id": "c7644e3c-19eb-4cd4-8cee-a8e1cb38a730", "choice_type": "single"} {"question": "Regarding PCOS and hyperinsulinaemia", "exp": "In PCOS\n\nHyperinsulinemia is seen in 50-70% cases of PCOS\nInsulin induces LH to cause thecal hyperplasia and secrete androgens\nHyperandrogenism lowers the level of hepatic sex hormone binding globulin\nMetformin treats the root causes, rectifies endocrine and metabolic function, improves fertility rate, used as insulin sensitiser\nIt reduces insulin level, delays glucose absorption, liver production of glucose and improves peripheral resistance\nMetformin do not cause hypoglycemia in a normoglycemic women", "cop": 2, "opa": "Hyperinsulinaemia is observed in about 40% to 80% of women with PCOS", "opb": "Metformin has many other health benefits", "opc": "Metformin causes hypoglycaemia in normoglycaemic women", "opd": "Hyperinsulinaemia stimulates hepatic synthesis of SHBG", "subject_name": "Gynaecology & Obstetrics", "topic_name": null, "id": "46fbb9c0-52f3-4599-b795-c12c403b1e23", "choice_type": "single"} {"question": "Best investigation to establish the diagnosis of endometriosis is", "exp": "Ans. is a i.e. Laparoscopy Diagnostic laparoscopy is the gold standard for diagnosing endometriosis. Typical lesion : Powder burn / gun shot lesions (black, dark brown or bluish cysts with old hemorrhage surrounded by variable degree of fibrosis). Other non typical findings could be : - Red implantsdeg - Serous or clear vesicles White plaques / scarring] - Yellowish brown discolouration of peritoneumdeg Sub ovarian adhesionsdeg. Histological confirmation of laparoscopic impression is essential for diagnosis of endometriosis. Danazol By its multiple effects, produces a high androgen & low estrogen environment that does not suppo the growth of endometriosis. It inhibits pituitary gonadotropins & produces pseudo menopause like state. Danazol is not more effective than other available medications to treat endometriosis. Due to its androgenic propey has significant side effect like weight gain, fluid retention, acne oily skin, hirsutism, hot flashes, atrophic vaginitis, reduced breast-size. reduced libido, fatigue, muscle cramps. Deepening of voice is an rreversible side effect. Gestrinone It is a 19 noesto sterone derivative. It acts centrally & peripherally to increase free testosterone, reduce SHBG. serum estradiol & LH which inturn obliterates LH & FSH surge. Its actions are similar to Danazol but is longer acting. So, can be given twice weekly. Side effects - similar to Danazol but less intense. Gonadotropin releasing hormone agonist When given continously suppress pituitary gonadotropin. They are inactive orally; & have to be given intramuscular, intra nasal or subcutaneous. Side effects due to hypo estrogen are hot-flashes, vaginal dryness, reduced libido and osteoporosis. GnRH should not be prescribed to girls who have not yet attained their maximum bone density (i.e., in age < 16 years).", "cop": 1, "opa": "Laparoscopy", "opb": "USG", "opc": "X-ray pelvis", "opd": "CT Scan", "subject_name": "Gynaecology & Obstetrics", "topic_name": null, "id": "002b6684-f29d-4be5-94cc-156d8a7fbe67", "choice_type": "single"} {"question": "The highest rate of transmission of toxoplasmosis in pregnancy is", "exp": "The incidence and severity of fetal toxoplasmosis infection depend on gestational age at the time of maternal infection. Risks for fetal infection increase with pregnancy duration. A meta-analysis estimated the risk to be 15 percent at 13 weeks, 44 percent at 26 weeks, and 71 percent at 36 weeks (SYROCOT Study Group, 2007). Conversely, the severity of fetal infection is much greater in early pregnancy, and these fetuses are much more likely to have clinical findings of infection.Ref: William&;s obstetrics; 24th edition; Chapter 64; Infectious Diseases; Protozoal infections", "cop": 2, "opa": "Puerperium", "opb": "3rd trimester", "opc": "2nd trimester", "opd": "1st trimester", "subject_name": "Gynaecology & Obstetrics", "topic_name": "Infections of the genital tract", "id": "dfe6afca-490e-47c4-ac76-48ee9d65f8cd", "choice_type": "single"} {"question": "Predisposing factor to carcinoma endometrium is", "exp": "i.e. (D M): (416-18-Shaw's 15th)PREDISPOSING FACTORS IN CARCINOMA ENDOMETRIUM* Un opposed and unsupervised administration of hormone replacement therapy (HRT) after menopause predispose to endometrial hyperplasia and cancer. Fortunately the malignancy is well-differentiated with good prognosis* Chronic non ovulatory cycle as seen in DUB* In some families, a strong familial predispotion is noticed. This may be due to genetic or dietic habits such as animal protein and fat. The oestrone is derived by peripherlal aromatization in the fat tissue from androstenedione and contributes to high level of oestrogen. Women with familial Lynch II syndrome suffering from anorectal and breast cancer are also likely to suffer from endometrial cancer* Tamoxifen given to women with breast cancer increases the risk of endometrial hyperplasia and cancer to 2- 3 folds* Combined oral hormonal pills have protective effect and reduce its risk by 40-50% and also adding progestogens for 12 days each cycle to oestrogen in HRT reduces its risks to 20%**** Obesity, hypertension and diabetes characterize this cancer in 30%* Infertile women and women with PCOS on account of non-ovulation have high oestrogen. They develop endometrial hyperplasia and endometrial cancer more than normal women. The uterine fibroid is associated with endometrial cancer in 3% after the age of 40 years* 15% women with endometrial cancer have feminizing ovarian tumour at the time of diagnosisOCP|| |Tumours associated Provides protection1. Cervical cancer 1. Ovarian tumours2. Hepatic adenoma 2. Uterine tumours3. Pituitary adenoma 3. Benign breast disease", "cop": 3, "opa": "Multiparity", "opb": "Early marriage", "opc": "DM", "opd": "OCP", "subject_name": "Gynaecology & Obstetrics", "topic_name": "Miscellaneous (Gynae)", "id": "77150eb9-6e3e-4db5-a8c0-eb46990b7770", "choice_type": "single"} {"question": "Mrs. X, 20years with 30weeks of pregnancy, her parity is", "exp": "Nullipara: a woman who has never completed a pregnancy beyond 20 weeks' gestation. She may not have been pregnant or may have had a spontaneous or elective aboion(s) or an ectopic pregnancy.Primipara: a woman who has been delivered only once of a fetus or fetuses born alive or dead with an estimated length of gestation of 20 or more weeks.Multipara: a woman who has completed two or more pregnancies to 20 weeks' gestation or more. Parity is determined by the number of pregnancies delivered reaching 20 weeks. It is not increased to a higher number if multiples are delivered in a given pregnancy. Moreover, stillbih does not lower this number.Reference: William's obstetrics; 25th edition", "cop": 1, "opa": "Nulliparous", "opb": "P1", "opc": "P2", "opd": "P3", "subject_name": "Gynaecology & Obstetrics", "topic_name": "General obstetrics", "id": "2e9a12b0-18de-482e-9c52-33ae1bedf0ec", "choice_type": "single"} {"question": "HIV transmission to the fetus is maximum during", "exp": "Ans. d (Vaginal Delivery) (Ref. Harrison 18th/ p 38, 1082)HUMAN IMMUNODEFICIENCY VIRUS (HIV)# Exposures, which increase the risk of mother-to-child transmission, include -- Vaginal delivery, Preterm delivery; Maternal bleeding, and Trauma to the fetal skin,# Additional factors that may increase the risk of transmission include:- Recent infection with high maternal viral load,- Low maternal CD4 T cell count, prolonged labor,- Prolonged length of membrane rupture, and- The presence of other genital tract infections, such as syphilis or herpes.# Maternal transmission to the fetus occurs most commonly in the perinatal period.# In the absence of prophylactic antiretroviral therapy to the mother during pregnancy, labor, and delivery, and to the fetus following birth, the probability of transmission of HIV from mother to infant/fetus ranges from 15 to 25% in industrialized countries and from 25 to 35% in developing countries.# HIV transmission from mother to the baby can be minimized by:A. Antepartum Zidovudine therapy to mother and to the neonate after birthB. Maternal Vitamin A therapyD. Avoidance of breast feeding# The rate of MOTHER-TO-CHILD TRANSMISSION IS APPROACHING 1 % OR LESS IN PREGNANT women who are receiving combination antiretroviral therapy for their HIV infection.WHO guidelines for PMTCT drug regimens in resource-limited settings:PREGNANCYLABOURAFTER BIRTH: MOTHERAFTER BIRTH: INFANT RecommendedAZT after 28 wkssingle dose nevirapine; AZT + 3TCAZT+3TC for seven daysSingle dose nevirapine; AZT for seven daysAlternative (higher risk of drug resistance)AZT after 28 wksSingle dose nevirapine-Single dose nevirapine; AZT for seven daysMinimum (less effective)-Single dose nevirapine; AZT + 3TCAZT+3TC for seven daysSingle dose nevirapineMinimum (less effective; higher risk of drug resistance)-Single dose nevirapine-Single dose nevirapineTreatment# The majority of cases of mother-to-child (vertical) transmission of HIV-1 occur during the intrapartum period.# Mechanisms of vertical transmission include infection after rupture of the membranes and direct contact of the fetus with infected secretions or blood from the maternal genital tract.# In women with HIV infection who are not receiving antiretroviral therapy, the rate of vertical transmission is approximately 25%.# Cesarean section and treatment with zidovudine, administered both before and during delivery, decrease the rate of vertical transmission.# In a meta-analysis, zidovudine treatment of both the mother during the prenatal and intrapartum periods and the neonate at birth reduced the risk of vertical transmission to 7.3%.# The combination of elective cesarean section plus zidovudine treatment reduced the risk of vertical transmission to 2%.CYTOMEGALOVIRUS INFECTION# The most common cause of congenital viral infection in the United States is cytomegalovirus (CMV).# Severe CMV disease in the newborn is characterized most often by petechiae, hepatosplenomegaly, and jaundice.# Chorioretinitis (\"tomato sause & cheese\" appearance), microcephaly, intracranial calcifications, hepatitis, hemolytic anemia, and purpura may also develop.# Central nervous system involvement, resulting in the development of psychomotor, ocular, auditory, and dental abnormalities over time, has been described.RUBELLA# First trimester rubella carries a high risk of fetal anomalies, though the risk decreases significantly later.# Congenital rubella may be diagnosed by PUBS with the detection of IgM antibodies in fetal blood.HERPESVIRUS# The acquisition of genital herpes during pregnancy is associated with spontaneous abortion, prematurity, and congenital and neonatal herpes.# Infection occurs equally in all three trimesters.# In women who acquired genital herpes shortly before delivery, the risk of transmission is high.# The risk of active genital herpes lesions at term can be reduced by prescribing acyclovir for the last 4 weeks of pregnancy to women who have had their first episode of genital herpes during the pregnancy.# It is recommended that pregnant women with active genital herpes lesions at the time of presentation in labor be delivered by cesarean section.PARVOVIRUS (human parvovirus B19)# It rarely causes sequelae.# But susceptible women infected during pregnancy may be at risk for fetal hydrops secondary to erythroid aplasia and profound anemia.TOXOPLASMOSIS# The diagnosis of congenital toxoplasmosis is possible through sampling of fetal umbilical blood.# If there is no evidence of placental/fetal infection, single-drug treatment with spiramycin is recommended.# Triple-drug therapy with spiramycin, pyrimethamine, and sulfa is recommended if there is evidence of fetal infection and the woman does not wish to terminate the pregnancy or cannot terminate it because of advanced gestational age.", "cop": 4, "opa": "1st trimester", "opb": "2nd trimester", "opc": "3rd trimester", "opd": "Vaginal delivery", "subject_name": "Gynaecology & Obstetrics", "topic_name": "Miscellaneous (Gynae)", "id": "193bdfb1-b10a-4315-a0fb-866b9ee38c1b", "choice_type": "single"} {"question": "The shoest diameter of fetal head is", "exp": "1. The occipitofrontal (11.5 cm), which follows a line extending from a point just above the root of the nose to the most prominent poion of the occipital bone. 2. The biparietal (9.5 cm), the greatest transverse diameter of the head, which extends from one parietal boss to the other. 3. The bitemporal (8.0 cm), which is the greatest distance between the two temporal sutures. 4. The occipitomental (12.5 cm), which extends from the chin to the most prominent poion of the occiput. 5. The suboccipitobregmatic (9.5 cm), which follows a line drawn from the middle of the large fontanel to the undersurface of the occipital bone where it joins the neck.Reference: William&;s Obstetrics; 24th edition; Chapter 7; Embryogenesis and Fetal Morphological Development", "cop": 4, "opa": "Biparietal diameter", "opb": "Suboccipito-Bregmatic diameter", "opc": "Suboccipito-Frontal diameter", "opd": "Bitemporal diameter", "subject_name": "Gynaecology & Obstetrics", "topic_name": "General obstetrics", "id": "6a09a527-cbcc-4afb-8e2d-d082b7a6cf40", "choice_type": "single"} {"question": "Most useful investigation for VVF", "exp": "Big fistula Fistula tract is available Obvious escape of urine Tiny fistula Dye test Knee chest position: Escape of bubbles on coughing Three-swab test Gold Standard Cystoscopy Ref: D.C.DUTTA&;S TEXTBOOK OF GYNAECOLOGY; 6th edition; Pg no: 420", "cop": 2, "opa": "Three swab test", "opb": "Cystoscopy", "opc": "Urine culture", "opd": "IVP", "subject_name": "Gynaecology & Obstetrics", "topic_name": "Urogynecology", "id": "f673a05b-cbcd-40c7-94c1-fe216033617b", "choice_type": "single"} {"question": "Use of valproate during pregnancy may result in", "exp": "Valproate crosses the placenta and has been repoed to have caused teratogenic effects, including neural tube defects (anencephaly, meningomyelocele, and spina bifida) in the fetus when the mother received valproate during the first trimester of pregnancy.", "cop": 4, "opa": "Mental retardation", "opb": "Respiratory depression", "opc": "Hydantoin syndrome", "opd": "Neural tube defect", "subject_name": "Gynaecology & Obstetrics", "topic_name": "Fetus", "id": "089d4add-3686-44c0-bf7f-5b6cc7f2e07c", "choice_type": "single"} {"question": "Peak value of Karyopyknotic index is reached on", "exp": "Normally, peak value of KPI is reached on the day of ovulation.", "cop": 2, "opa": "Proliferative phase", "opb": "Day of ovulation", "opc": "Secretory phase", "opd": "Menopause", "subject_name": "Gynaecology & Obstetrics", "topic_name": null, "id": "60264740-9f8c-4335-938d-66c1be4c7cba", "choice_type": "single"} {"question": "The progesterone of choice for emergency contraception is aEUR'", "exp": "Levonorgestrel [Ref: Dutra Obstetrics 6/e, p 549-550; S.K. Chaudhuri Feility Control 6/e, p 190-199; Novak's Gynaecology I4/e, p 283-385. Repeat from Nov 09 Q no.154", "cop": 3, "opa": "Norethisterone", "opb": "Medroxyprogesteronacetate", "opc": "Levonorgestrel", "opd": "Desogestrel", "subject_name": "Gynaecology & Obstetrics", "topic_name": null, "id": "911b7eb5-99e0-4059-9fa8-edf95824bd06", "choice_type": "single"} {"question": "According to FIGO staging, Women diagnosed with choriocarcinoma with metastasis to lungs will be staged under", "exp": "Anatomical stagingStage IGTN confined to the uterusStage IIGTN extends outside of the uterus but is limited to the genital structures Stage IIIGTN extends to the lungs, with or without genital tract involvementStage IVDistant metastasisReference: William's Obstetrics; 25th edition, chapter 20; Gestational trophoblastic diseases", "cop": 3, "opa": "Stage I", "opb": "Stage II", "opc": "Stage III", "opd": "Stage IV", "subject_name": "Gynaecology & Obstetrics", "topic_name": "General obstetrics", "id": "ab4e587c-0414-4e18-954a-bba31b92dc9b", "choice_type": "single"} {"question": "Best reproductive out come among mullerian duct anomalies is", "exp": "Best reproductive out come is seen is arcuate uterus > Didelphic uterus > Bicornuate uterus.", "cop": 3, "opa": "Bicornuate uterus > Arcuate uterus > Didelphic uterus", "opb": "Didelphic uterus > Bicornuate uterus > Arcuate uterus", "opc": "Arcuate uterus > Didelphic uterus > Bicornuate uterus", "opd": "Unicornuate uterus > Arcuate uterus > Didelphic uterus", "subject_name": "Gynaecology & Obstetrics", "topic_name": null, "id": "f030b36d-0d1a-47c8-a5ee-1241d108619b", "choice_type": "single"} {"question": "Endocervical polyps causing abnormal menstrual bleeding are seen most commonly in", "exp": "Endocervical polyps are seen more commonly in multiparous women.", "cop": 3, "opa": "Nulliparous women", "opb": "Pre-pubertal females", "opc": "Multiparous women", "opd": "Adolescents", "subject_name": "Gynaecology & Obstetrics", "topic_name": null, "id": "7bf51c58-b033-451c-80d4-e5d89e86f077", "choice_type": "single"} {"question": "Most common cause of postmenopausal bleeding is", "exp": "Causes of Postmenopausal Uterine BleedingCausesPercentageEndometrial atrophy60-80Estrogen replacement therapy15-25Endometrial polyps2-12Endometrial hyperplasia5-10Endometrial cancer10Reference: Novak's gynaecology; 14th edition; Chapter 35; Uterine cancer", "cop": 3, "opa": "Endometrial cancer", "opb": "Endometrial polyps", "opc": "Endometrial atrophy", "opd": "Endometrial hyperplasia", "subject_name": "Gynaecology & Obstetrics", "topic_name": "Gynaecological oncology", "id": "b48f2d88-46c5-414e-bbad-96b2a549959c", "choice_type": "single"} {"question": "A patient 45 years of age, non diabetic, presents with chronic pelvic pain of 1 year duration. She also complains of frequency, urgency and a sense of incomplete evacuation since 1 year without any significant finding on her past ultrasounds, urine examination and urine and high vaginal swab cultures. On pelvic examination there is no significant vaginal discharge. Cystoscopy is normal. Most probable diagnosis is", "exp": "Urethral syndrome Urethral syndrome is defined as a symptom complex including dysuria, frequency, urgency, suprapubic discomfo , post void fullness and dyspareunia in the absence of any abnormality of urethra or bladder. Cause is unceain and is attributed to subclinical infection, urethral obstruction, psychogenic and allergic factors. It is a diagnosis of exclusion, after ruling out UTI, malignancy, vulvovaginitis. Cystoscopy to rule out urethral diveiculum, stones and malignancy. Management: multidisciplinary approach, antibiotics for sterile pyuria. Post menopausal women may benefit from local estrogen. Physical therapy, cognitive-behavioral therapy and psycological suppo. Asymptomatic bacteriuria is ruled out with normal urine culture, Cystitis with normal urine r/m, c/s, and USG. Vulvovaginitis is ruled out by the given symptoms and normal local examination", "cop": 1, "opa": "Urethral syndrome", "opb": "Asymptomatic bacteriuria", "opc": "Vulvovaginitis", "opd": "Cystitis", "subject_name": "Gynaecology & Obstetrics", "topic_name": "Genitourinary Fistula", "id": "950d44a9-0d6c-4434-9782-afcaa1aa8bb9", "choice_type": "single"} {"question": "Cryptomenorrhea is seen in", "exp": "Cryptomenorrhea means monthly subjective symptoms of menstruation without the outflow of blood.In women with cryptomenorrhoea presenting as primary amenorrhoea, the common cause is an intact hymen or vaginal septum. A history of cyclic abdominal colickypain, retention of urine, the presence of a palpable abdominal lump and the visualization of a tense bluish bulging membrane on separation of the labia enables the diagnosis.Ultrasound scan of the pelvis confirms it. A simple cruciate incision of the hymen permits free drainage of the collected menstrual blood and leads to normal reproductive function.Ref: Shaw&;s Textbook of Gynecology; 16th edition; Page no: 324", "cop": 3, "opa": "Asherman's syndrome", "opb": "Testicular feminizing syndrome", "opc": "Imperforate hymen", "opd": "Uterine agenesis", "subject_name": "Gynaecology & Obstetrics", "topic_name": "Disorders of menstruation", "id": "81f29c5b-3d38-4009-a546-a7bc35861fe7", "choice_type": "single"} {"question": "The iron content in 100 gm of breast milk is", "exp": "(100 ng) Ref: 149, 455-D, 398-PK 17thPercentage composition of colostrums and breast milk ProteinFatCarbohydrateWaterColostrum8.62.33.286Breast milk1.23.27.587 Composition of Human and Cow's milk Lactose g/ 100mlFat g/ 100mlProtein g/100mlSodium mmol/LWaterCalories kcal/mHuman73.51.278975Cows4.53.53.4228867 MineralsBreast milk grammes per litreCow's milk grammes per litreCa0.331P0.151-Fe0.4 to 1.5 mg0.3 to 0.5 mg", "cop": 4, "opa": "1 mg", "opb": "10mg", "opc": "50 mg", "opd": "100 mg", "subject_name": "Gynaecology & Obstetrics", "topic_name": "Miscellaneous (Gynae)", "id": "d72cc4c1-8ccf-465a-bce9-a74337c88172", "choice_type": "single"} {"question": "In ovarian cycle increaed levels of LH are due to", "exp": "Luteinising Hormone \n\nLH is a water-soluble glycoprotein of high molecular weight secreted by beta cells of the anterior pituitary gland\nLH surge initiated by estrogen lasts for 48 h. LH level doubles up in 2 hrs and peak plateaus for 14 hours before declining.\nMinimum of 75 ng/ml is required for ovulation", "cop": 4, "opa": "Increased Progesterone", "opb": "Increased FSH", "opc": "Increased Androgens", "opd": "Increased Estrogen", "subject_name": "Gynaecology & Obstetrics", "topic_name": null, "id": "ec01fe30-c37d-4f4e-9789-b0aed9e0e6f6", "choice_type": "single"} {"question": "Earliest pregnancy is detected by", "exp": "A. i.e. (b hCG) (67-69- Dutta 6th)Immunological Test for Diagnosis of Pregnancy* Immunological Tests (Urine)Positive on* Agglutination inhibition test (Latex test)2 days after missed period* Direct latex agglutination test2-3 days after missed period* Card testOn the first day of the missed periodELISA**5 days before the first missed period* Radioimmunoassay (b-subunit)** 25th day of cycle* Immuno- radiometric assay (IRMA)8 days after conceptionULTRASONOGRAPHY - Intradecidual gestational sac (GS) is identified as early as 29 to 35 days of gestation* Gestational sac and yolk sac - 5 menstrual weeks* Fetal pole and cardiac activity - 6 weeks* Embryonic movements - 7 weeks* Fetal gestational age is best determined by measuring the (RL between 7 and 12 weeks (variation + 5 days)* Doppler effect of ultrasound can pick up the fetal heart rate reliably by 10th week.*** Active fetal movements can be felt at intervals by placing the hand over the uterus as early as 20th week**.It not only gives positive evidence ofpregnancy but of a live fetus* b- hCG- is marker of Dysgerminoma*** Inhibin is marker for 'Granulosa cell tumour*** The secreening test for gestational diabetes mellitus has the highest sensitivity is - 50 gram glucose challenge test**", "cop": 1, "opa": "b-HCG", "opb": "Ultrasound", "opc": "Foetal movements", "opd": "Foetal heart sound", "subject_name": "Gynaecology & Obstetrics", "topic_name": "Miscellaneous (Gynae)", "id": "715a2326-ddc4-4e5e-b369-8e1544a36315", "choice_type": "single"} {"question": "Secondary oocyte consists of", "exp": "Ans. d (23 X)Secondary oocyte consist of 23 X chromosomes.The chromosomal complement of a primary oocyte is 46 XX and that of secondary oocyte is 23 X.Sequence of maturation of ovarian primordial follicle after primary follicle stage:Primordial follicle(Single layered ovarian follicle)Spermatogonium (diploid, 2N) ||Primary Spermatocyte (diploid, 2N)Primordial follicle ||Secondary Spermatocyte (haploid, N)Secondary follicle (Antral follicle) ||Spermatid (haploid, N)(Spermatogenesis begins with Spermatogonia.Full development takes 2 months.It occurs in seminiferous tubules)Graffian follicle|Corpus luteum| Corpus albicans # While majority of oogonia divide, some enter into prophase of first meiotic division and are primary oocytes. These are surrounded by flat cells, which are called primordial follicles, and are present in cortex of the ovary.# At birth, there is no more mitotic division and all the oogonia are replaced by primary oocytes, which have finished prophase of first meiotic division and remain in resting phase between prophase and metaphase.# The first stage of maturation occurs with full maturation of ovarian follicle just prior to ovulation but the final maturation occurs only after fertilization.# The primary oocyte undergoes first meiotic division giving rise to secondary oocyte and one polar body.# Secondary oocyte has haploid number of chromosomes.# Ovulation occurs soon after formation of secondary oocyte.# The secondary oocyte completes the second meiotic division only after fertilization by the sperm in the fallopian tube.", "cop": 4, "opa": "46 XY", "opb": "46 XX", "opc": "23 Y", "opd": "23 X", "subject_name": "Gynaecology & Obstetrics", "topic_name": "The Ovarian Cycle", "id": "8ba2b730-39d7-41bc-af72-14a9097039e8", "choice_type": "single"} {"question": "Longest diameter of fetal skull is", "exp": "Mentoveical is the longest anteroposterior diameter of the fetal head measuring approximately 13.5cms It is the diameter in brow presentation", "cop": 4, "opa": "Biparietal", "opb": "Bitemporal", "opc": "Occipito frontal", "opd": "Mentoveical", "subject_name": "Gynaecology & Obstetrics", "topic_name": "General obstetrics", "id": "68d20d20-b1ba-43f5-95a9-b30070451724", "choice_type": "single"} {"question": "The peak level of chorionic gonadotropin in normal pregnancy occurs", "exp": "Peak maternal levels of chorionic gonadotropin in normal pregnancy occurs between 60th and 80th days after menses and is around 100,000 mIU/mL. Before 5 weeks of gestation, hCG is expressed in both cytotrophoblast and syncytiotrophoblast. Later during gestation, when maternal serum levels peak, hCG is produced almost solely in syncytiotrophoblasts. Plasma levels of hCG begin to decline at 10 to 12 weeks and a nadir is reached about 16 weeks. Plasma levels are maintained at this lower level for the remainder of pregnancy. Human Chorionic Gonadotropin is a glycoprotein with biological activity similar to luteinizing hormone. It has a molecular weight of 36,000 to 40,000 Da. It is structurally related to LH, FSH and TSH. Ref: Cunningham F.G., Leveno K.J., Bloom S.L., Hauth J.C., Rouse D.J., Spong C.Y. (2010). Chapter 3. Implantation, Embryogenesis, and Placental Development. In F.G. Cunningham, K.J. Leveno, S.L. Bloom, J.C. Hauth, D.J. Rouse, C.Y. Spong (Eds), Williams Obstetrics, 23e.", "cop": 2, "opa": "30-40 days", "opb": "60-70 days", "opc": "10-20 days", "opd": "100-110 days", "subject_name": "Gynaecology & Obstetrics", "topic_name": null, "id": "1a93b757-b031-4b1d-b2d8-c28604e598d9", "choice_type": "single"} {"question": "Kalavati, a 29 year old nulliparous women complains of severe menorrhagia and lower abdominal pain since 3 months. On examination there is a 14 wks size uterus with fundal fibroid. The treatment of choice is.", "exp": "Ans. is 'a' is i.e.,. Myomectomy (Ref.: Robert Shaw, Patrice Soutter, 3/e, p 487, 488, Shows, 73/e, p 346 (72/e, p 283;;Guidelines for the management of MvomaAbout medical t/t of myoma with GnRH analogues * For this we have consulted many books and consultants in gynecology. Allof them except 'Shaws gynae' were of the opinion that the medical t/t is not used as a curative therapy. Though it can reduce the size of the fibroid by upto 50%, the effect persists only for the duration of t/t. Once its stopped, the fibroid rapidly regrows to its pretreatment size. So It's only used in patients who are keen to avoid surgery or those with contraindications to surgery.Main aim of medical management in myomaRelief of symptomsReduction in fibroid size and vascularity preoperatively.", "cop": 1, "opa": "Myomectomy", "opb": "GnRh analogues", "opc": "Hysterectomy", "opd": "Wait and watch", "subject_name": "Gynaecology & Obstetrics", "topic_name": "Treatment and Recurrence", "id": "de71d500-bd1a-47ae-a93a-6193b4e6420a", "choice_type": "single"} {"question": "Drug contraindicated in patient with rheumatic heart disease in PPH is", "exp": "Ans. is 'b' i.e. Methyl ergometrine Cardiac patients should not receive ergotamine because these agents cause significant vasoconstriction and elevation of blood pressure which can be deleterious to the patientAlso know,In healthy women, an intravenous bolus of 10 units of oxytocin caused a transient but marked fall in arterial blood pressure that was followed by an abrupt increase in cardiac output.These hemodynamic changes could be dangerous to women already hypovolemic from hemorrhage or who had cardiac disease that limited cardiac output.The same danger is present for women with right to left cardiac shunts because the decrease in systemic resistance would further increase the shunt.So, - Oxytocin should not be given intravenously as a large bolus, but rather as a much more dilute solution by continuous intravenous infusion or as an intramuscular infection.Management of cardiac patient during pregnancy Antepartum Bed restThe most important measure for attenuating the impact of pregnancy on diseased heart is bed rest.Bed rest increases the venous return to the heart, improves renal perfusion, induces diuresis and promotes elimination of water.Dietary salt restrictionIt helps to prevent excess retention of sodium and water.DiureticsIt should be given to cardiac pregnant patient if moderate restriction in sodium intake is insufficient to limit the normal intravascular volume expansion that occurs during gestation.Prophylactic digitalizationIt is commonly used in patients with severe heart disease who are not in overt congestive failure.During labour and deliveryThe pregnant cardiac patient should always labor and deliver in lateral supine position to avoid hemodynamic impairment caused by dorsal decubitus position.Effective pain relief during labour with morphine.Continuous monitoring with pulse oximetry.Restriction of i.v. fluids to 75 ml/hr (Almost all cardiac patients in labour should be kept on the dry side).Antibiotic prophylaxis (This is given in patients with congenital or acquired heart lesions).Thrombosis prophylaxis.Patient should be placed in sitting position after delivery. This is done to avoid pulmonary edema (sitting will increase venous pooling in the lower extremities and decrease the venous return to the heart).", "cop": 2, "opa": "Oxytocin infusion", "opb": "Methyl ergometrine", "opc": "Misoprostol", "opd": "Carboprost", "subject_name": "Gynaecology & Obstetrics", "topic_name": "Cardiovascular Disorders", "id": "f086f8fc-9830-48d1-8ecf-54564ebbdf49", "choice_type": "single"} {"question": "Highest maternal mortality is seen in following congenital heart disease", "exp": "Ans. is 'a' i.e. Eisenmenger syndrome This syndrome is secondary pulmonary hypertension that can develop with any cardiac lesion in which pulmonary vascular resistance becomes greater than the systemic vascular resistance and there should also be some right to left shunting. The most common underlying defects are atrial or ventricular septal defects and persistent ductus arteriosus.Patients usually are asymptomatic but eventually pulmonary hypertension becomes severe enough to cause right to left shunting.The prognosis for pregnancy depends upon the severity of pulmonary hypertension.The mortality rate in Eisenmenger's syndrome is 50%. It is the maximum mortality for any cardiovascular disorder in pregnancy. So termination of pregnancy is done in most cases.Maternal tolerance to specific heart conditions;Well toleratedPulmonary stenosisAortic insufficiencyMitral insufficiencyCongenital heart blockWell tolerated if pulmonary hypertension is not presentAtrial septal defectsVentricular septal defectsPatent ductusVariable tolerance depending on the functional capacity of the heartUncomplicated Aortic coarctationAortic stenosisMitral regurgitationAortic regurgitationPoorly tolerated and source of significant problems during pregnancyMitral stenosisPeripartum cardiomyopathyPrimary pulmonary hypertensionEisenmenger's syndromeMarfan's syndrome with dilated aortic rootMetallic valve prosthesisCongenital cyanotic heart diseaseConditions listed in group C if pulmonary hypertension is present.Any class III or IV lesion.", "cop": 1, "opa": "Eisenmenger's syndrome", "opb": "Pulmonary stenosis", "opc": "Coarctation of aorta", "opd": "VSD", "subject_name": "Gynaecology & Obstetrics", "topic_name": "Cardiovascular Disorders", "id": "64f408ba-d1d6-40de-a66b-42618fd96f54", "choice_type": "single"} {"question": "Twin peak sign on obstetric ultrasound indicates the presence of", "exp": "(A) Dichorionic twins# TWIN PEAK SIGN indicates the presence of a dichorionic-diamniotic twin gestation. It represents the extension of placental villi into the potential space that is formed from the reflection of apposed amniotic and chorionic layers from each fetus.> By performing an obstetric ultrasound at a gestational age of 10-14 weeks, monochorionic-diamniotic twins are discerned from dichorionic twins.> Presence of a \"T-sign\" at the inter-twin membrane-placental junction is indicative of monochorionic-diamniotic twins (that is, the junction between the inter-twin membrane and the external rim forms a right angle), whereas dichorionic twins present with a \"lambda (I) sign\" (that is, the chorion forms a wedge-shaped protrusion into the inter- twin space, creating a rather curved junction).> \"Lambda sign\" is also called the \"twin peak sign\". At ultrasound at a gestational age of 16-20 weeks, the \"lambda sign\" is indicative of dichorionicity but its absence does not exclude it.", "cop": 1, "opa": "Dichorionic twins", "opb": "Monochorionic twins", "opc": "Conjoined twins", "opd": "Parasitic twins", "subject_name": "Gynaecology & Obstetrics", "topic_name": "Miscellaneous (Obs)", "id": "7935e44c-5492-4894-9fa7-34040e6c40ee", "choice_type": "single"} {"question": "Prevention of neural tube defect in the newborn is by", "exp": "(Folic acid): (865 H186th edition)* Folic acid supplementation of the time of conception and in the first 12 weeks of pregnancy reduce by -70% the incidence of neural tube defects (NTDs) (Anencephaly , meningomyelocele,encephalocele and spina bifida) in the fetus.* Most of this protective effect can be achieved by taking folic acid 0.4mg daily at the time of conception* The incidence of cleft palate and have lip also can be reduced by prophylactic folic acid* NTDs also can be caused by antifolate and antiepileptic drugs", "cop": 4, "opa": "Iron", "opb": "B6", "opc": "B12", "opd": "Folic acid", "subject_name": "Gynaecology & Obstetrics", "topic_name": "Miscellaneous (Gynae)", "id": "b369422c-8d4e-4f5d-8e1b-9dcf912d04c0", "choice_type": "single"} {"question": "The regimens of Mg So4 for the management of severe preclampsia and eclampsia is known as", "exp": "(Pritchard regime) (236-D)* Magnesium sulphate is the drug of choice in preeclampsia and eclampsia, Pritchard and Zuspan or Sibai regime are used* Lvtic cocktail regime (Menon 1961) using chloropromazine, phenergan and pethidine (CPP)* Diazepam therapy (Lean)* Pinard's maneuver - frank breech extraction", "cop": 2, "opa": "Menon regime", "opb": "Pritchard regime", "opc": "Lean regime", "opd": "Pinard's regime", "subject_name": "Gynaecology & Obstetrics", "topic_name": "Miscellaneous (Gynae)", "id": "2cd0b0ee-2b32-4f85-9874-c0dd9022c471", "choice_type": "single"} {"question": "A pelvis characterized by an anteroposterior diameter of the inlet greater than the transverse diameter is classified as", "exp": "The anthropoid pelvis is marked by an oval inlet (but the anteroposterior diameter exceeds the transverse), the pelvic side walls diverge, and the sacrum is inclined posteriorly. Its associated with Persistent occipito posterior presentation and Face to pubis delivery The android (male-like) pelvis, where the inlet is wedge shaped, the pelvic sidewalls are convergent, the ischial spines are prominent, the subpubic arch is narrow, and the sacrum is inclined anteriorly in its lower one third. It is likely to be associated deep transverse arrest dystocia. Gynaecoid Android Anthropoid Platypelloid Shape of inlet Round Hea-shaped/triangular Long oval Flat Sacrosciatic notch Wide Narrow Wide Narrow Side walls Straight Convergent Straight Straight Ischial spine Not prominent Prominent Not prominent Not prominent Subpubic angle Wide Narrow Medium Wide", "cop": 3, "opa": "Gynecoid", "opb": "Android", "opc": "Anthropoid", "opd": "Platypelloid", "subject_name": "Gynaecology & Obstetrics", "topic_name": "Obstetrics", "id": "94f8f873-8b38-4d2e-a8de-67dc22b9720a", "choice_type": "single"} {"question": "Commonest site for fibroids in uterus is", "exp": "Fibroids always sta intramural Outwards Inwards Pedunculated Fibroid Subserosal fibroid Mucosal fibroid Pedunculated Submucosal fibroid Intramural fibroids are the m/c Fibroids are the most common cause of hysterectomy in the world. Fibroids have genetic predisposition & estrogen cause | in size of fibroids Rx fibroids is Medically - Mifepristone Surgically can be removed by laparoscopy or laparotomy FIGO Classification Leiomyoma Sub classification System Submucosal 0. Pedunculated intra cavity 1. < 50 % intra moral 2. >= 50 % intra moral Others 3. Contacts endometrium 100% intra moral 4. Intramural 5. Sub serosal >= 50% Intra moral 6. Sub serosal < 50 % intra moral 7. Sub serosal pedunculated 8. Cervical, parasitic", "cop": 2, "opa": "Subserous", "opb": "Intramural", "opc": "Subserous", "opd": "Cervical", "subject_name": "Gynaecology & Obstetrics", "topic_name": "JIPMER 2019", "id": "1a835e7a-6094-4a0d-b54e-145c6df0a425", "choice_type": "single"} {"question": "Caudal regression syndrome is seen in", "exp": "Gestational diabetes is associated with caudal regression syndrome.", "cop": 2, "opa": "Preeclampsia", "opb": "Gestational diabetes", "opc": "Sickle-Cell anemia", "opd": "Systemic lupus erythematosus", "subject_name": "Gynaecology & Obstetrics", "topic_name": null, "id": "aa6c5cbe-a0d3-41dd-a133-86704b55af0e", "choice_type": "single"} {"question": "Cardinal movements of labour in order", "exp": "The cardinal movements of labour are engagement, descent, flexion, internal rotation, extension, restitution, external rotation and expulsion Reference: William&;s Obstetrics; 24th edition; Chapter 22; Normal labour", "cop": 1, "opa": "Engagement--Descent--Flexion--Internal Rotation--Extension--Restitution--External Rotation--Expulsion", "opb": "Engagement--Descent--Flexion--Internal Rotation--Restitution--Extension--External Rotation--Expulsion", "opc": "Engagement--Descent--Flexion--Internal Rotation--Extension--External Rotation--Restitution--Expulsion", "opd": "Engagement--Descent--Flexion--Restitution--Internal Rotation--Extension--External Rotation--Expulsion", "subject_name": "Gynaecology & Obstetrics", "topic_name": "General obstetrics", "id": "5c191465-3cdd-4870-8b5f-b9ebb4339b37", "choice_type": "single"} {"question": "Area of the mitral valve in severe mitral stenosis during pregnancy", "exp": "Normal area of the mitral valve is 4-6cm2\nSevere mitral stenosis is less than 1.5cm2\nCritical area is less than 1cm2", "cop": 2, "opa": "4-6 cm2", "opb": "1-1.5 cm2", "opc": "1.5-2.5 cm2", "opd": "0.8-1 cm2", "subject_name": "Gynaecology & Obstetrics", "topic_name": null, "id": "55323414-e941-42f3-b014-ac3896fed2e8", "choice_type": "single"} {"question": "A female presents with leaking and meconum stained liquior at 32 weaks. She is infected with", "exp": "“Discolored brownish or meconium stained amniotic fluid is common with fetal infection, even with preterm gestation”\nHence, the correct answer is Listeria, rest of the infections do not lead to preterm labour with meconium stained liquor.\nListeriosis\n\nCausative organism- listeria monocytogenes (facultative intracellular gram positive bacillus)\nMode of infection- Food-borne infection caused by eating food like raw vegetables, Coleslaw, Apple cider, Melons, Milk, Fresh mexican style cheese, Smoked fish, Processed foods.\n\nClinical features\n\nCan be asymptomatic\nCan cause febrile illness.\n\nMaternal complications which can occur due to listeria infection are:\n\nPreterm labor,\nChorioamnionitis,\nMenonium stained liquor,\nAbortions,\nPlacental Macroabscesses\n\nMaternal infection can lead to fetal infection characterised by:\n\nDisseminated granulomatous lesions with microabscess in skin\nStillbirth\nOverall perinatal mortality-50%.\n\nManagement\n\nDrug of choice is combination of Ampicillin + gentamicin or in case of pencillin allergy – Trimethoprim sulfamethoxazole\nMaternal treatment may be effective for fetal infection\nThere is no vaccine for listeriosis.", "cop": 2, "opa": "CMV", "opb": "Listeria", "opc": "Toxoplasma", "opd": "Herpes", "subject_name": "Gynaecology & Obstetrics", "topic_name": null, "id": "8a59373d-0a67-492f-a4f2-5c9e41721ab6", "choice_type": "single"} {"question": "Fetal position refers to", "exp": "Fetal Lie: The relation of the fetal long axis to that of the mother is termed fetal lie and is either longitudinal or transverse. The presenting pa is that poion of the fetal body that is either foremost within the bih canal or in closest proximity to it.In the later months of pregnancy, the fetus assumes a characteristic posture described as attitude or habitus. As a rule, the fetus forms an ovoid mass that corresponds roughly to the shape of the uterine cavity.Position refers to the relationship of an arbitrarily chosen poion of the fetal presenting pa to the right or left side of the bih canal.(Ref: William's Obstetrics; 25th edition)", "cop": 1, "opa": "The relationship of an arbitrarily chosen poion of the fetal presenting pa to the right or left side of the bih canal", "opb": "That poion of the fetal body that is either foremost within the bih canal or in closest proximity to it", "opc": "The relation of the fetal long axis to that of the mother", "opd": "The form of an ovoid mass that corresponds roughly to the shape of the uterine cavity", "subject_name": "Gynaecology & Obstetrics", "topic_name": "All India exam", "id": "08df1e55-ca02-457a-be09-1fd2d0563ebd", "choice_type": "single"} {"question": "Threshold values for Diagnosis of Gestational diabetes from a 75g of OGTT is", "exp": "Threshold values for diagnosis of Gestational diabetes is Fasting: 92mg/dl1-hour: 180mg/dl2-hour: 153mg/dlOne or more of these values must be equaled or exceeded for the diagnosis of gestational diabetes(Ref: William's Obstetrics; 25th edition)", "cop": 1, "opa": "92, 180, 153", "opb": "92, 180, 155", "opc": "95, 180, 153", "opd": "95, 180, 155", "subject_name": "Gynaecology & Obstetrics", "topic_name": "All India exam", "id": "ef1654b3-4d9f-43bf-a784-3546f4fb54d4", "choice_type": "single"} {"question": "In a case of Dysgerminoma of ovary one of the following tumor markers is likely to be raised;", "exp": "- dysgerminoma is the most common among the malignant germ cell tumors accounting for 40%. - tumors markers like placental alkaline phosphatase and lactic dehydrogenase may be elevated in dysgerminoma. Reference : textbook of gynaecology Sheila balakrishnan, 2nd edition, pg no: 283 <\\p>", "cop": 3, "opa": "Serum HCG", "opb": "Semm alpha fetoprotein", "opc": "Serum lactic dehydrogenase", "opd": "Semminhibin", "subject_name": "Gynaecology & Obstetrics", "topic_name": "Gynaecological oncology", "id": "df5b125a-34d8-41fe-9260-2bec04c517ed", "choice_type": "single"} {"question": "The commonest route of spread of puerperal sepsis is", "exp": "The organisms responsible are typically those that normally colonize the cervix, vagina, and perineum and are usually of low virulence. The uterine cavity is sterile before rupture of membranes but later becomes contaminated. Other factors which help to create a suitable environment for these ascending organisms are tissue trauma, devitalization of tissues and the alkaline lochia (refer pgno:434 Sheila 2 nd edition)", "cop": 2, "opa": "Lymphatic", "opb": "Direct invasion", "opc": "Skip lesions", "opd": "Hematogenous", "subject_name": "Gynaecology & Obstetrics", "topic_name": "General obstetrics", "id": "c22cb718-5ce9-432d-bd9d-6ed78992d895", "choice_type": "single"} {"question": "A lady presented with creamy white vaginal discharge with fishy odour, drug of choice is aEUR'", "exp": "Metronidazole This is a case of bacterial vaginosis D.O.C. of bacterial vaginosis Metronidazole Features Vulvovaginal candidiasis Trichomonal vaginitis Bacterial Vaginosis Etiology Candida albicans Trichomonas Trich vaginalis vaginitis Symptoms Vulval itching irritation Profuse purluent discharge M alodorus slightly increased discharge Colour White White or yellow White or gray Inflammation vulvar or vaginal epithelium Erythema of vaginal epithelium Erythema of vaginal and vulvar epithelium None Amine \"fishy\"odour with KOH None Sometimes Present", "cop": 3, "opa": "Doxycycline", "opb": "Ofloxacin", "opc": "Metronidazole", "opd": "Clindamycin", "subject_name": "Gynaecology & Obstetrics", "topic_name": null, "id": "21f237ae-19e2-41cf-b216-f1d41bb0ad8d", "choice_type": "single"} {"question": "Most accurate tool for gestational age assignment is", "exp": "A first-trimester crown rump length is the most accurate tool for gestational age assignment and is performed as clinically indicated. (Ref: William's Obstetrics; 25th edition)", "cop": 3, "opa": "Menstrual history", "opb": "Clinical assessment", "opc": "First trimester Crown-Rump length", "opd": "Femur length", "subject_name": "Gynaecology & Obstetrics", "topic_name": "All India exam", "id": "9191a340-451e-49ad-ac59-1889d7c9f9ec", "choice_type": "single"} {"question": "A major causal factor in some cases of hypogonadism is", "exp": "Reduced secretion of GnRH will result in extremely low levels of circulating LH and FSH, causing testicular atrophy, as in Kallmann's syndrome.", "cop": 1, "opa": "Reduced secretion of gonadotropin-releasing hormone (GnRH)", "opb": "Hypersecretion of pituitary LH and FSH as the result of increased GnRH", "opc": "Excess secretion of testicular activin by Seoli cells", "opd": "Failure of the hypothalamus to respond to testosterone", "subject_name": "Gynaecology & Obstetrics", "topic_name": "All India exam", "id": "5ce65f8c-c458-4a9e-a1ed-79e22578e9c1", "choice_type": "single"} {"question": "Increased estrone levels is PCOD is derived from", "exp": "Elevated androstenedione levels contribute to an increase in estrone levels through peripheral conversion of androgens to estrogens by aromatase.", "cop": 2, "opa": "Testosterone", "opb": "Androstenedione", "opc": "Dehydro epiandrosterone", "opd": "GnRH", "subject_name": "Gynaecology & Obstetrics", "topic_name": null, "id": "543d9545-c021-4bc1-98e7-9afea01c2bb6", "choice_type": "single"} {"question": "A patient at 17 weeks gestation is diagnosed as having an intrauterine fetal demise. She returns to your office 5 weeks later and has not had a miscarriage, although she has had some occasional spotting. This patient is at increased risk for", "exp": "In modern clinical medicine, once the diagnosis of fetal demise has been made, the products of conception are removed. If, however, the gestational age is over 14 weeks and the fetal death occurred 5 weeks ago, coagulation abnormalities may be seen. Septic abortions were more frequently seen during the era of illegal abortions, although occasionally sepsis can occur if there is incomplete evacuation of the products of conception in either a therapeutic or spontaneous abortion.", "cop": 3, "opa": "Septic abortion", "opb": "Recurrent abortion", "opc": "Consumptive coagulopathy with hypofibrinogenemia", "opd": "Future infertility", "subject_name": "Gynaecology & Obstetrics", "topic_name": "Obstructed Labour", "id": "c0cbe05a-aa11-422c-8ad0-3050118ae792", "choice_type": "single"} {"question": "Gold standard investigation for septate uterus is", "exp": "Septate uterus: Defined as uterus with normal outline and an internal indentation at the fundal midline Can be of two types- paial and complete Combined hysteroscopy and laproscopy help to differentiate between bicornuate uterus and septate uterus Ref: Shaw Gynecology 17 e pg 72.", "cop": 2, "opa": "HSG", "opb": "Hysteroscopy", "opc": "MRI", "opd": "USG", "subject_name": "Gynaecology & Obstetrics", "topic_name": "Congenital malformations", "id": "d93eec5f-bbbb-4bf8-80bf-ae3e6d989495", "choice_type": "single"} {"question": "Select the antibiotic most frequently associated with the Gray baby syndrome (SELECT 1 ANTIBIOTIC)", "exp": "Fetal exposure to an antibiotic depends on many factors such as gestational age, protein binding, lipid solubility, pH, molecular weight, degree of ionization, and concentration gradient. Some antibiotics are even concentrated in the fetal compartment. Tetracycline is contraindicated in all three trimesters. It has been associated with skeletal abnormalities, staining and hypoplasia of budding fetal teeth, bone hypoplasia, and fatal maternal liver decompensation. Sulfonamides are associated with kernicterus in the newborn. They compete with bilirubin for binding sites on albumin, thereby leaving more bilirubin free for diffusion into tissues. Sulfonamides should be withheld during the last 2 to 6 weeks of pregnancy. With prolonged treatment of tuberculosis (TB) in pregnancy, streptomycin has been associated with fetal hearing loss. Its use is restricted to complicated cases of TB. Nitrofurantoin can cause maternal and fetal hemolytic anemia if glucose- 6-phosphate dehydrogenase deficiency is present. Chloramphenicol is noted for causing the gray baby syndrome. Infants are unable to properly metabolize the drug, which reaches toxic levels in about 4 days and can lead to neonatal death within 1 to 2 days.", "cop": 4, "opa": "Tetracycline", "opb": "Streptomycin", "opc": "Nitrofurantoin", "opd": "Chloramphenicol", "subject_name": "Gynaecology & Obstetrics", "topic_name": "Fetus & New Born and their Diseases", "id": "1bd88de0-6bfc-4d47-b65c-3f62cf4a26cd", "choice_type": "single"} {"question": "Urine formation in intra uterine life stas at", "exp": "3 months", "cop": 1, "opa": "3 months", "opb": "4 months", "opc": "5 months", "opd": "6 months", "subject_name": "Gynaecology & Obstetrics", "topic_name": null, "id": "b63f4f27-9c54-49ed-91bb-cb9fad3c995f", "choice_type": "single"} {"question": "The Kleihauer Betke test for detecting foetal erythrocytes is based on the fact that", "exp": "The most commonly used quantitative test for fetal red cells in the maternal circulation is the acid elution or Kleihauer Betke test. Fetal erythrocytes contain HbF, which is more resistant to acid elution than HbA.", "cop": 3, "opa": "Adult erythrocytes are larger than those of foetus", "opb": "HbA has higher oxygen affinity than HbF", "opc": "HbF is more resistant acid elution than HbA", "opd": "HbA takes up erythrosin stain less than HbF", "subject_name": "Gynaecology & Obstetrics", "topic_name": "Intra Uterine Growth Restriction, Intrapaum and Antepaum Fetal Surviellance", "id": "7e3f5b0e-13fb-4243-9596-c49bb44018c1", "choice_type": "single"} {"question": "A 3.8 kg baby of a diabetic mother developed seizures 16 hours after bih. The most probable cause would be", "exp": "The neonate has seizures developed at around 16 hrs after bih. There are two possibilities in this neonate i.e. hypoglycemia and hypocalcemia. Hypoglycemia in first 18-36 hrs is usually due to transient neonatal hypoglycemia and after 36 hrs metabolic and other causes are likely. Hypoglycemia develops in about 25-50% of infants of diabetic mothers and 15-25% of infants of mothers with gestational diabetes, but only a small percentage of these infants become symptomatic.", "cop": 1, "opa": "Hypoglycemia", "opb": "Hypocalcemia", "opc": "Bih asphyxia", "opd": "Intraventricular hemorrhage", "subject_name": "Gynaecology & Obstetrics", "topic_name": "All India exam", "id": "ca576b84-96dc-407b-86a2-71ad4a3ab65d", "choice_type": "single"} {"question": "P57KIP2 immunostaining is helpful in diagnosing", "exp": "In pregnancies before 10 weeks, classic molar changes may not be apparent because villi may not be enlarged and molar stroma may not yet be edematous and avascularOne takes advantage of the differing ploidy to distinguish paial (triploid) moles from diploid entities. Another technique involves histological immunostaining to identify the p57KIP2 nuclear protein. Because the gene that expresses p57KIP2 is paternally imprinted, only maternally donated genes are expressed. Because complete moles contain only paternal genetic material, they cannot express this gene; do not produce p57KIP2; and thus, do not pick up this immunostain. In contrast, this nuclear protein is strongly expressed in paial moles and in non-molar pregnancies with hydropic change. As a result, the combined use of ploidy analysis and p57KIP2 immunostaining can be used to differentiate: (1) a complete mole (diploid/p57KIP2-negative), (2) a paial mole (triploid/p57KIP2-positive), and spontaneous aboion with hydropic placental degeneration (diploid/p57KIP2-positive).Reference: William's Obstetrics; 25th edition, chapter 20; Gestational trophoblastic diseases", "cop": 3, "opa": "Ectopic pregnancy", "opb": "Normal pregnancy", "opc": "Complete molar pregnancy", "opd": "Missed aboion", "subject_name": "Gynaecology & Obstetrics", "topic_name": "General obstetrics", "id": "fa30ae61-fdcf-4c98-8236-381c231ce52e", "choice_type": "single"} {"question": "Interpret the paogram", "exp": "ref : dutta 9th ed", "cop": 2, "opa": "Inadequate uterine contraction", "opb": "cephalopelvic dispropoion", "opc": "Prolonged latent phase", "opd": "normal paogram", "subject_name": "Gynaecology & Obstetrics", "topic_name": "All India exam", "id": "a4b0f754-0dd3-42eb-8da4-72cbc8019a18", "choice_type": "single"} {"question": "Most common cause of vaginal discharge in prepubeal girls is", "exp": "A foreign body in the vagina is a common cause of vaginal discharge in young girls, which may appear purulent or bloody.Ref: Berek and Novak&;s Gynecology; 15th edition; Chapter 14", "cop": 3, "opa": "Precocious pubey", "opb": "Urethral prolapse", "opc": "Foreign body", "opd": "Germ cell tumor", "subject_name": "Gynaecology & Obstetrics", "topic_name": "Infections of the genital tract", "id": "6e8da475-447b-4659-a710-63e422be7b3b", "choice_type": "single"} {"question": "For a woman with history of prior two molar pregnancies, the risk of having a third molar pregnancy is", "exp": "For those with a prior complete mole, the risk of another mole is 0.9 percent. With a previous paial mole, the rate is 0.3 percentAfter two prior molar pregnancies, approximately 20 percent of women had a third moleReference: William's Obstetrics; 25th edition, chapter 20; Gestational trophoblastic diseases", "cop": 4, "opa": "1%", "opb": "5%", "opc": "10%", "opd": "20%", "subject_name": "Gynaecology & Obstetrics", "topic_name": "General obstetrics", "id": "075257d0-63e2-4202-8f36-4f23980701db", "choice_type": "single"} {"question": "GnRH agonists used in treatment of leiomyoma suppress progesterone levels after", "exp": "After 1-2 weeks of GnRH agonist therapy, estrogen and progesterone levels start to decline.", "cop": 1, "opa": "2 weeks", "opb": "3 weeks", "opc": "4 weeks", "opd": "5 weeks", "subject_name": "Gynaecology & Obstetrics", "topic_name": null, "id": "202db397-1310-4c7d-a171-fbc1a61127f2", "choice_type": "single"} {"question": "Risk factor for Ca Cervix", "exp": "Ans. is a i.e. HPV; Smoking; and Early sexual intercourse", "cop": 1, "opa": "HPV", "opb": "Smoking", "opc": "Late Menarche", "opd": "Nulliparity", "subject_name": "Gynaecology & Obstetrics", "topic_name": null, "id": "bd759db1-803a-443f-b208-3b378b9b4d7a", "choice_type": "single"} {"question": "Ovulation occurs", "exp": "(Before 14 days of the menstruation) (27- Shaw's 14th)Ovulation is estimated to occur 14 days before the first days of the succeeding cycle* Ovulation occurs when the ovum surrounded by the corona radiata escapes out of the graffian follicle. It is quickly picked up by the tubal fimbria which hugs the ovary at ovulation* The peak level of 75 ng /ml of LH is required for ovulation.* Normally, one single ovum is discharged from the graffian follicle. However multiple ovulation can occur and results in a multiple dizygotic pregnancy.", "cop": 2, "opa": "After 14 days of menstruation", "opb": "Before 14 days of menstruation", "opc": "After 16 days of menstruation", "opd": "Before 16days of menstruation", "subject_name": "Gynaecology & Obstetrics", "topic_name": "Miscellaneous (Gynae)", "id": "3071eba0-fe63-4f10-bca9-70a154a0a656", "choice_type": "single"} {"question": "Most common twin presentation is", "exp": "(C) Vertex + vertex # Twin pregnancy:> Most common lie of the fetus is longitudinal (90%) but malpresentations are quite common.# Combinations of presentation of the fetuses are Both vertex (commonest) First vertex and second breech First breech and second vertex Both breech, first vertex and second transverse and so on but rarest one, being both transverse when the possibility of conjoined twins should be ruled out.# Combinations of presentations of twin fetuses are Both vertex 60% First vertex, secon breech - 20% First breech, second vertex 10% Both breech 10%", "cop": 3, "opa": "Vertex + breech", "opb": "Vertex + transverse", "opc": "Vertex + vertex", "opd": "Vertex + bro", "subject_name": "Gynaecology & Obstetrics", "topic_name": "Miscellaneous (Obs)", "id": "957838b3-6e28-43ac-9722-6af0d8abf0ff", "choice_type": "single"} {"question": "Most common CNS tumor to cause precocious puberty is", "exp": "Hamartoma is most common CNS tumor to cause precocious puberty.", "cop": 4, "opa": "Astrocytoma", "opb": "Neuro fibroma", "opc": "Ependymoma", "opd": "Hamartoma", "subject_name": "Gynaecology & Obstetrics", "topic_name": null, "id": "0fc49a1f-6fe8-4a3c-8b70-c6a3da594b90", "choice_type": "single"} {"question": "Most frequent site of tubal ectopic pregnancy is", "exp": "Nearly 95 percent of ectopic pregnancies are implanted in the various segments of the fallopian tube. The ampulla (70 percent) is the most frequent site, followed by isthmic (12 percent), fimbrial (11 percent), and interstitial tubal pregnancies (2 percent).The remaining 5 percent of non-tubal ectopic pregnancies implant in the ovary, peritoneal cavity, cervix, or prior cesarean scarReference: William's Obstetrics; 25th edition; Chapter 19; Ectopic pregnancy", "cop": 3, "opa": "Interstitial", "opb": "Isthmic", "opc": "Ampulla", "opd": "Infundibulum", "subject_name": "Gynaecology & Obstetrics", "topic_name": "General obstetrics", "id": "f85038e4-e70b-4a63-b316-5642f0b3f4fc", "choice_type": "single"} {"question": "Umbilical cord separation takes", "exp": "Umbilical cord separation usually takes place within first 2 weeks.", "cop": 2, "opa": "First 1 week", "opb": "First 2 weeks", "opc": "First 3 weeks", "opd": "First 4 weeks", "subject_name": "Gynaecology & Obstetrics", "topic_name": null, "id": "f2373b86-e1d2-49bd-b7da-45a4c07c4d1b", "choice_type": "single"} {"question": "Cesarean delivery has following advantage over vaginal delivery", "exp": "Elective cesarean delivery offers decreased risks for hemorrhage and chorio amnionitis compared to vaginal delivery.", "cop": 1, "opa": "Decreased risk of hemorrhage", "opb": "Decreased risk of hysterectomy", "opc": "Decreased rehospitalization rate", "opd": "Decreased risk of thromboembolism", "subject_name": "Gynaecology & Obstetrics", "topic_name": null, "id": "3418a6cc-c7e2-4bac-9544-8f1c3814de08", "choice_type": "single"} {"question": "Acanthosis Nigricans associated with malignancy has", "exp": "Acanthosis Nigricans associated with malignancy has extensive skin involvement and sudden onset.", "cop": 2, "opa": "Localised skin involvement", "opb": "Abrupt onset", "opc": "Resolves spontaneously", "opd": "Involves neck and axilla only", "subject_name": "Gynaecology & Obstetrics", "topic_name": null, "id": "6b3ed0ac-4391-4e7a-89ea-51cd1a5c194d", "choice_type": "single"} {"question": "Bandle ring on uterus suggests", "exp": "In Tonic Uterine Contraction and Retraction, a circular groove encircling the uterus is formed between the active upper segment and the distended lower segment, called pathological retraction ring (Bandl's ring). Reference: D C Dutta Textbook of Obstetrics 7th edition page 362", "cop": 4, "opa": "Cervical dystocia", "opb": "Cochleate uterus", "opc": "Hypeonic uterus", "opd": "Obstructed labour", "subject_name": "Gynaecology & Obstetrics", "topic_name": "General obstetrics", "id": "c6ffc556-ec59-480e-82a3-29ab6b3a1a91", "choice_type": "single"} {"question": "A 55 year old female presents with abdominal pain, distension, ascites and dyspnea. Her CA 125 levels are elevated The most likely diagnosis is a.", "exp": "– M/c age incidence for primary ovarian neoplasms is 40-60 years, peak age incidence being 55–60 years. Ovarian malignancies generally present with vague symptoms like abdominal pain, dyspepsia, and patient may also experience irregular menses and if pelvic mass compresses the bladder or rectum she may have urinary frequency or constipation.\nIn advanced stages patients have symptoms related to presence of ascites, omental metastasis or bowel metastasis like abdominal distension, dyspnea, bloating, nausea, anorexia or early satiety.\nIn the patient given in the question all these symptoms are present (which could be seen in other cancers as well) plus her Ca 125 levels are raised, which favors the diagnosis of ovarian cancer in her.Remember\nThe most important sign of epithelial ovarian cancer is the presence of a pelvic mass on physical examination.\nFor postmenopausal patients with an adnexal mass and a very high serum CA125 level (>200U/ml), there is 96% positive predictive value for ovarian malignancy.", "cop": 1, "opa": "Ca ovary", "opb": "Ca cervix", "opc": "Ca lung", "opd": "Symphoma", "subject_name": "Gynaecology & Obstetrics", "topic_name": null, "id": "1ea19f23-b864-4140-b62a-1fd1b3aa05c1", "choice_type": "single"} {"question": "DOC for candidiasis in pregnancy", "exp": "Ans. (c) FluconazoleRef. Shaws 15thed, /: 352CANDIDAL (MONILIAL) VAGINITIS* It is a fungal infection caused by yeast-like microorganisms called Candida or Monilia.* The commonest species causing human disease isCandida albicansClinical Features* Vulval itching is the most common symptom, accompanied by vaginal irritation, dysuria, or both* Thick curdy or flaky vaginal discharge.Diagnosis* Essentially based on clinical findings. But the diagnosis can be confirmed on microscopic examination of a smear of the vaginal discharge treated with 10% KOH solution.* Pap smear shows thick red-stained hyphae and dark red spores.Treatment* A single dose of fluconazole 150 mg has been found to be very effective.* Ideally, both partners should be treated and the underlying predisposing factor corrected to give long- term relief.* Recurrent infection requires fluconazole orally 150 mg every 72 hours for 3 doses.", "cop": 3, "opa": "Metronidazole 500mg", "opb": "Tinidazole 500mg", "opc": "Fluconazole", "opd": "Metronidazole 1 gm", "subject_name": "Gynaecology & Obstetrics", "topic_name": "Pharmacotherapeutic in Obstetrics", "id": "5e7c672f-e7f9-412d-ad9a-c535a2943e58", "choice_type": "single"} {"question": "Extreme preterm is defined as", "exp": "Preterm labor is defined as one where the labor stas before 37th completed week, counting from the first day of the last menstrual period Lower limit of gestation is - 20 weeks in developed countries 28 weeks in developing countries Ref: Dutta Obs 9e pg 294.", "cop": 4, "opa": "34-36 weeks", "opb": "32-33 weeks", "opc": "28-31 weeks", "opd": "< 28 weeks", "subject_name": "Gynaecology & Obstetrics", "topic_name": "Abnormal labor", "id": "8cc70960-8640-4e47-b79d-e2e608d951f8", "choice_type": "single"} {"question": "Value of Serum FSH in a normal adult female is", "exp": "Clinical StateSerum FSHSerum LHNormal adult female5-20 IU/L (midcycle peak 2 timesthe basal level in ovulatory women5-20 IU/L (midcycle peak 3 timesthe basal level in ovulatory womenHypogonadotropic state: Prepubeal,Hypothalamic or pituitary dysfunction<5 IU/L<5 IU/LHypergonadotropic state: Postmenopausal,Castrate, or Ovarian failure>20 IU/L>40 IU/LReference: Clinical gynecologic endocrinology; 10th edition; Chapter 9; Normal and Abnormal Sexual Development; Page no: 443", "cop": 2, "opa": "1-5 IU/L", "opb": "5-20 IU/L", "opc": "20-40 IU/L", "opd": "> 40 IU/L", "subject_name": "Gynaecology & Obstetrics", "topic_name": "Sexuality and intersexuality", "id": "ff2b7a19-55f0-4321-a573-a879138f6033", "choice_type": "single"} {"question": "A 25 yr. old girl admitted as a case of septic abortion with tricuspid valve endocarditis. Vegetation from the valve likely to affect will be", "exp": "Ans. D. LungBacteria present within the uterus colonizes dead conception products are responsible for the initiation of maternal infection and this infection may extend to cause endocarditis, peritonitis, parametritis and septicemia.Septic pelvic thrombophlebitis with or without septic pulmonary embolization is an uncommon but devastating complication of septic abortion. Right sided endocarditis often results in septic pulmonary emboli responsible for the cause of infarction and lung abscess.", "cop": 4, "opa": "Liver", "opb": "Spleen", "opc": "Brain", "opd": "Lung", "subject_name": "Gynaecology & Obstetrics", "topic_name": "Miscellaneous (Obs)", "id": "f470b8d9-1b3b-48c4-b34c-36dfe6005e44", "choice_type": "single"} {"question": "In Dysgerminoma stage 1A, treatment of choice is", "exp": "Surgical management of dysgerminoma Veical midline abdominal incision Peritoneal washings Exploration of abdominal and pelvic organs Unilateral salpingoopherectomy with preservation of uterus Pelvic and paraaoic lymph node sampling Infraacolic omentectomy Removal of the other tumor masses Ref: Shaw Gynecology 17 e pg 469.", "cop": 2, "opa": "TAH with BSO", "opb": "Unilateral ovariotomy with preservation of uterus", "opc": "Bilateral ovariotomy with preservation of uterus", "opd": "Bilateral salpingo-oophorectoy", "subject_name": "Gynaecology & Obstetrics", "topic_name": "Gynaecological oncology", "id": "ad8f7988-3d05-4b65-9e65-f84e5e536fe4", "choice_type": "single"} {"question": "Mechanism responsible for high rates of spontaneous abortion in septate uterus is", "exp": "The primary mechanism responsible for spontaneous abortion in septate uterus is partial or complete implantation on avascular septum.", "cop": 3, "opa": "Distorted uterine cavity", "opb": "Associated cervical abnormality", "opc": "Implantation on avascular septum", "opd": "Unfavourable endometrial lining", "subject_name": "Gynaecology & Obstetrics", "topic_name": null, "id": "092542ca-d4b9-4d95-8851-b7dec4afe195", "choice_type": "single"} {"question": "Maximum number of oogonia is seen at", "exp": "Maximal number of oogonia is achieved at 20th week of gestation.", "cop": 2, "opa": "Puberty", "opb": "20th week of gestation", "opc": "At birth", "opd": "20 years age", "subject_name": "Gynaecology & Obstetrics", "topic_name": null, "id": "720caea5-273d-490c-8b9d-5349a9f1773b", "choice_type": "single"} {"question": "A case of Gestational trophoblastic neoplasia is detected to have lung metastasis. She should be staged as", "exp": "FIGO staging system of GTN: Stage I: Disease confined to the uterus Stage II: Gestational trophoblastic tumor (GTT) extends outside of the uterus but is limited to the genital structures (ovary, tube, vagina, and broad ligament) Stage III: GTT extends to the lungs, with or without known genital tract involvement Stage IV: All other metastatic sites", "cop": 3, "opa": "Stage - I", "opb": "Stage - II", "opc": "Stage - III", "opd": "Stage - IV", "subject_name": "Gynaecology & Obstetrics", "topic_name": "Twin Pregnancy, Molar Pregnancy, Gestational Trophoblastic disease and contraception in special situations (Sour Grapes!)", "id": "fde4a5e6-aab2-46cf-8809-c32bc2c01b14", "choice_type": "single"} {"question": "With oral iron therapy, rise in hb% can be seen after", "exp": "The improvement should be evident within 3 weeks of the therapy. After a lapse of few days, the hemoglobin concentration is expected to rise at the rate of about 0.7 gm/100 mL per week. Reference: Textbook of Obstetrics; Sheila Balakrishnan; 2nd Edition; Page no: 287", "cop": 2, "opa": "1 week", "opb": "3 weeks", "opc": "4 weeks", "opd": "6 weeks", "subject_name": "Gynaecology & Obstetrics", "topic_name": "Medical, surgical and gynaecological illness complicating pregnancy", "id": "f1134e0b-61ed-4218-a91e-7069a5ef4de6", "choice_type": "single"} {"question": "The widest transverse diameter of the fetal skull is", "exp": "Transverse diameters of fetal skull are\n\nBiparietal diameter: 9.5cm\nSuper-subparietal: 8.5cm\nBitemporal: 8cm\nBimastoid: 7.5cm", "cop": 1, "opa": "Biparietal diameter", "opb": "Occipito-frontal diameter", "opc": "Bitemporal diameter", "opd": "Suboccipito-frontal diameter", "subject_name": "Gynaecology & Obstetrics", "topic_name": null, "id": "fba01264-4c46-40f8-8b51-ec66417f8363", "choice_type": "single"} {"question": "A patient is diagnosed with carcinoma of the breast. The most important prognostic factor in the treatment of this disease is", "exp": "Recognition of the high risk associated with axillary metastases for early death and poor 5-year survival has led to the use of postsurgical adjuvant chemotherapy in these patients. Patients who have estrogen- or progesterone-receptive tumors (i.e., receptor present or receptor-positive) are particular candidates for this adjuvant therapy, as 60% of estrogen-positive tumors will respond to hormonal therapy. Age and size of the tumor are certainly factors of importance, but they are secondary to the presence or absence of axillary metastases.", "cop": 3, "opa": "Age at diagnosis", "opb": "Size of tumor", "opc": "Axillary metastases", "opd": "Estrogen receptors on the tumor", "subject_name": "Gynaecology & Obstetrics", "topic_name": "Fetal Skull and Maternal Pelvis", "id": "d8cd1d0a-c3ce-4709-8038-d754e76a5ac7", "choice_type": "single"} {"question": "Most common congenital abnormality of uterus", "exp": "Types of fusion anomalies Arcuate - 18% Uterus didelphys- 8% Uterus bicornis - 26% Septate uterus- 35% Uterus unicornis- 10% D.C.DUTTA&;S TEXTBOOK OF GYNAECOLOGY Pg no:45,6th edition", "cop": 4, "opa": "Uterus didelphys", "opb": "Arcuate", "opc": "Unicornuate", "opd": "Septate", "subject_name": "Gynaecology & Obstetrics", "topic_name": "Congenital malformations", "id": "eaaed9a3-f683-4cb7-bf24-2964a0908d73", "choice_type": "single"} {"question": "Paial mole is", "exp": "Paial mole : Karyotype is triploid either 69XXY or 69XYY One maternal and usually two paternal haploid chromosomes Ref: Dutta Obs 9e pg 186.", "cop": 3, "opa": "Haploid", "opb": "Diploid", "opc": "Triploid", "opd": "Polypoid", "subject_name": "Gynaecology & Obstetrics", "topic_name": "General obstetrics", "id": "d3432ce5-b09f-4853-81b8-aeffb112ff91", "choice_type": "single"} {"question": "Side effect of MgSo4 used in the treatment of eclampsia is", "exp": "Complications : Pulmonary edema Anuria Hea failure Hyperpyrexia Psychosis Ref: Dutta Obs 9e pg 222.", "cop": 4, "opa": "Hypotension", "opb": "Polyuria", "opc": "Coma", "opd": "Pulmonary edema", "subject_name": "Gynaecology & Obstetrics", "topic_name": "Medical, surgical and gynaecological illness complicating pregnancy", "id": "6ea437a5-7dd2-4909-85f8-c94745c3c296", "choice_type": "single"} {"question": "Suspected ectopic pregnancy is best diagnosed by", "exp": "Ans. B. TVS* Investigation of choice in a case of ectopic pregnancy/suspected case of ectopic pregnancy is Transvaginal scan.* Findings raising suspicion of ectopic on TVS:* Empty uterine cavity: most important* Finding of complex adnexal mass* Fluid (echogenic) in the pouch of Douglas.* Adnexal mass clearly separated from the ovary.* Rarely cardiac motion may be seen in an unruptured tubal ectopic pregnancy.", "cop": 2, "opa": "MRI", "opb": "TVS", "opc": "X-ray abdomen supine", "opd": "HSG", "subject_name": "Gynaecology & Obstetrics", "topic_name": "Ecotopic Pregnancy", "id": "a1ef64a1-3c98-4837-9497-2391c764e8ae", "choice_type": "single"} {"question": "Most common uterine malformation associated with renal anomalies", "exp": "Unicornuate uterus is seen in 1% cases; Due to failure of development of one mullerian duct or failure of migration to proper location Most commonly associated with abnormality of urinary tract anomaly usually of kidney It is diagnosed by absence of round ligament and fallopian tube on the opposite side Associated with Endometriosis, Early spontaneous aboions, Ectopic pregnancy, Preterm labour and Fetal growth resriction SHAW&;S TEXTBOOK OF GYNAECOLOGY,PG NO:99,15th edition", "cop": 2, "opa": "Bicornuate", "opb": "Unicornuate", "opc": "Septate", "opd": "Didelphys", "subject_name": "Gynaecology & Obstetrics", "topic_name": "Congenital malformations", "id": "79e75dc7-5762-4840-8820-ec4efad4ac45", "choice_type": "single"} {"question": "Best tocolytic in a cardiac patient is", "exp": "Atosiban is oxytocin antagonist and best tocolytic in cardiac patients. B agonists and Calcium channel blockers are contraindicated in hea disease complicated pregnancies.", "cop": 1, "opa": "Atosbian", "opb": "Isoxsuprine", "opc": "Nifedipine", "opd": "MgSO4", "subject_name": "Gynaecology & Obstetrics", "topic_name": "Abnormal labor", "id": "9fc06bec-01a0-49b4-b5bb-6758edc76739", "choice_type": "single"} {"question": "A P2L2 poorly compensated cardiac patient delivered 2 days back. Contraceptive advise is", "exp": "If Patient is hemodynamically unstable better to advise vasectomy for the husband. Contraception in Hea disease Patients Steroidal contraception is contraindicated as it may precipitate thromboembolic phenomenon IUCD's are avoided in fear of infection Progestin only pills are SAFE and EFFECTIVE. They may cause Irregular bleeding if the patient is anticoagulated. Barrier methods is the BEST but failure rate is high Sterilisation should be considered with the completion of family at the end of first week in the puerperium under local anaesthesia through abdominal route by minilap technique. The procedure can be delayed up to several days to ensure that the mother has become hemodynamically near normal and that she is afebrile, not anemic, and ambulating normally. Reference: DC. Duttas textbook of OBG & GYN , 9th edition .Page 260", "cop": 3, "opa": "Undergo sterilization (tubectomy) after 1 week", "opb": "Undergo sterilization after 6 weeks", "opc": "Suggest her husband to undergo vasectomy", "opd": "Oral contraceptive pills after 6 months", "subject_name": "Gynaecology & Obstetrics", "topic_name": "Medical, surgical and gynaecological illness complicating pregnancy", "id": "007f0ccd-6b60-40a8-87a9-9c8576c40471", "choice_type": "single"} {"question": "Most common cause of early miscarriages is", "exp": "Ans. c (Chromosomal abnormality). (Ref. Dutta, Obstetrics, 5th ed., 211)ETIOLOGY OF SPONTANEOUS ABORTION/MISCARRIAGE1. Ova- fetal factors (60%)- operate in early abortion.# Chromosomal abnormality (commonest being autosomal trisomy, monosomy)# Gross congenital malformations.# Blighted ovum, hydropic degeneration of villi# Interference with circulation (cord twist)", "cop": 3, "opa": "Natural diabetes", "opb": "Cervical incompetence", "opc": "Chromosomal abnormality", "opd": "Genitourinary infections", "subject_name": "Gynaecology & Obstetrics", "topic_name": "Pathology of Conception", "id": "8f5bc169-e081-4650-8fca-5120b0dd632b", "choice_type": "single"} {"question": "Pure gonadal dysgenesis will be diagnosed in the presence of", "exp": "Pure gonadal dysgenesis is characterised by B/L streak gonads. The external phenotype is female and mullerian structures are present. Usually present with primary amenorrhea (hypergonadotropic hypogonadism)", "cop": 1, "opa": "Bilateral streak gonads", "opb": "Bilateral undescended testes in male", "opc": "One side streak and other side normal gonad", "opd": "One side dysgenetic testis and other side dysgenetic ovary", "subject_name": "Gynaecology & Obstetrics", "topic_name": "Mullerian Abnormalities", "id": "65e79f7d-b48f-4275-8688-978f5618588f", "choice_type": "single"} {"question": "HT indicated in menopausal women", "exp": "* Hormone Therapy (HT) is one of the government-approved treatments for relief of menopausal symptoms. * These symptoms, caused by lower levels of estrogen at menopause, include : * Hot flashes, * Sleep disturbances, and * Vaginal dryness. * HT is also approved for the prevention of osteoporosis REF : SHAW BOOK OF GYNECOLOGY", "cop": 1, "opa": "Hot flash", "opb": "Ca breast", "opc": "Endometriosis", "opd": "Uterine bleeding", "subject_name": "Gynaecology & Obstetrics", "topic_name": "All India exam", "id": "a1acf1a3-81bc-48e6-b640-91a32c645ae2", "choice_type": "single"} {"question": "Premature adrenarche is growth of axillary and pubic hair prior to the age", "exp": "Premature adrenarche is defined as growth of axillary and pubic hair prior to age of 8 years.", "cop": 2, "opa": "6 years", "opb": "8 years", "opc": "10 years", "opd": "12 years", "subject_name": "Gynaecology & Obstetrics", "topic_name": null, "id": "b0092639-59f4-4291-80a9-afbd55f3edef", "choice_type": "single"} {"question": "Grade 2 abruptio placenta is characterized by", "exp": "Grade 2 abruptio placenta is characterized by fetal distress. Page clinical classification of placental abruption: Grade 1 Grade 2 Grade 3 Vaginal bleeding is slight Vaginal bleeding mild to moderate Bleeding is moderate to severe or concealed Uterus irritable, minimal tenderness Uterine tenderness is always present Uterine tenderness is marked Maternal BP and fibrinogen unaffected Maternal pulse increased, BP maintained Shock is pronounced FHS good Fetal distress present Fetal death is the rule Maternal shock absent Maternal shock absent Associated coagulation defect or anuria may complicate Ref: DC Dutta&;s Textbook of Obsterics 8th edition Pgno: 297", "cop": 4, "opa": "Pronounced shock", "opb": "Severe bleeding", "opc": "Coagulation defect", "opd": "Fetal distress", "subject_name": "Gynaecology & Obstetrics", "topic_name": "General obstetrics", "id": "abc1a5c9-c375-403a-a24e-bd6512af8d51", "choice_type": "single"} {"question": "First sign of magnesium sulphate toxicity is", "exp": "Magnesium sulfate toxicity : Loss of deep tendon reflexes. - >7 mEq/L Respiratory depression more than 10 meq/L Cardiac arrest more than 25mEq/L Ref: Dutta Obs 9e pg 222.", "cop": 1, "opa": "Loss of deep tendon reflexes", "opb": "Cardiac arrest", "opc": "Respiory depression", "opd": "Decreased urinary output", "subject_name": "Gynaecology & Obstetrics", "topic_name": "Medical, surgical and gynaecological illness complicating pregnancy", "id": "248e6d86-3207-4585-805a-796842e8a9a7", "choice_type": "single"} {"question": "Amount of iron required for fetus in pregnancy tenure is", "exp": "Ans. c (0.3 gm). (Ref. Textbook of Obstetrics, D.C. Dutta, 6th /- Tab no. 19)The total iron requirement of pregnancy is 1,000 mg:# 500 mg increases the maternal red blood cell mass,# 300 mg is transported to the fetus and placenta, and# 200 mg compensates for blood loss at delivery.- The iron requirements of pregnancy increase steadily towards term but average 3.5 mg per day.Recommended supplementation for nonanemic gravidas is 300 mg of ferrous sulfate per day, which contains 60 mg of elemental iron.- Anemic gravidas (Hb of 8 or 9 g/dL) should take 300 mg ferrous sulfate 2 or 3 times a day.- Vitamin C facilitates iron absorption.- Therapeutic results can be expected after 3 weeks of therapy.- The severely anemic patient (Hb less than 8 g/dL) may require parenteral therapy in the form of intramuscular or intravenous iron dextran.- The total dose of iron required can be calculated using this formula:# Total dose of iron dextran (mL) = (0.0476 x body weight in kg x ) + 1 mL/5 kg of body weight up to a maximum of 14 mL.- Adequate parenteral therapy should result in a marked increase in the reticulocyte count within 7 to 14 days.", "cop": 3, "opa": "1 gm", "opb": "0.1 gm", "opc": "0.3 gm", "opd": "3 gm", "subject_name": "Gynaecology & Obstetrics", "topic_name": "Miscellaneous (Gynae)", "id": "570c75b4-17e2-4907-aaa1-b47791da682b", "choice_type": "single"} {"question": "Advantages of ultrasound nuchal translucency over biochemical screening for Down syndrome include", "exp": "Biochemistry does not work well for multiple gestations.", "cop": 3, "opa": "Uses transvaginal approach", "opb": "More consistent measurements than lab tests", "opc": "Better in multiple gestation", "opd": "Wide gestational age range", "subject_name": "Gynaecology & Obstetrics", "topic_name": "All India exam", "id": "0bbc9585-6d83-4750-a920-ef3e96065281", "choice_type": "single"} {"question": "Pseudogestational sac is seen in", "exp": "The gestational sac --an anechoic fluid collection that represents the exocoelomic cavity--may be seen by 4.5 weeks.A caveat is that a gestational sac may appear similar to other intrauterine fluid accumulations--the so-called pseudogestational sac. This pseudosac may be seen with ectopic pregnancy and is easier to exclude once a yolk sac is seen. Typically, the yolk sac is visible by 5.5 weeks and with a mean gestational-sac diameter of 10 mm. Thus, the diagnosis of a uterine pregnancy should be made cautiously if the yolk sac is not yet seen.Ref: William&;s Obstetrics; 24th edition; chapter 19", "cop": 4, "opa": "Missed aboion", "opb": "Threatened aboion", "opc": "Inevitable aboion", "opd": "Ectopic pregnancy", "subject_name": "Gynaecology & Obstetrics", "topic_name": "General obstetrics", "id": "d5c564e6-36c1-42b4-834b-04cdfd549f85", "choice_type": "single"} {"question": "Absolute proof of monozygosity is determined by", "exp": "Zygosity refers to the type of conception and can only be determined by DNA testing. Refer page no 179 of Text book of obstetrics,sheila balakrishnan,2 nd edition.", "cop": 1, "opa": "DNA finger printing", "opb": "Intervening membrane layers", "opc": "Sex of the babies", "opd": "Reciprocal skin grafting", "subject_name": "Gynaecology & Obstetrics", "topic_name": "General obstetrics", "id": "c7a033f5-910a-4d79-9b4e-21c135102606", "choice_type": "single"} {"question": "Unicollis bicornis", "exp": "In uterus bicornis unicollis:Two cornua of uterus are seperate in the region of the body but there is a single cervix and single vagina In uterus bicornis bicollis: Vagina is single but the two cornua of the uterus remains seperate and two complete cervices project into the vagina Uterus subseptus:This septum is restricted to the body of the uterus SHAW'S TEXTBOOK OF GYNAECOLOGY,Pg no:99,15th edition", "cop": 1, "opa": "Two uterine cavity with one cervix", "opb": "Single vagina with double uterus", "opc": "Incomplete septum of uterus", "opd": "Double uterus with double cervix", "subject_name": "Gynaecology & Obstetrics", "topic_name": "Congenital malformations", "id": "8f0f5b0a-9c1f-470c-a0e7-d640d7b5ad0b", "choice_type": "single"} {"question": "Hyperprolactinemia results in", "exp": null, "cop": 2, "opa": "Chronic anovulation", "opb": "Hypogonadotropic hypogonadism", "opc": "Androgen insensitivity syndrome", "opd": "Hypergonadotropic hypogonadism", "subject_name": "Gynaecology & Obstetrics", "topic_name": null, "id": "96fe7a0a-defa-4ffd-8c7f-70da4c58284f", "choice_type": "single"} {"question": "Placental site involution takes", "exp": "Complete placental site involution takes upto 6 weeks.", "cop": 3, "opa": "2 weeks", "opb": "3 weeks", "opc": "6 weeks", "opd": "8 weeks", "subject_name": "Gynaecology & Obstetrics", "topic_name": null, "id": "29f34247-3585-4c20-bd3a-4fb328ee80a4", "choice_type": "single"} {"question": "Maximum dose of PGF2 alpha in postpaum hemorrhage is", "exp": "Max dose of PGF2 alpha in postpaum haemorrhage is 2 mg. Carboprost tromethamine (Hemabate) is the 15-methyl derivative of Prostaglandin F2 alpha. It is approved for uternine atony treatment in a dose of 250mcg (0.25 mg) given intramuscularly. This dose can be repeated if necessary at 15- to 90- minutes intervals up to maximum of eight doses (2 mg) C/I- Asthmatic women and those with suspected amniotic fluid embolism (as they cause pulmonary airway and vascular condstriction) It is the best drug to control PPH. Is also used an aboifacient in second trimester. Ref: Williams Obsterics 24th edition PGno: 760", "cop": 2, "opa": "0.25 mg", "opb": "2 mg", "opc": "20 mg", "opd": "200 mg", "subject_name": "Gynaecology & Obstetrics", "topic_name": "General obstetrics", "id": "f93132c2-0f02-46bf-9748-f3436161d6b3", "choice_type": "single"} {"question": "Mean incubation period for primary syphilitic chancre is", "exp": "Mean incubation period is 3 weeks.", "cop": 3, "opa": "1 week", "opb": "2 weeks", "opc": "3 weeks", "opd": "4 weeks", "subject_name": "Gynaecology & Obstetrics", "topic_name": null, "id": "5a9bb7f8-a805-4da0-bfcc-614129f16568", "choice_type": "single"} {"question": "Ovarian hyper thecosis is characterized by", "exp": "Ovarian Hyperthecosis is severe variant of PCOD which exhibit severe hyperandrogenism.", "cop": 2, "opa": "Enlarged ovaries.", "opb": "Nests of lutenized theca cells in ovarian stroma.", "opc": "Less severe variant of PCOS.", "opd": "Less severe hyperandrogenism.", "subject_name": "Gynaecology & Obstetrics", "topic_name": null, "id": "d1a5c6c2-574f-4001-84e9-bc7e6d017cd0", "choice_type": "single"} {"question": "Infertility due to PCOD is", "exp": "Infertility due to PCOD is reversible and treatable using ovulation inducing drugs.", "cop": 2, "opa": "Irreversible", "opb": "Treatable by ovulation inducing drugs", "opc": "Due to Hyperandrogenism", "opd": "Resolves with age", "subject_name": "Gynaecology & Obstetrics", "topic_name": null, "id": "cb61a8ee-9d7a-4c02-ae8c-78d83db864ed", "choice_type": "single"} {"question": "Cut off value of length of cervix at 24 weeks of gestation for prediction of preterm delivery is", "exp": "Diagnosis of preterm labor : Regular uterine contractions with or without pain Dilatation more than 2cm and 80% effacement of cervix Length of the cervix <2.5 cm and funnelling of internal os Pelvic pressure, backache ,vaginal discharge Ref: Dutta Obs 9e pg 294.", "cop": 3, "opa": "0.5 cm", "opb": "1.5 cm", "opc": "2.5 cm", "opd": "3.5 cm", "subject_name": "Gynaecology & Obstetrics", "topic_name": "Abnormal labor", "id": "91822d89-4178-4c7f-9f3b-b4056ceb38a1", "choice_type": "single"} {"question": "Mirena is a", "exp": "LNG-IUS or Mirena\nIt is a levanorgestrel releasing device\nIt contains about 52 mg of levanorgestrel released at a rate approximately 20 microgram per day", "cop": 1, "opa": "Progesterone IUCD", "opb": "Antiprogesterone", "opc": "Used in abortions", "opd": "Hormonal implant", "subject_name": "Gynaecology & Obstetrics", "topic_name": null, "id": "19b7816e-b51c-4d4d-b851-793337ad9bdf", "choice_type": "single"} {"question": "Ha&;s rule is related to", "exp": "Elastic recoil of Levator Ani occurs bringing the occiput forwards in the midline. The process is repeated until the occiput is placed anteriorly. This is called as rotation by law of pelvic floor- Ha&;s rule.", "cop": 1, "opa": "Mechanism of Internal rotation.", "opb": "Mechanism of Crowning.", "opc": "Mechanism of Engagement.", "opd": "Mechanism of Restitution.", "subject_name": "Gynaecology & Obstetrics", "topic_name": "General obstetrics", "id": "273d2d9d-5dc6-430e-bbd1-c66447e45408", "choice_type": "single"} {"question": "In cervical cancer surgery is limited to patients with", "exp": "Radiation therapy can be used in all stages of disease, but surgery is limited to patients with stage I to IIa disease. The 5-year survival rate for stage I cancer of the cervix is approximately 85% with either radiation therapy or radical hysterectomy. Reference: Novak's gynaecology; 14th edition; Chapter 35; Uterine cancer", "cop": 1, "opa": "Stage I to IIA", "opb": "Stage I to IIB", "opc": "Stage I to IIIA", "opd": "Stage I to IIIB", "subject_name": "Gynaecology & Obstetrics", "topic_name": "Gynaecological oncology", "id": "12bb340c-93cd-4678-a7cc-74953ed21d14", "choice_type": "single"} {"question": "Yuzpe method is a type of", "exp": "Ans. a (Emergency post-coital hormonal contraception) (Ref. Shaw's Textbook of Gynaecology 13th ed:233)In Yuzpe method, a post-coital hormonal contraception, two doses of 100 meg ethinyl-oestradiol + 500 meg levonorgestrel 12 h apart are given in 72 hours of intercourse.EMERGENCY CONTRACEPTION (POSTCOITAL CONTRACEPTION)Mechanism of action1. Delaying/inhibiting ovulation2. Inhibiting fertilization3. Preventing implantation of fertilized egg.4. Degenartion of Corpus leuteumIndications1. Unprotected intercourse2. Condom rupture3. Missed pills4. Sexual assault or rape5. First time intercourse.Risk of pregnancy after single act of unprotected coitus around time of ovulation--8%.Methods1. Morning after pills (Conjugated estrogen and ethnyl estradiol)--use in 72 hr.2. Combined hormonal regimen (YUZPE method)23. POP (very successful)4. IUCD (should be inserted within 5 days)5. RU486 (Mifepristone)-single dose 600 mg irrespective of day and number of intercourse.ALSO KNOWVery low dose OC pills called Mircette - (desogestrel/ethinyl estradiol and ethinyl estradiol) Tablets provide an oral contraceptive regimen of 21 white round tablets each containing 0.15 mg desogestrel and 0.02 mg ethinyl estradiol.", "cop": 1, "opa": "Emergency post-coital hormonal contraception", "opb": "Emergency post-coital IUCD contraception", "opc": "Male contraception", "opd": "Reversible minilap", "subject_name": "Gynaecology & Obstetrics", "topic_name": "Emergency Contraception - Ulipristal acetate (UPA)", "id": "c4774cec-f3f1-4655-a268-8f4e71637c00", "choice_type": "single"} {"question": "A 18yr old girl comes to gynaec OPD presenting with 6months A amenorrhea with his low grade fever weight loss pain abdomen generalised weakness On clinical examination pelvic mass felt on left side with features of ascites Diagnosis", "exp": "ref : shaw gynecology", "cop": 1, "opa": "TB pelvic with tuboovasion mass", "opb": "ectopic pregnancy", "opc": "Granulosa cell tumour", "opd": "Fibroid with degeneration", "subject_name": "Gynaecology & Obstetrics", "topic_name": "All India exam", "id": "9351ace8-c88c-4536-895a-2afe55eb7e9d", "choice_type": "single"} {"question": "Dystocia dystrophia syndrome is seen in", "exp": "Dystocia dystrophia syndrome\nCharecteristic appearence\n\nShort \nStalky\nSubfertile \nMasculine hair distribution\nHistory of delayed menarche\nHas android pelvis", "cop": 3, "opa": "Anthropoid", "opb": "Platypelloid pelvis", "opc": "Android pelvis", "opd": "Gynaecoid pelvis", "subject_name": "Gynaecology & Obstetrics", "topic_name": null, "id": "f756dfcb-6285-4c43-ace4-4f889af1b9ab", "choice_type": "single"} {"question": "Earliest sign of intrauterine death is", "exp": "Intrauterine death : Presence of gas in the fetal large vessels - Robes sign is the earliest sign seen 12 hrs after fetal death Ball sign - Crumpled up spine of the fetus or hyperflexion of the spine Spalding's sign -Overlapping of the fetal skull bones due to shrinkage of cerebrum after death of fetus. Ref:Dutta Obs 9e pg 303.", "cop": 1, "opa": "Gas in blood vessels", "opb": "Hyperflexion of fetal spine", "opc": "Spalding's sign", "opd": "Aboion", "subject_name": "Gynaecology & Obstetrics", "topic_name": "Abnormal labor", "id": "90386e9b-8b3d-4788-96a6-61ca5532c89d", "choice_type": "single"} {"question": "Endodermal sinus tumour is assocaited with", "exp": "Schiller duval body [Ref. Robbins 7th/e p 11011 Endodermal sinus tumour (yolk sac tumour) This tumour is rare but it is the second most common malignant tumour of germ cell origin. It is thought to be derived from differentiation of malignant germ cells towards extraembryonic yolk sac structure. Similar to the yolk sac the tumour is rich is a feto protein and a1 antitrypsin. The characteristic histological feature of yolk sal tumour is schiller duval body. Schiller Duval body is a glomerulus like structure composed of central blood vessel enveloped by germ cells within a space lined by germ cells. Also know Call Exner body --> Granulosa cell tumour Reinke's crystal ---> Hilus cell tumour Signet ring cells Krukenberg tumour Hobnail cells Clear cell carcinoma Psammoma bodies -4 Papillary serous cystadenoma of ovary.", "cop": 1, "opa": ">Schiller duval body", "opb": ">Multinucleate giant cells", "opc": ">R-S cells", "opd": ">Plasma cells", "subject_name": "Gynaecology & Obstetrics", "topic_name": null, "id": "23053b76-9896-43fd-9ec1-58dc77b705f7", "choice_type": "single"} {"question": "Most common complication of vaginoplasty is", "exp": "Restenosis and dyspareunia are most common complication of vaginoplasty.", "cop": 2, "opa": "Vaginal bleeding", "opb": "Re - stenosis", "opc": "Dysmenorrhea", "opd": "Ectopic pregnancy", "subject_name": "Gynaecology & Obstetrics", "topic_name": null, "id": "065912d0-5b13-40f2-9a89-c06ec23e4d60", "choice_type": "single"} {"question": "Commonest cause of post partum haemorrhage is", "exp": "(Atonic Uterus) (411-Dutta 6th)* Atonic uterus (80%) is the commonest cause of PPH* Other causes of PPH are - Traumatic (20%), combination of atonic and traumatic causes, Blood coagulation disordes* The conditions where such disorders may occur are abruptioplacentae. Jaundice in pregnancy thrombocytopenia purpura, HELLP syndrome or in IVD.* Minimum duration between onset of symptoms and death is seen in PPH* Secondary PPH usually occur between 8th to 14th day of deliverySurgical method to control PPH(a) Ligation of ascending branch of uterine artery**(b) Ligation of ovarian & uterine artery anastomosis(c) Ligation of anterior division of internal iliac artery**(d) B- Lynch brace suture** & haemostatic suturing(e) Angiographic arterial embolisation (bleeding vessels)", "cop": 1, "opa": "Atonic uterus", "opb": "Traumatic", "opc": "Combination of atonic and traumatic", "opd": "Blood coagulation disorders", "subject_name": "Gynaecology & Obstetrics", "topic_name": "Miscellaneous (Gynae)", "id": "37596b5c-c229-41a8-a8d6-143a205c5277", "choice_type": "single"} {"question": "Cervical cerclage is advised in an antenatal woman with a prior preterm bih if the cervical length measures", "exp": "Cervical length screening is now recommended by both the American College of Obstetricians and Gynecologists (2016b) and the Society for Maternal-Fetal Medicine (2015) for women with prior preterm bih.Between 16 and 24 weeks' gestation, sonographic cervical measurement is completed every 2 weeks.If an initial or subsequent cervical length is 25 to 29 mm, then a weekly interval is considered.If the cervical length measures <25 mm, cerclage is offered to this group of women. Notably, for women without history of preterm bih but with a sho cervix incidentally identified sonographically, progesterone therapy is offered instead of cerclage.Reference: William's Obstetrics; 25th edition; Chapter 18; Aboion", "cop": 4, "opa": "Not related to cervical length", "opb": "Between 3-4 cm", "opc": "Between 2.5 to 2.9 cm", "opd": "< 2.5 cm", "subject_name": "Gynaecology & Obstetrics", "topic_name": "General obstetrics", "id": "fdd65acc-b8bd-49fd-af55-916a92fc4045", "choice_type": "single"} {"question": "Concerning fibroids", "exp": "EGF, IGF-1, TGF stimulates the growth of leiomyoma either directly or estrogen GnRH analogues reduces 50%-80% of myoma size when used for 6months Pregnancy rate following myomectomy is 40-60% Recurrence and persistance following myomectomy is 30-50% D.C.DUTTA'S TEXTBOOK OF GYNECOLOGY,Pg no:272,281,282,6th edition *Intramural fibroid : endometrial sampling next step of management", "cop": 4, "opa": "Use of GnRH analogues cause permanent reduction in size", "opb": "Pregnancy following myomectomy is about 80%", "opc": "Recurrence rate following myomectomy is about 20%", "opd": "Growth factors (IGF-L EGF) stimulates myoma to grow", "subject_name": "Gynaecology & Obstetrics", "topic_name": "Uterine fibroid", "id": "34116017-1171-4fc1-a78a-36bdac0de233", "choice_type": "single"} {"question": "One of the following features can be used to define contracted pelvis", "exp": "Minimal/Critical diameters of the Pelvis: If any of the following diameter is less than critical diameter, Pelvis is said to be contracted", "cop": 1, "opa": "Transverse diameter of inlet is 10 cm", "opb": "AP diameter of inlet is 12 cm", "opc": "Platypelloid pelvis", "opd": "Gynaecoid pelvis", "subject_name": "Gynaecology & Obstetrics", "topic_name": null, "id": "8d881f5e-5240-453f-b97b-eafce93ec13c", "choice_type": "single"} {"question": "Maximum chances of ureteric injury are with", "exp": "Weheim's hysterectomy:includes Radical abdominal hysterectomy Bilateral extra peritoneal dissection Excision of iliac and pelvic lymph nodes for cancer of the cervix Ureter injury is the major complication Ref : Shaw Gynecology 17e pg 526.", "cop": 2, "opa": "TAH", "opb": "Weheims hysteroctomy", "opc": "Anterior colporrhaphy", "opd": "Vaginal hsterectomy", "subject_name": "Gynaecology & Obstetrics", "topic_name": "Gynaecological diagnosis and operative surgery", "id": "5bf9a27a-8e75-4417-b78c-4c01fc2d5ab5", "choice_type": "single"} {"question": "The production of cervical mucus is stimulated by", "exp": "Cervical mucus is a glycoprotein gel that expand and contract in response to cyclic changes in the steroid hormone environment across the menstrual cycle to facilitate or inhibit the passage of sperm\n\nEstrogen stimulates cervical mucus production, and as levels rise during the follicular phase, mucus becomes more abundant and watery, less cellular, and more easily penetrated by sperm\nProgesterone inhibits cervical mucus production and renders it opaque, viscid, and impenetrable", "cop": 4, "opa": "Progesterone", "opb": "Pregnenolone", "opc": "Estriol", "opd": "Estradiol", "subject_name": "Gynaecology & Obstetrics", "topic_name": null, "id": "a2c1b8d2-6d39-4eba-a0da-6ef94e34a6b1", "choice_type": "single"} {"question": "The dose of betamethasone for fetal lung maturation is", "exp": "A single course of coicosteroids is currently recommended for women between 24 to 34 weeks who are at risk for delivery within 7days. Betamethasone and dexamethasone appear to be equivalent for fetal lung maturation. Dose: 12mg doses of betamethasone, each dose is given intramuscularly 24hours apa. With dexamethasone, 6mg doses are given intramuscularly every 12hours for four doses. (Ref: William's Obstetrics; 25th edition)", "cop": 2, "opa": "Two doses of 6mg, IM 24hours apa", "opb": "Two doses of 12mg, IM 24hours apa", "opc": "Four doses of 6mg, IM, 12hours apa", "opd": "Four doses of 12mg, IM, 12hours apa", "subject_name": "Gynaecology & Obstetrics", "topic_name": "All India exam", "id": "b256063d-a72a-4369-b498-266404ba5bc4", "choice_type": "single"} {"question": "Misoprostol is an analogue of", "exp": "Misoprostal: PGE-1 Given transvaginally and orally Dose of 25 micrograms every 4 hrs is found to be more effective than PGE -2 for cervical ripening and labor induction. Oral use of misoprostal is less effective than vaginal administration . Ref: Dutta Obs 9e pg 485.", "cop": 1, "opa": "PGE1", "opb": "PGE2", "opc": "PGF2alfa", "opd": "PGI2", "subject_name": "Gynaecology & Obstetrics", "topic_name": "Abnormal labor", "id": "f55764f6-a1de-404f-8fc2-296b52fdc15f", "choice_type": "single"} {"question": "The technique used for aspiration of sperm directly from testes for in vitro feilization is", "exp": "MESA and TESA are both techniques of sperm aspiration but since the question specifically asks about aspiration directly from the testes, TESA is the most appropriate answer here.Ref: Clinical Gynecologic Endocrinology and Infeility; 8th edition; Section IV; Infeility", "cop": 1, "opa": "TESA", "opb": "MESA", "opc": "GIFT", "opd": "IVF", "subject_name": "Gynaecology & Obstetrics", "topic_name": "Infertility", "id": "9c871139-ff3b-4ce6-8fd1-0251e8e1e32c", "choice_type": "single"} {"question": "Cochlear function in fetus develops between", "exp": "Cochlear function develops between 22 and 25 weeks, and its maturation continues for six months after delivery.Reference: William&;s Obstetrics; 24th edition; Chapter 7; Embryogenesis and Fetal Morphological Development", "cop": 3, "opa": "12 to 15 weeks", "opb": "18 to 20 weeks", "opc": "22 to 25 weeks", "opd": "30 to 34 weeks", "subject_name": "Gynaecology & Obstetrics", "topic_name": "General obstetrics", "id": "129b4d5c-6ea3-42e8-8bb6-d5c1c9dc94fd", "choice_type": "single"} {"question": "Extremely low bih weight refers to neonates weighing", "exp": "Low bihweight refers to neonates weighing 1500 to 2500 g; Very low bihweight are those between 1000 and 1500 g; and Extremely low bihweight refers to those between 500 and 1000 g.Reference: William's Obstetrics; 25th edition; Chapter 42; Preterm bih", "cop": 4, "opa": "< 2500 grams", "opb": "< 2000 grams", "opc": "< 1500 grams", "opd": "< 1000 grams", "subject_name": "Gynaecology & Obstetrics", "topic_name": "General obstetrics", "id": "74ba56b2-1cb8-4c2b-ba2d-342014e19006", "choice_type": "single"} {"question": "Dose of folic acid per day for treating megaloblastic anemia in pregnancy", "exp": "Folic acid deficiency in Pregnancy In nonpregnant women, the folic acid requirement is 50 to 100 mg/day During pregnancy, requirements are increased and 400 mg/day is recommended The earliest biochemical evidence is low plasma folic acid concentrations Normal serum folate levels Non Pregnant women: 5.4-18ng/ml 1st trimester: 2.6-15ng/ml 2nd trimester: 0.8-24ng/ml 3rd trimester: 1.4-20.7ng/ml Early morphological changes usually include neutrophils that are hypersegmented and newly formed erythrocytes that are macrocytic As the anemia becomes more intense, peripheral nucleated erythrocytes appear, and bone marrow examination discloses megaloblastic erythropoiesis The fetus and placenta extract folate from maternal circulation so effectively that the fetus is not anemic despite severe maternal anemia Treatment As little as 1mg of folic acid administered orally once daily produces a striking hematological response By 4 to 7 days after beginning folic acid treatment, the reticulocyte count is increased, and leukopenia and thrombocytopenia are corrected American College of Obstetricians and Gynecologists (2013c) have recommended that all women of childbearing age consume at least 400 mg of folic acid daily Increased need of Folic acid Multifetal pregnancy Hemolytic anemia Crohn disease Alcoholism Inflammatory skin disorders Reference: William&;s Obstetrics; 24th Edition; Chapter 56", "cop": 3, "opa": "400 mg", "opb": "5mg", "opc": "1 mg", "opd": "2 mg", "subject_name": "Gynaecology & Obstetrics", "topic_name": "Medical, surgical and gynaecological illness complicating pregnancy", "id": "d20ea009-ae34-41fb-9031-7af15250c746", "choice_type": "single"} {"question": "In embryo, Hemopoiesis is first demonstrable in", "exp": "In the early embryo, hemopoiesis is demonstrable first in the yolk sac, followed by the liver and finally bone marrow.Reference: William&;s Obstetrics; 24th edition; Chapter 7; Embryogenesis and Fetal Morphological Development", "cop": 1, "opa": "Yolk sac", "opb": "Liver", "opc": "Spleen", "opd": "Bone marrow", "subject_name": "Gynaecology & Obstetrics", "topic_name": "General obstetrics", "id": "cdc381b0-9797-4acd-ab13-08cabf361744", "choice_type": "single"} {"question": "Early hormonal sign of puberty", "exp": "Early sign of puberty is Sleep-associated increase in LH secretion.", "cop": 1, "opa": "Sleep associated increase in LH", "opb": "Early morning rise in temperature", "opc": "Increased FSH to LH ratio", "opd": "Elevated adrenal androgens.", "subject_name": "Gynaecology & Obstetrics", "topic_name": null, "id": "258c607c-70e6-4998-bdfe-945de6cd7c7b", "choice_type": "single"} {"question": "Low dose progestational contraceptives primarily act on", "exp": "Progesterone in bih control pills thickens cervical mucus (primary action): hinders the travel of sperm to the uterus and making it hard for sperm to enter the tubes and feilise the egg. thins the lining of the uterus: makes implantation unlikely. Progestogen only pills are generally less effective than other forms of oral contraceptive pill with a failure rate of 2 - 6 per 100 women years. The main side effects are Irregular, low-grade, breakthrough uterine bleeding but this may settle into a regular cycle after 6 - 9 months. Skin reactions, Breast fullness Depression Increased risk of ectopic pregnancy. Benefits include: Do not carry risk of thromboembolism Do not inhibit lactation - so can use when breast feeding Suitable for diabetics - as pills containing oestrogen exacerbate risk of aerial disease in such patients Suits smokers for whom oestrogen containing formulations are contra-indicated", "cop": 3, "opa": "Oviductal motility", "opb": "Uterine endometrium", "opc": "Cervix", "opd": "Pituitary", "subject_name": "Gynaecology & Obstetrics", "topic_name": "Contraceptives", "id": "2397c551-2f8b-46f3-b51e-f9e87213ffa7", "choice_type": "single"} {"question": "Leash sign is found in", "exp": "Ans. A. Ectopic PregnancyAll cases of ectopic pregnancies had a typical eccentric leash of vessels on color Doppler that showed a low resistance placental type of flow on spectral Doppler. This is called leash sign. This sign has a sensitivity of 100% and specifity of 99% a positive predictive value of 95% and negative predictive value of 100% thus helping in the diagnosis of early ectopic pregnancy, and resulting in earlier treatment with reduced morbidity and mortality.", "cop": 1, "opa": "Ectopic Pregnancy", "opb": "Vulvovaginitis", "opc": "Polyhydramnios", "opd": "Tubercular Salpingitis", "subject_name": "Gynaecology & Obstetrics", "topic_name": "Ecotopic Pregnancy", "id": "7c7c4853-9f28-4604-b83b-71fcfff2e026", "choice_type": "single"} {"question": "For the first 2hrs after delivery, the temperature should be recorded", "exp": "Care after delivery: For 2hrs after delivery, BP and pulse should be taken every 15 min. Temperature assessed every 4 hrs for the first 8hrs and then 8 hourly subsequently Amount of vaginal bleeding should be monitored Fundus of uterus palpated to ensure that it is well contracted. Ref: Williams 24e pg 675.", "cop": 2, "opa": "Every 5 mins", "opb": "Every 15 mins", "opc": "Every 30 mins", "opd": "Hourly", "subject_name": "Gynaecology & Obstetrics", "topic_name": "General obstetrics", "id": "27416dc0-b7f9-483c-a5de-fd8e7c2ee34a", "choice_type": "single"} {"question": "Cfitoris develops from", "exp": "Genital tubercle", "cop": 4, "opa": "Urogenital Sinus", "opb": "Labio Scrotal Swelling", "opc": "Genital folds", "opd": "Genital tubercle", "subject_name": "Gynaecology & Obstetrics", "topic_name": null, "id": "85fc6e82-4de9-40b5-8417-e1b8a6628422", "choice_type": "single"} {"question": "Normal Biophysical Profile (Manning Score) is", "exp": "Ans. a (8-10) (Ref. Dutta, Obstetrics, 6th ed., p 109, 464)BIOPHYSICAL PROFILE (MANNING SCORE)# The BPP measures- fetal heart rate,- muscle tone,- movement,- breathing, and the- amount of amniotic fluid around fetus.# A BPP is commonly done in the last trimester of pregnancy.# If patient has a high-risk pregnancy, a BPP may be done by 32 to 34 weeks or earlier.# Its 5 components are:Q- Non-stress test (NST)- Fetal breathing movements (at least 1 episode within 30 min lasting for > 30 seconds).- Fetal tone (> 1 episode of limb extension with return to flexion within 30 seconds).- Gross fetal movements (at least 3 discrete GFM within 30 minutes).- Amniotic fluid volume (> 1 large pocket more than 1 cm vertical diameter).# Score - In absence of oligohydramnios:- Normal score = 8-10,e- Equivocal = 6 and- Abnormal = < 4.Fetal heart rate in \"sinusoidal pattern\"Fetal heart rate in \"saltatory pattern\" is seen ina) Rh isoimmunizationb) Normal fetusc) Drugs-alphaprodinea) Acid-base imbalanceb) Labourc) Acute brief hypoxia in fetusd) Depressed fetus with low APGAR", "cop": 1, "opa": "10-Aug", "opb": "8-Jun", "opc": "6-Apr", "opd": "4-Mar", "subject_name": "Gynaecology & Obstetrics", "topic_name": "Miscellaneous (Gynae)", "id": "7b0ff59c-49ff-42f9-b390-e3ca55c138cb", "choice_type": "single"} {"question": "Page system of grading is used in", "exp": "Page system of grading: depends upon the degree of placental abruption 0- Clinical features are absent 1 - Vaginal bleeding is slight,FHS is good, uterine tenderness is minimal or absent 2 - Vaginal bleeding is mild or moderate, uterine tenderness is present,Fetal distress occurs, shock is absent 3 - Bleeding is moderate to severe, uterine tenderness is marked, fetal death Ref: Dutta Obs 9e pg 239.", "cop": 2, "opa": "Placenta pre", "opb": "Abruptio placenta", "opc": "Breech presentation", "opd": "Ectopic pregnancy", "subject_name": "Gynaecology & Obstetrics", "topic_name": "General obstetrics", "id": "86c61dd6-944d-4ebb-892c-3839738d0569", "choice_type": "single"} {"question": "Glomerular filtration rate increases in pregnancy by", "exp": "Both the glomerular filtration rate (GFR) and renal plasma flow increase early in pregnancy. The GFR increases as much as 25 percent by the second week after conception and 50 percent by the beginning of the second trimester(Ref: William's Obstetrics; 25th edition)", "cop": 4, "opa": "15%", "opb": "25%", "opc": "40%", "opd": "50%", "subject_name": "Gynaecology & Obstetrics", "topic_name": "All India exam", "id": "8b8c48c1-05d1-49a6-8faa-0ea9d4bcfe69", "choice_type": "single"} {"question": "Precocious puberty in females is defined as", "exp": "Ans. C. Appearance of Secondary sexual characteristics before 8 years* Precocious puberty: This is defined as the appearance of any of the secondary sexual characteristics before the age of 8 years or the occurrence of menarche before the age of 10 years* Precocious puberty in females is pubertal changes before the age of 8 years.* In males it is onset of puberty before the age of 9 years", "cop": 3, "opa": "Occurrence of menarche before age of 8 years", "opb": "Occurrence of menarche before age of 12 years", "opc": "Appearance of secondary sexual characteristics before 8 years", "opd": "Appearance of secondary sexual characteristics before 10 years", "subject_name": "Gynaecology & Obstetrics", "topic_name": "Miscellaneous (Obs)", "id": "26d5f7a1-425e-4ce0-b9d7-34c2b0f2fa57", "choice_type": "single"} {"question": "Endocervical sampling is recommended during colposcopy in the following situation", "exp": "Recommendations for Endocervical sampling:\n\nColposcopy in inadequate, or colposcopy is adequate but no lesion is identified. Endocervical sampling is acceptable in other cases at provider discretion.\nInitial evaluation of ASC - H, HSIL, AGC, or AIS cytology test results.\nSurveillance 4 to 6 months after excisional therapy if specimen margins are positive of HSIL.\nSurveillance after conization for AIS has been performed in women wishing fertility preservation. Negative endocervical curettage results add reassurance to this management.", "cop": 2, "opa": "Initial evaluation of LSIL in pregnant female", "opb": "Surveillance 4 to 6 months after excisional therapy if specimen margins are positive for HSIL", "opc": "Along with co-testing in post-menopausal LSIL", "opd": "Reassure LSIL patients in age group of 25 to 30 years.", "subject_name": "Gynaecology & Obstetrics", "topic_name": null, "id": "e9e074ae-ad20-4d24-af67-6ab012aaf161", "choice_type": "single"} {"question": "Pathology of endometriosis is best explained by", "exp": "Retrograde menstruation is the most acceptable theory", "cop": 3, "opa": "Coelemic metaplasia", "opb": "Endometrial hyperplasia", "opc": "Retrograde menstruation", "opd": "Intraperitoneal immunologic deficit", "subject_name": "Gynaecology & Obstetrics", "topic_name": "Gynaecology", "id": "c93af121-e31e-48b0-85ab-caa2f94288db", "choice_type": "single"} {"question": "The best time to do chorionic villous sampling is", "exp": "Ans. is d i.e. Between 11 - 13 weeks Chorionic villi sampling \"CVS is the second most common procedure for genetic prenatal diagnosis. Because this procedure is routinely performed at about 10 to 12 weeks of gestation, it allows for an earlier detection of abnormalities and a safer pregnancy termination, if desired.\"", "cop": 4, "opa": "Between 6-8 weeks", "opb": "Between 7-9 weeks", "opc": "Between 9-11 weeks", "opd": "Between 11-13 weeks", "subject_name": "Gynaecology & Obstetrics", "topic_name": null, "id": "5a5d10c9-7e37-494f-ad9f-e84852a6f091", "choice_type": "single"} {"question": "Cystitis is most commonly caused by", "exp": "(E. coli): Ref: 173-S* The organisms found in urine in cystitis are the E. coli, streptococci, staphylococci and B. proteus, the tubercle bacillus, and occasionally pseudomonas pyocyanea, Gonococcal cystitis is relatively rare, and almost invariably follows instrumentation.* The organism which is found most frequently is the E. coli* As the result of antibiotic treatments Pseudomonas, pyocyanea sometimes become the dominant infecting organism because of its resistance to antibiotics relative to the other infecting organisms.", "cop": 1, "opa": "E. coli", "opb": "Pseudomonas", "opc": "Proteus mirabilis", "opd": "Neisseria gonorrhea", "subject_name": "Gynaecology & Obstetrics", "topic_name": "Miscellaneous (Gynae)", "id": "0ee68dcf-42a7-4c5f-9b24-e86776afec5f", "choice_type": "single"} {"question": "The critical mitogen for uterine fibroid growth is", "exp": "Cell proliferation, extracellular matrix accumulation and cell hypertrophy which all lead to leiomyoma growth are controlled by progesterone directly.", "cop": 2, "opa": "Estrogen", "opb": "Progesterone", "opc": "IGF - I & II", "opd": "Corticosterone", "subject_name": "Gynaecology & Obstetrics", "topic_name": null, "id": "5b904afa-81c8-4311-ae12-945256e7fe0a", "choice_type": "single"} {"question": "Knots in umbilical cord", "exp": "Knots in umbilical cord are caused by fetal movements.\nCommon in monoamniotic twins (dangerous). C-section is not indicated always.", "cop": 1, "opa": "Caused by fetal movement", "opb": "Very high risk of still birth", "opc": "Dangerous in Diamniotic twins", "opd": "Always indication for C-section", "subject_name": "Gynaecology & Obstetrics", "topic_name": null, "id": "8a43d1bf-d46a-4225-8c75-0f530446ef7b", "choice_type": "single"} {"question": "Premature ovarian insufficiency is defined as hypogonadotropic hypogonadism and amenorrhea arising before the age of", "exp": "Premature ovarian failure (POF), traditionally defined as hypogonadotropic hypogonadism and amenorrhea arising before the age of 40.Reference: Clinical gynecologic endocrinology; 10th edition; Chapter 9; Normal and Abnormal Sexual Development; Page no: 463", "cop": 1, "opa": "40 years", "opb": "45 years", "opc": "50 years", "opd": "55 years", "subject_name": "Gynaecology & Obstetrics", "topic_name": "Sexuality and intersexuality", "id": "a860fd8d-2bd3-4830-90b2-e9263aca6214", "choice_type": "single"} {"question": "Most common cause of Breech presentation", "exp": "Ans. (a) Prematurity'", "cop": 1, "opa": "Prematurity", "opb": "Twins", "opc": "Android pelvis", "opd": "Previous LSCS", "subject_name": "Gynaecology & Obstetrics", "topic_name": "Breech Presentations ", "id": "72a9be45-1c1a-42e4-8147-3f3f49ccfbc2", "choice_type": "single"} {"question": "Most common degenerative change in fibroid seen in post menopausal women is", "exp": "Cystic degeneration is most common in post menopausal females.", "cop": 2, "opa": "Hyaline degeneration", "opb": "Cystic degeneration", "opc": "Red degeneration", "opd": "Calcareous degeneration", "subject_name": "Gynaecology & Obstetrics", "topic_name": null, "id": "2dbd5c25-69f4-4c63-836c-569342df799b", "choice_type": "single"} {"question": "The major contribution of the ammiotic fluid after 20 weeks of gestations", "exp": "(Foetal urine): Ref: 37, 214, 218, 464-D Obstetrics - G. Chamberlain 7thAMNIOTIC FLUIDProduced: In early pregnancy, from amnion over placenta and sacLate pregnancy: from fetal urine as wellVolume: Increase to 38 weeks (500 - 1500 ml)50 ml: 12 weeks500 ml: 24 weeks1000 ml: 36 weeksOsmolality: Decreases in late pregnancyCreatinine: Increases in late pregnancyAcid - base: Normally accumulation of CO2 and fixed acid causes a slight reduction in pH (About 7.15 - 7.20)* The foetus drinks about 400 ml of liquer every day at term and equal amount is excreted in the urine* The water in the amniotic fluid is completely changed and replaced in every 3 hours.* Abnormal colour- Meconium stained (green) - Foetal distress Breech or transverse- Golden colour - Rh. Incompatibility- Greenish yellow (Saffron) - Post maturity- Dark coloured - concealed accidental hemorrhage- Dark brown (tobacco juice) - IUD* Amniotic Fluid Index (AFI) - An AFI between 5 and 25 cm is normal and an AFI less than 5 cm (< 10 centile) indicates oligohydramnios and AFI more than 25 cm (> 95 centile for gestational age) indicates Polyhydramnios (464-D)", "cop": 2, "opa": "Ultrafiltrate and maternal plasma", "opb": "Foetal urine", "opc": "Foetal lung fluid", "opd": "Foetal skin", "subject_name": "Gynaecology & Obstetrics", "topic_name": "Miscellaneous (Gynae)", "id": "df5723d4-7935-41e5-9805-630db9cc1b50", "choice_type": "single"} {"question": "Zona hatching occurs", "exp": "Five days after feilisation, blastocyst performs zona hatching and it undergoes degeneration to be replaced by trophoblastic cells", "cop": 2, "opa": "4 days after feilisation", "opb": "5 days after feilisation", "opc": "6 days after feilisation", "opd": "8 days after feilisation", "subject_name": "Gynaecology & Obstetrics", "topic_name": "General obstetrics", "id": "f6f39934-2de5-4c82-9255-c63522bf6012", "choice_type": "single"} {"question": "Subfertility is seen most likely in", "exp": "Of myoma, subfertility is more closely associated with submucous leiomyoma.", "cop": 3, "opa": "Pedunculated leiomyoma", "opb": "Subserosal leiomyoma", "opc": "Submucosal leiomyoma", "opd": "Cervical leiomyoma", "subject_name": "Gynaecology & Obstetrics", "topic_name": null, "id": "16fa6135-4fc3-4034-a104-ce5bb4bf3c92", "choice_type": "single"} {"question": "After a full term normal delivery patient went into shock Most probable cause is", "exp": "Ans. a. Inversion of uterus", "cop": 1, "opa": "Inversion of uterus", "opb": "Post-paum hemorrhage", "opc": "Amniotic fluid embolism", "opd": "Eclampsia", "subject_name": "Gynaecology & Obstetrics", "topic_name": null, "id": "3c7535f8-11eb-4b27-84d2-36eb9f38701a", "choice_type": "single"} {"question": "Recruitment of follicles is caused by", "exp": "FSH is responsible for recruitment of follicles.", "cop": 2, "opa": "LH", "opb": "FSH", "opc": "Inhibin β", "opd": "GnRH", "subject_name": "Gynaecology & Obstetrics", "topic_name": null, "id": "f175cb06-0cdb-48da-a01b-9d85f235c8a8", "choice_type": "single"} {"question": "Risk factors for uterine fibroid is", "exp": "Age and race are risk factors for fibroid.\n\nReproductive age group\nAfrican-american women.", "cop": 2, "opa": "Pregnancy", "opb": "African-american woman", "opc": "Breast feeding", "opd": "Adolescence", "subject_name": "Gynaecology & Obstetrics", "topic_name": null, "id": "46953e3d-a4ce-4398-afa2-459fb0b6a8b6", "choice_type": "single"} {"question": "Dysgerminoma is", "exp": "Dysgerminoma is germ cell tumor of ovary.", "cop": 3, "opa": "Epithelial ovarian tumor", "opb": "Sex cord stromal tumor", "opc": "Germ cell tumor", "opd": "Metastatic ovarian tumor", "subject_name": "Gynaecology & Obstetrics", "topic_name": null, "id": "81624a2c-9c64-4be1-a433-bba4f79e2f25", "choice_type": "single"} {"question": "The double decidual sac sign (DDSS) is the best method for ultrasound diagnosis of", "exp": "Double decidual sac sign on USG - Intra-uterine sac (Eccentrically placed) Outer layer - Decidua Parietalis Inner layer - Decidua Capsularis Indicates - Intrauterine pregnancy Intradecidual ring sign is the first sign of pregnancy on USG.", "cop": 4, "opa": "Monoamniotic Dichorionic twins", "opb": "Diamniotic dichorionic twins", "opc": "Ectopic pregnancy", "opd": "Normal pregnancy", "subject_name": "Gynaecology & Obstetrics", "topic_name": "Diagnosis of Pregnancy", "id": "fe2903f5-7f35-48e5-8d8a-210380d3a1ac", "choice_type": "single"} {"question": "A young female presented with vaginal discharge. Endocervical biopsy revealed chlamydial antigen. Treatment given should be", "exp": "Treatment of Chlamydial infection: Azithromycin 1 g orally as a single doseor Doxycycline (100 mg bid for 7 days) Contact tracing, avoidance of sex or barrier contraceptive is necessary to avoid recurrence. Note: out of the given options, both azithromycin and doxycycline (not metronidazole) can be used for treatment of chlamydia, however since contact tracing is equally impoant, option 2 is preferred. Chlamydia infection :most common cause of tubal block in India", "cop": 2, "opa": "Doxycycline + metronidazole", "opb": "Azithromycin + contact tracing", "opc": "Fluconazole + Azithromycin", "opd": "Ampicillin + Probenecid", "subject_name": "Gynaecology & Obstetrics", "topic_name": "Genital Tract Infections (Too hot to handle!)", "id": "0975b992-26d9-479a-bb10-98343f8097d2", "choice_type": "single"} {"question": "Intrahepatic cholestatis liver disease of pregnancy, plan delivery at", "exp": "Management of Intrahepatic cholestatis of Pregnancy: 1. Ursodeoxycholic acid - increases excretion of bile acis. Also reduces neontal complications 2. Strict maternal and fetal surveillance 3. Termination of pregnancy - Definitive management. Done at 37 weeks after which fetal risk of morbidity and moality keeps on increasing", "cop": 3, "opa": "34 wks", "opb": "36 weeks", "opc": "38 weeks", "opd": "40 weeks", "subject_name": "Gynaecology & Obstetrics", "topic_name": "Labour - normal, abnormal, malposition, malpresentation and their management", "id": "a4f18755-c671-4c4d-9553-9a727cec7070", "choice_type": "single"} {"question": "The following curve is used in evaluation to", "exp": "Ans. C. Rh incompatibilityThe image shows Lily's curve, which is used to predict the severity of fetal hemolysis.The optical density of liquor containing bilirubin pigment is observed at wavelength 250-700nm. In presence of bilirubin, there is a deviation bulge/peak at 450nm. Bigger the bulge, more is the effect on the fetus.", "cop": 3, "opa": "Non progression of labor", "opb": "Fetal distress", "opc": "Rh incompatibility", "opd": "Fetal maturity", "subject_name": "Gynaecology & Obstetrics", "topic_name": "Diagnosis in Obstetrics", "id": "39a0259c-eda2-4093-a69c-e39758623257", "choice_type": "single"} {"question": "Normal junctional Zone thickness of uterus is", "exp": "Junctional zone thickness is normally 5mm.", "cop": 1, "opa": "5 mm", "opb": "10 mm", "opc": "15 mm", "opd": "20 mm", "subject_name": "Gynaecology & Obstetrics", "topic_name": null, "id": "a945448c-93ec-44dc-8423-7aecc0ef8baf", "choice_type": "single"} {"question": "GnRH agonists given along with progestins cause", "exp": "If progestins, are given simultaneously with GnRH agonists, there is typically increased leiomyoma growth.", "cop": 2, "opa": "Decrease in size of fibroid", "opb": "Increase in growth of fibroid", "opc": "No change in size of fibroid", "opd": "Decrease size of only vascular fibroids", "subject_name": "Gynaecology & Obstetrics", "topic_name": null, "id": "96de4f1d-cdb5-4ad1-bf5f-8517e842b5c5", "choice_type": "single"} {"question": "Chassar Moir surgery is done is a case of", "exp": "Chassar moir technique is widely used in the repair of vesicovaginal fistula.\nVagina and bladder are separated all around by flap splitting method and sutured separately in two layers.", "cop": 2, "opa": "Uterine inversion", "opb": "Vesicovaginal fistula repair", "opc": "Ureterovesical fistula repair", "opd": "Retroverted uterus", "subject_name": "Gynaecology & Obstetrics", "topic_name": null, "id": "f82853f9-a33b-4f9d-bffd-d1888f3dc112", "choice_type": "single"} {"question": "Pus discharge (crviitis/urethritis) in a female caused by", "exp": "(Chlamydia): (144-45- Dutta Gynae 6th edition; 123. 160 161 -Dutta Gynae 6th)CHLAMYDIAL INFECTIONS - The infection is mostly localized in the urethra, Bartholins gland and cervix* Mucopurulent cervical discharge, cervical oedema, cervical ectopy cervical friabilityComplication - Urethritis and bartholinitis are manifested by dysuria and purulent discharge, chlamydial cervicitis spreads upwards to produce endometritis and salpingitis.Gardnella (Bacterial) Vaginosis - Polymicrobial infectionCaused by Gard nerella vaginalis (Haermophilus vaginalis). The present concept is that long with G. vaginalis anaerobic organism such as Bacterioids species. Peptococcus species, mobiluncus and Mycoplasma hominis act synergistically to cause vaginal infection* Characterized by white milky (creamy) non viscous discharge adherent to vaginal wall* Fishy odour when mixed with 10% KOH* Presence of Clue cells - the epithelial cells have Puzzy border due to adherence of bacteriaAcute cervicitis - responsible organism are ryogenicAerobic(i) Gram positive organisms are. Staphylococcus(ii) Gram negative are - E.coli, pseudomonas, Klebsiella, N.gonnorhoeaeAnaerobes(i) Gram positive are - Streptococucus. Cl.welchii Cl.tetani(ii) Gram negative are mainly bacteroides group of which Bacteroides fragilisOther common pathogens are: Gonococcus, Chlamydia trachomatis, trichomona, bacterial vaginosis, mycoplasma and HP V. The first one being less common nowdaysClinical Features - Cervix is tender on touch or movements oedematous and congested. Mucopurulent discharge is seen escaping out through the external osChronic cervicitis - potential reservoir for N-gonorrhoea, Chlamydia. HPV, mycoplasma and bacterial vaginosisMucopurulent is the predominant symptomsCandida Vaginitis (Moniliasis) caused by Candida albicans* Pruritis is out of proportion to the discharge* Discharge is thick, curdy white and in flakes (cottage cheese type) often adherent to the vaginal wall", "cop": 2, "opa": "Papilloma", "opb": "Chlamydia", "opc": "Candida", "opd": "Bacterial Vaginosis", "subject_name": "Gynaecology & Obstetrics", "topic_name": "Miscellaneous (Gynae)", "id": "9dc5109b-0035-4aa5-a743-2dc2ca17339d", "choice_type": "single"} {"question": "Not an indication for blood transfusion", "exp": "Indications for Blood Transfusion To correct anemia due to blood loss and to combat postpaum hemorrhage Patient with severe anemia seen in later months of pregnancy (beyond 36 weeks) Refractory anemia: Anemia not responding to either oral or parenteral therapy in spite of correct treatment Associated infection Reference: Textbook of Obsteics; Sheila balakrishnan; 2nd edition; Page no: 288", "cop": 2, "opa": "Severe anemia at 36 weeks", "opb": "Moderate anemia at 24-30 weeks", "opc": "Blood loss anemia", "opd": "Refractory anemia", "subject_name": "Gynaecology & Obstetrics", "topic_name": "Medical, surgical and gynaecological illness complicating pregnancy", "id": "5de0e2e7-3b26-43ec-b8d2-e094a5734651", "choice_type": "single"} {"question": "Select the antibiotic most frequently associated with the Hypoplasia and staining of fetal teeth (SELECT 1 ANTIBIOTIC)", "exp": "Fetal exposure to an antibiotic depends on many factors such as gestational age, protein binding, lipid solubility, pH, molecular weight, degree of ionization, and concentration gradient. Some antibiotics are even concentrated in the fetal compartment. Tetracycline is contraindicated in all three trimesters. It has been associated with skeletal abnormalities, staining and hypoplasia of budding fetal teeth, bone hypoplasia, and fatal maternal liver decompensation. Sulfonamides are associated with kernicterus in the newborn. They compete with bilirubin for binding sites on albumin, thereby leaving more bilirubin free for diffusion into tissues. Sulfonamides should be withheld during the last 2 to 6 weeks of pregnancy. With prolonged treatment of tuberculosis (TB) in pregnancy, streptomycin has been associated with fetal hearing loss. Its use is restricted to complicated cases of TB. Nitrofurantoin can cause maternal and fetal hemolytic anemia if glucose- 6-phosphate dehydrogenase deficiency is present. Chloramphenicol is noted for causing the gray baby syndrome. Infants are unable to properly metabolize the drug, which reaches toxic levels in about 4 days and can lead to neonatal death within 1 to 2 days.", "cop": 1, "opa": "Tetracycline", "opb": "Streptomycin", "opc": "Nitrofurantoin", "opd": "Chloramphenicol", "subject_name": "Gynaecology & Obstetrics", "topic_name": "Fetus & New Born and their Diseases", "id": "aa69e4c6-2f9c-45dd-b7d1-5ff7cdd2dee0", "choice_type": "single"} {"question": "Most common symptom present in undisturbed ectopic", "exp": "Ans. is a i.e. Pain in lower abdomen", "cop": 1, "opa": "Pain in lower abdomen", "opb": "Amenorrhea", "opc": "Bleeding P/V", "opd": "Fainting attack", "subject_name": "Gynaecology & Obstetrics", "topic_name": null, "id": "a2b40f72-6a27-4161-a11e-b8abd0c59cc7", "choice_type": "single"} {"question": "Latzko procedure is done in case of", "exp": "Latzko procedure is surgical technique for vesicovaginal fistula repair.", "cop": 3, "opa": "Uterine inversion", "opb": "Retroverted uterus", "opc": "Vesicovaginal fistula repair", "opd": "Urethrocele repair", "subject_name": "Gynaecology & Obstetrics", "topic_name": null, "id": "6ed26a9d-569e-4999-becc-8d00307e55c7", "choice_type": "single"} {"question": "Investigation of choice to diagnose congenital malformations", "exp": "MRI is the investigation of choice to diagnose congenital malformaiton.", "cop": 3, "opa": "USG", "opb": "HSG", "opc": "MRI", "opd": "CT", "subject_name": "Gynaecology & Obstetrics", "topic_name": null, "id": "a524efbb-8a52-4298-9f4f-40790d95c8b9", "choice_type": "single"} {"question": "Investigation of choice in cholestasis of pregnancy", "exp": "Serum bile acid levels Obstetric cholestasis (also known as intrahepatic cholestasis of pregnancy or cholestasis of pregancy) is a liver disease unique to pregnancy which presents with pruritus. Clinical features Obstetric cholestasis most commonly presents in the third trimester (80% develop symptoms after 1st week). The pruritus is often severe and typically the palms of the hands and the soles of the feet are involved but the legs, thighs arms, backs, breasts and abdomen are also repoed. The diseaes tends to recur in every pregnancy and so a pa history of the condition is impoant. The disease is more common among multiple gestations. Other signs and symptoms of liver disease are not observed. Additional symptoms which may be repoed in common with other forms of cholestasis are right upper quadrant pain, pale stools and dark urine. Changes in liver function in obstetric cholestasis The most common abnormalities in liver function are - Increase in ALT (seen in 100% cases) - Increase in AST (seen in 99% cases) - Increase in bile acids (seen in 92% cases) - Elevated bilirubin (22% of cases) - Increased GGT (39% cases) Elevations in alkaline phosphatase are seen in cholestatic disease such as obstetric cholestasis as it is released from the damaged liver but because the enzyme is also released from the placenta in normal pregnancies, measurements are less helpful than outside the pregnancy. Serum bile acid test is the most accurate determinant of cholestasis of pregnancy. The problem with the bile acid test is that only a small number of labs around the world have the equipment necessary to perform it which can result in a considerable delay for women experiencing symptoms of the conditions. Liver biopsy in a case of obstetric cholestasis: ? Liver biopsy demonstrates: ? - Acinar cholestasis with centrilobular bile staining and bile plugs in the canaliculi. Liver biopsy is the definitive diagnostic test of the disease. Postnatal course in obstetric cholestasis - Biochemical abnormalities and pruritius in obsetric cholestasis persists until delivery following which resolution occurs. - Pruritus is typically absent within 48 hour after delivery usually improved within 1 week. - Liver function tests becomes normal within 6 weeks. Complications of obstetric cholestasis - Meconium stained liquor - Premature delivery - Fetal distress - Post paum hemorrhage Treatment of obstetric cholestasis : - i) Antihistamines - Chlorpheniramine can provide symptomatic relief in pruritus. ii) Topical treatments - If the skin is well moistured pruritus may be relieved. iii) Ursodeoxycholic acid iv) S-Adenosyl methiomine v) Dexamethasone - Dexamethasone in high dose results in resolution of all pruritus cases and significant reduction in total bile acids and ALT. vi) Vitamin K - In obstetric cholestasis there is malabsorption and subsequent deficiency of vitamin K. This will increase the risk of postpaum hemorrhage. - Vitamin K injection is given to reduce the risk of PPH.", "cop": 2, "opa": "Serum biliruhin levels", "opb": "Serum bile acids levels", "opc": "Serum alkaline phosphatase levels", "opd": "Serum glutathiones transferase levels", "subject_name": "Gynaecology & Obstetrics", "topic_name": null, "id": "f4234de3-67c2-4774-8e55-5bc34a7ec714", "choice_type": "single"} {"question": "The gene coding for androgen receptors is located on", "exp": "C i.e. Long arm of X-Chromosome The androgen receptor is a typical member of the steroid/ thyroid family and is encoded by a gene on the long arm of X-chromosomeQ.", "cop": 3, "opa": "Sho arm of X- chromosome", "opb": "Sho arm of Y- chromosome", "opc": "Long arm of X- chromosome", "opd": "Long arm of Y- chromosome", "subject_name": "Gynaecology & Obstetrics", "topic_name": null, "id": "58c63f00-4bfe-40e3-8f5a-171cd8f59544", "choice_type": "single"} {"question": "Maximum chances of ureteric injury are with;", "exp": "Weheim's hysterectomy:includes Radical abdominal hysterectomy Bilateral extra peritoneal dissection Excision of iliac and pelvic lymph nodes for cancer of the cervix Ureter injury is the major complication Ref : Shaw Gynecology 17e pg 526.", "cop": 2, "opa": "TAH", "opb": "Weheims hysterectomy", "opc": "Anterior colporrhaphy", "opd": "Vaginal hysterectomy", "subject_name": "Gynaecology & Obstetrics", "topic_name": "Gynaecological diagnosis and operative surgery", "id": "fb1cc4f3-84a2-4064-9260-413f30f9338d", "choice_type": "single"} {"question": "Most common uterine anomaly is", "exp": "Incidence of uterine anomalies: Bicornuate uterus - 35-40% Arcuate uterus - 15% Uterus didelphys - 10% Uterine septum - 5-10% Ref: Shaw Gynecology 17 e pg 72.", "cop": 4, "opa": "Uterus didelphys", "opb": "Uterus pseudodidelphys", "opc": "Arcuate uterus", "opd": "Bicornuate uterus", "subject_name": "Gynaecology & Obstetrics", "topic_name": "Congenital malformations", "id": "86cdf94e-f897-4d6f-bb48-cb1c0e5eedaf", "choice_type": "single"} {"question": "Risk factor for breech presentation is", "exp": "Risk factors for breech presentation\n\nOligohydramnios \nFundal placental implantation \nPlacenta previa \nHydrocephaly\nTwin gestation\nAnencephaly\nPrior breech delivery", "cop": 2, "opa": "Maternal diabetes", "opb": "Oligohydramnios", "opc": "Anterior placental implantation", "opd": "Preeclampsia", "subject_name": "Gynaecology & Obstetrics", "topic_name": null, "id": "7df170b2-9f13-4ebb-bf1a-b4b214f22e63", "choice_type": "single"} {"question": "Most common cause aof ambigious genitalia in a female child is", "exp": "CONGENITAL ADRENAL HYPERPLASIA\n\nClinical presentation: Ambiguity of sex detected at birth are due to androgenital syndrome unless proved otherwise. Hirsutism and amenorrhea may be presenting features around puberty in milder form\n\nDiagnosis at birth\n\nAn enlarged clitoris\nPresence of penile urethra or hypospadius\nHyponatremia,hypakalemia and hypotension\nFusion of labia minora", "cop": 4, "opa": "Placenta steriod sulfatase deficiency", "opb": "Fetal aromatase deficiency", "opc": "Wnt4 mutation", "opd": "Congenital adrenal hyperplasia", "subject_name": "Gynaecology & Obstetrics", "topic_name": null, "id": "4c3d71af-6e8c-4e22-a7e4-8567139b827f", "choice_type": "single"} {"question": "Total lung capacity during pregnancy is", "exp": "Total lung capacity: the combination of FRC and inspiratory capacity is unchanged or decreases by less than 5 percent at term(Ref: William's Obstetrics; 25th edition)", "cop": 1, "opa": "Unchanged", "opb": "Decreases", "opc": "Increases", "opd": "Increases in the early pregnancy followed by decreasing as the pregnancy progresses", "subject_name": "Gynaecology & Obstetrics", "topic_name": "All India exam", "id": "18e2c5d4-8bd7-403a-825f-aa127b6ccb3c", "choice_type": "single"} {"question": "Polyhydramnios is NOT seen with", "exp": "(A) Renal agenesis # POLYHYDRAMNIOS: The underlying cause of the excessive amniotic fluid volume is obvious in some clinical conditions and is not completely understood in others.> Causes include the following: Multiple pregnancy: Twin gestation with twin-to-twin transfusion syndrome (increased amniotic fluid in the recipient twin and decreased amniotic fluid in the donor) or multiple gestations> Fetal anomalies, including esophageal atresia (usually associated with a tracheoesophageal fistula), tracheal agenesis, duodenal atresia, and other intestinal atresias CNS abnormalities Anencephaly; Open spina bifida; and neuromuscular diseases that cause swallowing dysfunction; Congenital cardiac-rhythm anomalies associated with hydrops, fetal-to-maternal hemorrhage, and parvovirus infection; Facial clefts & neck masses; Hydrops fetalis> Chromosomal abnormalities, most commonly trisomy 21, followed by trisomy 18 and trisomy 13.> Fetal akinesia syndrome with absence of swallowing> Placental: Chorioangioma of the placenta> Maternal: Poorly controlled maternal diabetes mellitus (Oligohydramnios may also be seen if severe vascular disease is present.) & cardiac or renal disease> Oligohydramnios:> Fetal urinary tract anomalies, such as renal agenesis, polycystic kidneys, or any urinary obstructive lesion (E.g., posterior urethral valves)> PROM and chronic leakage of the amniotic fluid> Chorioamnionitis is an additional important maternal complication from oligohydramnios due to rupture of the membranes, which has an incidence of 21-74%.> The earlier chorioamnionitis occurs in pregnancy, the greater the fetal risk of bronchopulmonary dysplasia (BPD), neurologic complications, pulmonary hypoplasia, and, in severe cases, respiratory failure in the neonate.> Placental insufficiency, as seen in pregnancy-induced hypertension (PIH), maternal diabetes, or postmaturity syndro ne when the pregnancy extends beyond 42 weeks' gestation> Maternal use of prostaglandin synthase inhibitors or ACE inhibitorsHYDRAMNIOSOligo hydramnios (< 100 ml AF)Polyhydramnios (> 2000 ml AF)Potter syndrome (renal agenesis)Obstruction to GIT (E.e. Esophageal atresia) 100%Urinary tract obstructionAnencephaly (in 50%)Placental insufficiency with IUGROpen spina bifidaBreechMaternal DM", "cop": 1, "opa": "Renal agenesis", "opb": "Anencephaly", "opc": "Multiple pregnancy", "opd": "Breech", "subject_name": "Gynaecology & Obstetrics", "topic_name": "Miscellaneous (Obs)", "id": "92626a6e-22ff-43ab-86b8-4b648de605e6", "choice_type": "single"} {"question": "Difference between ectopic pregnancy and threatened aboion is", "exp": "In Threatened aboion , uterus corresponds to the period of amenorrhea In ectopic pregnancy, uterus is normal or slightly bulky. Ref: Dutta Obs 9e pg171.", "cop": 3, "opa": "Minimal vaginal bleeding", "opb": "Pain follows bleeding", "opc": "Uterine size corresponds to gestational age", "opd": "Manifests at an early age", "subject_name": "Gynaecology & Obstetrics", "topic_name": "General obstetrics", "id": "e8e4d723-55fc-40fe-8e77-95f9ae842f01", "choice_type": "single"} {"question": "During the active phase of labour, the minimum effective dilatation of the cervix in primigravida should be at the rate of", "exp": "Dilatation of cervix1cm/hr in primigravidae1.5cm/hr in multigravidaeD.C.DUTTA&;S TEXTBOOK OF GYNECOLOGY, Pg no;130,7th edition", "cop": 2, "opa": "0.5 cm/hour", "opb": "1 cm/hour", "opc": "1.5 cm/hour", "opd": "2.5 cm/hour", "subject_name": "Gynaecology & Obstetrics", "topic_name": "General obstetrics", "id": "69ed999b-e067-410e-83e2-98ae5196be48", "choice_type": "single"} {"question": "A 26 weeks pregnant female presented with HTN for the first time. There is no proteinuria. Diagnosis of such condition", "exp": "Gestational hypeension - bp > 140/90 , > 20weeks of gestation Pre eclamptic toxemia - gestational hypeension + proteinuria > 300mg Eclampsia - above 2 + convulsions Chronic hypeension - hypeension before she is pregnant", "cop": 3, "opa": "Chronic hypeension", "opb": "Eclampsia", "opc": "Gestational Hypeension", "opd": "Preeclampsia", "subject_name": "Gynaecology & Obstetrics", "topic_name": "FMGE 2019", "id": "f853eae9-8a48-464a-b9d5-b5bfe14316df", "choice_type": "single"} {"question": "Women with infeility with B/L cornual block on HSG, Next investigation for management is", "exp": "HSG is the initial test for tubal patency The cornua of the uterus may go into spasm, as a result of which the dye may not enter the tubes at all. This may be interpreted as a tubal block, when the tubes are actually open. If the spill is \"loculated\", the presence of adhesions can be suspected, but not confirmed. If the HSG shows that the tubes are closed, then one must do a laparoscopy to confirm this diagnosis. Laparoscopy with Chromopeubation is confirmative for making a diagnosis of tubal disease.", "cop": 4, "opa": "Tuboplasty", "opb": "IVF", "opc": "Hydrotubation", "opd": "Laparoscopy and Chromotubation", "subject_name": "Gynaecology & Obstetrics", "topic_name": "Infeility (Eggs'plantation!)", "id": "f7bc2e14-e94f-4bf9-b026-b351b65ecda9", "choice_type": "single"} {"question": "The usual dose of radiation delivered to point A in cervical cancer management is", "exp": "The usual doses delivered are 7,000 to 8,000 cGy to point A (defined as 2 cm superior to the external cervical os and 2 cm lateral to the internal uterine canal) and 6,000 cGy to point B (defined as 3 cm lateral to point A), limiting the bladder and rectal dosage to less than 6,000 cGy.To achieve this level, it is necessary to adequately pack the bladder and bowel away from the intracavitary sourceReference: Novak's gynaecology; 14th edition; Chapter 35; Uterine cancer", "cop": 3, "opa": "2000 to 4000", "opb": "4000 to 6000", "opc": "7000 to 8000", "opd": "8000 to 10000", "subject_name": "Gynaecology & Obstetrics", "topic_name": "Gynaecological oncology", "id": "bbaa857f-9858-4d68-a471-365b643ceb34", "choice_type": "single"} {"question": "Fetal blood loss occurs in", "exp": "Vasa pre occurs when there is velamentous inseion of placenta i.e. the umbilical cord vessels are splitting before getting attached to the placenta. The inseion of the cord may be close to the internal os. The fetal head can compress the cord leading to rupture of vessels. In this case the blood loss occurs from fetal vessels.", "cop": 1, "opa": "Vasa pre", "opb": "Placenta pre", "opc": "Cord prolapse", "opd": "Abruption", "subject_name": "Gynaecology & Obstetrics", "topic_name": "Antepaum Haemorrhage", "id": "69d9da0f-9878-4fe9-bb35-5b08a7ed9d03", "choice_type": "single"} {"question": "Indications of medical management In ectopic", "exp": "Criteria for selection Hemodynamically stable No intrauterine pregnancy on ultrasound No tubal rupture Size of ectopic less than 4 cm If there is foetal cardiac activity use with caution Beta hcg level preferably less than 3000 IU/L (refer pgno:162 sheila 2 nd edition)", "cop": 2, "opa": "Presence of fetal hea activity", "opb": "Size <4 cm", "opc": "Gestation <6 weeks", "opd": "a-hCG>1500", "subject_name": "Gynaecology & Obstetrics", "topic_name": "General obstetrics", "id": "3b448b21-1ae6-40bb-b6ee-4f4393a2fcad", "choice_type": "single"} {"question": "Veical transmission of HIV is most common in", "exp": "Veical transmission to the neonates is about 14-25% Transmission of HIV2 is less frequent than HIV1 Transplacental transmission 20% before 36 weeks, over 80% occur around the time of labor and delivery Risk of veical transmission is directly related to maternal viral load and inversely to the maternal immune status Maternal antiretroviral therapy reduces the risk of veical transmission by 70%. Ref: Dutta Obs 9e pg 282.", "cop": 4, "opa": "1st trimester", "opb": "2nd trimester", "opc": "3rd trimester", "opd": "During labour", "subject_name": "Gynaecology & Obstetrics", "topic_name": "Medical, surgical and gynaecological illness complicating pregnancy", "id": "86f7c3f8-1a62-4c46-a99b-ede99cdb12de", "choice_type": "single"} {"question": "Uterine contraction pressure is measured in terms of", "exp": "Montevideo units are calculated by subtracting the baseline uterine pressure from the peak contraction pressure for each contraction in a 10minute window and adding the pressures generated by each contraction.For example, three contractions in 10 minutes, each of 50 mm Hg intensity, would equal 150 Montevideo units.Clinical labor usually commences when uterine activity reaches values between 80 and 120 Montevideo units. This translates into approximately three contractions of 40 mm Hg every 10 minutes.Reference: William&;s Obstetrics; 24th edition; Chapter 23; Abnormal Labor", "cop": 3, "opa": "mmHg", "opb": "Cm of H2o", "opc": "Montevideo units", "opd": "Joules/kg", "subject_name": "Gynaecology & Obstetrics", "topic_name": "General obstetrics", "id": "362562d7-d4b3-42b0-8094-e13596267540", "choice_type": "single"} {"question": "The definite sign of placental separation in stage 3 of paurition is", "exp": "Placental separationMarked retraction reduces effectively the surface area at the placental site to about its half. But as the placenta is inelastic it cant keep pace with such an extent of diminution resulting in its buckling.The detachment of the placenta from the uterine attachment stas in the center resulting in opening up of few uterine sinuses and accumulation of blood behind the placenta.Marginal separation occursD.C.DUTTA&;S TEXTBOOK OF GYNECOLOGY, Pg no:122,7th edition", "cop": 3, "opa": "Uterine contraction", "opb": "Lengthening of cord", "opc": "Filling of placenta in vagina", "opd": "Increase of BP", "subject_name": "Gynaecology & Obstetrics", "topic_name": "General obstetrics", "id": "1455e433-33fb-4df0-addb-e3ed52c6ac4e", "choice_type": "single"} {"question": "Meconium ileus occurs in", "exp": "(A) Cystic fibrosis # CYSTIC FIBROSIS: CF is an autosomal resessive disorder with mutation in gene for the protein cystic fibrosis transmembrane conductance regulator (CFTR). This protein is required to regulate the components of sweat, digestive fluids, and mucus.> Symptoms include: Intestinal obstruction may be an initial manifestation in the neonatal period (meconium ileus). This may be due to the presence of abnormal protein and mucoprotein secreted by the pancreatic enzymes. Later these children tend to retain the food residue in the ileum, caecum and colon and these may form firm masses. Impaction of the faeces causes intestinal obstruction. lleocaecal intussusception may occur in less than 1 percent of the patients. Two cardinal symptoms of the disease are chronic diarrhea with massive steatorrhea and recurrent respiratory tract infections Failure to thrive is also a prominent feature. Rectal prolapse occurs frequently. Biliary cirrhosis of liver is also often observed.> Crohn's disease: The disease is most often seen in teenagers but it has been also reported during infancy. Crampy abdominal pain and dysphagia are the usual early symptoms. Pain may be paraumbilical or may be localized to the right lower quadrant of the abdomen. Chronic diarrhea is the presenting symptom in nearly one-third of cases. Anemia, weight loss, fever are complained of in most cases. Extra-intestinal symptoms of arthritis, uveitis, stomatitis and erythema nodosum may be seen in some cases.> Ulcerative colitis: There is history of diarrhea with passage of bloody stools, abdominal cramps, tenesmus, fever and anorexia. The onset may either be insidious or explosive with high fever, prostration and continuous bloody diarrhea. There may be extra-intestinal manifestation of arthralgia, liver disease or pyoderma.", "cop": 1, "opa": "Cystic fibrosis", "opb": "Hypothyroidism", "opc": "Ulcerative colitis", "opd": "Crohn's disease", "subject_name": "Gynaecology & Obstetrics", "topic_name": "Miscellaneous (Obs)", "id": "fc54455b-fe44-44f3-a4e3-f0cd0fa4bb54", "choice_type": "single"} {"question": "Postpaum decidual secretions present are referred to as", "exp": "Lochia refers to vaginal discharge for the first few weeks following delivery. It consists of blood from the placental site along with necrotic decidua. For the first few days following delivery, it is red in color and is termed as lochia rubra. After 3-4days - lochia serosa. After 10th day - lochia alba(refer to pgno: 422 Sheila 2nd edition)", "cop": 1, "opa": "Lochia", "opb": "Bleeding per vaginum", "opc": "Vasa pre", "opd": "Decidua capsularis", "subject_name": "Gynaecology & Obstetrics", "topic_name": "General obstetrics", "id": "0d94f4d2-6710-406e-96b1-74734e0530f9", "choice_type": "single"} {"question": "SRY gene is located on", "exp": "The elaboration of H-Y antigen complex in the sho arm of Y chromosome is known as sex determining region(SRY), which induces testicular development Ovarian determinant : Unless SRY is expressed ovarian development ensues in the presence of XX karyotype SHAW&;S TEXTBOOK OF GYNAECOLOGY,Pg no:106,15th edition", "cop": 1, "opa": "Sho arm of Y chromosome", "opb": "Long arm of Y Chromosome", "opc": "Sho arm of X chromosome", "opd": "Long arm of X chromosome", "subject_name": "Gynaecology & Obstetrics", "topic_name": "Congenital malformations", "id": "fe2def17-cf38-436c-97da-3dfb4eb1c5ea", "choice_type": "single"} {"question": "Definitive criteria for PID is", "exp": "Chronic pelvic inflammatory disease manifest as Hydrosalpinx Chronic pyosalpinx Tubo ovarian abscess or cyst Tubercular tubovarian masses Ref: Shaw Gynecology 17 e pg 340.", "cop": 1, "opa": "Tubo-Ovarian abscess on USG", "opb": "Cervical erosion", "opc": "Temparature > 37.5 degrees", "opd": "Infeility", "subject_name": "Gynaecology & Obstetrics", "topic_name": "Infections of the genital tract", "id": "979fea8a-cdd3-44b4-ade5-8467cfd73065", "choice_type": "single"} {"question": "Minimum effective dose of Ethinyl estradiol in combination oral pills is", "exp": "Ans. is a i.e. 20p.g -intensive pharmacological research clinical trials conducted to minimise the adverse effects of estrogen without reducing the contraceptive efficacy, resulted in lowering the dose of oestrogen to a minimum of 20gg or even 15gg.\" Examples of pills with 20p.g estrogen : Femilon : Loette Estrogen (EE) = 2opg Estrogen (EE) = 20pg Progestin (Desogestrel) = 0.15mg Progestin (Levonorgestrel) = 0.1mg Benefits of Low dose OCP's Decreased risk of Thromboembolic events with low dose OCP's.deg Note : Thrombosis risk is apparent by 4 months after staing estrogen containing OC's and does not increase fuher with continued use. Risk is highest during the first year of useq Decreased risk of high blood pressure (as compared to traditional high dose OCP's) Minimum adverse effect on lipid profile Less complains of Nausea and vomiting (as these complications are related to Estrogen component). The beneficial effects and efficacy of low dose OCP's is similar to traditional high dose OCP's whereas side effects have decreased. Extra Edge : Once a month (long acting pill). Contains : Ouniestrol (long acting estrogen) + sho acting progestin.", "cop": 1, "opa": "20 pgm", "opb": "35 pgm", "opc": "50 pgm", "opd": "75 pgm", "subject_name": "Gynaecology & Obstetrics", "topic_name": null, "id": "9c34bfe3-70c1-42e6-b286-9bb04e99b57e", "choice_type": "single"} {"question": "In stage III ovarian cancer patient with upper abdominal disease < 5cm, management of choice is", "exp": "Management of Advanced stage diseaseIncludes stage III and IVDebulking surgery versus neoadjuvant chemotherapy followed by Postoperative chemotherapyPrimary cytoreduction surgery is the preferred in1. Retroperitoneal only disease preoperatively 2. Patients with stage IIIC or IV disease with good performance status who have < 5cm upper abdominal disease 3. Patients who are considered ideal for intraperitoneal chemotherapy if optimally reducedNeoadjuvant chemotherapy and interval debulking surgery is preferred in1. Patients with bulky upper abdominal peritoneal disease >5cm in diameter2. Medically unfit for surgeryPostoperative chemotherapySystemic chemotherapy with 6 cycles of carboplatin and paclitaxelPostoperative radiotherapy1. Not effective compared to chemotherapy2. Used clear cell or mucinous carcinomasMAINTENANCE THERAPY1. No role after the complete clinical response 2. Drugs used: 3month or 12month paclitaxel every 28days", "cop": 1, "opa": "Debulking surgery", "opb": "Mantle cell irradiation", "opc": "Abdomino-pelvic radiotherapy", "opd": "Neo-adjuvant chemotherapy", "subject_name": "Gynaecology & Obstetrics", "topic_name": "Gynaecological oncology", "id": "fd011c29-dc35-41a9-ba97-23ffdcf9a75d", "choice_type": "single"} {"question": "Mrs. X, 25years old with a history of 2 previous pregnancies, and now 28weeks of gestation her gravidity is", "exp": "Nulligravida: a woman who currently is not pregnant and has never been pregnant.Gravida: a woman who currently is pregnant or has been in the past, irrespective of the pregnancy outcome. With the establishment of the first pregnancy, she becomes a primigravida, and with successive pregnancies, a multigravida.Reference: William's obstetrics; 25th edition", "cop": 4, "opa": "Nulligravida", "opb": "G1", "opc": "G2", "opd": "G3", "subject_name": "Gynaecology & Obstetrics", "topic_name": "General obstetrics", "id": "89449b3f-aee9-4894-a02e-125138c9a4c4", "choice_type": "single"} {"question": "Management of stage II endometrial cancer is", "exp": "Stage II endometrial cancer (cervix involved)\nModified radical hysterectomy is done.\nSimple hysterectomy will have high relapse rate . Chemotherapy is not indicated.", "cop": 3, "opa": "TAH + BSO", "opb": "Unilateral salpingo oophorectomy + TAH", "opc": "Modified radical hysterectomy", "opd": "Chemotherapy only", "subject_name": "Gynaecology & Obstetrics", "topic_name": null, "id": "a99f2af5-97ca-4765-840f-580eba64cdb6", "choice_type": "single"} {"question": "Twin Peak sign is characteristic of", "exp": "Determination of ZygositySonographic Determination:ChorionicityDividing inter-twin membrane thicknessSignDichorionicity> 2mmTwin peak sign or Lambda signMonochorionicity< 2mmT signReference: William's Obstetrics; 25th edition; Chapter 45; Multifetal pregnancy", "cop": 1, "opa": "Dichorionic Diamniotic", "opb": "Monochorionic Diamniotic", "opc": "Monochorionic Monoamniotic", "opd": "Conjoined twins", "subject_name": "Gynaecology & Obstetrics", "topic_name": "General obstetrics", "id": "49c53dfd-5e6c-4433-bb82-23e58094946a", "choice_type": "single"} {"question": "For hysteroscopy, following are/is used", "exp": "Distending Media Uterine cavity is distended with a media to seperate the uterine walls and have a panoramic view during hysteroscopy CO2: Commonly used for diagnostic purposes. It is solouble in blood and is safe Glycine1.5%: Excellent visualisation, hyperosmolar Mannitol 5%+ Glycine 2.2% Normal saline D.C.DUTTA&;S TEXTBOOK OF GYNECOLOGY; 6th edition; Pg no:620", "cop": 3, "opa": "Distilled Water", "opb": "Air", "opc": "Glycine", "opd": "O2", "subject_name": "Gynaecology & Obstetrics", "topic_name": "Gynaecological diagnosis and operative surgery", "id": "4bd68136-49b5-43e9-88ff-99fa03bd1d95", "choice_type": "single"} {"question": "MgSO4 have no role in prevention of", "exp": "Magnesium sulphate: - Mechanism of action:- * Reduced presynaptic release of the neurotransmitter glutamate * Blockade of glutaminergic n-methyl-d-aspaate (nmda) receptor * Potentiation of adenosine action * Improved calcium buffering by mitochondria * Blockage of calcium entry voltage-gated channels Used for: - * Severe preeclampsia * Recurrent eclamptic seizures * Neuroprotection Must check: - * Urine output * Respiratory rate * Patellar reflexes Ref:- Williams obstetrics 25th Edition; Pg num:- 38/68", "cop": 2, "opa": "Seizures in severe pre-eclampsia", "opb": "RDS in premature baby", "opc": "Recurrent seizures in eclampsia", "opd": "Neuroprotection", "subject_name": "Gynaecology & Obstetrics", "topic_name": "recent papers obst", "id": "212c0514-17d8-430c-a45b-10b2bdfc7c86", "choice_type": "single"} {"question": "Bilateral germ cell tumour is .", "exp": "Ans. is c i.e. Embryonal cell carcinoma Friends, this is one of those type of questions where we will derive a correct answer by ruling out other options. Let see, Option 'a' : Dysgerminoma \"They are bilateral in about 10% of cases\". Option 'b' : Immature teratoma \"In a premenopausal patient whose lesion appears to be confined to a single ovary, unilateral oophorectomy and surgical staging should be performed. For a postmenopausal patient, a total abdominal hysterectomy and bilateral salpingo-oophorectomy may he performed. Contralateral involvement is rare, and routine resection or wedge biopsy of the contralateral ovary is unnecessary.\" Coming on to Option 'c' with embryonal cell carcinoma. It is a rare type of Germ cell tumor and so is the information available on at. The only information I could lay my hands was : Options 'c': Embryonal cell carcinoma The primary lesions tend to be large, and about two thirds are confined to one ovary at the time of diagnosis Option 'd' : Endodermal Sinus Tumour (EST) It is unilateral in 100% of cases. Considering all above options I choose embryonal cell carcinoma as the option of choice I have provided you with all related references, you are free to have your opinion.", "cop": 3, "opa": "Dysgerminoma", "opb": "Immature teratoma", "opc": "Embryonal cell carcinoma", "opd": "Endodermal sinus tumour", "subject_name": "Gynaecology & Obstetrics", "topic_name": null, "id": "796055ca-b8b5-41b6-a03f-e12ff6ca451e", "choice_type": "single"} {"question": "A woman with the history of 2 pregnancies, her gravidity is", "exp": "Gravida: a woman who currently is pregnant or has been in the past, irrespective of the pregnancy outcome.With the establishment of the first pregnancy, she becomes a primigravida, and with successive pregnancies, a multigravida.Reference: William's obstetrics; 25th edition", "cop": 3, "opa": "G0", "opb": "G1", "opc": "G2", "opd": "G3", "subject_name": "Gynaecology & Obstetrics", "topic_name": "General obstetrics", "id": "c5d8b48a-4133-4eb4-a6df-2147178ce8f9", "choice_type": "single"} {"question": "Most common uterine malformation associated with poor obstetric outcome", "exp": "Septate Uterus Paial lack of resorption of the midline septum between the two mullerian ducts results in fibromuscular defects which range from a slight midline septum to a significant midline division of endometrial cavity Defect is not a cause of infeility, but once pregnant, greater the septum - greater the risk of recurrent spontaneous miscarriage, more in second trimester Complete septate uterus is associated with high risk of spontaneous miscarriage, Preterm labor, FGR and Breech presentation Outcomes are excellent with treatment by hysteroscopy Post treatment miscarriage rates are approximately 10% compared to 90% pretreatment with a complete septum Surgical procedure is not indicated in arcuate uterus Clinical Gynecologic Endocrinology and Infeility; Eigth Edition; Chapter 4", "cop": 3, "opa": "Bicornuate", "opb": "Unicornuate", "opc": "Septate", "opd": "Didelphys", "subject_name": "Gynaecology & Obstetrics", "topic_name": "Congenital malformations", "id": "0606cc0d-e8c2-44d0-bf65-1b384edc2325", "choice_type": "single"} {"question": "Gold standard investigation for uterine cavity evaluation is", "exp": "Hysteroscopy is considered the gold standard for uterine cavity evaluation because it allows for direct visualization. The procedure involves inseion of an endoscope through the cervical canal into the uterine cavity and instillation of distension media to allow for visualization.To optimize visualization of the endometrial cavity and avoid performing the procedure during early pregnancy, hysteroscopy is typically scheduled during the early- to mid follicular phase of the cycle. Disadvantages to the procedure include poor visualization when uterine bleeding is present and the inability to evaluate structures outside the uterine cavity, including those in the myometrium and adnexa. Reference: Novak's Gynecology; 14th edition; Chapter 32; Infeility and Assisted Reproductive Technology", "cop": 4, "opa": "Ultrasonography", "opb": "Saline sonography", "opc": "Hysterosalpingography", "opd": "Hysteroscopy", "subject_name": "Gynaecology & Obstetrics", "topic_name": "Infertility", "id": "93406edb-ce3e-4bb5-9ea9-cf66c8c817b9", "choice_type": "single"} {"question": "The triangular area bounded by clitoris, fourchette and labia minora is", "exp": "Vestibule : It is a triangular area bounded Anteriorly by clitoris Posteriorly by fourchette Either side by labia minora Ref: D.C. Dutta's Textbook of Gynaecology, 6th edition, page no: 2", "cop": 3, "opa": "Fossa navicularis", "opb": "Fouchette", "opc": "Vestibule", "opd": "Vulva", "subject_name": "Gynaecology & Obstetrics", "topic_name": "Anatomy of the female genital tract", "id": "f578dce5-32d3-4b1c-9d20-fd7ca2f4c7f1", "choice_type": "single"} {"question": "Laparoscopy detects", "exp": "Endometriosis", "cop": 1, "opa": "Endometriosis", "opb": "Ca uterus", "opc": "Ca cervix", "opd": "Ca rectum", "subject_name": "Gynaecology & Obstetrics", "topic_name": null, "id": "aa080f81-631e-4963-b512-622b581db66b", "choice_type": "single"} {"question": "Urge urinary incontinence is", "exp": "Urge urinary incontinence is urinary leakage preceeded by strong desire to void.", "cop": 2, "opa": "Involuntary urinary leak with increase in abdominal pressure", "opb": "Involuntary urinary leak preceeded by strong imminent need to void", "opc": "Urinary urgency with increased daytime frequency and nocturia", "opd": "Detrusor sphincter dyssynergia", "subject_name": "Gynaecology & Obstetrics", "topic_name": null, "id": "0f25b2ee-0593-4e20-a1f0-1da753373107", "choice_type": "single"} {"question": "Fetal marker of growth restriction in USG is", "exp": "One of the most frequent perinatal complications in high‐risk pregnancy is fetal growth retardation. \nThe aim of the study was the development of instrumental and laboratory criteria for the development of placental\ninsufficiency and IUGR on a preconception stage and in the early stages of gestation.", "cop": 1, "opa": "AC", "opb": "AFI", "opc": "FL", "opd": "BPD", "subject_name": "Gynaecology & Obstetrics", "topic_name": null, "id": "b4db2848-55ce-4115-8a86-f3e92ce0f19f", "choice_type": "single"} {"question": "A 26 years old female at 39 weeks gestation presents with gush of fluid and regular contractions. On examination, she is grossly ruptured, contraction every two minutes and a cervical dilatation of 4 cms. The fetal heart rate is 140/min and reactive. She was admitted for labor and delivery. Over the next four hours, the cervical dilatation progressed to 9 cms. In the past hour, the fetal heart rate increased from the baseline of 140/min to 160/min. There is moderate to severe variable decelerations are seen with each contraction. The fetal heart rate does not responds to scalp stimulation. It was decided to proceed for caesarean section. The most important reason for the decision is", "exp": "Answer: d) Non reassuring fetal heart rate tracingCardio toco graphy-Continuous electric fetal monitoring:CharacterNormalBaseline FHR110-150bpmBaseline variability5-25bpmAcceleration2 in 20 minDecelerationNone or earlyBaseline FHR is the mean level of FHR between the peaks and the depressions in beats per minute (bpm).Baseline variability is the oscillation of baseline FHR excluding the accelerations and decelerations. A base line variability of 5-25 bpm is a sign of fetal wellbeing.Accelerations are increased in FHR by 15 bpm or more lasting for at least 15 secs. Denotes healthy fetus.Deceleration is decrease in FHR below the baseline by 15 bpm or more. Three basic types of deceleration are observed:Early decelerationLate decelerationVariable decelerationType-1 dips.Due to headcompression.Type-ll dips.Due to utero-placentalinsufficiency and fetal hypoxiaIndicates cord compression and may disappear with the change in position of the patient.Pneumonic: (VEAL-CHOP)V- variable decelerationsC- cord compression/ prolapseE- early decelerationsH- Head compressionA- AccelerationsO-OKL- late acceelrationsP-Placental insuffiencyWith each uterine contraction, blood flow to the placenta decreases, and the fetus is exposed to transient hypoxia.As the labor progresses and more and more contractions occur, this hypoxia can eventually lead to a change from aerobic to anaerobic metabolism- fetal acidemia.However, the fetus has a variety of protective mechanisms, including a blood buffering system and the diving reflex (a lowering of the heart rate in times of hypoxic stress), to protect it from becoming dangerously acidemic.Electronic fetal monitoring is not a very specific tool for identifying fetal acidemia.Many fetuses with a non-reassuring fetal heart rate tracing do not have acidemia and are not in distress.However, it can be very difficult to distinguish non-acidemic fetuses with non-reassuring fetal heart rate tracings from acidemic fetuses with non-reassuring fetal heart rate tracings.Thus, the delivery of many fetuses is expedited because of the concern for fetal acidemia when, in fact, the fetus is not acidemic at all.Thus, it is most accurate to state, as is in this case, that the fetus was delivered because of the non-reassuring fetal heart rate tracing.Fetal acidemia is not the reason for delivery. In fact, there is a strong likelihood that this fetus is not acidemic at all.Fetal distress is not the reason for delivery. There is a strong likelihood that this fetus is perfectly healthy and will have high neonatal APGAR scores and no distress at all.Fetal hypoxic encephalopathy is not the reason for delivery. The desire to prevent hypoxic/acidemic damage to organs, including the brain, is the reason for expediting delivery.However, the non-reassuring fetal tracing does not indicate that hypoxic encephalopathy is necessarily occurring.Low neonatal APGAR scores can be a marker of fetal acidemia.However, many fetuses with non-reassuring fetal heart rate tracings do not have low neonatal APGAR scores.", "cop": 4, "opa": "Fetal distress", "opb": "Fetal acidemia", "opc": "Fetal hypoxic encephalopathy", "opd": "Non reassuring fetal heart rate tracing", "subject_name": "Gynaecology & Obstetrics", "topic_name": "Miscellaneous (Gynae)", "id": "8461818f-b58a-46e4-b381-5ad6890a9434", "choice_type": "single"} {"question": "In a female with appendicitis in pregnancy treatment of choice is", "exp": "Acute Appendicitis incidence is about 1 in 1000 pregnancies. It is most common nongyneocological cause of acute abdomen requiring surgery. Management of appendicitis Treatment consists of laparotomy at the earliest oppuunity.Once the diagnosis is suspected it is better to operate rather than to wait until generalised peritonitis has developed.Muscle splitting incision should be made at the point of maximum tenderness.Laparoscopic appendicectomy can be done before 28 weeks of gestation. D.C.DUTTA'S TEXTBOOK OF OBSTETRICS,Pg no:285,9th edition", "cop": 1, "opa": "Surgery at earliest", "opb": "Aboion with appendectomy", "opc": "Surgery after delivery", "opd": "Continue pregnancy with medical Rx", "subject_name": "Gynaecology & Obstetrics", "topic_name": "Medical, surgical and gynaecological illness complicating pregnancy", "id": "763b972e-9c06-4c95-bcb6-7af2e0b91c6f", "choice_type": "single"} {"question": "About Ovary", "exp": "The ovary is attached to Posterior layer of broad ligament by mesovarium To the lateral pelvic wall by infundibulopelvic ligament Uterus by ovarian ligament Ovarian aeries are direct branches of abdominal aoa Ovarian veins drain into Right side drains into inferior vena cava Left side drains into left renal vein Histology of Ovary Surface Epithelium: Single layer of flattened cuboidal to low columnar epithelium that is continuous with the peritoneum at the mesovarium Coex: Composed of specialized stroma and follicles in various stages of development or attrition Medulla: Small poion of ovary in its hilar region and is composed primarily of fibromuscular tissue and blood vessels Few hilar cells homologues to interstitial cells of testes are present in medulla and may rarely cause hilar cell tumour of the ovary", "cop": 1, "opa": "It is attached to the posterior layer of the broad ligament by mesovarium", "opb": "Has hilus cells in the coex", "opc": "Ovarian veins drain into internal iliac veins", "opd": "Is connected to the uterus by infundibulopelvic ligament", "subject_name": "Gynaecology & Obstetrics", "topic_name": "Anatomy of the female genital tract", "id": "c4a307da-8bf3-49ad-83d2-3db367f39141", "choice_type": "single"} {"question": "Common route of spread of puerperal sepsis", "exp": "Puerperal pyrexia – is defined as a rise of temperature reaching 100.4° F (38° C) or more (measured orally) on 2 separate occasions at 24 hours apart (excluding first 24 hours) within first 10 days following delivery.\nAny infection of genital tract which occurs as a complication of delivery is called as Puerperal sepsis.\nMost common site of Puerperal infection – Placental site. In vaginal delivery and uterine incision in cesarean section.\nMost common manifestation of Puerperal infection – Endometritis.\nMost common cause of Puerperal sepsis – Streptococcus.\nMost common route of infection – Direct spread.\nSingle most significant risk factor for development of puerperal sepsis (uterine infection) = Route of delivery (It is M/C in cesarean delivery than vaginal delivery)\nMgt: Clindamycin + Gentamycin ± Ampicillin", "cop": 2, "opa": "Lymphatic", "opb": "Direct invasion", "opc": "Skip lesion", "opd": "Hematogenous", "subject_name": "Gynaecology & Obstetrics", "topic_name": null, "id": "81f4324f-1d76-4bae-bfd7-a620fdaa0f82", "choice_type": "single"} {"question": "Prolonged pregnancy is the one that exceeds", "exp": "The international definition of prolonged pregnancy, endorsed by the American College of Obstetricians and Gynecologists (2016b,d) is one that exceeds 420/7 weeks, namely, 294 days or more from the first day of the last menstrual period.Reference: William's Obstetrics; 25th edition; Chapter 43; Postterm pregnancy", "cop": 4, "opa": "390/7", "opb": "400/7", "opc": "410/7", "opd": "420/7", "subject_name": "Gynaecology & Obstetrics", "topic_name": "General obstetrics", "id": "56d8a8f1-d298-4961-a6d9-786e1e30aed2", "choice_type": "single"} {"question": "Snow storm appearance is seen in", "exp": "Molar pregnancy is an abnormal form of pregnancy in which non - ble feilized egg implants in the uterus and will fill to come to term a molar pregnancy. It Is a gestational trophoblastic disease which grow into a mass in the uterus that has swollen villi on ultrasound mole resemble a bunch of grapes or snow - storm. There is increased trophoblast proliferation and enlarging of the chronic villi.", "cop": 2, "opa": "Ectopic pregnancy", "opb": "Molar pregnancy", "opc": "Incomplete aboion", "opd": "Heterotopic pregnancy", "subject_name": "Gynaecology & Obstetrics", "topic_name": "JIPMER 2018", "id": "c5259d9f-402f-4c1f-88b8-33c948876f37", "choice_type": "single"} {"question": "An elderly multiparous woman with intrauterine foetal death was admitted with strong labour pains The patient suddenly goes in shock with cyanosis respiratory disturbances and pulmonary oedema The most likely clinical diagnosis is;", "exp": "

Amniotic fluid enter the maternal circulation and sets up a DIC,leading to consumptive coagulopathy. Classically a woman in late labour gasps for air,has bronchospasm,become cyanotic and undergoes immediate collapse and cardiorespiratory arrest,usually accompanied by hemorrhage. Reference: Textbook of obstetrics,Sheila Balakrishnan,2nd edition,page no;408 <\\p>", "cop": 3, "opa": "Rupture of uterus", "opb": "Congestive hea failure", "opc": "Amniotic fluid embolism", "opd": "Concealed accidental hemorrhage", "subject_name": "Gynaecology & Obstetrics", "topic_name": "General obstetrics", "id": "2f97a6a5-d024-43b8-9ac9-786bd528402e", "choice_type": "single"} {"question": "Position of fetus definition", "exp": "(C) Oreientation of foetus in relation to pelvis before delivary# In obstetrics, position is the orientation of the fetus in the womb, identified by the location of the presenting part of the fetus relative to the pelvis of the mother. It is the position assumed by the fetus before the process of birth.", "cop": 3, "opa": "Vertex presentation before delivery", "opb": "Breech presentation before delivery", "opc": "Orientation of foetus in relation to pelvis before delivery", "opd": "Orientation of womb in relation to pelvis before delivery", "subject_name": "Gynaecology & Obstetrics", "topic_name": "Miscellaneous (Obs)", "id": "8a0ad558-2313-44b3-9f73-448b274445c8", "choice_type": "single"} {"question": "Endometrial biopsy in infertility\" is done at", "exp": "C. i.e. (21 - 23rd day of menstrual cycle) (222 - Dutta 4th) (142 - Shaw's 14th)ENDOMETRIAL BIOPSY - most reliable method to study the endometrium is by obtaining the material by curettage after dilatation of the cervix usually under general anaesthesiaIndication of Dilatation and CurettageDiagnosticInfertility, DUB, Pathological amenorrhoea, Endometrial tuberculosis, Endometrial carcinoma Postmenopausal bleeding, chorionepitheliomaTherapeuticDUB, Endometrial polyp. Removal of IUD, Incomplete abortion.CombinedDUB. Endometrial polyp* Dilatation and curettage should be carried out in the late premenstrual phase (142 - Shaw's 14th). This is so done because the tubercles are present in superficial layers and are shed during menstruation.* In Infertility biopsy is to be done on 21-23rd day of the cycle, Barrier contraceptive should be prescribed during the cycle to prevent accidental conception. However if the cycle is irregular. It is done within the 24 hours of the period**SubnucIear vacuolation is the earliest evidence appearing 36 -48 hours following ovulation* The curettage material is preserved in 10% formolsaline (Normal saline is suspected tubercular endometritritis) - Short history of the case and first day of last menstrual period specially in infertility cases and DUB should be positively mentioned. (534 - Dutta 4th)Endometrial biopsy - preferably 1 or 2 days before the onset of menstruation (Pre menstrual phase)* *** Post coital test is done usually at 12 - 13 days after LMP**", "cop": 3, "opa": "3-6 days of menstrual cycle", "opb": "5-7 days of menstrual cycle", "opc": "21 - 23rd day of the menstrual cycle", "opd": "24 - 26th days of the menstrual cycle", "subject_name": "Gynaecology & Obstetrics", "topic_name": "Miscellaneous (Gynae)", "id": "7b2a9322-e4fd-4282-ae41-d2db753f9114", "choice_type": "single"} {"question": "Metrorrhagia is", "exp": "The term intermenstrual bleeding is updated nomenclature for metrorrhagia.", "cop": 2, "opa": "Heavy menstrual bleeding", "opb": "Intermenstrual bleeding", "opc": "Break through bleeding", "opd": "Post coital bleeding", "subject_name": "Gynaecology & Obstetrics", "topic_name": null, "id": "58cb2f04-f0c1-4ff8-b693-ad8734746d6b", "choice_type": "single"} {"question": "An obese female having hirsutism on laboratory investigation has a high level of LH and androgens. Likely cause is", "exp": "The clinical and laboratory features of the patient described in the question match with those of PCOS as depicted in the table.\nSo the most appropriate answer is PCO", "cop": 1, "opa": "PCOS", "opb": "Exogenous steroid ingestion", "opc": "Turner syndrome", "opd": "Kleinfelter syndrome", "subject_name": "Gynaecology & Obstetrics", "topic_name": null, "id": "a16ae1c5-cfc4-4bae-8ea0-95679cd94ecd", "choice_type": "single"} {"question": "Progress of the labour is assessed with the help of March 2005, March 2013", "exp": "Ans. B: Paogram The paogram is a series of chaed measurements which are used to assess the progress of labour. Paograms are designed to encourage a universal manner of dealing with labour and to give an early, easy way of recognising problems in labour. Information recorded includes: Maternal - BP (2 hourly), hea rate (half hourly), temperature (once unless abnormal), Contractions - length, frequency, strength Baby - hea rate (2 hourly) State of membranes and colour of liquor. Also - maternal state of mind, drugs administered etc.", "cop": 2, "opa": "Cervicograph", "opb": "Paogram", "opc": "Dilatation cha", "opd": "Growth curve", "subject_name": "Gynaecology & Obstetrics", "topic_name": null, "id": "1a0f5b60-b55b-4713-b76c-7c165bb5a86b", "choice_type": "single"} {"question": "Preimplantataion genetic diagnosis is used for", "exp": "Pre implantation genetic diagnosis is used for daignosis of single gene disorders such as cystic fibrosis, β - thalassemia and hemophilia.", "cop": 2, "opa": "Fetal gender determination", "opb": "Single gene mutation analysis", "opc": "HLA typing", "opd": "Karyotyping", "subject_name": "Gynaecology & Obstetrics", "topic_name": null, "id": "abf516bd-aeec-4fee-a74e-4fcaf83bdeec", "choice_type": "single"} {"question": "The first change in precancerous lesion on colposcopy is", "exp": "The first change in precancerous lesion is punctuation", "cop": 2, "opa": "Mosaics", "opb": "Puntation", "opc": "Acetowhite lesions", "opd": "Cork-screw vessels", "subject_name": "Gynaecology & Obstetrics", "topic_name": null, "id": "092dc8d0-e4b5-4237-9afc-09c540fc1f10", "choice_type": "single"} {"question": "MC Type of female pelvis", "exp": "Ans. (a) Gynaecoid", "cop": 1, "opa": "Gynaecoid", "opb": "Anthropoid", "opc": "Android", "opd": "Platy pelloid", "subject_name": "Gynaecology & Obstetrics", "topic_name": "Fetal Skull and Maternal Pelvis", "id": "842bbde3-bb35-47f8-a227-bf91a608c47d", "choice_type": "single"} {"question": "Genital was in pregnancy treatment is", "exp": "Cryotherapy T/T of anogenital was Topical medication Trichloracetic acid or Podophyllin resin applied weekly. hniquinod or podofilox should also be used. Crvotherapy and laser therapy Cryotherapy (freezing) using nitrous oxide or carbon dioxide is used in was who do not respond to topical treatment. Carbon dioxide laser is used for larger and more numerous was. TIT of anogenital was during pregnancy :- \"Cryotherapy and laser therapy are the preferred t/t methods\" The only topical therapy which can be used during pregnancy is Trichloracetic acid. Podophyllin resin is not used during pregnancy.", "cop": 3, "opa": "Podophylin", "opb": "Salicylic acid & lactic acid", "opc": "Cryotherapy", "opd": "Imiquinod", "subject_name": "Gynaecology & Obstetrics", "topic_name": null, "id": "4c01d370-3e3b-499f-8fb1-d3b914d21d5c", "choice_type": "single"} {"question": "Duct of Bartholins gland opens", "exp": "Duct of Bartholins gland opens on the inner side of Labium minus external of hymen.", "cop": 4, "opa": "On outer side of Labium minus", "opb": "On inner side of Labium majus", "opc": "On outer side of Labium majus", "opd": "On inner side of Labium minus", "subject_name": "Gynaecology & Obstetrics", "topic_name": null, "id": "97ca9bd4-a163-45db-95bf-6b9808be0659", "choice_type": "single"} {"question": "Eye movements in fetus begins from", "exp": "16th Gestational WeekThe fetal crown-rump length is 12 cm, and the weight is 110grams. By 14 weeks, gender can be determined by experienced observers by inspection of the external genitalia. Eye movements begin at 16 to 18 weeks, coinciding with midbrain maturation.Reference: William&;s Obstetrics; 24th edition; Chapter 7; Embryogenesis and Fetal Morphological Development", "cop": 3, "opa": "6 to 9 weeks", "opb": "11 to 14 weeks", "opc": "16 to 18 weeks", "opd": "28 to 32 weeks", "subject_name": "Gynaecology & Obstetrics", "topic_name": "General obstetrics", "id": "db909dce-73a1-41ad-8b8d-05bf3312f7c2", "choice_type": "single"} {"question": "A primigravida is in second stage of labour since 1 hr. On examination the leading point of fetal skull is at +2 station and rotation is 450 sho. This can fuher be managed by", "exp": "The patient fulfills the criteria for applying low forceps The criteria for applying low forceps - (1) Leading point of fetal skull at station+ 2 cm and not on the pelvic floor, (2) Rotation is 45o or less. If the fetal head is at station below this or rotation doesn't exceed 45o, then one can apply outlet forceps. A: Unrotated head in transverse position B: 45 degree sho of complete rotation C: fully rotated head (ideal for application of forceps) In modern obstetrics there is no role of midcavity or high forceps, i.e. only station +2 and below are extracted with forceps (low and outlet forceps)", "cop": 2, "opa": "Applying outlet forceps", "opb": "Applying low forceps", "opc": "Applying mid forceps", "opd": "Applying high forceps", "subject_name": "Gynaecology & Obstetrics", "topic_name": "Labour - normal, abnormal, malposition, malpresentation and their management", "id": "2abaf64a-2643-4659-b503-caabf820f273", "choice_type": "single"} {"question": "Number of sperms required for fertilization per oocyte in invitro fertilization is", "exp": "100000 sperms per oocyte is required for IVF.", "cop": 4, "opa": "100", "opb": "1000", "opc": "10000", "opd": "100000", "subject_name": "Gynaecology & Obstetrics", "topic_name": null, "id": "f6da2908-e16a-41ed-95d2-134d1970be42", "choice_type": "single"} {"question": "Antidote of MgSO4 is", "exp": "Ans: b (Calcium gluconate) Ref: Dutta, 6th ed, p. 236; Davidson, 20th ed, p. 441Management of hyper magnesemia:Stop magnesium.Promote urinary excretion by loop diuretic with IV hydration in RFT normal cases.IV calcium gluconate to reverse overt cardiac effects.Dialysis if renal function is minimal.Magnesium sulphate:Drug of choice in pre-ecclampsiaMechanism of action:1. Reduction of motor end plate sensitivity to acetyl choline.2. It blocks neuronal calcium influx.3. Induces cerebral vasodilation.Adverse effects:Maternal - flushing, headache, muscle weakness, diplopia, pulmonary edema, cardiac arrest.Fetal and neonatal - lethargy, hypotonia and respiratory depression.", "cop": 2, "opa": "Vit K", "opb": "Calcium gluconate", "opc": "Insulin and dextrose", "opd": "NaHCO3", "subject_name": "Gynaecology & Obstetrics", "topic_name": "Miscellaneous (Gynae)", "id": "6082b6d0-3d66-4d81-ad96-ef2b93a9a849", "choice_type": "single"} {"question": "Specific gravity of amniotic fluid", "exp": "Specific gravity of the amniotic fluid is 1.010 Fluid is faintly alkaline It becomes highly hypotonic to maternal serum at term pregnancy Ref: Dutta Obs 9e pg 34.", "cop": 2, "opa": "1.213", "opb": "1.01", "opc": "1.51", "opd": "1.1", "subject_name": "Gynaecology & Obstetrics", "topic_name": "General obstetrics", "id": "fed2d236-0b62-4de1-9065-2fa829fecc92", "choice_type": "single"} {"question": "Failure rate is after tuba! sterilization with pomeroys technique", "exp": "0.40%", "cop": 3, "opa": "Zero", "opb": "0.10%", "opc": "0.40%", "opd": "1.00%", "subject_name": "Gynaecology & Obstetrics", "topic_name": null, "id": "91dafaad-5ec0-40ae-9f69-bc127dc1b92a", "choice_type": "single"} {"question": "Drug given to reduce the uterine contraction during preterm labour with least side effect", "exp": "Progesterone Among the given options progesterone is the drug with the least side effect. - Natural progesterone is completely innocuous for the mother and fetus. - The problem is that, the use of progesterone as tocolytics is controversial. Some studies have proved that progesterone can be used as tocolytics. Progesterone does have a role in prefer'', labour According to American college of obs and Gynae (ACOG) progesterone is given in patients with history of previous preterm labour and in women who are at risk for preterm labour (i.e., women with sho cervix). - In these patients progesterone administration stas from 16th month and is continued up to 36 months progesterone acts by reducing uterine contractions. An impoant point Progesterone reduces or prevents uterine contractions when the uterine contractions are not fully established but once the labour sets in i.e., regular uterine contractions sta progesterone has no role. - Progesterone cannot prevent regular uterine contractions of labour. We are not sure from the question, what they want to ask. The question does not clearly differentiate if they are asking about uterine contractions of pregnancy or labour. Once again remember If labour has begun, progesterone has no use. Among other tocolytics, calcium channel blockers have the least side effects. It is an effective tocolytic that is well tolerated by the majority of the pregnant patient. No significant fetal problems have been repoed after treatment with Nifedipine. The most common side effect seen with Nifedipine is lowering of B.P, facial flushing can be seen in ceain cases. Unlike the [3 adrenergic tocolytics they have no effect on carbohydrate metabolism and can be given to diabetic patient. Beta adrenergic agonists These are usually the tocolytics of choice for the treatment of preterm labour. They also have serious side effects which are Pulmonary edema - It is a serious and common side effect of 13 agonists. - It occurs in patients receiving oral or intravenous treatment. - It is common in patients having chorioamnionitis and in twins. Hyperglycemia - When 13 adrenergic drugs are given to patients with insulin dependent diabetes it causes significant hyperglycemia, glycosuria and ketonuria. - This occurs due to increased glycogenolysis and increased lipolysis. fi adrenergic agents are contraindicated in chorioamnionitis Side effects of magnesium Pulmonary edema Hypothermia Neuromuscular toxicity", "cop": 4, "opa": "Ritodrine", "opb": "Nifidipine", "opc": "Magnesium sulphate", "opd": "Progesterone", "subject_name": "Gynaecology & Obstetrics", "topic_name": null, "id": "43604b2c-8014-4819-bed9-14afce296606", "choice_type": "single"} {"question": "Enterocele formation is a common complication of", "exp": "BURCH COLPOSUSPENSION COMPLICATIONS Hemorrhage during dissection of the space Urethra and bladder neck injury due to injudicious placement of sutures Osteitis pubis voiding difficulty detrusor instability Enterocele formation Failure and recurrence D.C.DUTTA&;S TEXTBOOK OF GYNAECOLOGY,Pg no:404,6th edition", "cop": 3, "opa": "Suburethral sling surgery", "opb": "TVT", "opc": "Burch colposuspension", "opd": "TOT", "subject_name": "Gynaecology & Obstetrics", "topic_name": "Urogynecology", "id": "df552a3d-f426-4684-92ec-44931c38973a", "choice_type": "single"} {"question": "Intrahepatic cholestasis of pregnancy occurs in", "exp": "DisorderOnset in PregnancyHyperemesisEarlyCholestasisLateFatty liverLatePreeclampsiaMid to lateHepatitisVariable(Ref: William's Obstetrics; 25th edition)", "cop": 3, "opa": "First trimester", "opb": "Second trimester", "opc": "Third trimester", "opd": "Postpaum", "subject_name": "Gynaecology & Obstetrics", "topic_name": "All India exam", "id": "b0e2ba5c-0841-4fac-8a62-4c11ff27f30e", "choice_type": "single"} {"question": "In Swyers syndrome", "exp": "Swyers syndrome: Gonads are testis (46XY) but dysgenetic. Due to loss of negative feedback FSH and LH are elevated.", "cop": 1, "opa": "Testis is present", "opb": "Ovaries are present", "opc": "Short stature", "opd": "Decreased FSH and LH", "subject_name": "Gynaecology & Obstetrics", "topic_name": null, "id": "da2929df-acf8-4516-8abf-94a223190e14", "choice_type": "single"} {"question": "Sho structured primi gravida has height less then", "exp": "140 cm", "cop": 1, "opa": "140 cm", "opb": "145 cm", "opc": "150 cm", "opd": "135 cm", "subject_name": "Gynaecology & Obstetrics", "topic_name": null, "id": "0f810d3c-d7f0-4783-b806-e554b894b843", "choice_type": "single"} {"question": "In women having PCOD with irregular menstrual cycles, combined oral contraceptive pills", "exp": "In combined OCPS, estrogen component increases levels of SHBG, which binds free androgen.", "cop": 1, "opa": "Increase sex hormone binding globulin", "opb": "Cause endometrial hyperplasia", "opc": "Increases lutenizing hormone", "opd": "Increase insulin", "subject_name": "Gynaecology & Obstetrics", "topic_name": null, "id": "dcb21007-0c7c-4ec4-a35d-68a6b0dc085f", "choice_type": "single"} {"question": "A neonate born before 259th day is", "exp": "Term neonate: A neonate born any time after 37 completed weeks of gestation and up until 42 completed weeks of gestation (260 to 294 days).The American College of Obstetricians and Gynecologists (2013b) endorse and encourage specific gestational age designations. Early term refers to neonates born at 37 completed weeks up to 386/7 weeks. Full-term denotes those born at 39 completed weeks up to 406/7 weeks. Last, late-term describes neonates born at 41 completed weeks up to 416/7 weeks.Preterm neonate: A neonate born before 37 completed weeks (the 259th day).Postterm neonate: A neonate born anytime after completion of the 42nd week, beginning with day 295.Ref: Williams Obstetrics; 24th edition; Chapter 1; Page no: 55", "cop": 1, "opa": "Preterm neonate", "opb": "Early term neonate", "opc": "Late term neonate", "opd": "Postterm neonate", "subject_name": "Gynaecology & Obstetrics", "topic_name": "General obstetrics", "id": "dff4f634-8ae8-4545-8387-07bae7afdf90", "choice_type": "single"} {"question": "The recommended treatment for pregnant women with syphilis is", "exp": "Syphilis therapy during pregnancy is given to eradicate maternal infection and to prevent or treat congenital syphilis. Parenteral penicillin G remains the preferred treatment for all stages of syphilis during pregnancy.(Ref: William's Obstetrics; 25th edition)", "cop": 3, "opa": "Tetracycline", "opb": "Oral pencillin", "opc": "Parenteral pencillin", "opd": "Aminoglycosides", "subject_name": "Gynaecology & Obstetrics", "topic_name": "All India exam", "id": "97148ded-ac00-4d7e-9b4c-5689e0c0fc49", "choice_type": "single"} {"question": "Fourteen weeks pregnancy with third degree prolapse. Best M/n will be", "exp": "Ans. is 'c' i.e. . \"Ring Pessary\" 3rd degree prolapse means that cervix is outside uterus and such patients are managed by replacing the cervix inside the vagina and then it is kept in that position by a ring pessary.The pessary is to be kept there until 18-20 weeks of pregnancy when the body of the uterus will be sufficiency enlarge to sit on the brim of the pelvis.", "cop": 3, "opa": "Sling surgery", "opb": "Foot end elevation", "opc": "Ring Pessary", "opd": "No treatment", "subject_name": "Gynaecology & Obstetrics", "topic_name": "Management", "id": "092f29bc-0e28-4017-b4a9-976b3e276f6e", "choice_type": "single"} {"question": "Best contraceptive method during lactation", "exp": "(B) (Progesterone- only pill) (491- Dutta 6th)POP/MINIPILL* POP is contain very low dose of a progestin Leuonorgestrel, Norethisterone, desogestrel, lynestrenol or norgestrel* It has to be taken daily from the first day of the cycle*** Mechanism of action - it works mainly by making cervical mucus thick and viscous, thereby prevents sperm penetration**AdvantagesDisadvantages1. Side effects attributed to estrogen in the combined pill are totally eliminated1. Acne, mastalgia headache back through bleeding2. No adverse effect on lactation (''Lactation pill\")2. All the side effects, attributed to progestins may be evident3. Easy to take as there is no ''on and off' regime3. Simple cysts of the ovary may be seen, but they do not require any surgery4. It may be prescribed in patients having (medical disorder) Hypertension, fibroid DM, epilepsy, smoking and history of thrombo-embolism4. Failure rate is about o.5-2 per 100 women years of use5. Reduces the risk of PID and endometrial cancerCONTRAINDICATION1. Pregnancy2. Unexplained vaginal bleeding3. Recent breast cancer4. Arterial disease5. Thromboembolic disease* OCP do not act by interference with placental functioning* Contraceptive of choice for Rheumatic heart disease is Barrier method (condom)* Ideal contraceptive for newly married couple is combined OCP* Ideal contraceptive for a couple living in different cities meeting only occasionally - Barrier method* Best contraceptive for parous young women- IUCD* Progesterone of choice in emergency contraceptive is Levonorgestrel*** Effective life of device - Nova Cu-T (5 years) Multiload Cu-T 375 (5years) Cu-T 380A (10 years)* OCP should be avoided in patients on antiepileptic medication* The risk of chlamydial infections is increased in patients using OCP. In most patients such chlamydial infection is asymptomaticAzithromycin is drug of choice in chlaraydial infection in pregnancy* Fishy vaginal odor is most characteristic of Bacterial vaginosis due to Gardnella viginalis (Gram negative bacilli, decreased number of lactobacilli, clue cells, few leucocytes (polymorphs), PH of discharge is >4.5)", "cop": 2, "opa": "IUD", "opb": "Progesterone only pill (POP)", "opc": "Lactational amenorrhoea", "opd": "Barrier method", "subject_name": "Gynaecology & Obstetrics", "topic_name": "Miscellaneous (Gynae)", "id": "994e8efb-e7a7-4eaf-804e-cd2a6b2b6e16", "choice_type": "single"} {"question": "Treatment of carcinoma cervix stage MB include", "exp": "Ans. a. Concurrent chemoradiation", "cop": 1, "opa": "Concurrent chemoradiation", "opb": "Surgery", "opc": "Neoadjuvant chemotherapy followed by radiotherapy", "opd": "Only radiotherapy", "subject_name": "Gynaecology & Obstetrics", "topic_name": null, "id": "c12b88a5-f54a-4383-b9ff-e4a1938b4b7d", "choice_type": "single"} {"question": "Pregnancy rate with female condom as compared to male condom,", "exp": "Pregnancy rate with female condum is higher than with the male condom.", "cop": 2, "opa": "Lesser than male condom", "opb": "Higher than male condom", "opc": "Same as male condom", "opd": "Zero", "subject_name": "Gynaecology & Obstetrics", "topic_name": null, "id": "64c92a74-6502-4aa6-9555-22a72d31f219", "choice_type": "single"} {"question": "Uterine aery is a branch of", "exp": "The uterine aery arises from the anterior trunk of the internal iliac (or hypogastric aery). Its course is at first downwards and forwards until it reaches the parametrium when it turns medially towards the uterus. It reaches the uterus at the level of the internal os, where it turns upwards, at right angles, and follows a spiral course along the lateral border of the uterus to the region of the uterine cornua. Here it sends a branch to supply the fallopian tube and ends by anastomosing with the ovarian aery.Reference: Shaw's Textbook of Gynecology; 16th edition; Chapter 1; Anatomy", "cop": 2, "opa": "External iliac aery", "opb": "Anterior division of internal iliac aery", "opc": "Posterior division of internal iliac aery", "opd": "Ovarian aery", "subject_name": "Gynaecology & Obstetrics", "topic_name": "Anatomy of the female genital tract", "id": "8f464855-9284-44e0-8d9b-f7a27be9565e", "choice_type": "single"} {"question": "Grade III intraventricular hemorrhage is", "exp": "Grade I - Hemorrhage limited to the germinal matrix\nGrade II - Intraventricular hemorrhage\nGrade III - Hemorrhage with ventricular dilatation\nGrade IV - Parenchymal extension of hemorrhage.", "cop": 3, "opa": "Intraventricular hemorrhage alone", "opb": "Hemorrhage limited to germinal matrix", "opc": "Hemorrhage with ventricular dilatation", "opd": "Parenchymal extension of hemorrhage", "subject_name": "Gynaecology & Obstetrics", "topic_name": null, "id": "6200c3c6-7943-4c38-bc10-a6fea4f305e8", "choice_type": "single"} {"question": "Antithyroid drug of choice in pregnancy is", "exp": "Propylthiouracil", "cop": 1, "opa": "Propylthiouracil", "opb": "Carbimazole", "opc": "Propranolol", "opd": "Lugol's iodine", "subject_name": "Gynaecology & Obstetrics", "topic_name": null, "id": "ead2c681-0ea5-4cfd-b516-8ce416c38056", "choice_type": "single"} {"question": "Ophthalmic exam to screen for retinopathy of prematurity is mandatory for newborns with bih weight less than", "exp": ".", "cop": 2, "opa": "1000 grams", "opb": "1500 grams", "opc": "2000 grams", "opd": "2500 grams", "subject_name": "Gynaecology & Obstetrics", "topic_name": "All India exam", "id": "46fa9049-ea1b-40ab-9fca-2a7ad4b6f6dc", "choice_type": "single"} {"question": "A patient at 28 weeks pregnancy presents with low grade fever, malaise, vomiting since 1 week. On examination she is icteric, Her Hb is 10 gm%, bilirubin is 5 mg/dL, SGOT and SGPT at 630 and 600 with normal platelet count. Most probable cause of her jaundice is", "exp": "Patient here has presented with prodrome of viral hepatitis. Investigations to diiferentiate various causes of jaundice in pregnancy are IHCP AFLP HELLP Viral hepatiis Bilirubin (mg/dL) 1-5, direct 2-10, direct <2 >5 SGOT/ SGPT <200 <1000 <500 500-3000 ALP Raised Normal Normal Normal/raised Platelets Normal low Low Normal Prothrombin time Normal Prolonged prolonged prolonged", "cop": 2, "opa": "IHCP", "opb": "Viral hepatitis", "opc": "Hyperemesis gravidarum", "opd": "HELLP syndrome", "subject_name": "Gynaecology & Obstetrics", "topic_name": "Medical Illness Complicating Pregnancy", "id": "26979b55-e4f1-4c54-aef5-5ad49216f5c1", "choice_type": "single"} {"question": "Elevated androgens in PCOD", "exp": "Elevated androgen convert vellus hair into coarse terminal hair. This conversion is irreversible.", "cop": 2, "opa": "Convert terminal hair to vellus hair irreversibly.", "opb": "Convert vellus hair to terminal hair irreversibly.", "opc": "Reversible conversion of vellus hair to terminal hair.", "opd": "Reversible conversion of terminal hair to vellus hair.", "subject_name": "Gynaecology & Obstetrics", "topic_name": null, "id": "6961eeaf-07ce-4e87-a518-335feb78c0cf", "choice_type": "single"} {"question": "HPV vaccine Cervarix is protective against subtypes", "exp": "HPV vaccines have been developed from the capsid coat of the virus. It has high immunogenicity. Bivalent vaccine is against HPV 16 & 18. Quadrivalent vaccine is against 16, 18, 6 & 11. Effective in the prevention of 90% of cervical cancer. Both vaccines have some cross-protection against strains 31, 33 & 45. Vaccines are given at an age of 12-18years, safe and well tolerated. Three doses, IM over deltoid muscle. The impact is greatest when given before women are infected with HPV. Vaccine-induced neutralizing antibodies works locally on cervix by preventing attachment of the virus to the cervical epithelium. Immune defense is type specific & is effective only when given prophylactically. Vaccines are effective for at least 7.5years. Pap smear screening should be continued as vaccines are type specific and do not protect all types of HPV. Reference: Dutta's Textbook of Gynecology", "cop": 3, "opa": "6 and 11", "opb": "6 and 18", "opc": "16 and 18", "opd": "6, 11, 16 and 18", "subject_name": "Gynaecology & Obstetrics", "topic_name": "Gynaecological oncology", "id": "42fb0177-2836-45ef-8f27-f6d089835d1b", "choice_type": "single"} {"question": "Most common cause of abnormality in first trimester abortion", "exp": "(Chromosomal anomaly) (161 - Dutta 6th)Common Causes of AbortionFirst Trimester -1. Genetic factor (50%) of early miscarriage are due to chromosomal abnormality in the conceptus, Autosomal trisomy is the commonest (50%) cytogenic abnormality2. Endocrine disorders (LPD) thyroid abnormalities DM3. Immunological disorders (Autoimmune and Alloimmune)4. Infection5. UnexplainedSecond trimester1. Anatomic abnormalities(a) Cervical incompetance (congenital or acquired) is Most common(b) Mullerian fusion defects (Bicomuate uterus, septate uterus)(c) Uterine synechiae(d) Uterine fibroid2. Maternal medical illness3. Unexplained", "cop": 2, "opa": "Cervical incompetance", "opb": "Chromosomal ahnormolity", "opc": "Placental & membrane abnormality", "opd": "Retroversion", "subject_name": "Gynaecology & Obstetrics", "topic_name": "Miscellaneous (Gynae)", "id": "a3876104-90b8-42e2-804f-934ef54d1d78", "choice_type": "single"} {"question": "The drug being given is", "exp": "Ethacridine lactate (0.1%) is used only extra - amniotically.", "cop": 1, "opa": "Ethacrydine lactate (0.1%)", "opb": "20% mannitol", "opc": "50% Urea", "opd": "Misoprostol", "subject_name": "Gynaecology & Obstetrics", "topic_name": "General obstetrics", "id": "cae137a9-8bd1-4e6e-81f7-ed1c86412ccf", "choice_type": "single"} {"question": "Hypothyroidism in pregnancy is least likely associated with", "exp": "Ove or symptomatic hypothyroidism is repoed to complicate between 2 and 10 pregnancies per 1000. Clinical/ ove hypothyroidism is confirmed when an abnormally high serum TSH level is accompanied by an abnormally low thyroxine level. Most common cause is Hashimoto thyroiditis Subclinical hypothyroidism is defined by an elevated serum TSH level and normal serum thyroxine concentration Complications in pregnancy Aboion Placental abruption Pretem bih Low Bih weight Stillbih placental abruption Pre eclampsia Impaired psychomotor development of baby Also a cause of infeility, menstrual disturbances", "cop": 3, "opa": "PIH", "opb": "Recurrent aboions", "opc": "Polyhydramnios", "opd": "Preterm labour", "subject_name": "Gynaecology & Obstetrics", "topic_name": "Medical Illness Complicating Pregnancy", "id": "2cfc333b-8186-478b-bb0c-0e9eda6cb70a", "choice_type": "single"} {"question": "The most impoant indication for surgical repair of a bicornuate uterus is", "exp": "Bicornuate uterus Defective mullerian fusion such as double uterus,septate,or ,bicornuate results in recurrent aboion (12%). Aboions tend to occur beyond 12 weeks and the successive pregnancies are carried longer. Hysteroscopic resection of the uterine septa,synechae,improves the pregnancy outcome. Uterine unification operation (metroplasty)is done for bicornuate uterus D.C.DUTTA&;S TEXTBOOK OF OBSTETRICS,Pg no:169,7th edition", "cop": 4, "opa": "Infeility", "opb": "Dysmenorrhoea", "opc": "Menorrhagia", "opd": "Habitual aboion", "subject_name": "Gynaecology & Obstetrics", "topic_name": "Congenital malformations", "id": "978d3b16-4443-4b40-8adc-96bf052bd6f7", "choice_type": "single"} {"question": "Prolonged latent phase in nullipara is defined as one exceeding", "exp": "A prolonged latent phase was defined as one exceeding 20 hours in the nullipara and 14 hours in the multiparous women.(Ref: William's Obstetrics; 25th edition)", "cop": 4, "opa": "10 hours", "opb": "14 hours", "opc": "18 hours", "opd": "20 hours", "subject_name": "Gynaecology & Obstetrics", "topic_name": "All India exam", "id": "8d033ffb-6f9b-4ade-ab65-3983914e0b72", "choice_type": "single"} {"question": "Ovarian fibroma develops from", "exp": "Histologically fibroma has a cellular or myxomatous stroma which are scattered large signet ring cells. Ref: Shaw Gynecology 17 e pg 464.", "cop": 4, "opa": "Epithelium", "opb": "Germ cells", "opc": "Sex cord", "opd": "Stroma", "subject_name": "Gynaecology & Obstetrics", "topic_name": "Gynaecological oncology", "id": "a8c58517-2813-45c2-a5bb-a2a3912d9482", "choice_type": "single"} {"question": "Indication for Induction of labor is", "exp": "Induction is indicated when the benefits to either mother or fetus outweigh those of pregnancy continuation. The more common indications include Membrane rupture without labor, Gestational hypeension, Oligohydramnios, Nonreassuring fetal status, Postterm pregnancy, and Various maternal medical conditions such as chronic hypeension and diabetesRef: William&;s; 25th edition", "cop": 2, "opa": "Placenta pre", "opb": "Postterm pregnancy", "opc": "Preterm labor", "opd": "Breech", "subject_name": "Gynaecology & Obstetrics", "topic_name": "General obstetrics", "id": "e7b9e977-33d7-4dba-80b7-7dbc3dcfcd9b", "choice_type": "single"} {"question": "Sampson's Theory proposed to explain endometriosis is", "exp": "Sampson's theory: Reflux of menstrual endometrium through fallopian tubes causes endometriosis.", "cop": 3, "opa": "Embolization of menstrual fragments through lymphatic channels.", "opb": "Metaplastic changes in embryonal cell rests.", "opc": "Reflux of menstrual endometrium.", "opd": "Histogenesis by induction.", "subject_name": "Gynaecology & Obstetrics", "topic_name": null, "id": "9bd5d468-9668-41fb-a003-9c3eb987f81f", "choice_type": "single"} {"question": "Testosterone secreted by;", "exp": "Testosterone is secreted by Leydig cell In the testes, Leydig cells arise from the original mesoderm of the genital ridge and lie between the testicular cords. Leydig cells begin to secrete testosterone by 8-week gestation due to stimulation of the testes by human chorionic gonadotropin (hCG).", "cop": 1, "opa": "Leydig cell", "opb": "Seoli cell", "opc": "Theca lutein", "opd": "Granulosa cell", "subject_name": "Gynaecology & Obstetrics", "topic_name": "FMGE 2019", "id": "823c52b7-0ee8-4901-a8f4-7b19294dcd3c", "choice_type": "single"} {"question": "Hormone replacement therapy is not", "exp": "Answer- D (Prevention CAD)Hormone replacement therapy increases the risk of MI (coronary aery disease). The increased risk is attributed to progestin component.o H restore Ca++ balance, fuher bone loss is prevented and the excess fracture risk is nullified. o There is a higher incidence of breast cancer.o Risk for endometrial carcinoma is not increased as protective effect of progestin nullify the carcinogenic effect of estrogen.o There is increased risk for gall stone and migraine.o There is a small protective effective of H on colorectal cancer.Note :?o Usually in H combination of estrogen and progestin is given (combined H), except in hysterectimized women where estrogen alone is given.o With estrogen alone there is increased risk of endometrial cancer and decrease risk of coronary aery disease (as estrogen increases HDL and decreases LDL and triglyceride level).o So for H with estrogen alone the answer of this question will change. In that case it will be option 'a' i.e., increased risk of coronary aery disease.", "cop": 4, "opa": "Urogenital atrophy", "opb": "Vasomotor symptoms", "opc": "Prevention of osteoporosis", "opd": "Prevention of CAD", "subject_name": "Gynaecology & Obstetrics", "topic_name": null, "id": "cfb85f28-3c0a-4d2e-adb7-93e56ac6b843", "choice_type": "single"} {"question": "Amsel's criteria is used for;", "exp": "Amsel's criteria is used for diagnosis of Bacterial vaginosis MC Vaginitis Caused by Hemophilus vaginalis aka Gardnerella vaginalis Amsel's criteria - >= 3 out of 4 required for diagnosis of Bacterial vaginosis Creamy discharge Whiff test + Fishy odor Clue cells", "cop": 1, "opa": "Gardnerella vaginosis", "opb": "Candidiasis", "opc": "Trichomoniasis", "opd": "Chlamydia infection", "subject_name": "Gynaecology & Obstetrics", "topic_name": "FMGE 2019", "id": "52fb8dd8-43e6-4b01-9c47-3b5055290ec0", "choice_type": "single"} {"question": "Best Indicator of Ovarian reserve", "exp": "None of the ovarian reserve tests currently in use are accurate predictors for pregnancy\nWith current methods, basal FSH concentration is simplest and most widely used to test ovarian reserve. Levels greater than 10 IU/L (10-20 IU/L) have high specificity (80-100%) for predicting poor ovarian response to ovulation. Serum FSH levels vary significantly across the cycle hence FSH is best measured during the early follicular phase ( cycle day 2-4).\nBasal serum estradiol concentration has little value as ovarian reserve test.\nAMH: Antimullerian hormone is produced from preantral and small antral follicles hence AMH levels are gonadotropin-independent and exhibit little variation within and between menstrual cycles. AMH threshold levels (0.2-0.7ng/ml) has 40-97% sensitivity, 70-92% specificity, 22-88% PPV and 97-100% NPV for predicting poor response to stimulation (<3 follicles or < 2-4oocytes).", "cop": 1, "opa": "FSH", "opb": "Estradiol", "opc": "LH", "opd": "FSH/LH Ratio", "subject_name": "Gynaecology & Obstetrics", "topic_name": null, "id": "e0e43939-fc77-45cd-a1aa-45c01356d37f", "choice_type": "single"} {"question": "A lady with 16 weeks pregnancy presents with acute appendicitis,TLC of 24000. Management includes", "exp": "In pregnancy, if the patient develops symptoms of appendicitis and it has been confirmed with diagnostics. The preferable option is to do early surgery to avoid the complications like puerperial sepsis, aboion or intrauterine deaths and also complications of untreated appendicitis like rupture of appendix, pyoperitoneum etc", "cop": 2, "opa": "Conservative treatment always", "opb": "Do early surgery", "opc": "Appendecectomy with termination of pregnancy", "opd": "Medical treatment with surgery on recurrence", "subject_name": "Gynaecology & Obstetrics", "topic_name": "Medical Illness Complicating Pregnancy", "id": "67b059c3-85c5-47d5-9445-7f7153c5857e", "choice_type": "single"} {"question": "Percentage of cystic glandular hyperplasia turning to malignancy", "exp": "WHO classification and Progression to Carcinoma\nPercentage of cystic glandular hyperplasia turning to malignancy is 1%", "cop": 3, "opa": "0.10%", "opb": "2%", "opc": "1%", "opd": "10%", "subject_name": "Gynaecology & Obstetrics", "topic_name": null, "id": "e9f2e38c-d9db-4032-b3c0-5248dcf3142d", "choice_type": "single"} {"question": "Modified Biophysical profile is", "exp": "Modified biophysical profile: NST+AFI (takes only 10 minutes to perform) Biophysical profile : most accurate means of assessing fetal health as used 5 variables instead of 1.5 fetal biophysical components used are - a) Hea rate acceleration b) Breathing c) Movements d) Tone e) Amniotic fluid volumeThis test takes 30-60 minutes of examiner time.", "cop": 3, "opa": "NST+FETAL TONE", "opb": "FETAL TONE+AFI", "opc": "NST+AFI", "opd": "NST+FETAL TONE+AFI", "subject_name": "Gynaecology & Obstetrics", "topic_name": "Intra Uterine Growth Restriction, Intrapaum and Antepaum Fetal Surviellance", "id": "73919091-0abe-482e-b09c-350977e9f56c", "choice_type": "single"} {"question": "Tocolytic of choice in hea disease", "exp": "Atosiban [Ref: Ian Donald practical problems in obstetrics 6h/e Most tocolytics are contraindicated in women with cardiac disease because of their potential to precipitate cardiac failure. - As there are no studies performed to look at the safety of tocolytic drugs in setting of existing hea failure the recommendations for their use can be best made on the basis of their side effect profile in normal women. \"Atobisan, an oxytocin antagonist seems to be the drug of choice at present\" -In an animal study it was shown to have no central hemodynamic effect although some concern has been expressed over increased infant moality rate. Indomethacin has minimal effect on hea rate and mean aerial pressure and may be used as an alternative for sho term tocolysis. None of the other tocolytics are considered safe for cardiac disease. Beta inimetics probably are the worst They are contraindicated in :? - Cardiac arrhythinias - Valvular disease - Cardiac ischemia Nifedipine It is contraindicated in :? - Conduction defects - Left ventricular failure Magnesium sulfate - Status of magnesium sulfate as regards it safety in cardiac disease in unceain.", "cop": 2, "opa": "Nifedepine", "opb": "Atosiban", "opc": "MgSo4", "opd": "Salbutamol", "subject_name": "Gynaecology & Obstetrics", "topic_name": null, "id": "8cbcdb14-62c0-4253-b684-a0ec6d5a1e61", "choice_type": "single"} {"question": "Ligamentum teres is", "exp": "Circulatory Changes at Bih:After bih, the umbilical vessels, ductus aeriosus, foramen ovale, and ductus venosus normally constrict or collapse. The more distal poions of the hypogastric aeries, which course from the level of the bladder along the abdominal wall to the umbilical ring and into the cord as the umbilical aeries, undergo atrophy and obliteration within 3 to 4 days after bih. These become the umbilical ligaments, whereas the intraabdominal remnants of the umbilical vein form the ligamentum teres. The ductus venosus constricts by 10 to 96 hours after bih and is anatomically closed by 2 to 3 weeks, resulting in the formation of the ligamentum venosumReference: William&;s Obstetrics; 24th edition; Chapter 7; Embryogenesis and Fetal Morphological Development", "cop": 1, "opa": "Obliteration of umbilical vein", "opb": "Obliteration of ductus venosus", "opc": "Obliteration of ductus aeriosus", "opd": "Obliteration of hypogastric aery", "subject_name": "Gynaecology & Obstetrics", "topic_name": "Anatomy of the female genital tract", "id": "505bf04f-20b6-4870-a19b-477eca2abc0a", "choice_type": "single"} {"question": "Antihormonal substance used to induce ovulation", "exp": "Shaw's textbook of Gynaecology- 16th edn- pg no- 257. Clomiphene citrate is used for ovulation induction. Dose- 50mg/day startingfrom day 2 to day 6 of the cycle for 5 days.", "cop": 2, "opa": "Mifepristone", "opb": "Clomiphene citrate", "opc": "Tamoxifen", "opd": "Raloxifen", "subject_name": "Gynaecology & Obstetrics", "topic_name": null, "id": "c782d9dc-4e38-40ec-9a9c-8e7008089542", "choice_type": "single"} {"question": "Paracervical block is associated with", "exp": "Paracervical nerve block : Given at the end of the first stage of the labor It is injected at the site of the cervix, given on both sides It can be repeated to relieve pain Fetal bradycardia is the known complication. Ref: Dutta Obs 9e pg 480.", "cop": 2, "opa": "Inhibition of labour", "opb": "Fetal bradycardia", "opc": "Increased loss of blood", "opd": "Atonicity of uterus", "subject_name": "Gynaecology & Obstetrics", "topic_name": "General obstetrics", "id": "c60725e3-bfa8-4237-8f9d-2f4e26bac7c9", "choice_type": "single"} {"question": "Ovarian tumours are commonly arise from", "exp": "(Surface epithelium) (397-S) (333-S14th)Ovarian cancers* Epithelial ovarian cancers accounts for 80 to 90% of ovarian cancers* Non-epithelial cancers account for 10 to 20%These include malignancies of:I) Germ cell originII) Sex cord stromal cell originIII) Metastatic cancersIV) Rare malignancies like lipoid cell tumours sarcomas", "cop": 2, "opa": "Stroma", "opb": "Surface epithelium", "opc": "Germinal epithelium", "opd": "Endodermal", "subject_name": "Gynaecology & Obstetrics", "topic_name": "Miscellaneous (Gynae)", "id": "f37aa551-736e-4a77-8dc8-ff2f7d5633c3", "choice_type": "single"} {"question": "Urethral caruncle is best treated by", "exp": "Excision followed by repeated dilatation A urethral caruncle is a benign red polyp or leison covered by transitional epithelium.deg Usually found on the posterior aspect of the urethral meatus.deg Urethral caruncle is common in postmenopausal female.deg Due to atrophy of vulva and vagina : urethral meatus is exposed to infection. The posterior urethral mucosa becomes swollen, congested and pouts out like a cherry from the posterior wall of external meatus. Symptoms Dyspareuniadeg Post coital bleedingdeg Paindeg Dysuriadeg Investigation :In women complaining of post - menopausal bleeding, other genital malignancies should be ruled out. Management :Friends here there is some confusion : all books mention excision as the best line of management but no where it is mentioned that it is to be followed by repeated dilatation. ''Treatment is by excision and diathermy coagulation of the base of the stalk. The patient should be given antibiotics to treat the underlying chronic urethritis.\" \"Excision biopsy is the definitive surgery. The raw area may be sutured or cauterised. Continous bladder drainage is required for few days.\" \"Treatment is by excision biopsy followed by local or systemic oestrogens.\" As far as cauterisation is concerned - diathermy is done but chemical cauterisation is not mentioned anywhere. I have given you all references - decide for yourself.", "cop": 2, "opa": "Administration of antibiotics", "opb": "Excision followed by repeated dilatation", "opc": "Resection and end to end anastomosis", "opd": "Chemical uterization", "subject_name": "Gynaecology & Obstetrics", "topic_name": null, "id": "0e455d9c-1aa2-4622-8c23-f2a1f95ad99c", "choice_type": "single"} {"question": "Most common site for endometriosis in urinary system is", "exp": "Bladder is most common site for urinary endometriosis.", "cop": 3, "opa": "Kidney", "opb": "Ureter - lower segment", "opc": "Bladder", "opd": "Ureter - upper segment", "subject_name": "Gynaecology & Obstetrics", "topic_name": null, "id": "600120ca-051d-42e2-9d95-ac414a086196", "choice_type": "single"} {"question": "Drug of choice of ovulation induction in PCOD patients", "exp": "Clomiphene citrate is drug of choice for ovulation induction in PCOD patients.", "cop": 2, "opa": "Tamoxifen", "opb": "Clomiphene", "opc": "Raloxifene", "opd": "Metformin", "subject_name": "Gynaecology & Obstetrics", "topic_name": null, "id": "9ece0d91-3450-4e54-8da5-358fe0e65a4b", "choice_type": "single"} {"question": "Fetal alcohol syndrome is characterized by", "exp": "Fetal alcohol syndrome - dysmorphic facies\n\nSmall palpebral fissure\nThin vermillion border\nMicrosomia\nHypoplastic phltrum.", "cop": 2, "opa": "Large palpebral fissures", "opb": "Thin vermillion border", "opc": "Macrosomia", "opd": "Hyperplastic philtrum", "subject_name": "Gynaecology & Obstetrics", "topic_name": null, "id": "d6099d29-5e7e-4861-8a1d-b4dc520a18b1", "choice_type": "single"} {"question": "Treatment of choice for endocervical polyp is", "exp": "Polyp is categorized and defined by ultrasound, saline sonography, hysteroscopy with or without histopathology It is treated by polypectomy Ref: Shaw Gynecology 17 e pg 132.", "cop": 3, "opa": "Vaginal hysterectomy", "opb": "Conisation", "opc": "Polypectomy", "opd": "Radiotherapy", "subject_name": "Gynaecology & Obstetrics", "topic_name": "Disorders of menstruation", "id": "60c937a6-f37d-4577-a46d-ec58cfb4a61d", "choice_type": "single"} {"question": "Use of folic acid to prevent congenital malformation should be best initiated;", "exp": "Therapy recommendations suggest that women begin taking folic acid supplements before becoming pregnant and continue taking the supplements of at least 0.4 mg/day throughout the first trimester to reduce the risk of neural tube defects. recommended because it has been shown to reduce the occurrence and recurrence of neural tube defects . For low-risk women, folic acid supplementation of 400 micrograms per day currently is recommended because nutritional sources alone are insufficient. For women at high risk of NTDs or who have had a previous pregnancy with an NTD, folic acid supplementation of 4 mg per day is recommended.", "cop": 4, "opa": "During lst trimester of pregnancy", "opb": "During 2nd trimester of pregnancy", "opc": "During 3rd trimester of pregnancy", "opd": "Before conception", "subject_name": "Gynaecology & Obstetrics", "topic_name": "Anaemia in Pregnancy", "id": "973c7589-2999-4131-a247-7529588250a7", "choice_type": "single"} {"question": "Atonic PPH is not seen in", "exp": "Causes of Atonic PPH: Multiparity Overdistention of abdomen Malnutrition and anemia Antepaum hemorrhage Prolonged labor Anesthesia Initiation or augmentation of delivery by oxytocin Malformation of uterus Uterine fibroid Mismanaged third stage of labor Precipitate labor Ref: Dutta Obs 9e pg 385.", "cop": 3, "opa": "Multiple pregnancy", "opb": "Hydramnios", "opc": "Erythroblastosis fetalis", "opd": "Prolonged labour", "subject_name": "Gynaecology & Obstetrics", "topic_name": "General obstetrics", "id": "36c6d036-ce53-4ddd-8ced-012e82051c95", "choice_type": "single"} {"question": "Ovarian tumour staging if one ovary involved without surface involvement or rupture", "exp": "STAGE 1A: Ovarian cancer limited to one ovary without malignant ascites or positive peritoneal washings, surface involvement or rupture", "cop": 1, "opa": "Stage 1A", "opb": "Stage 1B", "opc": "Stage 1C1", "opd": "Stage 1C2", "subject_name": "Gynaecology & Obstetrics", "topic_name": "Gynaecological oncology", "id": "c025abea-8909-4fdc-9756-7f2fdec174d6", "choice_type": "single"} {"question": "Recurrent aboion in 1st trimester is most often due to.", "exp": "Ans. is a i.e. Chromosomal abnormalities Historically. recurrent pregnancy loss or \"habitual aboion\" was defined as 3 or more consecutive spontaneous miscarriages. Today, recurrent pregnancy is usually defined as 3 or more pregnancy losses (not necessarily cnnc,ecr_itive). Most obstetricians also consider clinical investigation and treatment should be considered in couples with ,,,,ntaneo-- miscarriages, especially when any of the following are present : Embryonic hea activity observed before any earlier pregnancy loss. Normal karyotype of products on conception from an earlier loss. Female paner age over 35 years. Infeility. cause rf recurrent aboions 1st trimester * Genetic factor / defective germplasm. Most common type of chromosomal abnormality is balanced translocation. 2nd trimester * Cervical incompetence", "cop": 1, "opa": "Chromosomal abnormalities", "opb": "Uterine anomaly", "opc": "Hormonal disturbance", "opd": "Infection", "subject_name": "Gynaecology & Obstetrics", "topic_name": null, "id": "23dfc81d-9808-4c2d-a9fc-5695fc556702", "choice_type": "single"} {"question": "The instrument used in", "exp": "REF : SHAW GYNECOLOGY", "cop": 1, "opa": "laparoscopic sterilization", "opb": "removal of ectopic", "opc": "myomectomy", "opd": "Endometrial polypectomy", "subject_name": "Gynaecology & Obstetrics", "topic_name": "All India exam", "id": "ab315004-d2a6-4272-ba8d-08ac3948bb97", "choice_type": "single"} {"question": "Chronic pelvic pain is defined as", "exp": "Chronic pelvic pain is defined as pain of greater than 6 months in duration, localized to the anatomic pelvis, and severe enough to cause functional disability or necessitating medical care", "cop": 3, "opa": "Pain of greater than 1 month", "opb": "Pain of greater than 3 months", "opc": "Pain of greater than 6 months", "opd": "Pain of greater than 12 months", "subject_name": "Gynaecology & Obstetrics", "topic_name": "Disorders of menstruation", "id": "eb49a342-d415-4ddb-ab92-523cac9aae35", "choice_type": "single"} {"question": "During pregnancy HIV transmission occurs mostly during", "exp": "Ans. is 'd' i.e. during labour HIV can be transmitted from an infected mother to the fetus during pregnancy, during delivery or by breastfeeding.HIV can be transmitted to the fetus as early as the first and second trimester of pregnancy.The transmission during birth (intrapartum) and the immediate period thereafter (peripartum) is considered to be the most common.In the absence of prophylactic antiretroviral therapy of the mother during pregnancy, labor and delivery and to the fetus following birth, the probability of transmission of HIV from mother to infant ranges from 15-25% in industrialized countries and 25-35% in developing countries.Higher risk of transmission is associated with high maternal viral load.Low CD4 count is also considered a risk factor.A prolonged interval between membrane rupture and delivery is another well documented risk factor for transmission.Other conditions that are potential risk factors but which have not been consistently demonstrated are:Presence of chorioamnionitis at deliverySTD's during pregnancyHard drug use during pregnancyCigarette smokingPreterm deliveryObstetrics procedures such as amniocentesis, amnioscopy fetal scalp electrodes, episiotomy.There is significant decrease in the risk of HIV transmission with elective cesarean.LSCS reduces the risk because during this procedure, the intrapartum period which accounts for significant proportion of transmission is by passed.", "cop": 4, "opa": "1st trimester", "opb": "2nd trimester", "opc": "3rd trimester", "opd": "During labour", "subject_name": "Gynaecology & Obstetrics", "topic_name": "Infectious Diseases", "id": "1338f975-13a9-4500-8b58-e5eb93091009", "choice_type": "single"} {"question": "For MTP, consent should be obtained from", "exp": "For medical termination of pregnancy : Only female consent is required Husband consent is not required If the age is less than 18 yrs, consent of parent or guardian is required Ref: Dutta Obs 9e pg 165.", "cop": 3, "opa": "Male paner", "opb": "Male and female paner", "opc": "Female paner", "opd": "No consent needed", "subject_name": "Gynaecology & Obstetrics", "topic_name": "General obstetrics", "id": "a39939ad-28f2-4767-a7e8-97427fd25046", "choice_type": "single"} {"question": "Regarding antibiotic of choice for urinary tract infection (UTI) during pregnancy at third trimester;", "exp": "Usually a combination of ampicillin of cephalosporin and gentamicin is commended empirically and changed if necessary ,after the urine and blood culture results are obtained. Any of the following drugs can be used: Amoxicillin(500mg tid) Nitrofurantoin (100mg bid) Cephalexin(500mg tid) Amoxicillin-clavulanic acid(875mg bid) A Course of 10-14days will cure 70-100% of cases. Refer page no 274 of Text book of obsteics,sheila balakrishnan,2 nd edition and Textbook of Dutta, 9th edition,page 279.", "cop": 1, "opa": "Cephalosporin", "opb": "Quinolones", "opc": "Aminoglycosides", "opd": "Tetracyclines", "subject_name": "Gynaecology & Obstetrics", "topic_name": "Medical, surgical and gynaecological illness complicating pregnancy", "id": "a8c38dc5-e507-48f2-85b1-5aa9a8974f9a", "choice_type": "single"} {"question": "Karyotype of Sweyer syndrome", "exp": "Sweyer syndrome is an uncommon form of gonadal dysgenesis, characterized by a 46, XY karyotype. Despite the presence of a Y chromosome, the phenotype is female because the dysgenetic (streak) gonads produce neither AMH nor androgens. Consequently, the vagina, cervix, uterus, and fallopian tubes develop normally and the internal and external genitalia do not masculinize. In approximately 10-15% of patients, the disorder results from an inactivating mutation in the SRY gene, but in most, no cause can be identified. Patients with Swyer syndrome generally present after the expected time of pubey with delayed sexual maturation, primary amenorrhea, normal pubic hair, and normal female internal and external genital anatomy. Evaluation reveals hypergonadotropic hypogonadism, prompting a karyotype that establishes the diagnosis. Gonadectomy is indicated soon after diagnosis due to the significant risk for development of germ cell tumours in occult testicular elements (20-30%). Reference: Clinical Gynecologic Endocrinology; 8th edition; Chapter 9; Normal and Abnormal Sexual Development", "cop": 1, "opa": "46XY", "opb": "46XX", "opc": "45XO", "opd": "47XXY", "subject_name": "Gynaecology & Obstetrics", "topic_name": "Disorders of menstruation", "id": "078926e0-7ee8-4a40-88e6-bc15268ddc2b", "choice_type": "single"} {"question": "Classical symptom of endocervical polyp is", "exp": "Intermenstrual bleeding is classical symptom of endocervical polyp.", "cop": 3, "opa": "Cervical prolapse", "opb": "Dyspareunia", "opc": "Intermenstrual bleeding", "opd": "Dysmenorrhea", "subject_name": "Gynaecology & Obstetrics", "topic_name": null, "id": "bd461837-3b0e-4a06-a7d5-1352431c2558", "choice_type": "single"} {"question": "Most common mass descending mass per vaginum", "exp": "Ans. is 'd' i.e., Uterovaginal prolapseMass descending per vaginum is a common gynaecological complaint and is most often due to uterovaginal prolapse.", "cop": 4, "opa": "Labial cyst", "opb": "Vaginal cyst", "opc": "Myoma", "opd": "Uterovaginal prolapse", "subject_name": "Gynaecology & Obstetrics", "topic_name": null, "id": "e812b0a3-0693-492e-a1f3-c2cdc2518216", "choice_type": "single"} {"question": "Villous pattern is lost in", "exp": "Choriocarcinoma : On histopathology , it is composed of cells of early cytotrophoblast and synctiotrophoblast There are no villi Has ability to metastatic Most common mode of spread is hematogenous Ref: Dutta Obs 9e pg 187.", "cop": 4, "opa": "Hydatidiform mole", "opb": "Invasive mole", "opc": "Tubal mole", "opd": "Choriocarcinoma", "subject_name": "Gynaecology & Obstetrics", "topic_name": "General obstetrics", "id": "03dc974e-032e-43b1-9a28-8a398738a9e0", "choice_type": "single"} {"question": "Endometriosis is best diagnosed by", "exp": "Endometriosis is best diagnosed by laparoscopy Role of Laparoscopy: To detect and diagnose pelvic endometriosis Locate site of endometriosis and staging To take biopsy To surgically treat endometriosis by ablation and removal", "cop": 2, "opa": "Ultrasound", "opb": "Laproscopy", "opc": "Laprotomy", "opd": "Endometerial curettage", "subject_name": "Gynaecology & Obstetrics", "topic_name": "Endometriosis & H", "id": "be4e95df-73cd-4273-9a59-c2ac86518d2d", "choice_type": "single"} {"question": "Large placenta is seen in", "exp": "Ans. a Twins. (Ref. Dutta, Obstetrics, 4th/pg.221)D/d of Large placenta/hyperplacentosis:# Twin pregnancy# Hydrops fetalis# Diabetes in pregnancy# IUGRHUMAN PLACENTA# It is Discoid, Deciduate and Haemchorial.# It develops from--- foetal (4/5th)--chorionic frondosum and- maternal (l/5th)--decidua basalis.# Its development begins at 6th week of gestation and is completed by 12th week.Q# Its weight is 500 gms and weight of blood in it is 500 ml.# Fetal blood flow through placenta = 400 ml/min# Decidua basalis retains its characteristic appearance till term and becomes the maternal portion of placenta.# Line of separation of placenta after birth of the baby is through decidua spongiosum.# Fetal surface has attachment of umbilical cord with ramification of umbilical vessels and aminon.# Only decidua basalis and blood in the inter villou space are of maternal origin.# Maternal surface has shaggy looks with cotyledons.# Placentome = fetal cotyledon.# Villi are the functional units of placenta. Umbilical arteryUmbilical veinO2 saturation50-60%70-80%pO2 (mm Hg)20-2530-40# Placental calcification is a feature of postmaturity.# Signs of Placental aging:- Appearance of syncytial knots.- Decreased stromal tissue including Hofbauer cells.- Partial deficiency of Langhan's cells.- Thinning of basement membrane layer of foetal endothelium.# Most common tumour of placenta is choriohaemangioma.QDiscriptionDiscriptionType of placentaPlacenta with umbilical cord attached to its marginPlacenta with chorionic plate smaller than basal platePlacenta associated with increased rate of prematurityPlacenta with accessory lobes in the membranes away from main body of placenta, connected by large vessels to main body which if it goes to cervix causes formation of \"Vasa previa\"--a vital cause of bleeding of fetal originBattledore placentaCircumvallate placenta.Circumvallate placenta.Succenturiate placenta.Placenta with functioning villi covering all foetal membranesPlacenta with central portion missingMembranous placenta.Fenestrated placenta.", "cop": 1, "opa": "Twins", "opb": "Oligohydramnios", "opc": "IUGR", "opd": "Pre-eclamptic toxemia", "subject_name": "Gynaecology & Obstetrics", "topic_name": "Miscellaneous (Gynae)", "id": "4c49f1f9-24e6-4c2d-93e4-31aa596557f2", "choice_type": "single"} {"question": "Female homologue of scrotum is", "exp": "Embryologically, Labia majora are homologous with male scrotum.", "cop": 1, "opa": "Labia majora", "opb": "Labia minora", "opc": "Vagina", "opd": "Vestibule", "subject_name": "Gynaecology & Obstetrics", "topic_name": null, "id": "6a9957e9-645c-4247-be3c-8a2db1634835", "choice_type": "single"} {"question": "Major source of estrogen in postmenopausal women is", "exp": "All postmenopausal and symptomatic per menopausal women who do not have contraindication paicularlyThose with high-risk factors osteoporosis.Those with premature menopauseEvaluation before staing H:History-personal, medical, gynecological, familyExamination -HT, wt., BMI, BP, breast, vaginal examinationPap smearMammographyLipid profileTransvaginal sonography-endometrial thicknessWork up relevant to symptoms, e.g. urinary continence, carbohydrate intolerance, psychoemotional states.Contraindications of H:PregnancyCA breastCA endometriumUndiagnosed abnormal vaginal bleedingActive liver diseasePresent deep vein thrombosis, thromboembolic disorderInvestigation not routinely needed before staing HEndometrial sampling-Only did if the history of abnormal vaginal bleeding, TVS shows endometrial thickness> 5 mm.Serum FSH-Only did if the diagnosis of menopause is not clear, e.g. vasomotor symptoms in absence of amenorrhoea Bone densitometry-Using DEXA (dual energy X-ray absorptiometry) of lumber veebrae, hip bone for women with risk factors for osteoporosis, persistent backache, bone aches, history of recent fracture, for research purpose", "cop": 1, "opa": "Peripheral conversion of androstenedione", "opb": "Secretion from adrenals", "opc": "Testosterone from the ovary", "opd": "H", "subject_name": "Gynaecology & Obstetrics", "topic_name": "Menopause and HRT", "id": "fb3400bd-f8b3-4bca-9419-aa25de6cfc4b", "choice_type": "single"} {"question": "Outlet forceps, meansa) Head at station \"0\"b) Full cervical dilatationc) Rupture of membraned) Rotation > 45", "exp": null, "cop": 4, "opa": "a", "opb": "c", "opc": "ac", "opd": "bc", "subject_name": "Gynaecology & Obstetrics", "topic_name": null, "id": "46aff9b5-4029-4595-8b1d-ca37d6471d36", "choice_type": "single"} {"question": "Endometrial hyperplasia is seen in", "exp": "Polycystic ovarian disease - endometrial hyperplasia due to high content of estrogen, largely estrone, by extraglandular aromatization of circulating androstenedione.", "cop": 3, "opa": "Endodermal sinus tumour", "opb": "Dysgerminoma", "opc": "Polycystic ovarian disease", "opd": "Carcinoma of cervix", "subject_name": "Gynaecology & Obstetrics", "topic_name": "All India exam", "id": "0204606d-0957-4071-a809-5625a878a840", "choice_type": "single"} {"question": "Most common Vagal carcinoma is", "exp": "Ans. is a i.e. Squamous cell Ca Vaginal carcinoma Primary cancer of vagina are very rare. Most common age group is Elderly females - 70 years. Most common histologic type is Squamous cell carcinoma Most common site is Upper third of Posterior wall of vagina. Mostly asymptomatic Patient may present with abnormal vaginal bleeding (including post coital bleeding). Foul smelling discharge per vaginum. P/S - Ulcerative/exophytic growth on vagina. Cervix appears normal. Lymphatic Drainage : Tumor arising in upper vagina : drain to pelvic lymph node. Tumor arising in lower pa : drain to inguinal lymph node. Management : Growth limited to middle third : Radiotherapy Growth limited to upper third and lower third : Surgery.", "cop": 1, "opa": "Squamous cell ca", "opb": "Adenocarcinorna", "opc": "Botroid's tumor", "opd": "Columnar hyperplasia", "subject_name": "Gynaecology & Obstetrics", "topic_name": null, "id": "c56b6482-d6c6-445f-b209-d455784aef1a", "choice_type": "single"} {"question": "Decreased vascularity of fibroid is seen with", "exp": "GnRH analogues used for 6months reduce the size by 50-80%. It is also useful in reducing the vascularity besides size preoperatively and thus causing amenorrhoea which restores hb level. Ref:shaw's textbook of gynaecology;15th edition; Pg :359", "cop": 1, "opa": "GnRH agonist", "opb": "Danazol", "opc": "Mifepristone", "opd": "Clomiphene citrate", "subject_name": "Gynaecology & Obstetrics", "topic_name": "Uterine fibroid", "id": "7b64d838-2c0f-4a0c-bf41-a17660f44872", "choice_type": "single"} {"question": "FSH to LH ratio during reproductive years", "exp": "During reproductive years, LH exceeds FSH levels.", "cop": 1, "opa": "LH > FSH", "opb": "FSH > LH", "opc": "FSH = LH", "opd": "FSH alone present, No LH.", "subject_name": "Gynaecology & Obstetrics", "topic_name": null, "id": "04faad6c-5da4-4f7b-a5bc-1ce9509e1adc", "choice_type": "single"} {"question": "Staging of ovarian tumor involving bladder", "exp": "-Stage II ovarian cancer: tumor involves one or both ovaries or fallopian tubes with pelvic extension or primary peritoneal cancer. IIA: ovarian tumor involving uterus or fallopian tubes IIB: ovarian tumor involving pelvic intraperitoneal organs ( bladder, rectum or pelvic side walls) Reference : textbook of gynaecology Sheila balakrishnan, 2nd edition, pg no:277 <\\p>", "cop": 1, "opa": "Stage IIB", "opb": "Stage IIIA", "opc": "Stage IIIB", "opd": "Stage IVA", "subject_name": "Gynaecology & Obstetrics", "topic_name": "Gynaecological oncology", "id": "c1c2edba-1fdf-4b2b-ab46-387b168dcc77", "choice_type": "single"} {"question": "Volume of amniotic fluid needed for fetal karyotyping.", "exp": "20 ML of amniotic fluid is required for fetal karyotyping.\n10 ML is required for FISH.", "cop": 2, "opa": "10 ML", "opb": "20 ML", "opc": "2 -3 ML", "opd": "3 - 5 ML", "subject_name": "Gynaecology & Obstetrics", "topic_name": null, "id": "58994b80-c863-44fc-bee6-b75fc0172979", "choice_type": "single"} {"question": "Uteroplacental blood flow per minute", "exp": "Uteroplacental blood flow per minute : 450 - 650 mL/min", "cop": 4, "opa": "200 ml / minute", "opb": "400 ml / minute", "opc": "300 ml / minute", "opd": "600 ml / minute", "subject_name": "Gynaecology & Obstetrics", "topic_name": "Maternal Anatomy and Physiology", "id": "0f1d9272-7da7-4b15-a39c-afd7923b4fe7", "choice_type": "single"} {"question": "Goodell sign in early pregnancy is", "exp": "In early pregnancy, increased vascularity within the cervix stroma beneath the epithelium creates an ectocervical blue tint that is characteristic of Chadwick sign. Cervical edemaleads to softening , whereas isthmic softening is Hegar sign.(Ref: William's Obstetrics; 25th edition)", "cop": 1, "opa": "Cervical softening", "opb": "Ectocervical blue tint", "opc": "Isthmic softening", "opd": "Uterine softnening", "subject_name": "Gynaecology & Obstetrics", "topic_name": "All India exam", "id": "7cc362fb-468d-4591-95fb-09587d7a4128", "choice_type": "single"} {"question": "P/V examation is contraindcated in", "exp": "(Placenta previa) (244- Dutta 7th)PLACENTA PREVIAVaginal examination must not be done outside the operation theater in the hospital, as it can provide further separation of placenta with torrential hemorrhage and may be fatal. It should only be done prior to termination of pregnancy in the operation theater under anesthesia, keeping everthing ready for casarean section* The regimen used for expectant management of placenta previa is- Macafee & Johnson regimen**Expectant Management of placenta previa* Rest (Absolute bed rest for up to 72 hours after bleeding stops)* Careful monitoring (clinical/ Laboratory/ USG every 2-3 weeks* Blood transfusion and Hematinics (maintain Hb atleast 10 g/dl and Hematocrit of 30%)* Antenatal steroids for foetal maturity (Between 24-34 weeks of gestation)* Anti D immunoglobulin (to all RH negative women)Use of Tocolysis (controversial) May be justified in selected cases but their use is not supported by strong evidenceCervical Encirclage - Not currently recommended insufficient evidence* First sign of magnesium toxicity is loss of deep tendon reflex* Bile acids represent best markers for intrahepatic cholestasis of pregnancy* Ursodiol (uricodeoxycholic acid) is treatment of choice for Intrahepatic cholestases in pregnancy* Methotrexate is absolutely contraindicated during pregnancy (class x)and lactation due to its potential for spontaneous abortion and teratogenecity* Transplacental transmission is rare with both Herpes simplex and Hepatitis B", "cop": 1, "opa": "Placenta previa", "opb": "Cord prolapsed", "opc": "Rupture membrane", "opd": "CPD", "subject_name": "Gynaecology & Obstetrics", "topic_name": "Miscellaneous (Gynae)", "id": "f9438e5f-2e68-4e02-a5b9-9a0ca60a6c61", "choice_type": "single"} {"question": "Ventouse in the 2nd stage of labour is contraindicated in", "exp": "1.Ventouse in the 2nd stage of labour is contraindicated in Prematurity as it can lead to - cephalohematoma, subgaleal hemorrhage, retinal hemorrhage, neonatal jaundice secondary to these hemorrhages and scalp lacerations. 2. A forceps is known to cage a small preterm head and in fact protect it.So, it is prefered", "cop": 4, "opa": "Persistent occipitoposterior position", "opb": "Uterine ineia", "opc": "Hea disease", "opd": "Prematurity", "subject_name": "Gynaecology & Obstetrics", "topic_name": "Labour - II", "id": "8f1b6243-533b-48b5-a482-9396d2473065", "choice_type": "single"} {"question": "Commonest diameter of fetal head engagement is", "exp": "The commonest diameter for fetal head engagement is suboccipitobregmatic, as it leads to veex presentation for normal vaginal delivery. Fetal skull diameters Measurements Suboccipitobregmatic Biparietal diameter Submentobregmatic 9.5 cm Suboccipitofrontal 10cm Occipitofrontal Submentoveical 11.5 cm Mentoveical 14 cm", "cop": 3, "opa": "Submentoveritcal", "opb": "Occipito frontal", "opc": "Suboccipito bregmatic", "opd": "Mentoveical", "subject_name": "Gynaecology & Obstetrics", "topic_name": "Labour - II", "id": "71df9e9b-150d-4f8f-962c-20c1b4e99e16", "choice_type": "single"} {"question": "Late hyperglycemia in pregnancy is associated with", "exp": "Fetal macrosomia Definition: Those infants exceeding the 90th percentile for a given gestational week are usually used as the threshold for macrosomia or large-for-gestational age (LGA) bihweight The primary effect attributed to gestational diabetes is excessive fetal size or macrosomia Maternal hyperglycemia prompts fetal hyperinsulinemia, paicularly during the second half of gestation . This in turn stimulates excessive somatic growth or macrosomia. Except for the brain, most fetal organs are affected by the macrosomia that characterizes the fetus of a diabetic woman. Infants are anthropometrically different from other large-forgestational age (LGA) infants with excessive fat deposition on the shoulders and trunk, which predisposes to shoulder dystocia or cesarean delivery The incidence of macrosomia rises significantly when mean maternal blood glucose concentrations chronically exceed 130 mg/dL Reference: William&;s Obstetrics; 24th edition; Chapter 57", "cop": 1, "opa": "Macrosomia", "opb": "lUGR", "opc": "Postmaturity", "opd": "Congenital malformation", "subject_name": "Gynaecology & Obstetrics", "topic_name": "Medical, surgical and gynaecological illness complicating pregnancy", "id": "0d519093-8331-441f-8118-45dcb2621092", "choice_type": "single"} {"question": "Isotretinoin to be stopped before pregnancy", "exp": "Isotretinoin iseffective for treating severe acne but it is is category X drug in pregnancy. It is recommended that a woman should wait at least 1 month after stopping Isotretinoin before trying to became pregnant but the desirable time is 3 months.", "cop": 3, "opa": "3 days", "opb": "2 week", "opc": "3 month", "opd": "3 years", "subject_name": "Gynaecology & Obstetrics", "topic_name": "Drugs in Pregnancy", "id": "1f39033d-a8c5-449a-a7aa-9637a8256dc6", "choice_type": "single"} {"question": "Retinal vessels develop by", "exp": "Ans: a (4th month) Ref: yanoff 3rd ed p- 606During development, retinal vessels migrate from the optic disc to the ora serrata beginning at 16 weeks of gestation.Mature vessels develop from these networks and extend to the nasal ora serrata by 36 weeks of gestation and to the temporal ora serrata by 39 - 41 weeks.", "cop": 1, "opa": "4th month", "opb": "6th month", "opc": "1st month", "opd": "3rd month", "subject_name": "Gynaecology & Obstetrics", "topic_name": "Miscellaneous (Obs)", "id": "cd1d33d1-08cb-43be-bedc-79d1a6abdd7f", "choice_type": "single"} {"question": "Endo metrial tuberculosis causing infertility is due to", "exp": "B. i.e. (Tubal blockage) (189 - Shaw's 14th) (215-130 Dutta 4th)GENITAL TUBERCULOSIS* Endometrial ulceration may lead to adhesions or synechiae formation (Asherman's syndrome). This may cause infertility, secondary amenorrhoea or recurrent abortion* In pelvic tuberculosis, blockage of tubes and endometrial tuberculosis causing As Herman syndromes (adhesions) are responsible* Breast milk is known to transmit - Untreated tuberculosis*** The uterine fibroid - which accounts for infertility is a corneal fibroid blocking the medial end of the fallopian tube, submucous fibroid, and cervical fibroid distorting the passage of the sperms**Isthumo- isthemic anastomosis is associated with maximum chances of re- canalization during surgery for reversal of tubal ligation.", "cop": 2, "opa": "Anovulation", "opb": "Tubal blockage", "opc": "Ciliary dysmotility", "opd": "Damage of endometrium", "subject_name": "Gynaecology & Obstetrics", "topic_name": "Miscellaneous (Gynae)", "id": "0d1b2245-1f59-40aa-b421-dacf5b68b90b", "choice_type": "single"} {"question": "Patient with NTD, dose of folic acid in next pregnancy", "exp": "Folic acid supplementation 4 mg daily 1 month before conception to about 12 weeks of pregnancy. Ref : Dutta book of obstetrics 8th Ed", "cop": 4, "opa": "0.5mg", "opb": "1mg", "opc": "2mg", "opd": "4mg", "subject_name": "Gynaecology & Obstetrics", "topic_name": "All India exam", "id": "5a336928-7822-4f69-bc12-438317d95dfd", "choice_type": "single"} {"question": "Burch technique of treating urinary incontinence is", "exp": "Burch colposuspension is procedure of retropubic urethropexy, wherein pubocervical fascia is attached to cooper ligament to lift anterior vaginal wall and periurethral fibromuscular tissue.", "cop": 2, "opa": "Pubovaginal sling", "opb": "Retropubic urethropexy", "opc": "Midurethral slings", "opd": "Needle suspension", "subject_name": "Gynaecology & Obstetrics", "topic_name": null, "id": "f1393574-95d6-4a03-bbfb-c82490c89002", "choice_type": "single"} {"question": "Treatment of choice for molar pregnancy is", "exp": "Regardless of uterine size, molar evacuation by suction curettage is usually the preferred treatment.Reference: William's Obstetrics; 25th edition, chapter 20; Gestational trophoblastic diseases", "cop": 3, "opa": "Expectant management", "opb": "Medical termination", "opc": "Dilatation and evacuation", "opd": "Chemotherapy", "subject_name": "Gynaecology & Obstetrics", "topic_name": "General obstetrics", "id": "8786fbe7-4cd9-4921-a207-8444769729e9", "choice_type": "single"} {"question": "Most commonly this tumour is due to carcinoma of", "exp": "Transperitoneal implantation of the carcinoma of the stomach may occur in which the carcinoma cells pass from, stomach into the peritoneal cavity; they \"gravitate\" into the pelvis where the secondaries palpable on rectal examination I may develop. In females, occasionally, they \"alight\" upon the ovaries giving rise to Krukenberg tumors which are I liable to produce diagnostic confusion. The term Krukenberg tumor is classically reserved\" for metastatic involvement of the ovary by a gastrointestinal tumor. The microscopic appearance of Krukenberg tumors shows characteristic mucin-containing signet-ring cells. This term is often used loosely to describe the metastatic involvement of the ovaries in general. These metastatic lesions often present clinically as primary ovarian lesions and may have a similar radiographic appearance to primary carcinoma of the ovary.", "cop": 1, "opa": "Stomach", "opb": "Breast", "opc": "Colon", "opd": "Uterus", "subject_name": "Gynaecology & Obstetrics", "topic_name": "Gynaecological oncology", "id": "60ae9b46-b76a-4986-b466-f4a62dc5e33b", "choice_type": "single"} {"question": "Pseudomyxoma peritonei occurs in", "exp": "i.e. (Mucinous cystadenocarcinoma of the ovary): (375-388-Shaw9s 15th) (1003-Bailey & Love 25th) (476-CSDT 13th)PSEUDOMYXOMA PERITONEI* Caused by low grade mucinous cystadenocarcinomas of the appendix or ovary* Recent studies suggest that most cases arise from a primary appendiceal tumour with secondary implantation on to one or both ovaries* The abdomen is filled with a yellow jelly, large quantities of which are often encysted* Painless and there is frequently no impairment of general health* Pseudomyxoma peritonei does not give rise to extraperitoneal metastasis* CT and USG shows a distinctive peritoneal scalloping of the liver margin, calcified plaques, ascites and low density masses* The appendix if present should be excised together with any ovarian tumour**** Adjuvant intracavitary radiotherapy - for residual disease**", "cop": 1, "opa": "Mucinous cystadenocarcinoma of ovary", "opb": "Carcinoma endometrium", "opc": "Ovarian dermoid", "opd": "Fibroid uterus", "subject_name": "Gynaecology & Obstetrics", "topic_name": "Miscellaneous (Gynae)", "id": "84a6b5cb-fb95-4745-9b38-e9d4674637a8", "choice_type": "single"} {"question": "Cause of death in breech delivery", "exp": "Ans. is a i.e. Intracranial hemorrhage Intracraniai hemorrhage is the most common cause of fetal loss. occurs due to tear of tentorium cere.belli and falx cerebri. It is caused by traumatic delivery of the after coming head of breech or too rapid delivery of the soft head of a premature baby.", "cop": 1, "opa": "lntracranial hemorrhage", "opb": "Aspiration", "opc": "Atlanto axial dislocation", "opd": "Asphyxia", "subject_name": "Gynaecology & Obstetrics", "topic_name": null, "id": "14770f5e-a613-41c0-b7f5-4f6aeb92a11e", "choice_type": "single"} {"question": "Absolute contraindication to Hormone replacement therapy", "exp": "Absolute conrtraindications to HRT\n\nSuspected pregnancy\nBreast cancer\nEndometrial cancer\nActive liver disease\nKnown thrombophilia\nOsteosclerosis.", "cop": 2, "opa": "Osteoporosis", "opb": "Otosclerosis", "opc": "Colon cancer", "opd": "Premature menopause", "subject_name": "Gynaecology & Obstetrics", "topic_name": null, "id": "d96b90f3-642a-416c-9001-eacdb474e9a8", "choice_type": "single"} {"question": "Most sensitive test for assessment of fetal well being is", "exp": "(Biparietal diameter) (85-Dutta 6th)Real ultrasound measurement of fetal growth* First trimester - Crown - rump length* Second trimester - Biparietal diameter (BPD)* Third trimester - Multiple foetal growth parameters are considered together. These are- Prior to 36 weeks - Combination of biparietal diameter, abdominal circumference and femur length (FL)- After 36 weeks - Combination of head circumference, abdominal circumference and femur lengthA- BPD greater than 9.2 cm indicates foetal pulmonary maturity corroborated by L:S ratio* Early measurement of BPD and FL are the most useful in establishing gestational age Later measurement of fetal growth are best achieved by measurement of the head circumference and the abdominal circumference (G. chamberlain 7th)* Foetal head: abdominal circumference ratioMaximum - 1.29 at 12 weeksEqual - 1.00 at 36 weeksMinimum - <1.00 after 37 weeks* Bishop score is used for induction of labour* Manning score is most sensitive & reliable test for fetal well beingDifferent diameter of skull in newborn are* Suboccipito-bregmatic =9.5cm* Suboccipito-frontal =10cm* Occipito frontal =11.5cm* Mento vertical =14cm* Sub mento vertical =11.5cm* Sub mento-bregmatic =9.5cm* Biparietal diameter =9.5cm* Super-subparietal =8.5cm* Bitemporal diameter =8cm* Bimastoid diameter =7.5cm* Twin peak sign - Ultrasonography shows extension of placental tissue into the inner twin membrane in diaminiotic dichorionic (DD)", "cop": 1, "opa": "Biparital diameter", "opb": "Arm circumference", "opc": "Abdominal circumference", "opd": "Head circumference", "subject_name": "Gynaecology & Obstetrics", "topic_name": "Miscellaneous (Gynae)", "id": "4ff6db83-2049-4da9-9bb0-56eb6a8db6db", "choice_type": "single"} {"question": "Lifespan of the fetal RBC approximately is", "exp": "The lifespan of fetal red cells is 80 days and Rh factor is developed by 38 days.", "cop": 3, "opa": "60 days", "opb": "100 days", "opc": "80 days", "opd": "120 days", "subject_name": "Gynaecology & Obstetrics", "topic_name": null, "id": "bd46a54b-5705-4a60-8e5a-610bc0f72a4c", "choice_type": "single"} {"question": "Tumor marker of Dysgerminoma", "exp": "Dysgerminoma is the most common malignant germ cell tumour, accounting for about 30% to 40% of all ovarian cancers of germ cell origin. Placental alkaline phosphatase (PLAP) and lactate dehydrogenase (LDH) are produced by up to 95% of dysgerminomas, and serial measurements of LDH may be useful for monitoring the disease. Reference: Berek and Novak's Gynecology; 14th edition; Chapter 37; Ovarian, Fallopian Tube, and Peritoneal Cancer", "cop": 4, "opa": "a-fetoprotein", "opb": "hCG", "opc": "CA-125", "opd": "Lactate dehydrogenase", "subject_name": "Gynaecology & Obstetrics", "topic_name": "Gynaecological oncology", "id": "ecfee3aa-2a33-4ca6-8e4a-1df630c00746", "choice_type": "single"} {"question": "Follistatin", "exp": "Follistatin is gonadal peptide, that suppresses FSH β gene expression thereby preventing interaction of activin with its receptor.", "cop": 3, "opa": "Decreased gonadotrope function", "opb": "Increases gonadotrope function", "opc": "Prevents interaction of activin with receptor", "opd": "Stimulates FSH β gene expression", "subject_name": "Gynaecology & Obstetrics", "topic_name": null, "id": "515558aa-49d8-4480-ad6a-bbbe6d9b26ec", "choice_type": "single"} {"question": "This organism may cause epidemics of puerperal sepsis.", "exp": "Most female pelvic infections are caused by bacteria indigenous to the genital tract. Over the past 20 years, there have been repos of group A b-hemolytic streptococcus causing toxic shock-like syndrome and life-threatening infection Gram-positive cocci--group A streptococci, enterococcus, Staphylococcus aureus, Staphylococcus epidermidis Gram-negative bacteria--Escherichia coli, Klebsiella, Anaerobes Cocci--Peptostreptococcus and Peptococcus species Others--Clostridium, Bacteroides, and Fusobacterium species, Mobiluncus species", "cop": 2, "opa": "Cytomegalovirus", "opb": "Group A b-hemolytic streptococci", "opc": "Group B b- hemolytic streptococci", "opd": "Toxoplasma gondii", "subject_name": "Gynaecology & Obstetrics", "topic_name": "Puerperium", "id": "686db3da-b8fa-4622-be0f-9ba798616f27", "choice_type": "single"} {"question": "The Point of distinction between paial mole to complete mole is", "exp": "Ref: Shaw&;s Textbook of Gynecology; 16th edition; Chapter 22; Gestational Trophoblastic Diseases", "cop": 4, "opa": "Paial mole show trophoblastic proliferation with absent villi", "opb": "Typical of paial mole is cellular atypia", "opc": "Paial mole is more prone to tumor malignancy", "opd": "Paial mole is triploid", "subject_name": "Gynaecology & Obstetrics", "topic_name": "General obstetrics", "id": "7f4cbefd-b73c-40bc-830f-ed18ac50eb73", "choice_type": "single"} {"question": "Transverse uterine incision is preferred to vertical incision because", "exp": "Compared with classical incision, transverse incision is easier to repair, is located in inactive segment and thus least likely to rupture in future pregnancies.\nIt causes less incision site bleeding and promotes less bowel adhesions.", "cop": 2, "opa": "Less risk of post partum endometritis.", "opb": "Lessens risk of rupture in subsequent pregnancy.", "opc": "Easier to deliver the fetus.", "opd": "Is located in active segment of uterus.", "subject_name": "Gynaecology & Obstetrics", "topic_name": null, "id": "3c9f9a57-7399-4229-960d-ee212d503239", "choice_type": "single"} {"question": "MgSO4 is/are indicated in", "exp": "The drug of choice for control and prevention of convulsion in eclampsia is magnesium sulphate ,usually given by Pritchard&;s regime.The mechanism of action thought to have a peripheral action at the neuromuscular junction,by competing with calcium for entry into the cells.It may also have central action. TEXT BOOK OF OBSTETRICS,Sheila Balakrishnan,2nd edition,page no.268", "cop": 1, "opa": "Severe pre eclampsia", "opb": "Gestational trophoblastic disease", "opc": "Gestation diabetes", "opd": "Placenta prae", "subject_name": "Gynaecology & Obstetrics", "topic_name": "Medical, surgical and gynaecological illness complicating pregnancy", "id": "bfcd698a-8c75-40c0-9c18-ef7fc86980ab", "choice_type": "single"} {"question": "Average blood loss in normal menstruation", "exp": "The duration of menstruation is 4-5 days and the amount of blood loss estimated to be 20-80ml with an average of 35ml. Reference: D.C.DUTTA&;S TEXTBOOK OF GYNECOLOGY; 6th edition; PG NO:82", "cop": 1, "opa": "50 mL", "opb": "80 mL", "opc": "100 mL", "opd": "120 mL", "subject_name": "Gynaecology & Obstetrics", "topic_name": "Disorders of menstruation", "id": "dc6e50ce-0c0a-4d7e-adac-ca6a0746af3f", "choice_type": "single"} {"question": "Area of mitral valve in severe mitral stenosis during pregnancy", "exp": "Normal area of mitral valve is 4-6cm2 Symptoms usually appear when stenosis <2.5cm2 Severe mitral stenosis is less than 1.5cm2 Critical area is less than 1cm2- High rate of Pulmoary edema(55%) and Arrhythmia(33%) Reference: William's Obstetrics; 24th edition; chapter 49 & DC.Duttas ,9th edition,page 260", "cop": 3, "opa": "4-6 cm2", "opb": "1.5-2.5 cm2", "opc": "1-1.5 cm2", "opd": "0.8-1 cm2", "subject_name": "Gynaecology & Obstetrics", "topic_name": "Medical, surgical and gynaecological illness complicating pregnancy", "id": "ae8b4582-5fa0-4aa5-9d2a-ec68f9a5b216", "choice_type": "single"} {"question": "Intermenstrual bleeding is frequent side effect seen with", "exp": "Intermenstrual bleeding is frequent with progestin - only pills throughout use and with combined OCPs during early months of use.", "cop": 4, "opa": "Progestin-only pills during early months of use.", "opb": "Combined OCPs throughout use.", "opc": "Progestin-only pills resultant AUB never occurs.", "opd": "Combined OCPs during early months of use.", "subject_name": "Gynaecology & Obstetrics", "topic_name": null, "id": "310ce282-65e2-4855-941c-e44f10dff4f0", "choice_type": "single"} {"question": "A forceps rotation of 30deg from left occiput anterior (LOA) to occiput anterior (OA) with extraction of fetus from +2 station", "exp": "FORCEPS OUTLET FORCEPS Scalp visible at introitus without seperating labia leading point of fetal skull is at the pelvic floor saggital suture in AP diameter or rotation<45 degree LOW FORCEPS leading point of fetal skull is at station +2cm or below,not on pelvic floor rotation not more than 45 degree MIDCAVITY FORCEPS fetal head 1/5 palpable per abdomen head engaged but station above +2cm rotation>45 degree HIGH FORCEPS never done TEXTBOOK OF OBSTETRICS,SHEILA BALAKRISHNAN,pg no:443,2nd edition", "cop": 3, "opa": "High forceps", "opb": "Mid forceps", "opc": "Low forceps", "opd": "Outlet forceps", "subject_name": "Gynaecology & Obstetrics", "topic_name": "Abnormal labor", "id": "65d86f5b-0a73-45e9-bbc3-ac420fd1264f", "choice_type": "single"} {"question": "Red degeneration most commonly occurs in", "exp": "Ans. is 'b' i.e. 2nd Trimester Red degeneration of the myomasRed degeneration of uterine myomas develops most frequently during pregnancy. It typically occurs during 2nd trimester (20-22) weeks of pregnancy.During red degeneration, the myomas become tense, tender and causes severe abdominal pain with constitutional upset and fever.Although the patient is febrile with moderate Leucocytosis and raised E.S.R., the condition is an aseptic one.It needs to be differentiated from appendicitis, twisted ovarian cyst, pyelitis and accidental hemorrhage.During the red degeneration, the tumour itself assumes a peculiar purple red colour and develops a fishy odour. If the tumour is carefully examined, some of the large veins in the capsule and the small vessels in the substance of the tumour will be found thrombosed. The discoloration is possibly caused by diffusion of blood pigments from thrombosed vessels.Conservative treatment with analgesia, reassurance and supportive therapy is almost always adequate.Also knowAnother complication of myoma which is seen during pregnancy is torsion of the myoma.Torsion of the myoma is also characterized by acute abdominal pain with point tenderness over the site of leiomyoma.Conservative t/t is adequate.", "cop": 2, "opa": "1st trimester", "opb": "2nd trimester", "opc": "3rd trimester", "opd": "Puerperium", "subject_name": "Gynaecology & Obstetrics", "topic_name": "Clinical Features, Diagnosis, and Pathology", "id": "318c223a-180d-44cd-b4c3-7e1ba73bea2b", "choice_type": "single"} {"question": "Compared with other mullerian duct defects, a transverse vaginal septum is associated with lower rate of", "exp": "In contrast to other mullerian duct defects, transverse vaginal septum is associated with few urologic abnormalities.", "cop": 2, "opa": "Endometriosis", "opb": "Renal anomalies", "opc": "Ectopic pregnancy", "opd": "Retrograde menstruation", "subject_name": "Gynaecology & Obstetrics", "topic_name": null, "id": "489d5015-fad1-4066-bd41-8ac7c15beb22", "choice_type": "single"} {"question": "Prelabour pains are mediated through", "exp": "T11-T12 Pain pathways during labour The pain of labour arises from - Contraction of myometrium, against the resistance of cervix and perineum - Progressive dilatation of the cervix and lower uterine segment - Stretching and compression of pelvic and perineal structures Pain during the, first stage of labour is mostly visceral pain resulting from uterine contractions and cervical dilatation. It is usually confined to the T11-T12 dermatomes during the latent phase but eventually involves the TIO-LI dermatomes as the labour enters the the active phase. The visceral afferent fibres responsible for labour pain travel with sympathetic nerve.fibres.first to the uterine and cervical plexes then through the hypogastric and aoic plexes before entering the spinal cord with the T10-L1 nerve roots. The onset of perineal pain at the end of fist stage signals the beginning of the fetal descent and the second stage of labour. Stretching and compression of pelvic and perineal structures intensify the pain. Sensory innervations of the perineum is provided by the pudendal nerve (S2-S4) so pain during the second stage of labour involves the T10-S4 dermatosomes.", "cop": 1, "opa": "Tit-T12", "opb": "T2-L3", "opc": "S1-S3", "opd": "L3-L4", "subject_name": "Gynaecology & Obstetrics", "topic_name": null, "id": "68b0418d-0260-445b-9518-c07108f41b41", "choice_type": "single"} {"question": "The immunoglobulin that crosses placental barrier is", "exp": "Ans. b (IgG). (Ref: Jawetz's Medical Microbiology, 24th/Table 8-l;Obstetrics by Dutta, 4th/677)# Antigens of MCH have been shown to be well expressed in newborn, in the fetus and in embryo of 10th week.# IgG abs can cross placenta. IgGIgAIgMIgEIgDHeavy chain symbolgabedMolecular weight (x 1000)150170 or 4001900190150Serum concentration (mg/mL)7-180.8-40.4-2.5< 0.0005< 0.003Serum half-life (days)217722Activates complementYes (+)NoYes (++)NoNoPercentage of total immunoglobulins in serum80136< 1< 1Crosses placentaYesNoNoNono", "cop": 2, "opa": "IgM", "opb": "IgG", "opc": "IgA", "opd": "IgD", "subject_name": "Gynaecology & Obstetrics", "topic_name": "Miscellaneous (Gynae)", "id": "b1eb0e51-f41a-42d6-80aa-bc262a6cb425", "choice_type": "single"} {"question": "FSH rise in menopause is attributed to", "exp": "Increased FSH levels reflect the reduced capability of aging follicles to secrete inhibin.", "cop": 2, "opa": "Diminished oestradiol feedback", "opb": "Decreased ovarian inhibin secretion", "opc": "Decreased GnRH frequency", "opd": "Increased AMH concentration", "subject_name": "Gynaecology & Obstetrics", "topic_name": null, "id": "2cea49a0-eceb-43bc-aa0a-c2c6b386131c", "choice_type": "single"} {"question": "In postmenopausal women, RANK ligand expression is", "exp": "In post menopausal women, decreased oestrogen levels lead to increased RANK ligand expression and decreased osteoprotegerin (OPG), resulting in bone resorption.", "cop": 2, "opa": "Decreased", "opb": "Increased", "opc": "Same as pre-menopause", "opd": "First decrease to increase later", "subject_name": "Gynaecology & Obstetrics", "topic_name": null, "id": "8ee80950-1b64-4fc8-9bcb-efc1d420a358", "choice_type": "single"} {"question": "Strassmann metroplasty is done in case of", "exp": "Strassman metroplasty is done in bicornuate or didelphic uterus.", "cop": 2, "opa": "Septate uterus", "opb": "Bicornuate uterus", "opc": "Arcuate uterus", "opd": "Unicornuate uterus with rudimentary horn", "subject_name": "Gynaecology & Obstetrics", "topic_name": null, "id": "3ba19e15-5a18-4fee-ae38-9133efca32cb", "choice_type": "single"} {"question": "Singer's alkali denaturation test is used to differentiate", "exp": "Singer's alkali denaturation test : It is a diagnostic test for vasa pre In this nucleated red blood cells are detected Ref: Dutta Obs 9e pg 243.", "cop": 4, "opa": "Aboion from early pregnancy", "opb": "Perimenopausal from pregnancy", "opc": "Uterine malignancy from pegnancy", "opd": "Vasa pre from placenta pre", "subject_name": "Gynaecology & Obstetrics", "topic_name": "General obstetrics", "id": "241aa260-ed2f-4fcc-bebb-1f6711a0cb4e", "choice_type": "single"} {"question": "Time required for ovulation from the onset of LH surge is", "exp": "From the onset of the LH surge, it takes 34-36 hours and from the peak it takes 10-12 hours to ovulate. The midcycle LH surge is responsible for a dramatic increase in local concentrations of prostaglandins and proteolytic enzymes in the follicle. These substances progressively weaken the follicular wall and ultimately allow a perforation to form.", "cop": 3, "opa": "12 hours", "opb": "24 hours", "opc": "36 hours", "opd": "48 hours", "subject_name": "Gynaecology & Obstetrics", "topic_name": "Normal Menstruation, Abnormal Menstruation, Menopausal Physiology and forsight of conception", "id": "db9182e7-a3e2-458b-9194-71b0883f749f", "choice_type": "single"} {"question": "Earliest detectable congenital malformation by USG", "exp": null, "cop": 1, "opa": "Anencephaly", "opb": "Spina bifida", "opc": "Meningocoele", "opd": "cystic hygroma", "subject_name": "Gynaecology & Obstetrics", "topic_name": null, "id": "500574a4-2576-4c59-b98e-bfbf89222101", "choice_type": "single"} {"question": "The recommended non surgical treatment of stress incontinence", "exp": "Conservative treatment is the first line of treatment especially in younger women Indications Milder forms of incontinence Patient medically unfit for the surgery Patient doesn't wish to undergo surgery Who have yet to complete their families Methods Pelvic floor muscle training : First line Periniometry Vaginal cones Maximum electrical stimulation Ref: SHAW&;S TEXTBOOK OF GYNAECOLOGY; 15th edition; Pg no:192", "cop": 1, "opa": "Pelvic Floor Muscle Exercises", "opb": "Bladder Training", "opc": "Electrical stimulation", "opd": "Vaginal cone\\/weights", "subject_name": "Gynaecology & Obstetrics", "topic_name": "Urogynecology", "id": "8f6fdbe8-6bf0-4fe3-8ec2-77b359f58d50", "choice_type": "single"} {"question": "Engaging diameter in brow presentation is", "exp": "Brow presentation : Engaging diameter of head is Mentoveical which is 14 cm The position is commonly unstable and conves to either veex or face presentation Position can be confirmed by Palpating supraorbital ridges and anterior fontanel. Ref: Dutta Obs 9e pg 367.", "cop": 1, "opa": "Mento-veical", "opb": "Mento-bregmatic", "opc": "Suboccipito-bregmatic", "opd": "Occipito-frontal", "subject_name": "Gynaecology & Obstetrics", "topic_name": "Abnormal labor", "id": "8c53d075-4a14-4364-9c9f-ddeb2e158c7f", "choice_type": "single"} {"question": "Commonest site of Tubal pregnancy is", "exp": "(Ampulla) (240- Shaw's 14th) (181-Dutta 6th)* Ampullary portion is the site of ectopic pregnancy in 80%, fimbrial end is the site in 6% isthmus in 12% and interstial in 2%* Commonest site - Ampulla. Least - Interstitial* In a tubal pregnancy, the most frequent inplantation site is the ampulla, because the plicae are most numerous in this situation, and previous salpingitis is more likely to produce crypts here than else where along the fallopian tube* Tubal abortion is the common mode of termination if implantation occurs in the ampulla or infundibulum* Tubal rupture is predominantly common in isthmic and interstitial implantation* Isthmic rupture usually occurs at 6 - 8 weeks the ampullary one at 8-12 weeks and the interstitial one at about 4 months", "cop": 1, "opa": "Ampulla", "opb": "Isthmus", "opc": "Fimbria", "opd": "Interstitial", "subject_name": "Gynaecology & Obstetrics", "topic_name": "Miscellaneous (Gynae)", "id": "4daa9866-599a-4d28-8fa2-20df6a8d8161", "choice_type": "single"} {"question": "A patient treated for infeility with clomiphene citrate presents with sudden onset of abdominal pain and distension with ascites the probable cause is", "exp": "Ovarian Hyperstimulation syndrome It is an iatrogenic complication of ovulation induction with exogenous gonadotropins and clomiphene-induced cycles Charecteristic feature of OHSS is an increase in capillary permeability resulting in fluid shift from intravascular to extravascular spaces Mechanism of Action Probably mediated by increased ovarian secretion of vasoactive substances including Vascular endothelial growth factor , elements of Renin-Angiotensin system and other cytokines Risk factors Young age, low body weight, PCOS, higher doses of gonadotropins and previous episodes of hyperstimulation Mild illness Charecterised by Ovarian enlargement, lower abdominal discomfo, mild nausea and vomiting, diarrhea and abdominal discomfo. Managed by outpatient basis with analgesics and bedrest; monitored by daily weight checkup, urinary frequency, clinical examination to detect ascites, lab tests of hematocrit, electrolytes and serum creatinine Serious illness Characterised by severe pain, rapid weight gain, tense ascites, hemodynamic instability, respiratory diffiulty, progressive oliguria leadng to renal failure, ovarian rupture and thromboembolic phenomenon Hospitalization and treated according to symptoms Ref: Clinical Gynecologic Endocrinology and Infeility; Eigth Edition; Chapter 31", "cop": 4, "opa": "Uterine repture", "opb": "Ectopic pregnancy rupture", "opc": "Multifetal pregnancy", "opd": "Hyperstimulation syndrome", "subject_name": "Gynaecology & Obstetrics", "topic_name": "Reproductive physiology and hormones in females", "id": "2ccede39-a077-45a2-b0ca-79e637bdaa59", "choice_type": "single"} {"question": "Cystic degeneration is seen commonly in", "exp": "Cystic degeneration is seen commonly seen in interstitial fibroid.", "cop": 3, "opa": "Submucous fibroid", "opb": "Subserosal fibroid", "opc": "Interstitial fibroid", "opd": "Cervical fibroid", "subject_name": "Gynaecology & Obstetrics", "topic_name": null, "id": "1a126118-5c22-4e95-b38c-623adbe8fa00", "choice_type": "single"} {"question": "Regarding IUGR", "exp": "‘AC is the single most sensitive parameter to detect IUGR’\nDutta Obs 7/e, p 462\n‘Serial measurements of AC (not BPD) and estimation of fetal weight are more diagnostic to fetal growth restriction’\nDutta Obs 7/e, p 462\nHead circumference (HC) and abdominal circumference (AC) ratios: In a normally growing fetus the HC/AC ratio exceeds 1.0 before 32 weeks. It is approximately 1.0 at 32 to 34 weeks. After 34 weeks, it falls below 1.0. If the fetus is affected by asymmetric IUGR, the HC remains larger. The HC/AC is then elevated. In symmetric IUGR, both the HC and AC are reduced. The HC/AC ratio remains normal. Using HC/AC ratio, 85% of IUGR fetuses are detected\nPathophysiology: Basic pathology in small for gestational age is due to reduced availability of nutrients in the mother or its reduced transfer by the placenta to the fetus. It may also be due to reduced ultilization by the fetus. Brain cell size (asymmetric–SGA) as well as cell numbers (symmetric-SGA) are reduced. Liver glycogen content is reduced. AC indirectly reflects the decreased fetal liver size and glycogen content.", "cop": 4, "opa": "Abdominal circumference (AC) is the least sensitive parameter for detection of IUGR", "opb": "In asymmetric IUGR head circumference/abdominal circumference (HC/AC) is reduced", "opc": "Serial biparietal diameter (BPD) is the only important measurement in IUGR", "opd": "AC indirectly reflects fetal liver size and glycogen storage", "subject_name": "Gynaecology & Obstetrics", "topic_name": null, "id": "f7834c25-ec73-4da0-ba42-b3df9f290db8", "choice_type": "single"} {"question": "Subnuclear vacuolation is a characteristic sign of", "exp": "The most characteristic signs of secretory phase are found in the glands. Their epithelial cells develop spherical translucent areas between the nuclei and the basement membrane which contain the precursors of the glandular secretion and which persist until about the 21st day of the cycle. This characteristic appearance is called subnuclear vacuolation and is presumptive evidence of progesterone activity and therefore of ovulation.Reference: Shaw's Textbook of Gynecology; 16th edition; Chapter 2; Normal Histology", "cop": 2, "opa": "Proliferative phase of endometrium", "opb": "Secretory phase of endometrium", "opc": "Menstruation", "opd": "Regeneration of endometrium", "subject_name": "Gynaecology & Obstetrics", "topic_name": "Reproductive physiology and hormones in females", "id": "22d6a4ef-8a73-4f31-9222-5f46ba79a20c", "choice_type": "single"} {"question": "M/c cause of APH is", "exp": "Abruptio placenta is defined as the haemorrhage occurring in pregnancy due to separation of a normally situated placenta.It is also called accidental haemorrhage or premature separation of placenta (refer pgno:133 sheila textbook of obstetrics 2 nd edition)", "cop": 2, "opa": "Placenta pre", "opb": "Abruptio placenta", "opc": "Vasa pre", "opd": "Placenta accreta", "subject_name": "Gynaecology & Obstetrics", "topic_name": "General obstetrics", "id": "8861f03f-4055-4d8a-911f-8e90b9c19d3f", "choice_type": "single"} {"question": "Temporary method of contraception among the following is", "exp": "Temporary contraceptive methods are\n\nHormonal contraception\nIntrauterine devices\nBarrier contraception\nNatural family planning methods", "cop": 2, "opa": "Vasectomy", "opb": "OCP", "opc": "Tubectomy", "opd": "Postpartum sterilisation", "subject_name": "Gynaecology & Obstetrics", "topic_name": null, "id": "610a3ff8-bcae-4356-8d3a-6ea6cf029972", "choice_type": "single"} {"question": "The nerve root blocked in pudendal block is", "exp": "Pudendal nerve block : Pudendal nerve arises from S2,3,4. Doesn't relieve the pain of labor Cause perineal analgesia and relaxation Ref: Dutta Obs 9e pg 480.", "cop": 3, "opa": "L 1,2,3", "opb": "L 2,3", "opc": "S 2,3,4", "opd": "S 4", "subject_name": "Gynaecology & Obstetrics", "topic_name": "General obstetrics", "id": "00b3f0c1-994f-4eb2-9caa-e37dd57c19c9", "choice_type": "single"} {"question": "Risk of ureteric injury is more with", "exp": "Maximum chances of injuring ureter is seen in case of wertheim's (modified radical) hysterectomy, as medial half of cardinal ligament is removed.", "cop": 2, "opa": "Vaginal hysterectomy", "opb": "Wertheims hysterectomy", "opc": "Radical hysterectomy", "opd": "Total abdominal hysterectomy", "subject_name": "Gynaecology & Obstetrics", "topic_name": null, "id": "b47f9317-6b57-446f-bf40-1bee042546eb", "choice_type": "single"} {"question": "Main hormone acting upon uterus to initiate labour", "exp": "Oxytocin is a nonapeptide,\n It is secreted by posterior pituitary along with vasopressin (ADH).\nBoth oxytocin and ADH are synthesized within the nerve cell bodies in supraoptic and paraventricular nuclei of the hypothalamus; are transported down the axon and stored in the nerve endings within the neurohypophysis.\nThey are stored in separate neurones as complexes with their specific binding proteins (Neurophysin).\n\n Actions \n\nUterus It increases the force and frequency of uterine contraction. Estrogens sensitize the uterus to oxytocin, while progestins decrease the sensitivity. The increased contractility is restricted to the fundus and body„ lower segment is not contracted, may even be relaxed at term.\nBreast Oxytocin contracts myoepithelium of mammary alveoli and forces milk into bigger sinusoids - Milk ejection or milk letdown reflex.\nCVS Higher doses cause vasodilation → Brief fall in BP, reflex tachycardia and flushing. The umbilical vessels are constricted oxytocin may help their clossure at birth.\nKidney Oxytocin in high doses exerts ADH like action —» water retention can occur.", "cop": 1, "opa": "Oxytocin", "opb": "Estrogen", "opc": "Progesterone", "opd": "Cortisole", "subject_name": "Gynaecology & Obstetrics", "topic_name": null, "id": "7b780a26-4ff0-441a-90ea-3803177517d0", "choice_type": "single"} {"question": "Multiple bih are commonest among .", "exp": "Negroes", "cop": 4, "opa": "Indians", "opb": "Mongol", "opc": "Caucasians", "opd": "Negroes", "subject_name": "Gynaecology & Obstetrics", "topic_name": null, "id": "e502d008-2b2d-4d51-bf8d-16e223930803", "choice_type": "single"} {"question": "Internal podalic version is done in", "exp": "Internal podalic version is done for only delivery of second twin (Transverse lie).", "cop": 3, "opa": "Complete breech delivery", "opb": "Frank breech delivery", "opc": "Delivery of aftercoming twin", "opd": "Preterm breech delivery", "subject_name": "Gynaecology & Obstetrics", "topic_name": null, "id": "4c9e60dd-28ea-48e8-a40c-4311354e7213", "choice_type": "single"} {"question": "Menopause is diagnosed by", "exp": "Menopause - Permanent cessation of mensturation. Clinical diagnosis- Stoppage of menses for 1 year without any other pathology Cessation of menses before the age of 40 is known as premature ovarian failure Investigation- Increase FSH> 40mIU/ml ( 3 values at weekly interval required) LH>20 IU Estradiol <20 pg Vaginal cytology: maturation index : 10/85/5", "cop": 3, "opa": "Estradiol >20 pg/ml", "opb": "Progesterone <40 ng/dl", "opc": "FSH>40 IU/L", "opd": "LH<20 IU/L", "subject_name": "Gynaecology & Obstetrics", "topic_name": "Normal Menstruation, Abnormal Menstruation, Menopausal Physiology and forsight of conception", "id": "2e79d3db-034e-4ecd-a1d4-99df69a60430", "choice_type": "single"} {"question": "Thickness of placental barrier in early pregnancy", "exp": "Placental barrier in early pregnancy consists of Synctiotrophoblast Cytotrophoblast Basement membrane Stromal tissue Endothelium of fetal capillary wall It is about 0.025 mm thick Ref: Dutta Obs 9e pg 30.", "cop": 3, "opa": "0.5 mm", "opb": "0.05 mm", "opc": "0.025 mm", "opd": "0.25 mm", "subject_name": "Gynaecology & Obstetrics", "topic_name": "General obstetrics", "id": "db84953b-65fc-4b47-8030-2c6ff8502cda", "choice_type": "single"} {"question": "Progesterone of choice in emergency contraception is", "exp": "Levonorgestrel Levonorgestrel alone, 0.75 mg stat F/B another 0.75 mg 12 hrs later, taken within 72 hrs of unprotected intercourse is the method of choice .for emergency contraception Emergency contraception is also k/a postcoital morning after contraception. Basically the methods used interfere with the physiological events before implantation, for e.g. inhibition or delaying of ovulation or interference with postovulatory events necessary for implantation & longevity of the blastocyst. Agents used for emergency contraception are known as 'interceptives'. Methods of emergency contraception Drugs Dosage Time frame Preg. rate * High dose estrogens (not used now) - Ethinyl estradiol (EE) 2.5 mg BD x 5 days Within 72 hrs of coitus L-0.15% - Conjugated equine estrogen (CEE) 15 mg BD x 5 days 0-0.6% * Estrogen & progestin combination pill ,, - Yuzpe method EE 50 rg + d, l-NG 0.5 mg (Ovral) 2 tabs stat & 2 tabs after 12 hrs ,, ,, 0-2% (Total dose EE 0.2 mg & LNG 2 mg) - Low dose pills ,, II 0-2% EE 30 lig + LNG 0.15mg (Ovral L, Mala-N etc.) 4 tabs stat & 4 tabs after 12 hors * Levonorgestrel (LNG) alone (Ecee 2, i- 0.75 mg stat F/B Preferably 0-1% pill, E-pill, unwanted-72, Plan-B) 0.75 mg after 12 hrs or 1.5 mg stat within 72 hrs but can be given upto 5 days * Centchroman 30mg 2 tab BD x 1 day Within 5 days not known (Saheli) * Danazol 400/800 mg BD x Within 72 hrs 0.8 - 1.7% 3 days or 1200 mg BD x 2 days * Mifepristone (RU 486) 600 mg/200 mg / Within 5 days to - 1.3% 10 mg single dose 27th day of cycle * Copper IUD (more effective than steroids) To be inseed within 7 days < 1%", "cop": 4, "opa": "Norethisterone", "opb": "Medroxy progesterone", "opc": "Desogestrel", "opd": "Levonorgestrel", "subject_name": "Gynaecology & Obstetrics", "topic_name": null, "id": "5b2ffe64-e23a-4ce7-bb9a-baea5de8cc0d", "choice_type": "single"} {"question": "The dose of folic acid per day for treating megaloblastic anaemia in pregnancy", "exp": "Folic acid deficiency in Pregnancy\n\nIn nonpregnant women, the folic acid requirement is 50 to 100 μg/day\nDuring pregnancy, requirements are increased and 400 μg/day is recommended \n\nThe earliest biochemical evidence is low plasma folic acid concentrations \nNormal serum folate levels\n\nNon Pregnant women: 5.4-18ng/ml\n1st trimester: 2.6-15ng/ml\n2nd trimester: 0.8-24ng/ml\n3rd trimester: 1.4-20.7ng/ml\n\n\nEarly morphological changes usually include neutrophils that are hypersegmented and newly formed erythrocytes that are macrocytic\n As the anaemia becomes more intense, peripheral nucleated erythrocytes appear, and bone marrow examination discloses megaloblastic erythropoiesis\nThe fetus and placenta extract folate from maternal circulation so effectively that the fetus is not anaemic despite severe maternal anaemia\n\nTreatment\n\n As little as 1mg of folic acid administered orally once daily produces a striking haematological response\nBy 4 to 7 days after beginning folic acid treatment, the reticulocyte count is increased, and leukopenia and thrombocytopenia are corrected\nAmerican College of Obstetricians and Gynecologists (2013c) have recommended that all women of childbearing age consume at least 400 μg of folic acid daily\n\nIncreased need of Folic acid\n\nMultifetal pregnancy\nHemolytic anaemia\nCrohn disease\nAlcoholism\nInflammatory skin disorders", "cop": 2, "opa": "400 μg", "opb": "1 mg", "opc": "5mg", "opd": "2 mg", "subject_name": "Gynaecology & Obstetrics", "topic_name": null, "id": "3a521f10-8fa0-432a-86cc-289132744ecc", "choice_type": "single"} {"question": "Post delivery uterus becomes pelvic organ by", "exp": "Following delivery, the fundus lies about 13.5 cm (5 1/2\") above the symphysis pubis. During the first 24 hours, the level remains constant; thereafter, there is a steady decrease in height by 1.25 cm (1/2\") in 24 hours, so that by the end of the second week the uterus becomes a pelvic organ. The rate of involution thereafter slows down until by 6 weeks, the uterus becomes almost normal in sizeReference: DC Dutta&;s Textbook of Obstetrics; Chapter 13; Normal Puerperium", "cop": 3, "opa": "48hours", "opb": "1week", "opc": "2weeks", "opd": "6weeks", "subject_name": "Gynaecology & Obstetrics", "topic_name": "General obstetrics", "id": "579524fb-ba05-45c6-b3e8-67faec1a6ef1", "choice_type": "single"} {"question": "Oxytocin will not do the following action", "exp": "Ans. (a) Milk productionRef : D.C. Dutta 8th ed. /136, 574Milk production is by Prolactin.", "cop": 1, "opa": "Milk production", "opb": "Milk let down", "opc": "Contraction of myo-epithelial cells", "opd": "Vascular contraction", "subject_name": "Gynaecology & Obstetrics", "topic_name": "Breasts and Lactation", "id": "0b931813-184b-449f-b3e7-b6e3759e78e4", "choice_type": "single"} {"question": "Bleeding associated with anovulation is due to", "exp": "As ovulation does not occur, proliferative endometrium persists causing bleeding.", "cop": 1, "opa": "Persistent proliferative endometrium.", "opb": "Alterations in endometrial vascular structure.", "opc": "Increased endometrial responsiveness to vasodilating prostaglandins.", "opd": "Impaired prostaglandins concentration.", "subject_name": "Gynaecology & Obstetrics", "topic_name": null, "id": "fbc8051e-4abb-42ec-a883-ef8d3e9c8958", "choice_type": "single"} {"question": "Gonadotropin levels decline to prepubertal levels by age", "exp": "Gonadotropin levels gradually decline to reach prepubertal levels by age 1 to 2 years.", "cop": 2, "opa": "6 month to 1 year of age", "opb": "1 year to 2 years of age", "opc": "2 years to 3 years of age", "opd": "3 years to 4 years of age.", "subject_name": "Gynaecology & Obstetrics", "topic_name": null, "id": "ac3e3574-d48c-4618-be7c-75ce9a16454b", "choice_type": "single"} {"question": "Augmented retraction ring in labour is known as", "exp": "Ans. b (Bandl's ring). (Ref. Williams, Obstetris 21st/pg.836, Dutta, Obstetrics 4th/pg.367)BANDL'S (PATHOLOGICAL RETRACTION) RING# A circular groove encircling the uterus, formed between active upper segment and distended lower segment.SCHROEDER'S (CONSTRICTION) RING# Situated at the junction of upper and lower segment.# Usually reversible and complete.# May appear in all stages of labour.# Ring is not felt per abdomen, round ligament also not felt.", "cop": 2, "opa": "Shroder's ring", "opb": "Bandl's ring", "opc": "Hour glass uterus", "opd": "Normal ring", "subject_name": "Gynaecology & Obstetrics", "topic_name": "Miscellaneous (Gynae)", "id": "b5794332-912c-474c-b2c0-95fc79c2ec77", "choice_type": "single"} {"question": "Minimum dose of estrogen in combined OCP&;s", "exp": ".", "cop": 1, "opa": "20mcg", "opb": "30cg", "opc": "35mcg", "opd": "50mcg", "subject_name": "Gynaecology & Obstetrics", "topic_name": "All India exam", "id": "94e76f87-afc5-4396-ad4f-7088ed21b2a7", "choice_type": "single"} {"question": "Most dangerous hepatitis in pregnancy", "exp": "Ans. is 'd' i.e., Hepatitis EHas been explained in previous sessions.", "cop": 4, "opa": "Hepatitis A", "opb": "Hepatitis B", "opc": "Hepatitis C", "opd": "Hepatitis E", "subject_name": "Gynaecology & Obstetrics", "topic_name": "Miscellaneous (Gynae)", "id": "cfbafda0-96b8-49aa-9c3b-b2fc608aad52", "choice_type": "single"} {"question": "Tocolytic of choice in pregnancy with hea disease (OBG)", "exp": "Calcium-channel blockers, especially nifedipine, are safer and more effective tocolytic agents than are B-agonists Nifedipine is the best first-line tocolytic agent available at this time. The initial dose should be 30 mg and subsequent doses 20 mg every 6 hours. MgSO4 which is an excellent drug for ecclampsia prevention and management is also used for tocollyis. This has a'Neuroprotective effect' on the preterm fetus and prevent s cerebral palsy instances main subjective maternal complaints to Mgs04 are related to the mental effects (\"out of it\" or \"felt like a zombie\") and the overall muscular weakness produced by the drug. The most frequent side effects of magnesium sulfate are pulmonary congestion, respiratory depression,hypothermia, and neuromuscular toxicity. Infants of mothers treated with IV magnesium sulfate sholy before delivery are frequently Hypotonic . neonatal/fetal complications are hypocalcemia & Intraventricular hage NOTE: FDA had denied approval of Atosiban (Oxytocin receptor antagonist) in view of concerns regarding efficacy and fetal-newborn safety.", "cop": 1, "opa": "Nifedipine", "opb": "MgSO4", "opc": "Atosiban", "opd": "Alcohol", "subject_name": "Gynaecology & Obstetrics", "topic_name": "Preterm Labour", "id": "05601938-e7e7-4d71-a3d2-85c4b66f5732", "choice_type": "single"} {"question": "Deep transverse arrest is seen in", "exp": "Ans. a (Occipitoposterior position). (Ref. Dutta, Obstetrics 4th/367, Ref. Williams Obstetrics, 21st/305)OCCIPITO-POSTERIOR POSITION# A vertex presentation with occiput placed posteriorly over SI joint or directly over the sacrum.# During delivery when occiput rotates through l/8th of a circle, anterior and sagital sutures come to lie in bispinous diameter, further anterior rotation becomes unlikely and arrest occurs in this position, and there occurs no progress of descent of head even after Vi hour following full dilatation of cervix, called as deep transverse arrest.# Etiology:- Lax pelvic floor- Android pelvis,- Weak uterine contraction,# Treatment:- Ventouse is ideal- Application of forceps can be useful- LSCS, if big baby, inadequate pelvis.", "cop": 1, "opa": "Occipito-posterior position", "opb": "Occipito-anterior position", "opc": "Breech delivery", "opd": "Face presentation", "subject_name": "Gynaecology & Obstetrics", "topic_name": "Miscellaneous (Gynae)", "id": "9c5c95b8-feb3-471e-8760-34fa92c53ee1", "choice_type": "single"} {"question": "The cause of &;postpaum blues&; is", "exp": "Also called maternity blues Affects women within 3-5 days of delivery Symptoms include insomnia, unexplained weepiness, depression, anxiety, headache, poor concentration, and fatigue.The condition is self-limiting and requires only reassurance and emotional suppo especially family suppo (refer to pgno:438 Sheila 2 nd edition)", "cop": 4, "opa": "Decreased estrogen", "opb": "Decreased progesterone", "opc": "Increased prolactin", "opd": "Decreased estrogen and progesterone", "subject_name": "Gynaecology & Obstetrics", "topic_name": "General obstetrics", "id": "6665e920-b16b-4994-8587-10622cf116b0", "choice_type": "single"} {"question": "70yrs old patient diagnosed with ovarian cancer. Surgical staging involving inguinal lymph nodes", "exp": "Stage IV: distant metastasis excluding peritoneal metastasis. IVA: pleural effusion with positive cytology. IVB: hepatic and/ or splenic parenchyma metastasis, metastasis to extra abdominal organs (including inguinal lymph nodes and lymph nodes outside of the abdominal cavity) Reference: textbook of gynaecology Sheila balakrishnan, 2nd edition, pg no:277 <\\p>", "cop": 4, "opa": "Stage IIIA", "opb": "Stage IIIB", "opc": "Stage IVA", "opd": "Stage IV B", "subject_name": "Gynaecology & Obstetrics", "topic_name": "Gynaecological oncology", "id": "91f385a2-8ba3-40ae-ad22-3e708c9d2f20", "choice_type": "single"} {"question": "Chassarmoir surgery is done in", "exp": "Chassarmoir technique Used for repair of vesicovaginal Fistula Technique: Flap splitting method Bladder and vaginal are widely separated and sutured in two layers Ref: SHAW&;S TEXTBOOK OF GYNECOLOGY; 15th edition; Pg no:187", "cop": 2, "opa": "Uterine inversion", "opb": "Vesicovaginal fistula repair", "opc": "Ureterovesical fistula repair", "opd": "Retroveed uterus", "subject_name": "Gynaecology & Obstetrics", "topic_name": "Gynaecological diagnosis and operative surgery", "id": "babe116f-4ed1-46c2-8530-de773d828d52", "choice_type": "single"} {"question": "Preferred procedure for obtaining endometrial sample for histopathology is", "exp": "Endometrial aspiration should be the preferred procedure for obtaining endometrial sample for histopathology. If endometrium is thick on imaging, but where HPE is inadequate or atrophic, hysteroscopy should be performed to rule out polyps. Dilatation and curettage should not be the procedure of choice for endometrial assessment. Reference: Novak's gynaecology; 14th edition; Chapter 14; Benign Diseases of the Female Reproductive Tract", "cop": 3, "opa": "Dilatation and curettage", "opb": "Dilatation and evacuation", "opc": "Endometrial aspiration", "opd": "Hysteroscopy", "subject_name": "Gynaecology & Obstetrics", "topic_name": "Disorders of menstruation", "id": "079c2309-c5aa-41bf-85ca-7b783a988728", "choice_type": "single"} {"question": "Incidence of preterm delivery in twin pregnancy is", "exp": "Incidence of preterm in twin pregnancy is 50% Mean gestational age for twins is 37weeks Causes of prematurity: Overdistention of uterus Hydromnios Premature rupture of membranes Ref: Dutta Obs 9e pg 193.", "cop": 2, "opa": "25%", "opb": "50%", "opc": "75%", "opd": "100%", "subject_name": "Gynaecology & Obstetrics", "topic_name": "General obstetrics", "id": "2cc70655-c45e-443f-8bd8-0b9b46c37bc4", "choice_type": "single"} {"question": "In a post paum patient the blood volumes becomes normal after non pregnant state in", "exp": "2 weeks", "cop": 1, "opa": "2 weeks", "opb": "4 weeks", "opc": "6 weeks", "opd": "10 weeks", "subject_name": "Gynaecology & Obstetrics", "topic_name": null, "id": "f5debd65-f0c0-4360-887d-4a87dd35e88e", "choice_type": "single"} {"question": "The recurrent risk for postpaum psychosis in next pregnancy is", "exp": "Postpaum psychosis has a 50% recurrence risk in the next pregnancy. Impoant risk for postpaum psychosis is a history of bipolar disease. (Ref: William's Obstetrics; 25th edition)", "cop": 4, "opa": "10%", "opb": "20%", "opc": "35%", "opd": "50%", "subject_name": "Gynaecology & Obstetrics", "topic_name": "All India exam", "id": "d18fd359-477d-4d24-8942-c256560ff631", "choice_type": "single"} {"question": "Pregnancy is contraindicated in", "exp": "WHO 4 Category: Very high risk of maternal moality or severe morbidity; Pregnancy contraindicated and termination discussedPulmonary aerial hypeensionSevere systemic ventricular dysfunction Previous peripaum cardiomyopathy with any residual impairment of left ventricular functionSevere left hea obstructionMarfan syndrome with aoa dilated > 40mm(Ref: William's Obstetrics; 25th edition)", "cop": 3, "opa": "Hea transplantation", "opb": "Mechanical valves", "opc": "Pulmonary aerial hypeension", "opd": "Marfan syndrome without aoic dilatation", "subject_name": "Gynaecology & Obstetrics", "topic_name": "All India exam", "id": "d9c3aa29-605b-44ee-98d0-8cabdac28f36", "choice_type": "single"} {"question": "Most common nerve injured during normal vaginal delivery is", "exp": "Most common nerve injured during vaginal delivery - Lateral femoral cutaneous nerve followed by femoral nerve Most common nerve injured during cesarean section - Iliohypogastric nerve and Ilioinguinal nerve Ref: Williams Obs 24e pg 677.", "cop": 2, "opa": "Femoral nerve", "opb": "Lateral femoral cutaneous nerve", "opc": "Iliohypogastric nerve", "opd": "Ilioinguinal nerve", "subject_name": "Gynaecology & Obstetrics", "topic_name": "General obstetrics", "id": "34369666-202e-4bdb-a432-215fa9dec085", "choice_type": "single"} {"question": "A diabetic female at 40 wks of gestation delivered a baby by elective cesarean section. Soon after bih the baby developed respiratory distress. The diagnosis is", "exp": "Ans. is a i.e. Transient tachypnea of the new born Friends don't get shocked by the answer, even I was perplexed when I went through the texts given in all the standard reference books. Let's see what they have said. Respiratory distress syndrome (RDS) or hyaline membrane disease (HMD) RDS almost always occurs in nreterm babies often less than 34 weeks of gestation. It is the commonest cause of respiratory distress in a preterm neonate. According to Ghai -- RDS is seen in preterm babies and not the term babies (as is given in our question). Now let's see what Ghai says about transient tachypnea of newborn : Transient tachypnea of newborn (TTN) Transient tachypnea of the newborn is a benign self-limiting disease occurring usually in term neonates and is due to delayed clearance of lung fluid. These babies have tachypnea with minimal or no respiratory distress. Chest X-ray may show prominent vascular marking and prominent interlobar fissure. Oxygen treatment is often adequate and ventilatory suppo is necessary and prognosis is good. But Ghai didn't mention any correlation between TTN and Diabetes. So, I had to search other books for more information. Neonatal complications : RDS and transient tachypnea are more common in infants of women with poorly controlled diabetics. Respiratory distress : \"Conventional obstetrical teaching through the late 1980s generally held that fetal lung maturation was delayed in diabetic pregnancies. Thus, these infants were at increased risk for respiratory distress (Gluck and Kulovich, 1973). Subsequent observations have challenged this concept, and gestational age rather than ove diabetes is likely the most significant factor associated with neonatal respiratory distress (Berkowitz and colleagues, 1996; Kjos and colleagues. 1990b)\". So, it is the gestational age and not diabetes which is the main factor causing neonatal respiratory distress. In our question the baby is delivered at 40 weeks gestation (Full term) so, the answer can not be Hyaline membrane disease rather it is transient tachypnea of newborn (i.e. option 'a is correct).", "cop": 1, "opa": "Transient tachypnea of the new born", "opb": "Congenital diaphragmatic hernia", "opc": "Tracheo oesophageal fistula", "opd": "Hyaline membrane disease", "subject_name": "Gynaecology & Obstetrics", "topic_name": null, "id": "1031c23d-b7a0-4a0a-ad8f-aa0caad3d162", "choice_type": "single"} {"question": "Oligohydramnios is associated with", "exp": "Etiology of oligohydramnios: \n\nIntrauterine growth restriction\nPost-term pregnancy \nPremature rupture of membrane \nDrugs like ACE inhibitors and prostaglandin inhibitors \nRenal anomalies \nRenal Agenesis \nInfantile polycystic disease \nUpper urinary tract obstructions \nLower urinary tract obstructions", "cop": 1, "opa": "Renal agenesis", "opb": "Neural tube defects", "opc": "Postmature birth", "opd": "Premature birth", "subject_name": "Gynaecology & Obstetrics", "topic_name": null, "id": "fd043291-4a52-4b8a-ada9-1f98bbcf394d", "choice_type": "single"} {"question": "Lactional amenorrhea is due to", "exp": "High levels of prolactin inhibits GnRH release from Hypothalamus and thus indirectly FSH and LH secretion from anterior pituitary Reference: Textbook of Gynaecology; Sheila balakrishnan; 2nd edition; Page no 27", "cop": 1, "opa": "Prolactin induced inhibition of GnRH", "opb": "Prolactin induced inhibition of FSH", "opc": "Oxytocin induced inhibition of GnRH", "opd": "Oxytocin induced inhibition of FSH", "subject_name": "Gynaecology & Obstetrics", "topic_name": "Disorders of menstruation", "id": "f20a1442-016f-4877-bb11-7b62737689b0", "choice_type": "single"} {"question": "\"Tule neck sign\" is associated with", "exp": "Inability to feel the neck of the baby, when fetal head retracts/recoils against the maternal perineum and external rotation is not accomplished as such there is inability to deliver the shoulders after the head has been delivered. This is an impoant sign to diagnose &;shoulder dystocia&; after full dilatation of cervix. And this sign is known as tule neck sign.", "cop": 3, "opa": "Anencephaly", "opb": "Extended breech", "opc": "Shoulder dystocia", "opd": "Congenital goiter", "subject_name": "Gynaecology & Obstetrics", "topic_name": "All India exam", "id": "7fd38747-0fdc-4333-8a72-8033a3a97f64", "choice_type": "single"} {"question": "Impoance of the plane of midpelvis is", "exp": "The pelvis is described as having four imaginary planes:The plane of the pelvic inlet--the superior strait. 2. The plane of the pelvic outlet--the inferior strait. 3. The plane of the midpelvis--the least pelvic dimensions. 4. The plane of greatest pelvic dimension--of no obstetrical significance.(Ref: William's Obstetrics; 25th edition)", "cop": 3, "opa": "Superior strait", "opb": "Inferior strait", "opc": "Plane of least pelvic dimensions", "opd": "Plane of greatest pelvic dimensions", "subject_name": "Gynaecology & Obstetrics", "topic_name": "All India exam", "id": "f8bf2897-6230-4d1a-8cff-0277bab6655b", "choice_type": "single"} {"question": "The first trimester of pregnancy completes by", "exp": "TrimestersIt has become customary to divide pregnancy into three equal epochs of approximately 3 calendar months. Historically, the first trimester extends through completion of 14 weeks, the second through 28 weeks, and the third includes the 29th through 42nd weeks of pregnancy. Thus, there are three periods of 14 weeks each.Reference: William's Obstetrics; 24th edition; Chapter 9; Prenatal care", "cop": 4, "opa": "10weeks", "opb": "12weeks", "opc": "13weeks", "opd": "14weeks", "subject_name": "Gynaecology & Obstetrics", "topic_name": "General obstetrics", "id": "ebd5529f-b971-4bdb-85dc-3c7bf29db084", "choice_type": "single"} {"question": "M/c cause of vaginal discharge among reproductive age females", "exp": "Ans. is c, i.e. Bacterial vaginosisRef: William Gynae 3rd/ed, p51\"Bacterial vaginosis is the most common cause of vaginal discharge among reproductive age women.\" - Ref: Williams Gynae 3rd/ed, p51.", "cop": 3, "opa": "Trichomonas", "opb": "Candida", "opc": "Bacterial vaginosis", "opd": "Gonorrhea", "subject_name": "Gynaecology & Obstetrics", "topic_name": "Sexually Transmitted Disease in the Female", "id": "0dbdefd8-9721-4bf8-a536-7857bd518031", "choice_type": "single"} {"question": "The contraceptive method with a failure rate of <5 is", "exp": "The oral contraceptives are highly effective contraceptive method\nThe failure rate is very low ( 0.3℅), as ovulation is inhibited", "cop": 1, "opa": "Oral contraceptive pills", "opb": "Copper-T", "opc": "Vaginal sponge", "opd": "Condom", "subject_name": "Gynaecology & Obstetrics", "topic_name": null, "id": "35e917ef-c442-49c4-ae42-1ef33b823133", "choice_type": "single"} {"question": "Instrumental delivery system is", "exp": "Ventouse - is an instrumental device designed to assist delivery by creating a vaccum between it and the fetal scalp. In case of failed ventouse, forceps may be used if 1. Head is almost completely rotated 2. Cervix is fully dilated 3. There is some degree of descent", "cop": 2, "opa": "Forceps only used in full cervical dilation", "opb": "Forceps used when ventouse fails", "opc": "Ventouse cannot be used in rotational deliveries of occipito-posterior and transverse presentations", "opd": "Forceps can be used in breech deliveries", "subject_name": "Gynaecology & Obstetrics", "topic_name": "All India exam", "id": "14242458-0cee-4e05-b169-bc865d9e600f", "choice_type": "single"} {"question": "Potential candidate for high risk of GDM is. .", "exp": "High-risk of gestational diabetes History of stillbih History of neonatal death History of fetal macrosomia Concomitant obesity and/or hypeension Diabetes in a first-degree relative Overweight before pregnancy Strong family history of type II DM Previous history of GDM", "cop": 2, "opa": "Young Primigravida", "opb": "Unexplained perinatal loss", "opc": "Presence of oligohydramnios", "opd": "Previous low bih weight baby", "subject_name": "Gynaecology & Obstetrics", "topic_name": "Gestational Diabetes Mellitus", "id": "8820e047-d175-4fed-9248-d60814c9498c", "choice_type": "single"} {"question": "'Double set up examination' is concerned with", "exp": "- 'Double set up of examination' is done when there is suspicion of placenta pre and patient is having bleeding. - It is done under anesthesia, in operation theater, keeping everything ready for cesarean section. - A Digital examination through the cervix is done. Palpation of placenta on lower segment not only conclusively confirms the clinical diagnosis but also identifies its degree. *If the placenta is not felt within approximate 10 cms diameter of the internal os, then the patient can be taken up for a trial of normal vaginal delivery *If the placenta is felt within 10 cms of the cervical rim or if there is bleeding due to the examination, then an immediate cesarean section is performed", "cop": 1, "opa": "Placenta pre", "opb": "Manual removal of placenta", "opc": "Twin pregnancy", "opd": "Bicornuate uterus", "subject_name": "Gynaecology & Obstetrics", "topic_name": "Antepaum Haemorrhage", "id": "1d24340a-e8d7-4b22-b8c8-797bbcae8c88", "choice_type": "single"} {"question": "Shortest diameter of fetal skull is", "exp": "(B) (Submentobregmatic) (85-Dutta 7th)FETAL SKULL AND MATERNAL PELVISDimetersMeasurement in cm (inches)Attitude of the headPresentation1. Suboccipito- bregmatic- extends from the nape of the neck to the center of the bregma9.5 cm 3 3/4 \")Complete flexionVertex2. Suboccipito-frontal- extends from the nape of the neck to the anterior of the anterior fontancll or center of the sinciput10 cm (4\")Incomplete flexionVertex3. Occipito-frontal- extends from the occipital eminence to the root of the nose (Glabella).11.5 cm (4 1/2 \")Marked deflexionVertex4. Mento-vertical- extends from the mid point of the chin to the highest point on the sagittal suture.14 cm (5 1/2 \")Partial extensionBrow5. Submcnto-vcrtical-extends from junction of floor of the mouth and neck to the highest point on the sagittal suture11.5 cm 4 1/2 \")IncompleteextensionFace6. Submento- bregmatic- extends from junction of floor of the mouth and neck to the centre of the bregma9.5 cm (3 3/4 \")Complete extensionFace* Transverse diameters- Biparietal diameter - 9.5 cm (3 3/4 \")- Super- subparietal - 8.5 cm (3 1/2 \")- Bitemporal dimeter - 8.cm (3 11/4\")- Bimastoid diameter - 7.5 cm (3\")A. Diameters of pelvic inlet1. Antero-posterior (True conjugate/Anatomical conjugate 11cm2. Obstetric conjugate 10 cm3. Diagonal conjugate 12 cm4. Transverse diameter 13 cm5. Oblique diameter 12 cm6. Sacroclyloid (smallest) 9.5 cmB. Diameter of pelvic outlet1. Obstetrical* Anteroposterior; from inferior border of symphysis pubis to tip of sacram 11 cm* Transverse (bispinous) interspinous distance between two ischial spines 10.5 cm2. Anatomical outlet* From lower borders of symphysis pubic to tip of coccyx 13 cm* Transverse diameter (inter tuberous diamcter/distance between inner borders of ischial tuberosities 11cm* Diagnal conjugate measured clinically **", "cop": 2, "opa": "Mentovertical", "opb": "Submentobregmatic", "opc": "Mentobregmatic", "opd": "Occipitofrontal", "subject_name": "Gynaecology & Obstetrics", "topic_name": "Miscellaneous (Gynae)", "id": "1e3df6fb-6291-4a91-98be-d46c0455e2e9", "choice_type": "single"} {"question": "Presence of endometrial tissue within the myometrium is defined as", "exp": "Endometriosis is defined as the presence of endometrial tissue (glands and stroma) outside the uterus. The most frequent sites of implantation arethe pelvic viscera and the peritoneum.Adenomyosis is defined as the presence of endometrial stroma and glands within the myometrium, at least one low-power field from the basis of theendometrium, whereas endometriosis is characterized by ectopic endometrium appearing within the peritoneal cavityRef: Berek and Novak&;s Gynecology; 15th edition; Chapter 17", "cop": 1, "opa": "Adenomyosis", "opb": "Endometriosis", "opc": "Myoma", "opd": "PID", "subject_name": "Gynaecology & Obstetrics", "topic_name": "Disorders of menstruation", "id": "0b16f94f-4ea8-4129-8a12-6877e9bcdd67", "choice_type": "single"} {"question": "Reinke crystals are found in", "exp": "Hilus cell tumor: Rare virilizing tumor Arises from cells in the ovarian hilum Presence of Reinke crystals in the cells is the distinguishing feature of leydig or interstitial cells of testis Ref: Shaw Gynecology 17e pg 449.", "cop": 4, "opa": "Arrhenoblastoma", "opb": "Granulosa cell tumor", "opc": "Dysgerminoma", "opd": "Hilus cell tumor", "subject_name": "Gynaecology & Obstetrics", "topic_name": "Gynaecological oncology", "id": "6330085e-40a2-4fd8-8fc0-663e6ccb3ff1", "choice_type": "single"} {"question": "Not used as emergency contraception", "exp": "Emergency contraception(EC) is based on the fact that protection is provided within 72 hours of unprotected intercourse. Mifepristone and LNG are proven easily administered oral therapy for EC whereas Cu T can be used effectively and cost effectively even till the fifth day after unprotected intercourse. However, LNG intrauterine device is more useful for disorders like menorrhagia, irregular bleeding, hyperplasia of the endometrium, etc .", "cop": 1, "opa": "Intrauterine LNG system", "opb": "Mifepristone", "opc": "Oral LNG", "opd": "Copper intrauterine device", "subject_name": "Gynaecology & Obstetrics", "topic_name": "Aboions, Spontaneous & Induced Emergency Contraception (Hey,whats the hurry !)", "id": "3dd7883b-b180-41b4-8c16-5ac95aeb159e", "choice_type": "single"} {"question": "Increased acidosis and hypoxaemia is seen in", "exp": "(Increased fetal diastolic flow in the middle cerebral artery with absent diastolic flow in the aorta)(648- D)Doppler - direction as well as velocity of blood flow can be measured by doppler ultrasound.* Reduced diastolic flow indicates high resistance in the downstream vessel and low tissue perfusion. Presence of'notch' in the early diastolic waveform also indicates high resistance in the downstream vessel and low tissue perfusion.* *Presence of notch in uterine artery when confirmed bilaterally at 24 weeks indicates the possible development of preclampsia and fetal growth restriction* Absent end diastolic flow in the umbilical artery was associated with 16% fetal death rate and reversed diastolic flow with 50% fetal death rates.* * Increased fetal diastolic flow in the middle cerebral artery (centralization of flow) with absent diastolic flow in the aorta implies fetal acidemia** Abnormal venous waveforms (ductus venosus, IVC) indicates fetal cardiac dysfunction (failure)Single umbilical artery - It is more common in twins and in babies bom of diabetic mothers or in polyhydramnios.* It is frequently associated with congenital malformation of the fetus (10-20%). Renal and genital anomalies, Trisomy 18 are common, (228-D)* There is increased chances of abortion, prematurity, IUGR, and increased perinatal mortality.", "cop": 2, "opa": "Normal doppler wave form", "opb": "Increased fetal diastolic flow in the middle cerebral artery with absent diastolic flow in the aorta", "opc": "Presence of the 'notch' in the uterine artery", "opd": "Absent umbilical artery", "subject_name": "Gynaecology & Obstetrics", "topic_name": "Miscellaneous (Gynae)", "id": "7e36f867-4915-4458-bf6b-c5c6a08c3cd7", "choice_type": "single"} {"question": "The weight of placenta and fetus are equal at", "exp": "In the first trimester, placental growth is more rapid than that of the fetus. But by approximately 17 weeks, placental and fetal weights are approximately equal. Ref:Williams Obs 24e pg 95.", "cop": 3, "opa": "14 weeks", "opb": "16 weeks", "opc": "17 weeks", "opd": "21 weeks", "subject_name": "Gynaecology & Obstetrics", "topic_name": "General obstetrics", "id": "efbc9b70-0d24-4574-bc20-1fcc4ecfac5e", "choice_type": "single"} {"question": "Maximum oogonia can be seen In ovaries at", "exp": "Oogonia multiply in large numbers by mitosis averaging about 6-7 million around 18-20 weeks of intrauterine life", "cop": 2, "opa": "7th month of intrauterine life", "opb": "5th month of intrauterine life", "opc": "At birth", "opd": "At puberty", "subject_name": "Gynaecology & Obstetrics", "topic_name": null, "id": "943bf5d8-e7da-4ad8-a3ef-015d1ee6fd20", "choice_type": "single"} {"question": "Not a feature of diabetes mellitus in pregnancy", "exp": "Neonatal complications Respiratory distress syndrome Hypoglycemia Polycythemia Hyperbilirubinemia Hyperviscosity Cardiomyopathy Bih trauma Reference: Textbook of Obstetrics; Sheila Balakrishnan, 2nd edition; Page no: 241", "cop": 3, "opa": "Postdatism", "opb": "Hydramnios", "opc": "Neonatal hyperglycemia", "opd": "congenital defect", "subject_name": "Gynaecology & Obstetrics", "topic_name": "Medical, surgical and gynaecological illness complicating pregnancy", "id": "e3f23718-cc7c-4369-be21-2e85ecc4ea1c", "choice_type": "single"} {"question": "A primipara is in labor and an episiotomy is about to be cut. Compared with a midline episiotomy, an advantage of mediolateral episiotomy is", "exp": "Midline episiotomies are easier to fix and have a smaller incidence of surgical breakdown, less pain, and lower blood loss. The incidence of dyspareunia is somewhat less. However, the incidence of extensions of the incision to include the rectum is considerably higher than with mediolateral episiotomies. Regardless of technique, attention to hemostasis and anatomic restoration is the key element of a technically appropriate repair.", "cop": 4, "opa": "Ease of repair", "opb": "Fewer breakdowns", "opc": "Less blood loss", "opd": "Less extension of the incision.", "subject_name": "Gynaecology & Obstetrics", "topic_name": "Episiotomy", "id": "78503239-fe99-4f51-9ed3-7620b2e06737", "choice_type": "single"} {"question": "Most common type of pelvis in women is", "exp": "Caldwell (1939) repoed that the gynecoid pelvis was found in almost half of women.(Ref: William's Obstetrics; 25th edition)", "cop": 1, "opa": "Gynecoid", "opb": "Anthrapoid", "opc": "Android", "opd": "Platypelloid", "subject_name": "Gynaecology & Obstetrics", "topic_name": "All India exam", "id": "0ffc49d8-fdf0-4e5a-b836-14b853c091da", "choice_type": "single"} {"question": "Intrauterine seizure in foetus is seen due to deficiency of", "exp": "(D) Glutamic acid", "cop": 4, "opa": "Biotin", "opb": "Riboflavin", "opc": "Thiamine", "opd": "Glutamic acid", "subject_name": "Gynaecology & Obstetrics", "topic_name": "Miscellaneous (Obs)", "id": "66833a4b-5449-4e63-a225-149169a0c253", "choice_type": "single"} {"question": "One of the following is used for sex chromatin testing", "exp": "Chromosomal sex can be determined by the study of the leucocytes or by simply taking a smear from the buccal mucosa\nThe nuclei of the female chromosome contain a stainable body called the sex chromatin, hence female cells are termed as chromatin positive.\nIn epithelial cell nuclei, this small peripherally situated darkly staining nodule is called Barr body\n\nMale cell nuclei lack this body and termed chromatin negative.", "cop": 1, "opa": "Barr body", "opb": "Testosterone receptors", "opc": "Hormone levels", "opd": "Phenotypic features", "subject_name": "Gynaecology & Obstetrics", "topic_name": null, "id": "13b1f651-0238-43be-b722-2722d2d270f7", "choice_type": "single"} {"question": "HIV transmission to the newborn is most commonly and effectively by", "exp": "Ans. b (Vaginal delivery). (Ref. Harrison's Principles of Internal Medicine, 16th/pg.38, 1082)MATERN AL-FETAL/INFANT TRANSMISSION# The predominant cause of HIV infection in children is transmission of the virus from the mother to the newborn during the perinatal period.# The majority of cases of mother-to-child (vertical) transmission of HIV-1 occur during the intrapartum period.# Exposures, which increase the risk of mother-to-child transmission, include -- Vaginal delivery,- Preterm delivery,- Maternal bleeding, and# Trauma to the fetal skin.# Perinatal HIV transmission can be most accurately correlated with measurement of maternal plasma HIV RNA burden.# In the absence of prophylactic antiretroviral therapy to the mother during pregnancy, labor, and delivery, and to the fetus following birth (see below), the probability of transmission of HIV from mother to infant/fetus ranges from 15- 25% in industrialized countries and from 25-35% in developing countries.# In developed countries, current recommendations to reduce perinatal transmission of HIV include- universal voluntary HIV testing and counseling of pregnant women,- antiretroviral prophylaxis with one or more drugs in cases in which the mother does not require therapy for her HIV infection,- combination therapy for women who do require therapy,- obstetric management that attempts to minimize exposure of the infant to maternal blood and genital secretions, and- avoidance of breast-feeding.# Certain studies have demonstrated that truncated regimens of zidovudine alone or in combination with lamivudine given to the mother during the last few weeks of pregnancy or even only during labor and delivery, and to the infant for a week or less, significantly reduced transmission to the infant compared to placebo.# Short-course prophylactic antiretroviral (ARV) regimens, such as a single dose of nevirapine given to the mother at the onset of labor and a single dose to the infant within 72 h of birth, are of particular relevance to low- to mid-income nations because of the low cost and the fact that in these regions perinatal care is often not available and pregnant women are often seen by a health care provider for the first time at or near the time of delivery.# Indeed, short-course ARV regimens have now been used for several years in developing nations for the prevention of mother-to-child transmission.", "cop": 2, "opa": "LSCS", "opb": "Vaginal delivery", "opc": "Perinatal", "opd": "Breast feeding", "subject_name": "Gynaecology & Obstetrics", "topic_name": "Miscellaneous (Gynae)", "id": "f588a6e8-4f5b-4372-a417-d6dabcfacbcb", "choice_type": "single"} {"question": "Diethylstilbesterol causes the following defects to expect", "exp": "Diethylstilbesterol defects\n\nNeural tube defects\nVaginal and cervical adenosis \nClear cell adenocarcinoma\nUterine anomalies \nCryptorchidism\nTesticular hypoplasia", "cop": 2, "opa": "Incompetent cervix", "opb": "Renal anomalies", "opc": "T saped uterus", "opd": "Vaginal adenosis", "subject_name": "Gynaecology & Obstetrics", "topic_name": null, "id": "a49c0be0-b487-4e15-836e-2b6eba2ea388", "choice_type": "single"} {"question": "Pierre robin syndrome is a", "exp": "Sequence describes anomalies that all develop sequentially from one insult.\nPierre robin sequence. Micrognathia → glossoptosis → cleft palate.", "cop": 4, "opa": "Deformation", "opb": "Malformation", "opc": "Disruption", "opd": "Sequence", "subject_name": "Gynaecology & Obstetrics", "topic_name": null, "id": "77a93c61-a87f-4d2b-9c7b-77b5a793e383", "choice_type": "single"} {"question": "A case of Gestational trophoblastic neoplasia belongs to high risk group if disease develops after", "exp": "WHO SCORING FOR GESTATIONAL TROPHOBLASTIC DISEASE (Score > 6 is high risk) PROGNOSTIC SCORING 0 1 2 4 1. Age <=39 >39 2. Antecedent pregnancy mole aboion term pregnancy 3. Interval (pregnancy and sta of CT) <4 4-6 7-12 >12 4. Hcg <10,000 1000 - 10,000 10,000 - 1 lakh >1 lakh 5. Blood group O or A B or AB 6. Largest tumor size <3 3-5 >5 7. Site of metastasis spleen, kidney GIT Brain, liver 8. No. of metastasis 1-3 4-8 >8 9. Prior chemotherapy 1 drug >=2 drugs", "cop": 2, "opa": "Hydatidiform mole", "opb": "Full term pregnancy", "opc": "Spontaneous", "opd": "Ectopic pregnancy aboion", "subject_name": "Gynaecology & Obstetrics", "topic_name": "Twin Pregnancy, Molar Pregnancy, Gestational Trophoblastic disease and contraception in special situations (Sour Grapes!)", "id": "1cbfa7db-0414-412d-a5ee-b2cf11c2c8f2", "choice_type": "single"} {"question": "Of the Following, most orable breech presentation is", "exp": "Most orable breech presentation is Extended / Frank breech as it has least risk of cord prolapse among all the breech types. Risk of Cord prolapse in Breech - Footling breech > Complete breech > Frank breech", "cop": 1, "opa": "Extended breech", "opb": "Complete breech", "opc": "Footling", "opd": "Extended Head", "subject_name": "Gynaecology & Obstetrics", "topic_name": "Labour - II", "id": "4eb38033-dbc1-441f-89c5-e81237148714", "choice_type": "single"} {"question": "Oral contraceptive acts by", "exp": "Neither estrogen nor progestin hormone dose used in combination hormonal oral contraceptives alone would be able to suppress ovulation but together the estrogen and progestin hormones work synergistically to suppress ovulation \"successfully\". The synergistic antiovulatory effect from the combined use of estrogen and progestin directly decreases the secretion of the gonadotropin-releasing hormone from the hypothalamus and is considered the main action. This negative feedback mechanism disrupts ovulation by interfering with the hypothalamus-pituitary-ovary axis and gonadotropin secretion from the pituitary. Specifically, the progestin component suppresses luteinizing hormone release and the LH surge, which is necessary for ovulation, and the estrogen component suppresses the follicle stimulating hormone, which prevents the selection and maturation of the dominant follicle.", "cop": 4, "opa": "Aseptic inflammation", "opb": "Change in endometrial fluid", "opc": "Foreign body reaction", "opd": "Prevent ovulation", "subject_name": "Gynaecology & Obstetrics", "topic_name": "Contraception", "id": "d44b685f-a945-4547-842a-79b9934332af", "choice_type": "single"} {"question": "Drug contraindicated in patient with rheumatic hea disease in PPH is", "exp": "Methyl ergometrine Cardiac patients' should not receive ergotamine because these agents cause significant vasoconstriction and elevation of blood pressure which can be deleterious to the patient. Also know, In healthy women, an intravenous bolus of 10 units of oxytocin caused a transient but marked fall in aerial blood pressure that was followed by an abrupt increase in cardiac output. These hernodynamic changes could be dangerous to women already hypovolemic from hemorrhage or who had cardiac disease that limited cardiac output. The same danger is present for women with right to left cardiac shunts because the decrease in systemic resistance would fuher increase the shunt. So, - Oxytocin should not be given intravenously as a large bolus, but rather as a much more dilute solution by continuous intravenous infusion or as an intramuscular infection. Management of cardiac patient during pregnancy Antepaum Bed rest The most impoant measure for attenuating the impact of pregnancy on diseased hea is bed rest. Bed rest increases the venous return to the hea, improves renal perfusion, induces diuresis and promotes elimination of water. Dietory salt restriction It helps to prevent excess retention of sodium and water. Diuretics It should be given to cardiac pregnant patient if moderate restriction in sodium intake is insufficient to limit the normal intravascular volume expansion that occurs during gestation. Prophylactic digitalization It is commonly used in patients with severe hea disease who are not in ove congestive failure. During labour and delivery The pregnant cardiac patient should always labor and deliver in lateral supine position to avoid hemodynamic impairment caused by dorsal decubitus position. Effective pain relief during labour with morphine. Continuous monitoring with pulse oximetry. Restriction of i.v. fluids to 75 ml/hr (Almost all cardiac patients in labour should be kept on the dry side). Antibiotic prophylaxis (This is given in patients with congenital or acquired hea lesions). Thrombosis prophylaxis. Patient should be placed in sitting position after delivery. This is done to avoid pulmonary edema (sitting will increase venous pooling in the lower extremities and decrease the venous return to the hea).", "cop": 2, "opa": "Oxytocin infusion", "opb": "Methyl ergometrine", "opc": "Misoprostol", "opd": "Carboprost", "subject_name": "Gynaecology & Obstetrics", "topic_name": null, "id": "2ed11eaf-d214-4c7e-80ff-5bb358119801", "choice_type": "single"} {"question": "Factors that protect against endometrial cancer", "exp": "Factors that increase\n\t\t\tendometrial cancer\n\t\t\trisk\nFactors that protect\n\t\t\tagainst endometrial\n\t\t\tcancer\n\n\nObesity\nHysterectomy\n\n\nDiabetes\nCombined oral contraceptive pill\n\n\nNulliparity\nProgestin - based\n\t\t\tContraceptives, including injectables\n\n\nLate menopause\n\t\t\t> 52 years\nIntrauterine device, including Cu-IUD and LNG-IUS\n\n\nUnopposed oestrogen\n\t\t\ttherapy\nPregnancy\n\n\nTamoxifen therapy\nSmoking\n\n\n\nFamily history of\n\t\t\tcolorectal and\n\t\t\tendometrial cancer", "cop": 2, "opa": "Obesity", "opb": "Smoking", "opc": "Nulliparity", "opd": "Diabetes", "subject_name": "Gynaecology & Obstetrics", "topic_name": null, "id": "09cf1f83-8f19-41be-8559-5a5138ff3bb7", "choice_type": "single"} {"question": "Pseudo gestational sac is seen in the USG of", "exp": "Pseudo gestational sac : Irregular in outline Usually, uterus is centrally located No double decidual sign Sac remains empty Seen in ectopic pregnancy Ref: Dutta Obs 9e pg 601.", "cop": 2, "opa": "Missed aboion", "opb": "Ectopic gestation", "opc": "Complete aboion", "opd": "Hematometra", "subject_name": "Gynaecology & Obstetrics", "topic_name": "General obstetrics", "id": "910b1354-0a99-469f-a654-336d754ed8f9", "choice_type": "single"} {"question": "Puerperal sepsis is last for", "exp": "(2 weeks): Ref: 433-36-DPUERPERAL SEPSIS - An infection of the genital tract which occurs as a complication of delivery is termed puerperal sepsisPuerperal pyrexia: A rise of temperature reaching 100deg.4degF (38degC) or more (measured orally) on 2 separate occasions at 24 hours apart (excluding first 24 hours) within first 10 days following delivery is called puerperal pyrexia* Commonest responsible organism are anaerobic streptococcus* Uterus is the most common site of puerperal infection (Placental site are the common sites to be infected first, and uterus usually occurs between the 3rd and 6th day after delivery)", "cop": 1, "opa": "2 weeks", "opb": "4 weeks", "opc": "6 weeks", "opd": "8 weeks", "subject_name": "Gynaecology & Obstetrics", "topic_name": "Miscellaneous (Gynae)", "id": "b7fb7768-8ca3-496b-a8a8-622cf4996399", "choice_type": "single"} {"question": "Congenital adrenal hyperplasia is most likely a result of", "exp": ".", "cop": 1, "opa": "Defects in adrenal steroidogenic enzymes", "opb": "Addison's disease", "opc": "Defects in ACTH secretion", "opd": "Defects in coicosteroid-binding globulin", "subject_name": "Gynaecology & Obstetrics", "topic_name": "All India exam", "id": "f0b493c2-051e-42c3-83aa-fd86a6161e0c", "choice_type": "single"} {"question": "A female presents with 8 weeks amenorrhea with pain left lower abdomen. On USG, there was thick endometrium with mass in lateral adnexa. Most probable diagnosis", "exp": "Ans. a. Ectopic pregnancy A female with 8 weeks amenorrhea with pain left lower abdomen and on USG, thick endometrium with mass in lateral adnexa is suggestive of ectopic pregnancy", "cop": 1, "opa": "Ectopic pregnancy", "opb": "Torsion of dermoid cyst", "opc": "Tubo-ovarian mass", "opd": "Hydrosalpinx", "subject_name": "Gynaecology & Obstetrics", "topic_name": null, "id": "9899177f-0774-4fc6-ab1b-ce275dd5afff", "choice_type": "single"} {"question": "The lymphatics of the vulva", "exp": "Lymphatic Drainage of Reproductive Tract:Uterine Corpus - 1. Fundus-Para-aoic lymph nodes2. Cornu-Superficial inguinal3. Body-External Iliac L NCervix 1. Parametrial2. Internal iliac group3. Obturator group4. External iliac group5. SacralAll these drains into common iliac and superior lumbar group Fallopian tube and ovary-Superior lumbar groupVagina - Upper 2/3-Like cervix Lower 1/3-Inguinal and external iliac nodesVulva- Superficial inguinal Deep inguinal External iliac", "cop": 3, "opa": "Do not freely communicate with each other", "opb": "Do not cross the labiocrual fold", "opc": "Transverse the labia from medial to lateral margin", "opd": "Drain directly into the deep femoral glands", "subject_name": "Gynaecology & Obstetrics", "topic_name": "Anatomy of the female genital tract", "id": "b68810c3-e81c-48a1-bd8a-f143b418cdee", "choice_type": "single"} {"question": "First histological sign of ovulation is", "exp": "After ovulation, glycogen accumulates in basal portion of glandular epithelium creating subnuclear vacuoles and pseudostratification.", "cop": 4, "opa": "Vacuoles at apical portion of secretory nonciliated cells.", "opb": "Cessation of glandular cell mitosis.", "opc": "Absent glycoprotein in secretory non ciliated cells.", "opd": "Subnuclear vacuoles and pseudostratification in basal portion of glandular epithelium.", "subject_name": "Gynaecology & Obstetrics", "topic_name": null, "id": "d8a7a3fd-8e41-412b-b4a6-9b8d88fefbc2", "choice_type": "single"} {"question": "Primary amenorrhea is absence of menstruation by the age of", "exp": "Primary amenorrhea refers to the failure of the onset of menstruation beyond the age of 16 years regardless of the development of secondary sexual characters Ref: Shaw Gynecology 17 e pg 141.", "cop": 3, "opa": "13 years", "opb": "15 years", "opc": "16 years", "opd": "18 years", "subject_name": "Gynaecology & Obstetrics", "topic_name": "Disorders of menstruation", "id": "bd902238-a496-4596-95e3-60ac76787f0c", "choice_type": "single"} {"question": "The pH ot vagina in adults is 1", "exp": "4.5 - 5.5 The pH of vagina in an adult woman is 4 - 5.5 with an average of 4 5 The pH of vagina varies with age Age Vaglnal pH - In a newborn infantdeg Between 4.5 - 7deg - 6 weeks old child deg 7deg - Pubeydeg Changes from alkaline to acidicdeg - Reproductive age groupdeg 4S 5.5deg - Pregnancydeg 3.5 - 4.5deg -- LoLL'4 - Late post menopausaldeg 6 - 8deg pH of vagina also varies along its length, being highest in the upper pa because of admixture of alkaline cervical mucus. The acidity of vagina is because of the action of doderlein's bacilli (lactobacilli) a normal inhabitant of vagina which acts on the glycogen contained in the vaginal cells to produce lactic acid. Vagina is lined by squamous epithelium in adults and by transitional epithelium in newborn.deg No glands open in vagina.deg The squamous cells of vagina and cervix stain deep brown in colour after being painted with iodine solution, owing to the presence of glycogen in healthy cells (positive schiller test).deg", "cop": 2, "opa": "3.5 - 4.5", "opb": "4.5 - 5.5", "opc": "5.5 - 6.5", "opd": "6.5 5 7.5", "subject_name": "Gynaecology & Obstetrics", "topic_name": null, "id": "9f40be25-0897-4ca9-aa70-d287131f70d0", "choice_type": "single"} {"question": "Diagnosis of Gestational hypeension is made if blood pressure is", "exp": "Diagnostic Criteria for Pregnancy-Associated HypeensionConditionCriteriaGestational HypeensionBp > 140/90 mmHg after 20weeks in previously normotensive womenPreeclampsiaHypeension andProteinuria orThrombocytopenia Renal Insufficiency cerebral symptoms Pulmonary edemaReference: William&;s Obstetrics; 24th edition; Chapter 40; Hypeensive disorders", "cop": 2, "opa": "> 130/90 mmHg after 20weeks in previously normotensive patient", "opb": "> 140/90 mmHg after 20weeks in previously normotensive patient", "opc": "> 150/90 mmHg after 20weeks in previously normotensive patient", "opd": "> 160/90 mmHg after 20weeks in previously normotensive patient", "subject_name": "Gynaecology & Obstetrics", "topic_name": "Medical, surgical and gynaecological illness complicating pregnancy", "id": "6102ad48-d662-46ad-a77f-435c40726353", "choice_type": "single"} {"question": "After an initial pregnancy resulting in a spontaneous loss in the first trimester, your patient is concerned about the possibility of this recurring. An appropriate answer would be that the chance of recurrence", "exp": "An initial spontaneous aboion, irrespective of the karyotype of sex of the child, does not change the risk of recurrence in the next pregnancy. The rate of aboion is commonly quoted as 15% of all known pregnancies. Recurrent pregnancy loss is seen in around 1 in 300 pregnancies. For patients with a history of RPL, the risk of subsequent pregnancy loss is estimated to be: 24% after two clinically recognized losses, 30% after three losses 40% to 50% after four losses", "cop": 2, "opa": "Depends on the genetic makeup of the prior abous", "opb": "Is no different than it was prior to miscarriage", "opc": "Is increased to approximately 25%", "opd": "Is increased most likely to greater than 50%", "subject_name": "Gynaecology & Obstetrics", "topic_name": "Aboions, Spontaneous & Induced Emergency Contraception (Hey,whats the hurry !)", "id": "38213043-d1bf-418a-8c93-59ed47969bdd", "choice_type": "single"} {"question": "Likely size of uterus at 8 weeks post paum is", "exp": "Ans. is a i.e. 100 gm Weight of uterus : Immediately after delivery -- 1000 gm. At the end of 1 week -- 500 gm. At the end of 2 weeks -- 300 gm. At the end of 6 weeks it weighs -- 60 - 100 gm (Pre-pregnant state). Also Know : Immediately following delivery, the fundus is just below the umbilicus (13.5 cms above the symphysis pubis/ 20 weeks gestational age size.)deg Involution of the uterus After 24 hours of delivery, height of uterus decreases by 1.25 cm/day.deg Uterus is a pelvic organ.deg by the end of 2 weeks. Uterus returns almost to its normal size (pre pregnant size) by the end of 6 weeks. The process by which the post paum uterus returns to its pre pregnant state is called as Involution. Involution is achieved by decrease in the size of muscle fibres.deg (and not in the number)deg", "cop": 1, "opa": "100 gm", "opb": "500 gm", "opc": "700 gm", "opd": "900 gm", "subject_name": "Gynaecology & Obstetrics", "topic_name": null, "id": "62acc108-7fc4-42e3-a056-7afa8c838b95", "choice_type": "single"} {"question": "Duration of second stage of labour in a multipara is", "exp": "Ans. is 'b' i.e., 20 minutes o Second stage begins with full dilatation of the cervix and ends with expulsion of the fetus.o Mean duration of second stage is 50 minutes for nullipara and 20 minutes in multipara. Prolonged second stage is diagnosed if the duration exceeds 2 hours in nullipara and 1 hour in multipara when no regional anaesthesia is usedo One hour or more is permitted in both the groups when regional anaesthesia is used during labour.", "cop": 2, "opa": "10 minutes", "opb": "20 minutes", "opc": "40 minutes", "opd": "l hour", "subject_name": "Gynaecology & Obstetrics", "topic_name": "Management of the Second Stage of Labor", "id": "e9035531-7c64-4d95-bad3-72a9279e698c", "choice_type": "single"} {"question": "In a young female of reproductive age an absolute contraindication for prescribing OCP&;s is", "exp": "OCP act by - causing atropic changes of endometrium preventing nidation - Thickening cervical mucus - Increasing the tubal motility. The hormones of the OCP are metabolized in the liver. Hence, chronic liver diseases and liver failure are absolute CI of OCP.", "cop": 4, "opa": "Diabetes", "opb": "Hypeension", "opc": "Obesity", "opd": "Impaired liver function", "subject_name": "Gynaecology & Obstetrics", "topic_name": "Contraception", "id": "6485c86d-5b66-4c4b-9229-63e5cd6d0261", "choice_type": "single"} {"question": "Second stage of labor begins with", "exp": "Second Stage of LaborThis stage begins with complete cervical dilation and ends with fetal delivery. The median duration is approximately 50 minutes for nulliparas and about 20 minutes for multiparas, but it is highly variable Reference: William's Obstetrics; 25th edition; Chapter 22; Normal Labor", "cop": 2, "opa": "Fully effaced cervix", "opb": "Fully dilated cervix", "opc": "Fetal delivery", "opd": "Placental delivery", "subject_name": "Gynaecology & Obstetrics", "topic_name": "General obstetrics", "id": "5577d23e-09c3-4ef5-a480-3e7630b0eecb", "choice_type": "single"} {"question": "Clomiphene citrate used in initial treatment for most anovulatory infertile women is", "exp": "Clomiphene citrate is non-steroidal derivative given on 3rd to 5th day of cycle. It has both oestrogen agoinst and antagonist properties.", "cop": 3, "opa": "Steroidal triphenylethylene derivative", "opb": "Given on 7th day to 10th day after onset of spontaneous menses", "opc": "Can be administered if no follicle is >20mm and endometrium is <5mm", "opd": "Is oestrogen antagonist only", "subject_name": "Gynaecology & Obstetrics", "topic_name": null, "id": "fa0606f5-8a6e-424f-96ba-d142d67d8ec5", "choice_type": "single"} {"question": "Keyhole sign is seen in", "exp": "Distal obstruction of the urinary tract is more frequent in male fetuses, and the most common etiology is posterior urethral valves. Characteristically, there is dilatation of the bladder and proximal urethra, termed the\"keyhole\" sign, and the bladder wall is thick (Ref: William's Obstetrics; 25th edition)", "cop": 4, "opa": "Polycystic kidney disease", "opb": "Renal agenesis", "opc": "Ureteropelvic junction obstruction", "opd": "Bladder outlet obstruction", "subject_name": "Gynaecology & Obstetrics", "topic_name": "All India exam", "id": "92c50a45-ae75-4378-baa8-f6f3643b0186", "choice_type": "single"} {"question": "Paramesonephric duct develop into", "exp": ".", "cop": 4, "opa": "Vas deferens", "opb": "Seminal vesicle", "opc": "Ureter", "opd": "Uterus", "subject_name": "Gynaecology & Obstetrics", "topic_name": "All India exam", "id": "e0a19d7a-7075-4d8f-bdc0-94b471b62cfd", "choice_type": "single"} {"question": "Proteinuria in preeclampsia is due to", "exp": "Endothelial cell dysfunction and increased free radicals cause activation of microvascular coagulation that manifest by thrombocytopenia and increased capillary permebility manifest as edema and proteinuria.", "cop": 2, "opa": "Increased glomerular filtration rate", "opb": "Increased capillary permeability", "opc": "Increased renal artery resistance", "opd": "Increased systemic vascular resistance", "subject_name": "Gynaecology & Obstetrics", "topic_name": null, "id": "895144c1-5391-4f61-98e5-2f1de05e8fb9", "choice_type": "single"} {"question": "Menorrhagia is defined as", "exp": "Ans. c (Increased loss of blood than normal during menstrual cycle). (Ref. Shaw's textbook of Gynaecology 13th ed. 291)A normal menstrual blood loss is 50-80 ml, & does not exceed 100 ml. Menorrhagia is increase in duration & quantity of menstrual loss (> 100 ml).", "cop": 3, "opa": "Intermenstrual bleeding", "opb": "Menses in less than 28 days", "opc": "Increased loss of blood than normal during menses", "opd": "Menses every more than 45 days.", "subject_name": "Gynaecology & Obstetrics", "topic_name": "Heavy menstrual bleeding ", "id": "29416e9c-158a-4eaf-9b9b-c529c16cbea3", "choice_type": "single"} {"question": "Herpes gestatioinalis is", "exp": "Herpes gestationalis (pemphigoid gestationis ) is characterized by eosinophilic degranulation at dermal-epidermal junction. Becomes severe with subsequent pregnancies.", "cop": 2, "opa": "Caused by Herpes virus", "opb": "Autoimmune reaction to collagen XVII in basement membrane of skin", "opc": "Neutrophilic degranulation at dermal - epidermal junction", "opd": "Less severe in subsequent pregnancies.", "subject_name": "Gynaecology & Obstetrics", "topic_name": null, "id": "4ffc7805-8676-4f41-a359-c77cceb2ac4a", "choice_type": "single"} {"question": "Calories in human milk (gm per 100 mL) are", "exp": "Ans. d (67 Kilocalories). (Ref. Park PSM, 19th ed., 429; OP Ghai Paediatrics, 6th/pg 97)MILKNutrientContent/100 mL of the human/breast milkCalories67 kcal.... (Ref. OP Ghai Paediatrics); 70 kcal.... (Ref. Park PSM)Proteins1.1 gWhey/casein ratio80:20Fat3.5 gLactose7.0 gSodium0.9 mEqPotassium1.4 mEqCalcium35 mgPhosphorus15 mgIron30-50 mgZinc120 mgVitamin A60 mgVitamin C5.2 mg % of contents inProteinsFatCarbohydrateWaterColostrum8.62.33.286Breast milk1.23.27.587", "cop": 4, "opa": "37", "opb": "47", "opc": "57", "opd": "67", "subject_name": "Gynaecology & Obstetrics", "topic_name": "Miscellaneous (Gynae)", "id": "004f10e9-0519-4cdd-b2b0-a9fe5f42b77a", "choice_type": "single"} {"question": "A 55 year old lady presenting to OPD with postmenopausal bleeding for 3 months has a 1 x1cm nodule on the anterior lip of cervix, The most appropriate investigation to be done subsequently is", "exp": "Post menopausal bleeding with a visible cervical growth needs a tissue diagnosis, done by a punch biopsy of the lesion. Paps smear is a screening test, done in asymptomatic patients. Colposcopy + biopsy is a diagnostic test, indicated in symptomatic patients, like those with postmenopausal or post coital bleeding, or in asymptomatic patients where paps screening was done and turned out abnormal Endocervical curettage is indicated in patients with suspected endocervical lesion, or if entire transformation zone could not be evaluated at colposcopy", "cop": 2, "opa": "Pap smear", "opb": "Punch biopsy", "opc": "Endocervical currettage", "opd": "Colposcopy & biopsy", "subject_name": "Gynaecology & Obstetrics", "topic_name": "Cervical Carcinoma", "id": "c6bb5942-411e-4ae5-ac78-7e5f51280c62", "choice_type": "single"} {"question": "Surfactant appears in amniotic uid at the gestational age of", "exp": "Read below what Nelson has to say about the appearance of surfactant.\n\"Surfactant is present in high concentrations in fetal lung homogenates by 20 wk gestation* but does not reach the surface of the lung until later. It appears in the amniotic uid between 28-32 wks. Mature level of pulmonary surfactant is usually present after 35 wks\".\nNow I am leaving it to you to decide between 28 and 32 wks. I personally feel the answer should be 28 wks", "cop": 2, "opa": "20 wks", "opb": "28 wks", "opc": "36 wks", "opd": "40 wks", "subject_name": "Gynaecology & Obstetrics", "topic_name": null, "id": "61d33d24-2786-4276-bc85-2c7e5b2945b8", "choice_type": "single"} {"question": "In a case of Dysgerminoma of ovary one of the following tumor markers is likely to be raised.", "exp": "Placental alkaline phosphatase (PLAP) and lactate dehydrogenase (LDH) are produced by up to 95% of dysgerminomas, and serial measurements of LDH may be useful for monitoring the disease. A karyotype should be obtained preoperatively for all premenarcheal girls, paicularly those with dysgerminomas, because of the propensity of these tumors to arise in dysgenetic gonads", "cop": 3, "opa": "Serum HCG.", "opb": "Serum alphafetoprotein.", "opc": "Serum lactic dehydrogenase.", "opd": "Serum inhibin.", "subject_name": "Gynaecology & Obstetrics", "topic_name": "Ovarian Tumors", "id": "92a2d249-451f-4bfb-8977-04c5401e3e62", "choice_type": "single"} {"question": "The weight of term uterus is", "exp": "Non pregnant state of uterus Weight - 60g Length - 7.5 cm Capacity - 5-10 ml Pregnant uterus Weight - 900-1000g Length - 35 cm Capacity - increased to 500-1000 times Ref: Dutta Obs 9e pg 42.", "cop": 2, "opa": "500 grams", "opb": "1000 grams", "opc": "1500 grams", "opd": "2000 grams", "subject_name": "Gynaecology & Obstetrics", "topic_name": "General obstetrics", "id": "0f70c8c9-cfee-467e-9994-93f33e756f17", "choice_type": "single"} {"question": "Least pregnancy failure rate in", "exp": "(Oestrogen + Progesterone) (547-Dutta 6th)Pregnancy Failure Rate (H,W,Y) (PEARL - INDEX)**Number of accidental pregnancies x 1200Number of patients in 100 year1200 = Number of months in 100 years* Condome = 14 * IUCD = 2* Diaphragm = 20 LNG- IUS (0.2 H.W.Y)]* Rhythm method = 20-30 = 1-2 H.W.Y* Coitus interruptus = 20 * OCP = * Progesterone alone [0.5-2 H.W.Y", "cop": 2, "opa": "IUCD", "opb": "Oestrogen + Progesterone", "opc": "Oestrogen alone", "opd": "Progesterone alone", "subject_name": "Gynaecology & Obstetrics", "topic_name": "Miscellaneous (Gynae)", "id": "a0ba1f42-2f64-47d8-a245-aea7916dcfe4", "choice_type": "single"} {"question": "The best gas used for pneumoperitoneum laparoscopy is", "exp": "Carbon dioxide is the most widely used peritoneal distention medium, because it is non-flammable, colourless and has a higher blood solubility than air & the rapid absorption of CO2 in blood reduces the incidence of gas emboli.", "cop": 3, "opa": "N2", "opb": "O2", "opc": "CO2", "opd": "NO", "subject_name": "Gynaecology & Obstetrics", "topic_name": "Operative Gynecology", "id": "7770d28c-282b-4944-8f11-4cbc3a84d6aa", "choice_type": "single"} {"question": "MIRENA is", "exp": "Ans. is 'a' i.e. Progesterone IUCD MIRENA is an intrauterine device.There are two types of IUD'sChemically inert IUD's - They are composed of a nonabsorbable material, most often polyethylene and impregnated with barium sulfate for radiopacity.Chemically active IUD's * They either haveContinuous elution of copper orProgestational agent.At present time only chemically active IUD's are available.MIRENA is a progesterone containing IUDThe progesterone agent present in it is levonorgestrel.MIRENA releases levonorgestrel into the uterus at a relatively constant rate of 20 mg/day.It works by causing endometrial atrophy.Other beneficial effect:The MIRENA device reduces menstrual blood loss and can even be used to treat menorrhagia.", "cop": 1, "opa": "Progesterone IUCD", "opb": "Hormonal implant", "opc": "Anti progesterone", "opd": "Used for MTP", "subject_name": "Gynaecology & Obstetrics", "topic_name": "Methods - Progestogen-Only Contraception", "id": "0adf2f89-0fec-4e26-a2cd-999b9e2cf66d", "choice_type": "single"} {"question": "The best method of cuailing the second stage of labour in hea disease is by", "exp": "Seccond stage management in a Hea disease complicating Pregnancy Maternal pushing and the tendency to delay in the second stage of labour is to be cuailed by forceps or ventouse under pudendal /perineal block Ventouse is preferable to forceps as it can be applied without putting patient in lithotomy position IV ergometrine after the delivery should be withheld to prevent sudden overloading of the hea by additional blood squeezed out from the uterus Reference: D.C.Dutta&;s Textbook of Obstetrics; 7h edition; Page no: 278", "cop": 2, "opa": "Prophylactic forceps", "opb": "Prophylactic ventouse", "opc": "Spontaneous delivery with episiotomy", "opd": "Caesarean section", "subject_name": "Gynaecology & Obstetrics", "topic_name": "Medical, surgical and gynaecological illness complicating pregnancy", "id": "3b6927de-6b66-47f0-80a8-a9213470d2ea", "choice_type": "single"} {"question": "A 40 year old multiparous women complains of involuntary loss of urine associated with coughing, laughing, lifting weight or standing. The history is most likely suggestive of", "exp": "Stress incontinence When the woman voids a small quantity of urine involuntarily while sneezing, coughing or laughing. The condition also develops during pregnancy and soon after delivery There is no desire to pass urine, but dribbles a small quantity on stress Grading: Grade 0:Incontinence without leakage Grade I: Incontinence caused only with severe stress such as coughing , sneezing and jogging Grade II: Incontinence caused by moderate stress such as fast walk,going up and down the stairs Grade III: Incontinence caused by mild stress such as standing Other options Fistula: a fistula is a communication between two cavities, such as vesico-vaginal fistula, urethrovaginal, uretero-vaginal fistulas. These fistulas have will present with incontinence, unrelated to activity or raised intra-abdominal pressure Urge incontinence :associated with sudden and abrupt desire to void. It is associated with detrusor instability (overactive bladder) or detrusor hyperreflexia secondary to neurological disease Overflow incontinence: leakage of small amount of urine resulting from mechanical forces on overdistended bladder", "cop": 2, "opa": "Fistula", "opb": "Stress incontinence", "opc": "Urge incontinence", "opd": "Urethral diveiculum", "subject_name": "Gynaecology & Obstetrics", "topic_name": "Genital Prolapse - various presentation of prolapse and management of urinary stress incontinence (I am hanging by my nails !)", "id": "a26d7d91-9c3d-46b4-b9cd-1ba2d4122569", "choice_type": "single"} {"question": "Failure rate of vasectomy is", "exp": null, "cop": 2, "opa": "0.20%", "opb": "0.10%", "opc": "3%", "opd": "10%", "subject_name": "Gynaecology & Obstetrics", "topic_name": null, "id": "b60a1bab-3549-4118-b325-bb825639f9ff", "choice_type": "single"} {"question": "A 8th week pregnant women presents with mild vaginal bleeding and pain in the lower abdomen. The size of the uterus is 8 weeks. The cause is", "exp": "A i.e. (Threatened abortion) (161 - 165 - Dutta 6th)THREATENED ABORTIONINEVITABLE ABORTIONINCOMPLETE ABORTION* Clinical entity where the process of abortion has started but has not progressed to a state from which recovery is impossible* Bleeding per vaginam is usally slight and bright red in colour. Bleeding stops spontaneously* Bleeding is painless mild backache or dull pain in lower abdomen**. Pain appears usually following haemorrhage(i) Closed external os(ii) Uterine size corresponds to the period of amenorrhoea**(iii) Uterus and cervix feel soft* A state from where continuation of pregnancy is impossible* Increased vaginal bleeding* Aggravation of pain in the lower abdomen (colicky in nature)* Dilated internal os of the cervix through which the products of conception are felt* The general condition of the patient is proportionate to the visible blood loss.* When the entire products of conception are not expelled, instead a part of it is left inside the uterine cavity* This is the commonest type met amongst women, hospitalized for abortion complications* History of expulsion of a fleshy mass per vaginum*** Continuation of pain lower abdomen coliky in nature* Persistence of vaginal bleeding of varying magnitude* Uterus smaller than the period of amonorrhea*** Patulous cervical is often admitting tip of the finger* Varying amounts of bleeding* Expelled mass is found incomplete* Mifepristone is used in Threatened abortion*** HIV is associated with minimal teratogenic risk to the fetus**", "cop": 1, "opa": "Threatened abortion", "opb": "Inevitable abortion", "opc": "Incomplete abortion", "opd": "Induced abortion", "subject_name": "Gynaecology & Obstetrics", "topic_name": "Miscellaneous (Gynae)", "id": "7592a1b0-a499-4a96-8fc7-335ef39410a4", "choice_type": "single"} {"question": "In post menopausal women, there is an increased level of", "exp": "FSH", "cop": 1, "opa": "FSH", "opb": "LH", "opc": "Oestrogens", "opd": "HCG", "subject_name": "Gynaecology & Obstetrics", "topic_name": null, "id": "0a7e3cee-834a-4cbc-8921-3a87e940e9a0", "choice_type": "single"} {"question": "A 9 year girl presents with menarche. History reveals thelarche at the age of 8 years the most common cause of this condition in girls is", "exp": "Precocious pubey:The term is reserved for girls who exhibit any secondary sexual charecters before the age of 8 or menstruate before the age of 10 Causes GnRH Dependent: 80% Constitutional : Activation of HPO axis Juvenile Primary Hypothyroidism Intracranial Lesions GnRH Independent Ovary Adrenal Liver Iatrogenic Granulosa cell tumour Theca cell tumor Leydig cell tumor Androblastoma McCune Albright syndrome Hyperplasia Tumor Hepatoblastoma Exogenous Estrogen or Androgen intake Ref: D.C.DUTTA&;S TEXTBOOK OF GYNAECOLOGY; 6th edition; Pg no:51", "cop": 1, "opa": "Idiopathic", "opb": "Gonandal tumor", "opc": "McCune albright syndrome", "opd": "Hypothyroidism", "subject_name": "Gynaecology & Obstetrics", "topic_name": "Sexuality and intersexuality", "id": "6b3e010e-a28c-477e-a946-4823fb1929c1", "choice_type": "single"} {"question": "Peripartum cardiomyopathy occurs at", "exp": "(D) Within 5 months[?]PPCM is a structural heart muscle disease that occurs in women either at the end of pregnancy or up to five months after giving birth.Peripartum Cardiomyopathy:Peripartum cardiomyopathy is a cardiomyopathy that occurs in the third trimester or in the months following delivery and presents with congestive heart failure.The etiology is poorly understood.Treatment must include anticoagulation because of the high risk of thromboembolism.Over one third of patients have complete recovery.A risk of recurrence exists in subsequent pregnancies.Peripartum Cardiomyopathy - Pathophysiology:Peripartum cardiomyopathy (PPCM) is a dilated cardiomyopathy documented with echocardiographic left ventricular dysfunction occurring in the last month of pregnancy or within 5 months of delivery.Patients with a prior history of myocardial disease are excluded from this definition.Risk factors include multi-parity, being black, older maternal age and preeclampsia. In a retrospective study of 123 women with PPCM, a history of hypertension was obtained in 43% of patients and twin pregnancies were reported in 13%.Idiopathic cardiomyopathy presenting with heart failure secondary to LV systolic dysfunction from 1 month antepartum, up to 5 months after delivery, when no other causes of heart failure is evident.Incidence of 1:300 to 1:4000 pregnancies.Predisposing factors:Multi-parity, Multiple Births, Smoking, diabetes, Pregnancy complicated by pre-eclampsia, eclampsia or hypertension, Advanced age or teenage pregnancy.Criteria for Peripartum Cardiomyopathy:Classic:Development of Cardiac failure in the last month of pregnancy or within 5 months after delivery.Absence of an identifiable cause for the cardiac failure.Absence of recognizable heart disease prior to the last month of pregnancy.Additional:Left ventricular systolic dysfunction demonstrated by classic Echo Cardio Graphic criteria such as depressed shortening fraction or ejection fraction.Peripartum Cardiomyopathy:Important diagnostic criteria are:Cardiac failure within last month of pregnancy or within 5 months postpartum.No determinable cause for failure.Absence of previous heart disease.Left ventricular dysfunction as evidenced on echocardiography-Ejection fraction <45%-Left ventricular end diastolic dimension >2.7cm/m2. Peripartum cardiomyopathy is a diagnosis of exclusion. The patients are usually multiparous & young (20-35 years). They complain of weakness, shortness of breath, cough, nocturnal dyspnoea & palpitation. Examination reveals - tachycardia, arrhythmia, peripheral oedema & pulmonary rales. Pregnancy is poorly tolerated in women with dilated cardiomyopathy.The treatment is bed rest, digitalis, diuretics (preload reduction), hydralazine or ACE inhibitors (postpartum) (after load reduction), beta blocker & anticoagulant therapy. Vaginal delivery is preferred. Epidural anaesthesia is ideal. There is no contraindication of breast feeding. Mortality is high (20-50%) due to pulmonary embolism & cerebral thrombosis. It may recur in subsequent pregnancies.Myocardial infarction is rare in pregnancy. Management is mostly as in on pregnant state. Coronary angioplasty, stenting and thrombolytic therapy has been done in pregnancy when indicated. Supine position and hypotension should be avoided. Labor: managed as with standard cardiac care. Elective delivery within two weeks of infarction should be avoided. Regional analgesia for pain in labor and p blockers for tachycardia may be used. Maternal pushing is avoided and second stage is shortened by forceps or vacuum. Syntocinon should be used in the third stage management as ergometrine may cause coronary artery spasm. Percutaneous transluminant coronary angioplasty can be done successfully around 36 weeks of pregnancy if needed.", "cop": 4, "opa": "Within 7 days", "opb": "Within 6 weeks", "opc": "Within 24 months", "opd": "Within 5 months", "subject_name": "Gynaecology & Obstetrics", "topic_name": "Miscellaneous (Obs)", "id": "24752b77-feb4-448f-b64b-2ca6bedc7100", "choice_type": "single"} {"question": "Most common infection associated with IUCD use is", "exp": "Ans. a.Chlamydia Infections with chlamydia and actinomyces are most common with use of IUCD", "cop": 1, "opa": "Chlamydia", "opb": "Staphylococcus epidermidis", "opc": "Staphylococcus aureus", "opd": "Group D Streptococcus", "subject_name": "Gynaecology & Obstetrics", "topic_name": null, "id": "08d7f768-be51-41bb-8b1c-dfae931c68a5", "choice_type": "single"} {"question": "Kelly&;s plication operation is done in", "exp": "Stress Incontinence Management Surgical: Kelly&;s repair : Plication of bladder neck Pacey&;s repair : apposing medial fibres of the puborectalis muscles in the midline under the bladder neck region to elevate the same Both may be combined with anterior colporrhaphy Ref: SHAW&;S TEXTBOOK OF GYNAECOLOGY;15th edition; Pg no:193", "cop": 1, "opa": "Stress incontinence", "opb": "Vault prolapse", "opc": "Rectal prolapse", "opd": "Uterine prolapse", "subject_name": "Gynaecology & Obstetrics", "topic_name": "Urogynecology", "id": "ec295459-39b0-4d16-b36d-68545b5e1885", "choice_type": "single"} {"question": "Initial site of RBC production in fetus", "exp": "Hematopoiesis - 1st demonstrable in yolk sac followed by liver and then finally spleen and bone marrow life span of fetal RBCs at term - 90 days", "cop": 2, "opa": "Gestational sac", "opb": "Yolk sac", "opc": "Placenta", "opd": "Fetal bones", "subject_name": "Gynaecology & Obstetrics", "topic_name": "Anaemia in Pregnancy", "id": "1fe8d213-7dc1-4aea-ab70-02692354fa9e", "choice_type": "single"} {"question": "Infertility issues with leiomyoma can be addressed by", "exp": "GnRH agonist treatment is approved for treating infertility associated with fibroid uterus.", "cop": 3, "opa": "Combined oral contraceptive pills", "opb": "DMPA", "opc": "GnRH agonist", "opd": "Ulipristal", "subject_name": "Gynaecology & Obstetrics", "topic_name": null, "id": "df6a272b-7829-430a-8a33-e4699311c2f7", "choice_type": "single"} {"question": "A 21 yrs old girl with 8 weeks amenorrhoea, now comes in shock. The likely diagnosis is", "exp": "Ans. is a i.e. Ruptured ectopic pregnancy Always remember : History of acute abdominal catastrophe with fainting attack and collapse i.e. shock following sho period of amenorrhea, in a woman of child bearing age always points towards ectopic pregnancy (ruptured) and no other diagnosis.", "cop": 1, "opa": "Ruptured Ectopic Pregnancy", "opb": "Incarcerated Amnion", "opc": "Twisted Ovarian cyst", "opd": "Threatened Aboion", "subject_name": "Gynaecology & Obstetrics", "topic_name": null, "id": "054244f1-b78b-42e8-bc34-9d812662cb63", "choice_type": "single"} {"question": "Most sensitive diagnostic technique for Trichomonas infection", "exp": "Ans. is a, i.e. Culture in Diamond mediaRef. Williams Gynae 3rd/ed, p63\"The most sensitive diagnostic techniques is culture which is impractical because special media (Diamond media) is required.\"", "cop": 1, "opa": "Culture in Diamond media", "opb": "Culture in Thayer Martin media", "opc": "Pap Smear", "opd": "NAAT", "subject_name": "Gynaecology & Obstetrics", "topic_name": "Sexually Transmitted Disease in the Female", "id": "f13e9f8c-f291-4787-bfec-49b53bd0b19b", "choice_type": "single"} {"question": "Best time to do quadruple test", "exp": "* The quad screen is done in the second trimester, usually between 15 and 20 weeks of pregnancy. * Ideally, the test should be performed in conjunction with first-trimester screening tests. REF : DUTTA BOOK OF OBSTETRICS", "cop": 3, "opa": "8-12 weeks", "opb": "11-15 weeks", "opc": "15-20 weeks", "opd": "18-22 weeks", "subject_name": "Gynaecology & Obstetrics", "topic_name": "All India exam", "id": "7395ce38-139a-4d7b-b79e-38cacea8a965", "choice_type": "single"} {"question": "Prostaglandin that can be used to induce labour at term is", "exp": "Ans. a. PGE2. (Katzung Clinical Pharmacology 11th edition./, Ch.l8/page 410)Dinoprostone, a synthetic prep of PGE2, is administered vaginally for oxytocic use. It is used for inducing abortion in the second trimester of pregnancy, for missed abortion, for benign hydatidiform mole, and for ripening of the cervix for induction of labor in patients at or near term. An analog of PGF2a (carboprost tromethamine) is also used in obstetrics - used to induce second-trimester abortions and to control PPH not responding to conventional methods of Rx.PROSTAGLANDINS# Have very short half-life.# Arachidonic acid is common precursor (acylic fatty acid derivatives).# The subscript numeral after the letter (e.g., PGE2) indicates degree of unsaturation.# PGF2 alpha acts predominantly on myometrium, while PGE2 on cervix due to its collagenolytic property. (Hint: F for fundus and E for effacement of cervix.).# Carboprost is effective in refractory cases of PPH.# Misoprostol is methyl ester of PGE1, rapidly absorbed and more effective than oxytocin and prostaglandins in induction of labour, also effective in treatment in NSAID induced peptic ulcer.# Inflammatory mediators from arachidonic are:- TxA2, PGD2, PGE2, PGF2 alpha, PGI2 (Cycloxygenase pathway) leukotrienes (Lipoxygenase)--LTB4 is a neutrophil chemotactic agent. LTC4, D4 and E4 (SRS-A) function in broncho-constriction, vasoconstriction, contractions of smooth muscle and increased vascular permeability.PgActionsPGE2Bronchoconstriction, platelet aggregation.PGE2Potent inducer of bone resorption and releases Ca++ from bones and cervical effacement.PGE2Bronchoconstriction, role in uterine and ovarian function (fundal contraction).PGI2Bronchodilation, vasoconstriction, and platelet aggregation.TXA2Bronchodilation, vasoconstriction, and platelet aggregation stimulation (Bradykinin and histamine are potent vasodilators)USES OF PROSTAGLANDINS IN OBSTETRICS# Induction of abortion (MTP and missed abortion); Antiprogestins (eg, mifepristone) have been combined with an oral oxytocic synthetic analog of PGE1 (misoprostol) to produce early abortion.# Induction of labour# Termination of molar pregnancy# Cervical ripening before induction of labour/abortion# Acceleration of labour# Management of atonic PPH# Management of tubal ectopic pregnancyActionArachidonic Acid Metabolites (Eicosanoids)VasodilationPGI2 (prostacyclin), PGE1, PGE2, PGD2VasoconstrictionThromboxane A2, leukotrienes C4, D4, E4Increased vascular permeabilityLeukotrienes C4, D4, E4Chemotaxis, leukocyte adhesionLeukotriene B4", "cop": 1, "opa": "PGE2", "opb": "PGF2", "opc": "PGI2", "opd": "PGD2", "subject_name": "Gynaecology & Obstetrics", "topic_name": "Miscellaneous (Gynae)", "id": "da14134d-9afc-4b5e-ada6-6231d81eba07", "choice_type": "single"} {"question": "Betamethasone in pregnancy is given to prevent", "exp": "Respiratory distress syndrome can be prevented by administration of Betamethasone to the mother two doses IM 24hrs apa given before 34 weeks Ref: Dutta Obs 9e pg 444.", "cop": 4, "opa": "Prematurity", "opb": "Neonatal convulsions", "opc": "Cerebral palsy", "opd": "Respiratory distress syndrome", "subject_name": "Gynaecology & Obstetrics", "topic_name": "Abnormal labor", "id": "435d58a3-8b6b-437b-a0c4-aeb75a6fe2cc", "choice_type": "single"} {"question": "The use of combined oral contraceptive pill is associated with an increased incidence of", "exp": "Ans. is b i.e. Chlamydial endocervicitis This is a tricky question as some believe Option \"b should be the answer while others believe Option \"c\" should be concerned. As far as candidial (monilial) vaginitis is concerned, OCP's use increase their incidence. But for Chlamydial infections : \"Persons who use barrier contraception are less frequently infected by C. Trachomatis than those who use no contraception, and women who use oral contraceptives may have a higher incidence of cervical infection than women not using oral contraceptives\". As if replying to CGDT \"Chlamydial colonization of the cervix appears more likely in OC users than in non users, but despite this. there is a 40-50% reduction in risk for Chlamydial PID\" I then had to confirm the answer from Clinical Gynaecologic Endocrinology and Infeility 7,/e by Leon Speroff (It is the most authentic and reliable book for all problems related to Endocrinology, Contraception and Infeility) \"Fifteen of the 17 published studies repoed a positive association of oral contraception with lower Genital tract infections caused by Chlamydial cervicitis. Because lower genital tract infection are on the rise (now the most prevalent STI in the US) and the rate of hospitalization for PID is also increased, it is wohwhile for both patients and clinicians to be ale for symptoms of cervicitis or salpingitis in women on oral contraceptives who are at high risk of sexually transmitted infections.\" As far as HPV infection i.e. Vaginal was is concerned 'The viral sexually transmitted infections (STI's) include HIV, Human papilloma virus (HPV), Herpes simplex virus (HSV) and hepatitis B (HBV). At the present time, no known associations exist between oral contraception and the viral STI'S' ... Leon Speroff 7/e, p 904 So, now we can be sure that the answer is Chlamydial endocervicitis. Also know : Infections and Oral contraception : Use of OCP is associated with Increased risk of infection - Candida (Moniliasis) - Gonorrheadeg - Viral STI's i.e. HIVdeg . HPVdeg - Chlamydia - Trichomonasdeg - Hepatitis B virus - Urinary tract infections - Bacterial vaginosis - Herpes simplex virus Note : If question says PID and does not specify any organism. Then OCP'S overall not only decrease the incidence of PID but also risk of hospitalisation and severity of the disease is decreased. For protection against PID, at least 12 months of continuous use is necessary and this protection is limited only to current users.", "cop": 2, "opa": "Bacterial vaginosis", "opb": "Chlamydial endocervicitis", "opc": "Vaginal was", "opd": "Genital herpes", "subject_name": "Gynaecology & Obstetrics", "topic_name": null, "id": "a740f90e-864f-423e-bba0-8b74a07b4810", "choice_type": "single"} {"question": "A pregnant woman with fibroid uterus develops acute pain in abdomen with low grade fever and mild leucocytosis at 28 weeks ,the most likely diagnosis is", "exp": "Red degeneration of fibroid: It results from softening of surrounding connective tissue The capillaries tend to rupture and blood effused out into the myoma causing a diffuse reddish discoloration The tumor it self assumes a peculiar purple red colour and develops a fishy odour,if the tumor is carefully examined We may find some vessels and large veins in the capsule thrombosed This is the most common complication with fibroid during pregnancy . And the patient is usually febrile and with moderate leucocytosis and raised ESR, the condition is aseptic one. And ultrasound is useful for diagnosis Ref Shaw 16/e pg 395", "cop": 3, "opa": "Preterm labour", "opb": "Torsion of fibroid", "opc": "Red degeneration of fibroid", "opd": "Infection in fibroid", "subject_name": "Gynaecology & Obstetrics", "topic_name": "Anatomy of the female genital tract", "id": "de396e96-6668-4cca-9bd7-e69620d07ee5", "choice_type": "single"} {"question": "Tumor marker of ca ovary for follow up", "exp": "Ans. (c) CA 125Ref : Shaws 15th ed. 1422,, Shaws 15th ed./ 928* Tissue markers such as CA-125 and NB/70k are useful mainly in the follow-up of certain tumors.* CA-125 is a glycoprotein and surface cell antigen which is secreted by the malignant epithelial tumors.* Level of CA-125 more than 35 U/ml suggests malignant and residual tumor, and indicates the need of chemotherapy.* CA-125 is also raised in abdominal tuberculosis and endometriosis.* CEA (carcinoembryonic antigen) more than 5 mg/1 is seen in mucinous ovarian tumour.* NOTE: CA-125 is raised in only 50% cases in Stage I and 90% in Stage II ovarian cancer.", "cop": 3, "opa": "CEA", "opb": "PSA", "opc": "CA 125", "opd": "BETA HCG", "subject_name": "Gynaecology & Obstetrics", "topic_name": "Tumor Markers", "id": "4a2444ce-3b34-4e24-8930-94e679cb072c", "choice_type": "single"} {"question": "Pseudogestational sac is seen in ultrasonography is suspicious of", "exp": "Anechoic fluid collections, which might normally suggest an early intrauterine gestational sac, may also be seen with ectopic pregnancy. These include pseudogestational sac and decidual cyst.A pseudosac is a fluid collection between the endometrial layers and conforms to the cavity shape. If a pseudosac is noted, the risk of ectopic pregnancy is increased Reference: William's Obstetrics; 25th edition; Chapter 19; Ectopic pregnancy", "cop": 3, "opa": "Missed aboion", "opb": "Incomplete aboion", "opc": "Ectopic pregnancy", "opd": "Complete aboion", "subject_name": "Gynaecology & Obstetrics", "topic_name": "General obstetrics", "id": "67278fe4-9382-4616-880a-dd5198c852a6", "choice_type": "single"} {"question": "Estrogen administration in a menopausal woman increases the", "exp": "Ans. is c i.e Bone mass Before seeing any Reference for the Question let's rule out some options. Estrogen administration will exe a negative feedback on gonadotropin secretion and decreases Gonadotropin secretion rather than increasing it (Option 'a' ruled out). As I have already discussed in previous Question -Estrogen decreases LDL and not increases it (ruling out Option `b). Now we are left with 2 options, Option 'c' i.e. Bone mass and Option 'rile. Muscle mass. Estrogen given as hormone replacement therapy is most beneficial in preventing osteoporosis i.e it must be increasing bone mass. So, Option 'c'seems to be correct. Now have a look what texts have to say : Estrogen helps to maintain bone mass and skeletal integrity thereby protecting against osteoporosis. Effect of estrogen on bones : Estrogen causes increased osteoblastic activity in the bones. It is impoant in maintaining bone mass primarily by retarding bone resorption. The major action of estrogen is directed at reducing the maturation and activity of osteoclasts, by modifying regulatory cytokine signals from osteoblasts. The action of estrogen and progesterone result in increased expression of bone matrix proteins such as osteonectin, osteocalcin, collagen and alkaline phosphatase. Extra Edge : An overveiw of osteoporosis Risk factors for osteoporosis Non modifiable Modifiable Associated Medical Conditions * Age * Race (caucasian, asian) * Small body frame * Early menopause * Prior fracture * Family history of osteoporosis Inadequate intake of calcium and vitamin D Smoking Low body weight Excess alcohol use Sedentary lifestyle Hypehyroidism Hyperparathyroidism Chronic renal disease Conditions requiring systemic coicosteroid use Diagnosis : Bone Mineral Density Measurement (BMD) Dual X-ray Absorptiometry (DXA) of hip and spine is the primary technique for BMD assessment. BMD is expressed as a T score, which is the number of standard detions from the mean for a young healthy woman. A T score above --1 is considered normal, a value between --1 and --2.5 denotes osteopenia and a score below --2.5 indicates osteoporosis. Evaluation of BMD by DXA is recommended for all women aged 65 and older, regardless of risk factors and for younger postmenopausal women with 1 or more risk factors. Prevention : Women should receive 1000 to 1500 mg of calcium and 400 to 800 IU of vitamin D daily Options for osteoporosis prevention and treatment BisphosphonatesO Alendronate : 35 - 70 mg/week Risedronate : 35 mg/week lbandronate : 150 mg/month Additional potential benefits : none Potential risks : esophageal ulcers Side effects : gastrointestinal distress, ahralgia / myalgia Hormone therapy Estrogendeg or estrogen / progestin therapydeg Additional potential benefits : treatment of vasomotor symptoms and urogenital atrophy Potential risk : breast cancer, gallbladder disease, venous thromboembolic events, cardiovascular disease, stroke. Side effects : vaginal bleeding, breast tenderness Selective estrogendeg receptor modulators (SERMSP Raloxifenedeg : 60 mg/day Additional potential benefits : reduced risk of breast cancer Potential risks : venous thromboembolic events Side effects : Vasomotor symptoms, leg cramp Calcitonindeg : 200 IU/day intranasally or 100 IU/day subcutaneously or intramuscularly Additional potential benefits : non Potential risks : none Side effects : rhinitis, back pain Tariparatidedeg : 20 mg/day subcutaneously Additional potential benefits : none Potential risks : osteosarcoma after long - term use in rodents, hypercalcemia Side effects : leg cramps", "cop": 3, "opa": "Gonadotrophin secretion", "opb": "LDL -- cholesterol", "opc": "Bone mass", "opd": "Muscle mass", "subject_name": "Gynaecology & Obstetrics", "topic_name": null, "id": "d37f030c-97e0-492f-9627-0d67d1282e45", "choice_type": "single"} {"question": "Grade 3 endometrial tumors according to FIGO staging is", "exp": "The differentiation of a carcinoma, expressed as its grade, is determined by architectural growth pattern and nuclear features. In the International Federation of Gynecology and Obstetrics (FIGO) grading system proposed in 1989, tumors are grouped into three grades: grade 1, 5% or less of the tumor shows a solid growth pattern; grade 2, 6% to 50% of the tumor shows a solid growth pattern; and grade 3, more than 50% of the tumor shows a solid growth pattern. The presence of notable nuclear atypia that is inappropriate for the architectural grade increases the tumor grade by one.Histopathologic degree of differentiation:G1 <5% non squamous or non morular growth patternG2 6%-50% non squamous or non morular growth patternG3 >50% non squamous or non morular growth patternReference: Novak's gynecology; 14th edition; Chapter 35; Uterine cancer", "cop": 3, "opa": "<5% non squamous or non morular growth pattern", "opb": "6%-50% non squamous or non morular growth pattern", "opc": ">50% non squamous or non morular growth pattern", "opd": ">75% non squamous or non morular growth pattern", "subject_name": "Gynaecology & Obstetrics", "topic_name": "Gynaecological oncology", "id": "85b8234a-7fcb-4fab-afde-a45b9ab566f8", "choice_type": "single"} {"question": "Follow uo in a patient of H mole is done", "exp": "Ans. is a i.e. Serum Beta HCG monitoring The chances of persistent trophoblastic disease and choriocarcinoma are high after evacuation of H.mole therefore regular follow up is mandatory. \"After molar evacuation, patients should be monitored with weekly determinations of 0-subunit HCG levels until these levels are normal for 3 consecutive weeks, followed by monthly determinations until the levels are normal for 6 consecutive months. The average time to achieve the first normal HCG level after evacuation is about 9 weeks. At the completion of follow-up, pregnancy may be undeaken. After a patient achieves a nondetectable HCG level, the risk of developing tumour relapse is very low and may approach zero.\" Besides Serum Beta HCG at follow up following investigations are done : Pelvic examination -- To rule out any vulval and vaginal metastasis. -- To see the size of the uterus. -- To note the presence of any ovarian cysts and their sizes. It should be repeated every four weeks in the observation period. unest X-ray To detect lung metastasis Repeated at four weeks till spontaneous remission occurs and then at 3 months interval in the observation period. Pelvic Ultrasound : To detect any residual or locally invasive tumour and presence of ovarian cysts. Also Know : Patients are encouraged to use effective contraception during the entire interval of HCG follow up (i.e. atleast for 1 year). Choice of contraception : IUCD : should not be used because of the potential risk of uterine perforation and IUCDs cause irregular uterine bleeding which can be confused with persistent trophoblastic disease. Progestin pills : cause irregular bleeding therefore should not be used. OCP's : Earlier it was believed that OCP's should not be used till HCG values return to normal. But now it is said - \"OCP's may be used safely after molar evacuation during the entire interval of hormonal follow up\". \"Oral contraceptives are found to be superior to Barrier method or use of IUCD in decreasing the risk of developing gestational trophoblastic neoplasia\". Barrier method : Earlier they were the contraception of choice in case of GTDs. (Now it is combined pills).", "cop": 1, "opa": "Serum Beta-HCG monitoring", "opb": "Serum CEA level estimation", "opc": "Serum amylase level", "opd": "Serum a feto protein estimation", "subject_name": "Gynaecology & Obstetrics", "topic_name": null, "id": "53696ca6-f1d6-4f71-950b-c9ee0952a7bb", "choice_type": "single"} {"question": "Most commonly gas used in Laparoscopy is", "exp": "B. i.e. (Carbon - dioxide) (560 - Dutta 4th) (217- Shaw's 14th)* Pneumoperitoneum - Most commonly used gas is carbon dioxide (1L - 4L)* For diagnostic purposes nitous oxide or room air or oxygen can be used* CO2 is safer than air and nitrous oxide which can cause air embolism and accidental explosion respectively* The flow rate of the gas is about 1 L/minutc with a pressure not exceeding 20 mm HgCONTRAINDICATIONS OF LAPAROSCOPY* Severe cardiopulmonary disease* Extensive peritoneal adhesion* Patient haemodynamically unstable* Large pelvic tumour* Generalized peritonitis* Pregnancy > 16 weeks* Significant haemoperitoneum* Previous periumbilical surgery (relative)* Intestinal obstruction* Extreme obesity", "cop": 2, "opa": "Oxygen", "opb": "Carbondioxide", "opc": "Nitrous oxide", "opd": "Room air", "subject_name": "Gynaecology & Obstetrics", "topic_name": "Miscellaneous (Gynae)", "id": "1af0b2c9-6a1a-4b69-a142-3a190a095b1d", "choice_type": "single"} {"question": "Stage of Ovarian tumor with surgical spill", "exp": "Stage 1: tumor confined to ovaries or fallopian tubes. 1c: tumor limited to one or both ovaries. : Surgical spill : Capsule rupture before surgery or tumor on ovarian surface : Malignant cells present in ascites or peritoneal washings Reference : textbook of gynaecology Sheila balakrishnan, 2nd edition, pg no:277 <\\p>", "cop": 3, "opa": "Stage 1A", "opb": "Stage 1B", "opc": "Stage 1C1", "opd": "Stage1C2", "subject_name": "Gynaecology & Obstetrics", "topic_name": "Gynaecological oncology", "id": "e891b482-a8c4-4a19-b2f4-64bbd4fa8daa", "choice_type": "single"} {"question": "Point B in cervical radiotherapy represents", "exp": "Point A: 2cm cephalic and lateral to the external os It is the point of crossing of uterine aery and ureter It receives 7000-8000cGy Point B: 2cm cephalic and 5cm lateral at the same plane and is approximately the site of obturator gland It receives 2000cGy initially and rest of dose supplemented by external beam irradiation of 4000cGy spreading over another 3 weeks total of 6000cGy Ref: D.C.DUTTA'S TEXTBOOK OF GYNECOLOGY; 6th edition; Pg no:350", "cop": 4, "opa": "Sentinel Lymph nodes", "opb": "Internal iliac Lymph nodes", "opc": "External iliac Lymph nodes", "opd": "Obturator Lymph nodes", "subject_name": "Gynaecology & Obstetrics", "topic_name": "Gynaecological oncology", "id": "23459e13-6928-485b-9006-aebbc6e75f44", "choice_type": "single"} {"question": "Surest sign of salpingitis is", "exp": "[Cl (Seropuralent discharge from the part of tubes) (419-S) (166-Gynae -Dutta 5th)Acute salpingitis, the fallopian tube is swollen, oedematous, and hyperaemic with visible dilated vessels on the peritoneal surface. Some degree of serous exudation is seen around the fallopian tubes.* The sure sign of salpingitis is the discharge of seropurulent fluid from the fimbrial end of the tubes, without which the diagnosis cannot be justified at laparotomy, as the peritoneal surface may be inflamed in pelvic peritonitis due to any other cause.", "cop": 3, "opa": "Edema of tubes", "opb": "Enlargement of tubes", "opc": "Seropurelent discharge from the part of the tube", "opd": "Hyperaemic tubes", "subject_name": "Gynaecology & Obstetrics", "topic_name": "Miscellaneous (Gynae)", "id": "b043d56b-2c26-4434-aac2-052e26b548e3", "choice_type": "single"} {"question": "The hormone responsible for lactation is", "exp": "(Prolactin): Ref: 63-D, 4/4-K. Sembulingeum 3rdEndocrine control of lactation can be divided in to following stage(a) Preparation of breast (mammogenesis)(b) Synthesis and secretion of milk by breast alveoli (lactogenesis)(c) Ejection of milk (galactokinesis)(d) aintenance of lactation (galactopoiesis)* Immediately after the delivery of the baby and expulsion of placenta, there is sudden loss of estrogen and progesterone. Now the prolactin is free to exert its action on breast and to promote lactogenesis.* Prolactin is important for initiation of milk secretion rather than maintenance of milk secretion. Galactopoiesis depends upon other hormonal factors like growth hormone, thyroxine and cortisol* Milk ejection is a reflex phenomenon. It is called milk ejection reflex or milk let down reflex. It is a neuroendocrine reflex* Milk ejection in most species requires oxytocin (247-Ganong 22th)", "cop": 1, "opa": "Prolactin", "opb": "FSH", "opc": "LH", "opd": "Progesterone", "subject_name": "Gynaecology & Obstetrics", "topic_name": "Miscellaneous (Gynae)", "id": "25de866a-d4ac-4849-8969-a84fc435eee8", "choice_type": "single"} {"question": "The given procedure is used to diagnose", "exp": "Trattner - double balloon catheter is used to diagnose urethral diverticula.", "cop": 2, "opa": "Vesicovaginal fistula", "opb": "Urethral diverticula", "opc": "Stop post - partum hemorrhage", "opd": "Pelvic organ prolapse", "subject_name": "Gynaecology & Obstetrics", "topic_name": null, "id": "2971b374-0a28-498b-9e3d-b85d2c99ec73", "choice_type": "single"} {"question": "Not an absolute contraindication of laparoscopy is", "exp": "Contraindications Severe cardiopulmonary disease Hemodynamically unstable Generalised peritonitis Diaphramatic hernia Hemoperitoneum Intestinal obstruction Excessive peritoneal adhesions Large pelvic tumour Pregnancy>16weeks Advanced malignancy Anticoagulation therapy D.C.DUTTA&;S TEXTBOOK OF GYNECOLOGY,Pg no:615,6th edition", "cop": 4, "opa": "Diaphragmatic hernia", "opb": "Generalized peritonitis", "opc": "Patient on anticoagulant therapy", "opd": "Previous incomplete laparoscopy", "subject_name": "Gynaecology & Obstetrics", "topic_name": "Gynaecological diagnosis and operative surgery", "id": "62f374c1-aa84-42bc-85fa-605ee6aeaea7", "choice_type": "single"} {"question": "Prophylactic chemotherapy in GTD is done for", "exp": "Prophylactic chemotherapy : If the hcG levels fails to become normal by the stipulated time or re-elevation at 4-8 weeks Rising beta hcG levels after reaching normal levels Post evacuation hemorrhage Follow up facilities are not adequate Evidences of metastasis Ref: Dutta Obs 9e pg 186.", "cop": 3, "opa": "Theca leutein cyst", "opb": "Hypehyroidism", "opc": "High initial beta Hcg", "opd": "Paial Mole", "subject_name": "Gynaecology & Obstetrics", "topic_name": "General obstetrics", "id": "ff5a3d7f-be9b-4369-907c-e6c4cdca65eb", "choice_type": "single"} {"question": "Clue cells are seen in..", "exp": "Ans. is \"a' i.e., BV Bacterial (Gardnerellal vaginosiso It is not a sexually transmitted condition. It is termed as vaginosis rather than vaginitis because it is associated with alteration of vaginal flora rather than any specific infection. There is considerable decrease in the number of lactobacilli and lOOtimes increase in the growth of other anaerobic bacteria. These are Haemophilus vaginalis, Gardnerella, Mobiluncus, Mycoplasma hominis. Half of the females are asymptomatic carriers but majority complain of vaginal discharge without itching.o Characteristic of vaginal discharge are as follows:White, milky, non-viscous discharge adherent to vaginal wall.pH of discharge is more than 4.5.Fishy odour when mixed with 10% KOH is due to amino-metabolites (amine or whiff test positive).Presence of clue cells - the epithelial cell have fuzzy border due to adherence of bacteria,o Diagnosis is based on wet smear examination & culture.TreatmentMetronidazole 500mg bd for 7 daysAmpicillin 500 mg bd or cephalosporine 500 mg bd for 7 daysTetracycline, doxycycline, clindamycin and sulphafurazole are other options.Clindamycin (2%) cream, lacteal gel (5% lactic acidjand omidazole vaginal tabs are for local use.", "cop": 1, "opa": "BV", "opb": "Chlamydia", "opc": "Moniliasis", "opd": "Trichomoniasis", "subject_name": "Gynaecology & Obstetrics", "topic_name": "Specific Infections", "id": "1427782a-26fa-4e9d-8105-53b6bd560b80", "choice_type": "single"} {"question": "The least common type of female pelvis is", "exp": ".", "cop": 4, "opa": "Gynecoid", "opb": "Android", "opc": "Anthropoid", "opd": "Platypelloid", "subject_name": "Gynaecology & Obstetrics", "topic_name": "All India exam", "id": "d9cecefc-27b5-4113-9846-07a8002ae60b", "choice_type": "single"} {"question": "Multipara with LSCS, Presents With Cyclical Hematuria, Diagnosis can be", "exp": "VESICO-UTERINE FISTULA Usually caused during caesarean section or uterine rupture Symptoms: Cyclical hematuria Gold standard for diagnosis: Cystoscopy Treatment is abdominal repair Ref: SHAW&;S TEXTBOOK OF GYNAECOLOGY; 15th edition; Pg no:188", "cop": 1, "opa": "VUF", "opb": "UVF", "opc": "Bladder Endometriosis", "opd": "Carcinoma Cervix", "subject_name": "Gynaecology & Obstetrics", "topic_name": "Urogynecology", "id": "ca5cc8ef-c3fd-4f43-9f06-7b583b5af7d7", "choice_type": "single"} {"question": "Select the most appropriate response of Pill forgotten for 10 continuous days (SELECT 1 RESPONSE)", "exp": "Common side effects of birth control pills include nausea, breakthrough bleeding, bloating, and leg cramps. If these side effects are experienced in the first two or three cycles of pills-when they are most common-the pills may be safely continued, as these effects usually remit spontaneously. On occasion, following correct use of a full cycle of pills, withdrawal bleeding may fail to occur (silent menses). Pregnancy is a very unlikely explanation for this event; therefore, pills should be resumed as usual (after 7 days) just as if bleeding had occurred. However, if a second consecutive period has been missed, pregnancy should be more seriously considered and ruled out by a pregnancy test, medical examination, or both. Women occasionally forget to take pills; however, when only a single pill has been omitted, it can be taken immediately in addition to the usual pill at the usual time. This single-pill omission is associated with little if any loss in effectiveness. If three or more pills are omitted, the pill should be resumed as usual, but an additional contraceptive method (e.g., condoms) should be used through one full cycle. Although most side effects caused by birth control pills can be considered minor, serious side effects do sometimes occur. A painful, swollen calf may signal thrombophlebitis.", "cop": 3, "opa": "Stop pills and resume after 7 days", "opb": "Continue pills as usual", "opc": "Continue pills and use an additional form of contraception", "opd": "Take an additional pill", "subject_name": "Gynaecology & Obstetrics", "topic_name": "Contraceptives", "id": "d7295d73-ca2e-4111-9ba7-3f1d29835039", "choice_type": "single"} {"question": "Method of conservative management of placenta pre", "exp": "Mcafee and Johnson Regimen- It is a regimen used for placenta pre when pregnancy is <34 weeks(lungs are not maturated) and there is not active bleeding. In it, following is done: 1. Resuscitation 2. Steroids for fetal lung maturity 3. Sedation Other options Crede's method is the act of vigourous massaging and squeezing of the fundus to force the separation of the placenta. Lilly's Cha is the plotting of optical density of the amniotic fluid in Rh isoimmunisation. Brandt Andrew method is the technique of delivering the placenta with the \"Controlled cord traction\" method.", "cop": 2, "opa": "Brandt andrew method", "opb": "Mcafee and Johnson", "opc": "Lilly's method", "opd": "Crede's method", "subject_name": "Gynaecology & Obstetrics", "topic_name": "Antepaum Haemorrhage", "id": "dcec1d3b-385a-4ba0-90db-5e6ca4b9d350", "choice_type": "single"} {"question": "Paogram helps in detecting", "exp": "Obstructed labour", "cop": 2, "opa": "Abruptio placenta", "opb": "Obstructed labour", "opc": "Incoordinate uterine action", "opd": "PPH", "subject_name": "Gynaecology & Obstetrics", "topic_name": null, "id": "34043be8-1453-49c9-a9ed-8f607067da34", "choice_type": "single"} {"question": "In comparison to breast milk, colostrum has a higher content of", "exp": "Colostrum is the milk secreted during the first three days. It is yellow, thick and contains more of antibodies and fat-soluble vitamins.Transitional milk is that secreted during the following 2 weeks and contains plenty of immunoglobulins.This is followed by mature milk which is thin and watery.(refer to pgno: 430 Sheila 2nd edition)", "cop": 2, "opa": "Carbohydrates", "opb": "Fat soluble vitamins", "opc": "Water", "opd": "Potassium", "subject_name": "Gynaecology & Obstetrics", "topic_name": "General obstetrics", "id": "11e07dff-2940-4e37-a1f6-361bd427b5b1", "choice_type": "single"} {"question": "Hysteroscopy should be done in", "exp": "Hysteroscopy should be done in Pre-ovulatory phase when the endometrium is thin and bleeding is less likely to occur.", "cop": 3, "opa": "Secretory phase", "opb": "Immediate post menstrual phase", "opc": "Pre-ovulatory phase", "opd": "Menstrual phase", "subject_name": "Gynaecology & Obstetrics", "topic_name": null, "id": "5d3f94a8-4f80-4152-92ba-e4570715bd5d", "choice_type": "single"} {"question": "A patient with spontaneous aboion came with history of amenorrhea and her FSH is 6 IU/mL, Diagnosis is", "exp": "Asherman&;s Syndrome Results from intrauterine adhesions that obstruct or obliterate the uterine cavity, as a consequence of trauma There is a formation of adhesion following post aboal and puerperal curettage and diagnostic curettage in AUB Menstrual abnormalities include dysmenorrhea, hypomenorrhea, oligomenorrhea and amenorrhea May present with Infeilty and Recurrent Pregnnacy Loss Combined Estrogen and Progesterone challenge test will be negative Hysterosalpingography shows honeycomb appearance Operative hysteroscopy is the primary method for treatment of intrauterine adhesions that may be lysed by scissors, electrodissection, or with a laser Reference: Clinical Endocrinologic Gynecology and Infeility; Eigth Edition; Chapter 11", "cop": 2, "opa": "Ovarian failure", "opb": "Uterine Synechiae", "opc": "Pregnancy", "opd": "Pituitary failure", "subject_name": "Gynaecology & Obstetrics", "topic_name": "Disorders of menstruation", "id": "d39d3e10-e1bb-4dba-89d0-9bc9469a17c9", "choice_type": "single"} {"question": "Karyotype of complete molar pregnancy is", "exp": "Complete moles most often have a diploid chromosomal composition. These usually are 46, XX and result from androgenesis, meaning both sets of chromosomes are paternal in origin. The chromosomes of the ovum are either absent or inactivatePaial moles usually have a triploid karyotype .Reference: William's Obstetrics; 25th edition, chapter 20; Gestational trophoblastic diseases", "cop": 1, "opa": "46 XX", "opb": "69 XXX", "opc": "69 XXY", "opd": "69 XYY", "subject_name": "Gynaecology & Obstetrics", "topic_name": "General obstetrics", "id": "3850c3f9-af65-492b-984f-4730303d9a4f", "choice_type": "single"} {"question": "Earliest detection of pregnancy by ultrasound is by", "exp": "The first definitive sonographic finding to suggest pregnancy is visualization of the gestational sac. The first sign of intrauterine pregnancy is presence of yolk sac within the gestational sac.", "cop": 4, "opa": "Fetal skeleton", "opb": "Fetal node", "opc": "FSH", "opd": "Gestation sac", "subject_name": "Gynaecology & Obstetrics", "topic_name": null, "id": "f8ae7f13-e82a-446b-8229-bd95831b411e", "choice_type": "single"} {"question": "After an initial pregnancy resulted in a spontaneous loss in the first trimester, your patient is concerned about the possibility of this recurring. An appropriate answer would be that the chance of recurrence", "exp": "An initial spontaneous abortion, irrespective of the karyotype or sex of the child, does not change the risk of recurrence in a future pregnancy. The rate is commonly quoted as 15% of all known pregnancies.", "cop": 2, "opa": "Depends on the genetic makeup of the prior abortus", "opb": "Is no different than it was prior to the miscarriage", "opc": "Is increased to approximately 50%", "opd": "Is increased most likely to greater than 50%", "subject_name": "Gynaecology & Obstetrics", "topic_name": "Fetus & New Born and their Diseases", "id": "1f9de46b-cf4a-4471-b523-c3d37a6efd05", "choice_type": "single"} {"question": "After delivery, gonadotropin levels in the neonate", "exp": "After delivery, gonadotropin levels in the neonate rise abruptly due to separation from the placenta.", "cop": 2, "opa": "Decrease abruptly", "opb": "Rise abruptly", "opc": "No changes", "opd": "Variable", "subject_name": "Gynaecology & Obstetrics", "topic_name": null, "id": "2ed04ca2-7bca-4584-902d-dd882beb0b38", "choice_type": "single"} {"question": "Moschcowitz repair is done in case of", "exp": "Enterocele is repaired by Moschcowitz repair.", "cop": 2, "opa": "Urethrocele", "opb": "Enterocele", "opc": "Vaginal vault prolapse", "opd": "Genital prolapse in pregnancy", "subject_name": "Gynaecology & Obstetrics", "topic_name": null, "id": "4b60dce9-ae5f-4caa-9102-85f67a36dd18", "choice_type": "single"} {"question": "Amount of liquor is maximum at", "exp": "The volume of amniotic fluid in a normal pregnancy is about 1000 ml at 34-36 weeks and thereafter it slowly decreases. The amniotic epithelium is the major source of amniotic fluid", "cop": 2, "opa": "32-34 weeks", "opb": "34-36 weeks", "opc": "36-38 weeks", "opd": "38-40 weeks", "subject_name": "Gynaecology & Obstetrics", "topic_name": null, "id": "2e4e1ee8-5d6e-488c-8a6f-7ba50afb6001", "choice_type": "single"} {"question": "Liley curve for evaluation of amnionic fluid bilirubin in Rh sensitized patient stas at", "exp": "The main limitation of the liley curve is that it stas at 26weeks gestation and extrapolation of the lines to earlier gestational ages is inaccurate. Queenan has developed a curve for fetal assessment from 14-40weeks, divided into 4zones", "cop": 4, "opa": "14weeks", "opb": "16weeks", "opc": "20weeks", "opd": "26weeks", "subject_name": "Gynaecology & Obstetrics", "topic_name": "Medical, surgical and gynaecological illness complicating pregnancy", "id": "430b7174-7a56-4e01-9b16-9a2d7865883f", "choice_type": "single"} {"question": "Women undergoing infertility treatment are advised to avoid NSAIDs in preovulatory period because", "exp": "Prostaglandin stimulates smooth muscle contraction in ovary, contributing to ovulation.  prostaglandin synthetase inhibitors if taken in pre ovulatory period can cause luteinized unruptured follicle syndrome (LUFS).", "cop": 1, "opa": "Prostaglandins contribute to ovulation", "opb": "NSAIDs inhibit LH surge", "opc": "NSAIDs cause premature rupture of follicle", "opd": "NSAIDs like aspirin cause endometrial shedding.", "subject_name": "Gynaecology & Obstetrics", "topic_name": null, "id": "244adc66-622f-47c1-9f4d-4f328199edb5", "choice_type": "single"} {"question": "A women at 8 months of pregnancy complains of abdominal pain and slight vaginal bleed. On examination the uterus is tender with size more than expected for dates with absent fetal hea sounds. The diagnosis", "exp": "This is a case of abruption with concealed retroplacental bleed which makes the uterus tender and also increase the uterine height. Please note that there may be some amount of vaginal bleeding in concealed hemorrhage as well. The uterine tenderness is also because of bruising of the uterine surface. With abruption the fetal hea sounds are often not heard due to the taut abdominal wall and hypeonic uterus, even when the fetus is alive. If the fetal hea rate is normal, the best management here is to induce labour,", "cop": 2, "opa": "Hydramnios", "opb": "Concealed haemorrhage", "opc": "Active labour", "opd": "Uterine rupture", "subject_name": "Gynaecology & Obstetrics", "topic_name": "Antepaum Haemorrhage", "id": "6096b752-956b-4672-8b16-22dfb9e3cc9d", "choice_type": "single"} {"question": "Absolute contraindication of IUCD is", "exp": null, "cop": 2, "opa": "Endometriosis", "opb": "Pelvic tuberculosis", "opc": "Dysmenorrhea", "opd": "Iron deficiency anemia", "subject_name": "Gynaecology & Obstetrics", "topic_name": null, "id": "50b954a6-de2b-4f45-ad03-5dfc063fba30", "choice_type": "single"} {"question": "Mature graffian follicle preovulatory is less than", "exp": "18 mm", "cop": 3, "opa": "12mm", "opb": "1.4mm", "opc": "18 mm", "opd": "25mm", "subject_name": "Gynaecology & Obstetrics", "topic_name": null, "id": "af54ed54-3609-4bdf-b9ca-87f85d268dcd", "choice_type": "single"} {"question": "The first trimester completes by", "exp": "Historically, the first trimester extends through completion of 14 weeks, the second through 28 weeks, and the third includes the 29th through 42nd weeks of pregnancy. Thus, there are three periods of 14 weeks each. (Ref: William's Obstetrics; 25th edition)", "cop": 3, "opa": "12 completed weeks", "opb": "13 completed weeks", "opc": "14 completed weeks", "opd": "16 completed weeks", "subject_name": "Gynaecology & Obstetrics", "topic_name": "All India exam", "id": "dab20607-d2df-4244-8305-3563ca71e41b", "choice_type": "single"} {"question": "Lamellar bodies in amniotic fluid, used to assess fetal lung maturity are derived from", "exp": "Lamellar body count is rapid, simple and accurate method to assess fetal lung maturity that is comparable to L/S ratio.", "cop": 3, "opa": "Syncytiotrophoblast", "opb": "Pulmonary vascular endothelium", "opc": "Type II pneumocytes", "opd": "Chorionic epithelium", "subject_name": "Gynaecology & Obstetrics", "topic_name": null, "id": "94e320c0-833e-4eb4-8d37-699d1ef42d02", "choice_type": "single"} {"question": "The first line of treatment of infertility in PCOD is", "exp": "Clomifene, also known as clomiphene, is a medication used to treat infertility in women who do not ovulate. \nThis includes those who have polycystic ovary syndrome. Use results in a greater chance of twins. It is taken by mouth once a day.", "cop": 3, "opa": "Cabergoline", "opb": "GNRH", "opc": "Clomiphene", "opd": "FSH", "subject_name": "Gynaecology & Obstetrics", "topic_name": null, "id": "4ec23564-fa22-4ae3-a19a-773e7adb220b", "choice_type": "single"} {"question": "Eclampsia occurs in Antepaum in", "exp": "Eclampsia occurs antepaum in 35-45%, intrapaum 15-20%, and postpaum in 35-45% of the cases.Reference: Practical guide to High-risk pregnancy and delivery; Chapter 16; Hypeensive disorders in pregnancy", "cop": 3, "opa": "< 10%", "opb": "15-20%", "opc": "35-45%", "opd": "> 50%", "subject_name": "Gynaecology & Obstetrics", "topic_name": "Medical, surgical and gynaecological illness complicating pregnancy", "id": "a64b2fe7-0387-419f-a612-f21780d00eed", "choice_type": "single"} {"question": "Best gas used for creating pneumoperitonium at laparoscopy is", "exp": "Ans. (c) CO2Ref: Shaws 15th ed. 1492Pnenumoperitoneum in created with CO2 or nitrous oxide. CO2 is preferred because nitrous oxide can cause explosion in the presence of volatile anaesthetic drug.", "cop": 3, "opa": "N2", "opb": "O2", "opc": "CO2", "opd": "N2O", "subject_name": "Gynaecology & Obstetrics", "topic_name": "Operative Gynaecology", "id": "484b66a0-2b5f-49c0-ac97-4274933ed579", "choice_type": "single"} {"question": "Hormones decreased in PCOD", "exp": "In PCOD, Progesterone is decreased due to anovulation.", "cop": 2, "opa": "Androgens", "opb": "Progesterone", "opc": "Estrone", "opd": "Insulin", "subject_name": "Gynaecology & Obstetrics", "topic_name": null, "id": "fafd0e91-90e3-4e99-843b-ec5f45998570", "choice_type": "single"} {"question": "An ectopic pregnancy is shed as", "exp": "The endometrial lining of the uterus is called decidua during pregnancy, If there is feilization and pregnancy occurs, the decidua differentiated intoDecidua basalis - becomes the maternal poion of the placentaDecidua - capsularis - the thin layer covering the ovumDecidua vera or parietalis - which is the rest of the decidua lining the uterine cavity outside the site of implantation.", "cop": 1, "opa": "Decidua vera", "opb": "Decidua basalis", "opc": "Decidua capsularis", "opd": "Decidua rubra", "subject_name": "Gynaecology & Obstetrics", "topic_name": "All India exam", "id": "ea32a776-76f9-4b5f-b44a-fcbe94b16fc6", "choice_type": "single"} {"question": "The indication for amnioinfusion is", "exp": "Amnioinfusion with warm saline can be performed in the presence of variable decelerations due to cord compression in oligohydramnios. \nThe role of amnioinfusion in cases of meconium-stained amniotic fluid is not clear. It is routinely used to dilute the meconium and reduce the chance of meconium aspiration syndrome.", "cop": 4, "opa": "Fetal distress", "opb": "Suspected Renal anomalies", "opc": "To facilitate labour", "opd": "Oligohydramnios", "subject_name": "Gynaecology & Obstetrics", "topic_name": null, "id": "9b23afc9-b422-45e4-8e48-50f9db9866d6", "choice_type": "single"} {"question": "In long protocol IVF, GnRH agonist is staed on", "exp": "Gonadotropin-Releasing Hormone Agonist ProtocolsIn the long protocol, a GnRH agonist is staed in the luteal phase (day 21) of the previous cycle. This diminishes the GnRH agonists flare effect and suppresses endogenous FSH and dominant follicle selection to promote synchronous follicular growth. After 10 to 14 days of GnRH agonist administration, a pelvic ultrasound and estradiol level are used to confirm suppression and gonadotropin stimulation begins. The GnRH agonist is continued (the dose may be halved or unchanged) throughout the cycle until the hCG trigger.The long protocol provides for better oocyte yields and pregnancy rates in normal responders when compared with shoer protocols that use later administration or early cessation of agonists. GnRH microdose flare protocols have been developed that may improve oocyte yield in poor responders. Microdose flare regimens involve pretreatment with 14 to 21 days of combination oral contraceptives.Four days following the cessation of the oral contraceptive pills, a microdose (leuprolide 25 mg) of agonist is added in the early follicular phase to take advantage of the agonist flare effect. Gonadotropin stimulation is initiated 1 to 2 days later while continuing the agonist.Reference: Novak's Gynecology; 14th edition; Chapter 32; Infeility and Assisted Reproductive Technology", "cop": 2, "opa": "Day 1 of previous cycle", "opb": "Day 21 of previous cycle", "opc": "Day 1 of IVF cycle", "opd": "Day 21 of IVF cycle", "subject_name": "Gynaecology & Obstetrics", "topic_name": "Infertility", "id": "ca90b7c6-a821-4417-a734-8ac01745aff1", "choice_type": "single"} {"question": "Simple vesicovaginal fistula is having following feature", "exp": "Simple vesicovaginal fistula is located near the cuff (supratrigonal)", "cop": 1, "opa": "Supratrigonal", "opb": "Short vaginal length", "opc": "Associated pelvic malignancy", "opd": "Due to pelvic radiation", "subject_name": "Gynaecology & Obstetrics", "topic_name": null, "id": "b6319420-5aff-4970-9448-d0f3cd826cd7", "choice_type": "single"} {"question": "Le Fort's operation is done is", "exp": "(Elderly menopausal patients with advanced prolapse) (329-S) (307-S14th)Operative treatment of ProlapseAnterior colporrhaphyOperation is performed to repair a cystocele and cystourethrocelePosterior-colporrhaphy and colpoperineorrhaphyOperation is done to correct a rectocele and repair a deficient perineumFothergill's repair (Manchester operation)In this operation, the surgeon combines an anterior colporrhaphy with amputation of cervix* It is suitable for women under 40 years who are desirous of retaining their menstrual and preporductive functionsVaginal hysterectomy with pelvic floor repairThis operation is suitable for women over the age of 40 years, those who have completed their families and are no longer keen on retaining their childbearing and menstrual functionsLe Fort's repairOperation is reserved for the very elderly menopausal patient with an advanced prolapse or for those women who are poor medical risks and are considered unfit for any major surgical procedureAbdominal sling operationsI) AbdominocervicopexyII) Shirodkar's abdominal sling- operationIII) Khanna's abdominal sling operation* This operation is designed for young women suffering from second or third degree uterovaginal prolapse", "cop": 1, "opa": "Elderly menopausal patients with advanced prolase", "opb": "Young woman suffering from second or third degree prolapse", "opc": "Women under 40 years who are desirous of retaining their menstrual and reproduction function", "opd": "Women over 40 years, those who have completed their families", "subject_name": "Gynaecology & Obstetrics", "topic_name": "Miscellaneous (Gynae)", "id": "85f84b62-63ae-44c7-8dbf-de05c73fcba3", "choice_type": "single"} {"question": "Turner syndrome is", "exp": "Ans. (a) 45 XORef: Shaws 15th ed. H10-111; Harrison 19/e 635; William's Gynecology Ch 16", "cop": 1, "opa": "45 XO", "opb": "47 XXY", "opc": "Trisomy 13", "opd": "Trisomy 18", "subject_name": "Gynaecology & Obstetrics", "topic_name": "Choriocarcinoma", "id": "059329c7-1cb3-4a71-8339-d8e995c39d40", "choice_type": "single"} {"question": "Most common twin after assisted reproduction is", "exp": "(C) Dizygotic twin[?]Prevalence of Dizygotic twins is related to:Race: The frequency is highest amongst Negroes, lowest amongst Mongols & intermediate amongst Caucasians.Hereditary: There is hereditary predisposition likely to be more transmitted through the female (maternal side).Advancing age of the mother: Increased incidence of twinning is observed with the advancing age of the mother, the maximum being between the age of 30-35 years. The incidence of twins is markedly reduced thereafter.Influence of parity: The incidence is increased with increasing parity specially from 5th gravida onwards.Iatrogenic: Drugs used for induction of ovulation may produce multiple fetuses to the extent of 20-40% following gonadotrophin therapy, although to a lesser extent (5-6%) following clomiphene citrate.Dizygotic twins:There are two placentae, either completely separated or more commonly fused at the margin appearing to be one (9 out of 10). There is no anastomosis between the two fetal vessels.Each fetus is surrounded by a separate amnion & chorion.As such, intervening membranes consist of 4 layers - amnion, chorion, chorion & amnion. In fact in early pregnancy the decidua capsularis of each sac may be identified under the microscope in between the chorionic layers. Other Options[?]Monozygotic twins:The placenta is single. There is varying degree of free anastomosis between the two fetal vessels.Each fetus is surrounded by a separate amniotic sac with the chorionic layer common to both (Diamniotic-monochorionic).As such the intervening membranes consist of two layers of amnion only. However, on rare occasions, the uniovular twins may be diamniotic-dichorionic or mono-amniotic-monochorionic.[?]Fetus acardiacus:It occurs only in monozygotic twins. Part of one fetus remains amorphous & becomes parasitic without a heart.", "cop": 3, "opa": "Acardiac twin", "opb": "Monozygotic twin", "opc": "Dizygotic twin", "opd": "Conjoined twin", "subject_name": "Gynaecology & Obstetrics", "topic_name": "Multiple Pregnancy", "id": "975861eb-2afe-4687-bc38-ec45693d2fe4", "choice_type": "single"} {"question": "Chromosomal number of primary cnermatecvte is", "exp": "Ans. is a i.e. 44XY", "cop": 1, "opa": "44 XY", "opb": "22 XY", "opc": "22 XX", "opd": "46 XX", "subject_name": "Gynaecology & Obstetrics", "topic_name": null, "id": "143da966-f4fa-4e3b-a805-ce5b421b97c0", "choice_type": "single"} {"question": "Child with history of Rh incompatibility is brought to OPD with jaundice, irritability and convulsions most common cause", "exp": "(A) Kernicterus # Kernicterus is the pathological findings of bilirubin (un-conjugated) toxicity within the brain.> The main transporter of bilirubin is albumin therefore increase in albumin causes less bilirubin to be transported to the brain> This includes staining & necrosis of neurons in the basal ganglia, hippocampal cortex, subthalamic nuclei and cerebellum followed by gliosis of these areas.> The cerebral cortex is generally spared, but 50% of babies have extra neuronal lesions with necrosis of renal tubular cells, intestinal mucosa and pancreatic cells.> These may manifest as gastrointestinal hemorrhage or hematuria.> Clinically, kernicterus is described in phases, which may progress over 24 hours to 7 days", "cop": 1, "opa": "Kemicterus", "opb": "Physiological jaundice", "opc": "Hyperpyraxia", "opd": "Meningitis", "subject_name": "Gynaecology & Obstetrics", "topic_name": "Miscellaneous (Obs)", "id": "e872ee6f-e413-44d4-ab9a-4508da3da15f", "choice_type": "single"}